*NURSING > EXAM > A&E I Comprehensive Testbank (Rated A) (All)

A&E I Comprehensive Testbank (Rated A)

Document Content and Description Below

A&E I Comprehensive Testbank Table of Contents Week 1 Care of Older Adults: Culture, Spirituality, Communication, Sexuality, Infection Control Chronic Illness and Older Adults_____________________... _____________________________3 Cultural Awareness____________________________________________________________10 Older Adult__________________________________________________________________18 Communication_______________________________________________________________28 Patient Education_____________________________________________________________39 Infection Prevention and Control_________________________________________________51 Sexuality____________________________________________________________________69 Spiritual Health______________________________________________________________78 Week 2 Critical Thinking, The Nursing Process, Loss, Death, and Grief Critical Thinking in Nursing Practice_____________________________________________86 Nursing Assessment___________________________________________________________94 Nursing Diagnosis___________________________________________________________102 Planning Nursing Care________________________________________________________110 Implementing Nursing Care____________________________________________________119 Evaluation__________________________________________________________________128 The Experience of Loss, Death and Grief__________________________________________136 Week 3 Safety and Fall Prevention among Older Adults, Preventing Complications of Immobility Patient Safety and Quality_____________________________________________________146 Immobility__________________________________________________________________163 Activity and Exercise__________________________________________________________181 Week 4 Skin and Wound Care Hygiene, Introduction to Pharmacology and Medication Administration Medication Administration_____________________________________________________194 Hygiene____________________________________________________________________212 Skin Integrity and Wound Care__________________________________________________231 Week 5 Fluid & Electrolytes, Dehydration Fluid, Electrolyte and Acid-Base Balance_________________________________________251 Week 6 Pain and Sleep 1 A&E I Comprehensive Testbank Pain Management____________________________________________________________268 Sleep______________________________________________________________________280 Week 7 Concepts related to Oxygenation, Circulation, and Tissue Perfusion, Chronic Obstructive Pulmonary Disease Oxygenation________________________________________________________________295 Obstructive Pulmonary Diseases________________________________________________314 Week 8 Diabetes Mellitus Diabetes Mellitus____________________________________________________________331 Week 9 Hypertension, Stroke Hypertension________________________________________________________________341 Stroke_____________________________________________________________________356 Week 10 Documentation and Informatics Documentation and Informatics______________________________________368 Week 11 Nutrition, Dysphagia_____________________________________382 Week 12 Care of the Surgical Patient________________________________398 Week 1 Care of Older Adults: Culture, Spirituality, Communication, Sexuality, Infection Control Chapter 05: Chronic Illness and Older Adults Lewis: Medical-Surgical Nursing, 10th Edition MULTIPLE CHOICE 1. When caring for an older patient with hypertension who has been hospitalized after a transient ischemic (TIA), which topic is the most important for the nurse to include in the discharge teaching? a) Effect of atherosclerosis on blood vessels b) Mechanism of action of anticoagulant drug therapy c) Symptoms indicating that the patient should contact the health care provider d) Impact of the patient’s family history on likelihood of developing a serious stroke ANS: C One of the tasks for patients with chronic illnesses is to prevent and manage a crisis. The patient 2 A&E I Comprehensive Testbank needs instruction on recognition of symptoms of hypertension and TIA and appropriate actions to take if these symptoms occur. The other information may also be included in patient teaching but is not as essential in the patient’s self-management of the illness. 2. The nurse performs a comprehensive assessment of an older patient who is considering admission to an assisted living facility. Which question is the most important for the nurse to ask? a) “Have you had any recent infections?” b) “How frequently do you see a doctor?” c) “Do you have a history of heart disease?” d) “Are you able to prepare your own meals?” ANS: D The patient’s functional abilities, rather than the presence of an acute or chronic illness, are more useful in determining how well the patient might adapt to an assisted living situation. The other questions will also provide helpful information but are not as useful in providing a basis for determining patient needs or for developing interventions for the older patient. 3. An alert older patient who takes multiple medications for chronic cardiac and pulmonary diseases lives with a daughter who works during the day. During a clinic visit, the patient verbalizes to the nurse that she has a strained relationship with her daughter and does not enjoy being alone all day. Which nursing diagnosis should the nurse assign as the priority for this patient? a) Social isolation related to fatigue b) Risk for injury related to drug interactions c) Caregiver role strain related to family employment schedule d) Compromised family coping related to the patient’s care needs ANS: B The patient’s age and multiple medications indicate a risk for injury caused by interactions between the multiple drugs being taken and a decreased drug metabolism rate. Problems with social isolation, caregiver role strain, or compromised family coping are not physiologic priorities. Drug–drug interactions could cause the most harm to the patient and are therefore the priority. 4. Which method should the nurse use to gather the most complete assessment of an older patient? a) Review the patient’s health record for previous assessments. b) Use a geriatric assessment instrument to evaluate the patient. c) Ask the patient to write down medical problems and medications. d) Interview both the patient and the primary caregiver for the patient. 3 A&E I Comprehensive Testbank ANS: B The most complete information about the patient will be obtained through the use of an assessment instrument specific to the geriatric population, which includes information about both medical diagnoses and treatments and about functional health patterns and abilities. A review of the medical record, interviews with the patient and caregiver, and written information by the patient are all included in a comprehensive geriatric assessment. 5. Which intervention should the nurse implement to provide optimal care for an older patient who is hospitalized with pneumonia? a) Plan for transfer to a long-term care facility. b) Minimize activity level during hospitalization. c) Consider the preadmission functional abilities. d) Use an approved standardized geriatric nursing care plan. ANS: C The plan of care for older adults should be individualized and based on the patient’s current functional abilities. A standardized geriatric nursing care plan will not address individual patient needs and strengths. A patient’s need for discharge to a long-term care facility is variable. Activity level should be designed to allow the patient to retain functional abilities while hospitalized and also to allow any additional rest needed for recovery from the acute process. 6. The nurse cares for an older adult patient who lives in a rural area. Which intervention should the nurse plan to implement to meet this patient’s needs? a) Suggest that the patient move closer to health care providers. b) Obtain extra medications for the patient to last for 4 to 6 months. c) Ensure transportation to appointments with the health care provider. d) Assess the patient for chronic diseases that are unique to rural areas. ANS: C Transportation can be a barrier to accessing health services in rural areas. The patient living in a rural area may lose the benefits of a familiar situation and social support by moving to an urban area. There are no chronic diseases unique to rural areas. Because medications may change, the nurse should help the patient plan for obtaining medications through alternate means such as the mail or delivery services, not by purchasing large quantities of the medications. 7. Which nursing action will be most helpful in decreasing the risk for drug-drug interactions in an older adult? a) Teach the patient to have all prescriptions filled at the same pharmacy. b) Make a schedule for the patient as a reminder of when to take each medication. c) Instruct the patient to avoid taking over-the-counter (OTC) medications or supplements. d) Ask the patient to bring all medications, supplements, and herbs to each appointment. 4 A&E I Comprehensive Testbank ANS: D The most information about drug use and possible interactions is obtained when the patient brings all prescribed medications, OTC medications, and supplements to every health care appointment. The patient should discuss the use of any OTC medications with the health care provider and obtain all prescribed medications from the same pharmacy, but use of supplements and herbal medications also need to be considered in order to prevent drug–drug interactions. Use of a medication schedule will help the patient take medications as scheduled, but will not prevent drug–drug interactions. 8. A patient who has just moved to a long-term care facility has a nursing diagnosis of relocation stress syndrome. Which action should the nurse include in the plan of care? a) Remind the patient that making changes is usually stressful. b) Discuss the reason for the move to the facility with the patient. c) Restrict family visits until the patient is accustomed to the facility. d) Have staff members write notes welcoming the patient to the facility. ANS: D TestBankWorld.org Having staff members write notes will make the patient feel more welcome and comfortable at the long-term care facility. Discussing the reason for the move and reminding the patient that change is usually stressful will not decrease the patient’s stress about the move. Family member visits will decrease the patient’s sense of stress about the relocation. 9. An older patient complains of having “no energy” and feeling increasingly weak. The patient has had a 12-lb weight loss over the past year. Which action should the nurse take initially? a) Ask the patient about daily dietary intake. b) Schedule regular range-of-motion exercise. c) Discuss long-term care placement with the patient. d) Describe normal changes associated with aging to the patient. ANS: A In a frail older patient, nutrition is frequently compromised, and the nurse’s initial action should be to assess the patient’s nutritional status. Active range of motion may be helpful in improving the patient’s strength and endurance, but nutritional assessment is the priority because the patient has had a significant weight loss. The patient may be a candidate for long-term care placement, but more assessment is needed before this can be determined. The patient’s assessment data are not consistent with normal changes associated with aging. 10. The nurse is admitting an acutely ill, older patient to the hospital. Which action should the nurse take? a) Speak slowly and loudly while facing the patient. b) Obtain a detailed medical history from the patient. 5 A&E I Comprehensive Testbank c) Perform the physical assessment before interviewing the patient. d) Ask a family member to go home and retrieve the patient’s cane. ANS: C When a patient is acutely ill, the physical assessment should be accomplished first to detect any physiologic changes that require immediate action. Not all older patients have hearing deficits, and it is insensitive of the nurse to speak loudly and slowly to all older patients. To avoid tiring the patient, much of the medical history can be obtained from medical records. After the initial physical assessment to determine the patient’s current condition, then the nurse could ask someone to obtain any assistive devices for the patient if applicable. 11. The nurse cares for an alert, homeless older adult patient who was admitted to the hospital with a chronic foot infection. Which intervention is the most appropriate for the nurse to include in the discharge plan for this patient? a) Teach the patient how to assess and care for the foot infection. b) Refer the patient to social services for assessment of resources. c) Schedule the patient to return to outpatient services for foot care. d) Give the patient written information about shelters and meal sites. ANS: B An interprofessional approach, including social services, is needed when caring for homeless older adults. Even with appropriate teaching, a homeless individual may not be able to maintain adequate foot care because of a lack of supplies or a suitable place to accomplish care. Older homeless individuals are less likely to use shelters or meal sites. A homeless person may fail to keep appointments for outpatient services because of factors such as fear of institutionalization or lack of transportation. 12. The home health nurse cares for an older adult patient who lives alone and takes several different prescribed medications for chronic health problems. Which intervention, if implemented by the nurse, would best encourage medication compliance? a) Use a marked pillbox to set up the patient’s medications. b) Discuss the option of moving to an assisted living facility. c) Remind the patient about the importance of taking medications. d) Visit the patient daily to administer the prescribed medications. ANS: A Because forgetting to take medications is a common cause of medication errors in older adults, the use of medication reminder devices is helpful when older adults have multiple medications to take. There is no indication that the patient needs to move to assisted living or that the patient does not understand the importance of medication compliance. Home health care is not designed for the patient who needs ongoing assistance with activities of daily living or instrumental ADLs. 6 A&E I Comprehensive Testbank 13. The home health nurse visits an older patient with mild forgetfulness. Which new information is of most concern to the nurse? a) The patient tells the nurse that a close friend recently died. b) The patient has lost 10 lb (4.5 kg) during the past month. c) The patient is cared for by a daughter during the day and stays with a son at night. d) The patient’s son uses a marked pillbox to set up the patient’s medications weekly. ANS: B A 10-pound weight loss may be an indication of elder neglect or depression and requires further assessment by the nurse. The use of a marked pillbox and planning by the family for 24-hour care are appropriate for this patient. It is not unusual that an 86-yr-old would have friends who have died. 14. Which statement, if made by an older adult patient, would be of most concern to the nurse? a. “I prefer to manage my life without much help from other people.” a) “I prefer to manage my life without much help from other people.” b) “I take three different medications for my heart and joint problems.” c) “I don’t go on daily walks anymore since I had pneumonia 3 months ago.” d) “I set up my medications in a marked pillbox so I don’t forget to take them.” ANS: C Inactivity and immobility lead rapidly to loss of function in older adults. The nurse should develop a plan to prevent further deconditioning and restore function for the patient. Selfmanagement is appropriate for independently living older adults. On average, an older adult takes seven different medications so the use of three medications is not unusual for this patient. The use of memory devices to assist with safe medication administration is recommended for older adults. 15. The nurse assesses an older patient who takes diuretics and has a possible urinary tract infection (UTI). Which action should the nurse take first? a) Palpate over the suprapubic area. b) Inspect for abdominal distention. c) Question the patient about hematuria. d) Request the patient empty the bladder. ANS: D Before beginning the assessment of an older patient with a UTI and on diuretics, the nurse should have the patient empty the bladder because bladder fullness or discomfort will distract from the patient’s ability to provide accurate information. The patient may seem disoriented if distracted by pain or urgency. The physical assessment data are obtained after the patient is as comfortable as possible. 7 A&E I Comprehensive Testbank 16. Which patient is most likely to need long-term nursing care management? a) 72-yr-old who had a hip replacement after a fall at home b) 64-yr-old who developed sepsis after a ruptured peptic ulcer c) 76-yr-old who had a cholecystectomy and bile duct drainage d) 63-yr-old with bilateral knee osteoarthritis who weighs 350 lb (159 kg) ANS: D Osteoarthritis and obesity are chronic problems that will require planning for long-term interventions such as physical therapy and nutrition counseling. The other patients have acute problems that are not likely to require long-term management. 17. An older adult being admitted is assessed at high risk for falls. Which action should the nurse take first? a) Use a bed alarm system on the patient’s bed. b) Administer the prescribed PRN sedative medication. c) Ask the health care provider to order a vest restraint. d) Place the patient in a “geri-chair” near the nurse’s station. ANS: A The use of the least restrictive restraint alternative is required. Physical or chemical restraints may be necessary, but the nurse’s first action should be an alternative such as a bed alarm. 18. An older adult patient presents with a broken arm and visible scattered bruises healing at different stages. Which action should the nurse take first? a) Notify an elder protective services agency about possible abuse. b) Make a referral for a home assessment visit by the home health nurse. c) Have the family member stay in the waiting area while the patient is assessed. d) Ask the patient how the injury occurred and observe the family member’s reaction. ANS: C The initial action should be assessment and interviewing of the patient. The patient should be interviewed alone because the patient will be unlikely to give accurate information if the abuser is present. If abuse is occurring, the patient should not be discharged home for a later assessment by a home health nurse. The nurse needs to collect and document data before notifying the elder protective services agency. 19. The family of an older patient with chronic health problems and increasing weakness is considering placement in a long-term care (LTC) facility. Which action by the nurse will be most helpful in assisting the patient to make this transition? a) Have the family select a LTC facility that is relatively new. b) Ask the patient’s preference for the choice of a LTC facility. 8 A&E I Comprehensive Testbank c) Explain the reasons for the need to live in LTC to the patient. d) Request that the patient be placed in a private room at the facility. ANS: B The stress of relocation is likely to be less when the patient has input into the choice of the facility. The age of the long-term care facility does not indicate a better fit for the patient or better quality of care. Although some patients may prefer a private room, others may adjust better when given a well-suited roommate. The patient should understand the reasons for the move but will make the best adjustment when involved with the choice to move and the choice of the facility. 20. The nurse manages the care of older adults in an adult health day care center. Which action can the nurse delegate to unlicensed assistive personnel (UAP)? TestBankWorld.org a) Obtain information about food and medication allergies from patients. b) Take blood pressures daily and document in individual patient records. c) Choose social activities based on the individual patient needs and desires. d) Teach family members how to cope with patients who are cognitively impaired. ANS: B Measurement and documentation of vital signs are included in UAP education and scope of practice. Obtaining patient health history, planning activities based on the patient assessment, and patient education are all actions that require critical thinking and will be done by the registered nurse. MULTIPLE RESPONSE 1. Which nursing actions will the nurse take to assess for possible malnutrition in an older adult patient (select all that apply)? a) Assess for depression. b) Review laboratory results. c) Determine food preferences. d) Inspect teeth and oral mucosa. e) Ask about transportation needs. ANS: A, B, D, E The laboratory results, especially albumin and cholesterol levels, may indicate chronic poor protein intake or high-fat or high-cholesterol intake. Transportation affects the patient’s ability to shop for groceries. Depression may lead to decreased appetite. Oral sores or teeth in poor condition may decrease the ability to chew and swallow. Food likes and dislikes are not necessarily associated with malnutrition. 9 A&E I Comprehensive Testbank Chapter 09: Cultural Awareness Potter et al.: Fundamentals of Nursing, 9th Edition MULTIPLE CHOICE 1. A nurse is working at a health fair screening people for liver cancer. Which population group should the nurse monitor most closely for liver cancer? ANS: B While Asian Americans generally have lower cancer rates than the non-Hispanic Caucasian population, they also have the highest incidence rates of liver cancer for both sexes compared with Hispanic, non-Hispanic Caucasians, or non-Hispanic African-Americans. 2. A nurse is caring for an immigrant with low income. Which information should the nurse consider when planning care for this patient? ANS: B Populations with health disparities (immigrant with low income) have a significantly increased incidence of disease or increased morbidity and mortality when compared with the general population. Although Americans’ health overall has improved during the past few decades, the health of members of marginalized groups has actually declined. 3. A nurse is assessing the health care disparities among population groups. Which area is the nurse monitoring? ANS: A While health disparities are the differences among populations in the incidence, prevalence, and outcomes of health conditions, diseases and related complications, health care disparities are differences among populations in the availability, accessibility, and quality of health care services (e.g. screening, diagnostic, treatment, management, and rehabilitation) aimed at prevention, treatment, and management of diseases and their complications. 4. A nurse is providing care to a patient from a different culture. Which action by the nurse indicates cultural competence? a. Hispanic b. Asian Americans c. Non-Hispanic Caucasians d. Non-Hispanic African-Americans a. There is a decreased frequency of morbidity. b. There is an increased incidence of disease. c. There is an increased level of health. d. There is a decreased mortality rate. a. Accessibility of health care services b. Outcomes of health conditions c. Prevalence of complications d. Incidence of diseases a. Communicates effectively in a multicultural context b. Functions effectively in a multicultural context c. Visits a foreign country d. Speaks a different language 10 A&E I Comprehensive Testbank ANS: B Cultural competence refers to a developmental process that evolves over time that impacts ability to effectively function in the multicultural context. Communicates effectively and speaking a different language indicates linguistic competence. Visiting a foreign country does not indicate cultural competence. 5. The nurse learns about cultural issues involved in the patient’s health care belief system and enables patients and families to achieve meaningful and supportive care. Which concept is the nurse demonstrating? ANS: D The nurse is demonstrating culturally congruent care. Culturally congruent care, or care that fits a person’s life patterns, values, and system of meaning, provides meaningful and beneficial nursing care. Marginalized groups are populations left out or excluded. Health care disparities are differences among populations in the availability, accessibility, and quality of health care services (e.g. screening, diagnostic, treatment, management, and rehabilitation) aimed at prevention, treatment, and management of diseases and their complications. Transcultural nursing is a comparative study of cultures in order to understand their similarities (culture that is universal) and the differences among them (culture that is specific to particular groups). 6. A nurse is beginning to use patient-centered care and cultural competence to improve nursing care. Which step should the nurse take first? ANS: A Becoming more aware of your biases and attitudes about human behavior is the first step in providing patientcentered care, leading to culturally competent care. It is helpful to think about cultural competence as a lifelong process of learning about others and also about yourself. Learning about the world view, developing cultural skills, and understanding organizational forces are not the first steps. 7. A nurse is performing a cultural assessment using the ETHNIC mnemonic for communication. Which area will the nurse assess for the “H”? ANS: B The “H” in ETHNIC stands for healers: Has the patient sought advice from alternative health practitioners? While health, history, and homeland are important, they are not components of “H.” 8. The nurse is caring for a patient of Hispanic descent who speaks no English. The nurse is working with an interpreter. Which action should the nurse take? a. Marginalized groups b. Health care disparity c. Transcultural nursing d. Culturally congruent care a. Assessing own biases and attitude b. Learning about the world view of others c. Understanding organizational forces d. Developing cultural skills a. Health b. Healers c. History d. Homeland a. Use long sentences when talking. b. Look at the patient when talking. 11 A&E I Comprehensive Testbank ANS: B Direct your questions to the patient. Look at the patient, instead of looking at the interpreter. Pace your speech by using short sentences, but do not break your sentences. Observe the patient’s nonverbal and verbal behaviors. 9. Which action indicates the nurse is meeting a primary goal of cultural competent care for patients? ANS: A Although cultural competence and patient-centered care both aim to improve health care quality, their focus is slightly different. The primary aim of cultural competence care is to reduce health disparities and increase health equity and fairness by concentrating on people of color and other marginalized groups, like transgender patients. Patient-centered care, rather than cultural competence care, provides individualized care and restores an emphasis on personal relationships; it aims to elevate quality for all patients. 10. The nurse is caring for a Chinese patient using the Teach-Back technique. Which action by the nurse indicates successful implementation of this technique? ANS: C The Teach-Back technique asks open-ended questions, like what will you tell your spouse about changing the dressing, to verify a patient’s understanding. When using the Teach-Back technique do not ask a patient, “Do you understand?” or “Do you have any questions?” Does this make sense and do you think you can do this at home are closed-ended questions. Would you tell me if you don’t understand something so we can go over it is not verifying a patient’s understanding about the teaching. 11. A nurse is using core measures to reduce health disparities. Which group should the nurse focus on to cause the most improvement in core measures? ANS: B To improve results, the nurse should focus on the highest disparity. Poor people received worse care than highincome people for about 60% of core measures. American Indians and Alaska Natives received worse care than Caucasians for about 30% of core measures. 12. A nurse is designing a form for lesbian, gay, bisexual, and transgender (LGBT) patients. Which design should the nurse use? c. Use breaks in sentences when talking. d. Look at only nonverbal behaviors when talking. a. Provides care to transgender patients b. Provides care to restore relationships c. Provides care to patients that is individualized d. Provides care to surgical patients a. Asks, “Does this make sense?” b. Asks, “Do you think you can do this at home?” c. Asks, “What will you tell your spouse about changing the dressing?” d. Asks, “Would you tell me if you don’t understand something so we can go over it?” a. Caucasians b. Poor people c. Alaska Natives d. American Indians a. Use partnered rather than married. 12 A&E I Comprehensive Testbank ANS: A Include LGBT-inclusive language on forms and assessments to facilitate disclosure, knowing that disclosure is a choice impacted by many factors. For example, provide options such as “partnered” under relationship status. For parents, use parent/guardian, instead of mother/father. Use neutral and inclusive language when talking with patients (e.g., partner or significant other), listening and reflecting patient’s choice. Remember that some LGBT patients are also legally married. 13. A nurse is assessing population groups for the risk of suicide requiring medical attention. Which group should the nurse monitor mostclosely? ANS: A Gay, lesbian, and bisexual young people have a significantly increased risk for depression, anxiety, suicide attempts, and substance use disorders, being 4 times as likely as their straight peers to make suicide attempts that require medical attention. Caucasian youth, Asian Americans, and African-Americans are not as likely to attempt suicide resulting in medical attention. 14. A nurse is assessing a patient’s ethnohistory. Which question should the nurse ask? ANS: B An ethnohistory question is the following: How different is your life here from back home? Caring beliefs and practice questions include the following: Which caregivers do you seek when you are sick and How different is what we do from what your family does when you are sick? The language and communication is the following: What language do you speak at home? 15. A nurse is teaching patients about health care information. Which patient will the nurse assess closely for health literacy? ANS: B About 9 out of 10 people in the United States experience challenges in using health care information. Patients who are especially vulnerable are the elderly (age 65+), immigrants, persons with low incomes, persons who do not have a high-school diploma or GED, and persons with chronic mental and/or physical health conditions. A 35-year-old patient and patients with high-school and college education are not identified in the vulnerable populations. b. Use mother rather than father. c. Use parents rather than guardian. d. Use wife/husband rather than significant other. a. Young bisexuals b. Young caucasians c. Asian Americans d. African-Americans a. What language do you speak at home? b. How different is your life here from back home? c. Which caregivers do you seek when you are sick? d. How different is what we do from what your family does when you are sick? a. A patient 35 years old b. A patient 68 years old c. A patient with a college degree d. A patient with a high-school diploma 13 A&E I Comprehensive Testbank 16. A nurse works at a hospital that uses equity-focused quality improvement. Which strategy is the hospital using? ANS: D Organizations can implement equity-focused quality improvement by recognizing disparities and committing to reducing them. Staff diversity is a priority for equity-focused quality improvement, not staff satisfaction. While the family is important, the focus is on the patients. Organizations should start by implementing a change on a small scale (pilot testing), learning from each test, and refining the intervention through performance improvement cycles (e.g., plan, do, study, and act). 17. A nurse is providing care to a culturally diverse population. Which action indicates the nurse is successful in the role of providing culturally congruent care? ANS: A The goal of transcultural nursing is to provide culturally congruent care, or care that fits the person’s life patterns, values, and system of meaning. Patterns and meanings are generated from people themselves, rather than from predetermined criteria. Discovering patients’ cultural values, beliefs, and practices as they relate to nursing and health care requires you to assume the role of learner (not become the leader) and to partner with your patients and their families to determine what is needed to provide meaningful and beneficial nursing care. Culturally congruent care is sometimes different from the values and meanings of the professional health care system. 18. A nurse is assessing the patient’s meaning of illness. Which area of focus by the nurse is priority? ANS: A To provide culturally congruent care, you need to understand the difference between disease and illness. Illness is the way that individuals and families react to disease, whereas disease is a malfunctioning of biological or psychological processes. The way a patient interacts to family/social interactions is communication processes and family dynamics. MULTIPLE RESPONSE 1. A nurse is using Campinha-Bacote’s model of cultural competency. Which areas will the nurse focus on to become competent? (Select all that apply.) a. Document staff satisfaction. b. Focus on the family. c. Implement change on a grand scale. d. Reduce disparities. a. Provides care that fits the patient’s valued life patterns and set of meanings b. Provides care that is based on meanings generated by predetermined criteria c. Provides care that makes the nurse the leader in determining what is needed d. Provides care that is the same as the values of the professional health care system a. On the way a patient reacts to disease b. On the malfunctioning of biological processes c. On the malfunctioning of psychological processes d. On the way a patient reacts to family/social interactions a. Cultural skills b. Cultural desire c. Cultural transition d. Cultural knowledge 14 A&E I Comprehensive Testbank ANS: A, B, D, E Campinha-Bacote’s model of cultural competency has five interrelated components: cultural awareness; cultural knowledge; cultural skills; cultural encounters; and cultural desire. Cultural transition is not a component of this model. 2. A nurse is using the RESPECT mnemonic to establish rapport, the “R” in RESPECT. Which actions should the nurse take? (Select all that apply.) ANS: A, C The “R” in RESPECT stands for rapport and includes the following behaviors: connect on a social level; seek the patient’s point of view; and consciously attempt to suspend judgment. The “S” stands for support and includes the behavior of helping the patient overcome barriers. The “P” stands for partnership and includes the following behaviors: be flexible with regard to issues of control and stress that you will be working together to address medical problems. The “C” stands for cultural competence and includes the behavior of knowing your limitations in addressing medical issues across cultures. 3. A nurse is using the explanatory model to determine the etiology of an illness. Which questions should the nurse ask? (Select all that apply.) ANS: B, C, E The questions for etiology include “What do you call your problem?” and “What name does it have?” Recommended treatment is asked by the question “How should your sickness be treated?” Pathophysiology is asked by the question “How does this illness work inside your body?” The course of illness is asked by the question “What do you fear most about your sickness?” MATCHING A nurse is using Campinha-Bacote’s model of cultural competency to improve cultural care. Which actions describe the components the nurse is using? e. Cultural encounters a. Connect on a social level. b. Help the patient overcome barriers. c. Consciously attempt to suspend judgment. d. Stress that they will be working together to address problems. e. Know limitations in addressing medical issues across cultures. a. How should your sickness be treated? b. What do you call your problem? c. How does this illness work inside your body? d. What do you fear most about your sickness? e. What name does it have? a. In-depth self-examination of one’s own background b. Ability to assess factors that influence treatment and care c. Sufficient comparative understanding of diverse groups d. Motivation and commitment to continue learning about cultures e. Cross-cultural interaction that develops communication skills 15 A&E I Comprehensive Testbank Chapter 14: Older Adult Potter et al.: Fundamentals of Nursing, 9th Edition MULTIPLE CHOICE 1. A nurse is obtaining a history on an older adult. Which finding will the nurse most typically find? ANS: B In 2012, 57% of older adults in non-institutional settings lived with a spouse (45% of older women, 71% of older men); 28% lived alone (35% of older women, 19% of older men); and only 3.5% of all older adults resided in institutions such as nursing homes or centers. Most older adults have lost a spouse due to death rather than divorce. 2. A nurse is developing a plan of care for an older adult. Which information will the nurse consider? ANS: B Every older adult is unique, and the nurse needs to approach each one as a unique individual. The nursing care of older adults poses special challenges because of great variation in their physiological, cognitive, and psychosocial health. Aging does not automatically lead to disability and dependence. Chronological age often has little relation to the reality of aging for an older adult. 3. Which information from a co-worker on a gerontological unit will cause the nurse to intervene? ANS: A Most older people remain functionally independent despite the increasing prevalence of chronic disease; therefore, this misconception should be addressed. It is critical for you to respect older adults and actively involve them in care decisions and activities. You also need to identify an older adult’s strengths and abilities during the assessment and encourage independence as an integral part of your plan of care. 4. A nurse suspects an older-adult patient is experiencing caregiver neglect. Which assessment findings are consistent with the nurse’s suspicions? ANS: A a. Lives in a nursing home b. Lives with a spouse c. Lives divorced d. Lives alone a. Should be standardized because most geriatric patients have the same needs b. Needs to be individualized to the patient’s unique needs c. Focuses on the disabilities that all aging persons face d. Must be based on chronological age alone a. Most older people have dependent functioning. b. Most older people have strengths we should focus on. c. Most older people should be involved in care decision. d. Most older people should be encouraged to have independence. a. Flea bites and lice infestation b. Left at a grocery store c. Refuses to take a bath d. Cuts and bruises 16 A&E I Comprehensive Testbank Caregiver neglect includes unsafe and unclean living conditions, soiled bedding, and animal or insect infestation. Abandonment includes desertion at a hospital, nursing facility, or public location such as a shopping center. Self-neglect includes refusal or failure to provide oneself with basic necessities such as food, water, clothing, shelter, personal hygiene, medication, and safety. Physical abuse includes hitting, beating, pushing, slapping, kicking, physical restraint, inappropriate use of drugs, fractures, lacerations, rope burns, and untreated injuries. 5. A nurse is teaching a group of older-adult patients. Which teaching strategy is best for the nurse to use? ANS: C Teaching strategies include the use of past experiences to connect new learning with previous knowledge, focusing on a single topic to help the patient concentrate, giving the patient enough time in which to respond because older adults’ reaction times are longer than those of younger persons, and keeping the tone of voice low; older adults are able to hear low sounds better than high-frequency sounds. 6. An older patient has fallen and suffered a hip fracture. As a consequence, the patient’s family is concerned about the patient’s ability to care for self, especially during this convalescence. What should the nurse do? ANS: D Nurses help older adults and their families by providing information and answering questions as they make choices among care options. Some older adults deny functional declines and refuse to ask for assistance with tasks that place their safety at great risk. The decision to enter a nursing center is never final, and a nursing center resident sometimes is discharged to home or to another less-acute residence. What defines quality of life varies and is unique for each person. 7. What is the best suggestion a nurse could make to a family requesting help in selecting a local nursing center? ANS: C a. Provide several topics of discussion at once to promote independence and making choices. b. Avoid uncomfortable silences after questions by helping patients complete their statements. c. Ask patients to recall past experiences that correspond with their interests. d. Speak in a high pitch to help patients hear better. a. Stress that older patients usually ask for help when needed. b. Inform the family that placement in a nursing center is a permanent solution. c. Tell the family to enroll the patient in a ceramics class to maintain quality of life. d. Provide information and answer questions as family members make choices among care options. a. Have the family members evaluate nursing home staff according to their ability to get tasks done efficiently and safely. b. Make sure that nursing home staff members get patients out of bed and dressed according to staff’s preferences. c. Explain that it is important for the family to visit the center and inspect it personally. d. Suggest a nursing center that has standards as close to hospital standards as possible. 17 A&E I Comprehensive Testbank An important step in the process of selecting a nursing home is to visit the nursing home. The nursing home should not feel like a hospital. It is a home, a place where people live. Members of the nursing home staff should focus on the person, not the task. Residents should be out of bed and dressed according to their preferences, not staff preferences. 8. A 70-year-old patient who suffers from worsening dementia is no longer able to live alone. The nurse is discussing health care services and possible long-term living arrangements with the patient’s only son. What will the nurse suggest? ANS: C Some family caregivers consider nursing center placement when in-home care becomes increasingly difficult or when convalescence from hospitalization requires more assistance than the family is able to provide. An apartment setting and the use of home health visits are not appropriate because living at home is unsafe. Dementia is not a time of inactivity but an impairment of intellectual functioning. 9. A nurse is caring for an older adult. Which goal is priority? ANS: C Adjusting to retirement is one of the developmental tasks for an older person. A young or middle-aged adult has to adjust to career and/or divorce. A middle-aged adult has to adjust to grandchildren. 10. A nurse is observing for the universal loss in an older-adult patient. What is the nurse assessing? ANS: B The universal loss for older adults usually revolves around the loss of relationships through death. Life transitions, of which loss is a major component, include retirement and the associated financial changes, changes in roles and relationships, alterations in health and functional ability, changes in one’s social network, and relocation. However, these are not the universal loss. 11. A nurse is discussing sexuality with an older adult. Which action will the nurse take? a. An apartment setting with neighbors close by b. Having the patient utilize weekly home health visits c. A nursing center because home care is no longer safe d. That placement is irrelevant because the patient is retreating to a place of inactivity a. Adjusting to career b. Adjusting to divorce c. Adjusting to retirement d. Adjusting to grandchildren a. Loss of finances through changes in income b. Loss of relationships through death c. Loss of career through retirement d. Loss of home through relocation a. Ask closed-ended questions about specific symptoms the patient may experience. b. Provide information about the prevention of sexually transmitted infections. c. Discuss the issues of sexuality in a group in a private room. d. Explain that sexuality is not necessary as one ages. 18 A&E I Comprehensive Testbank ANS: B Include information about the prevention of sexually transmitted infections when appropriate. Open-ended questions inviting an older adult to explain sexual activities or concerns elicit more information than a list of closed-ended questions about specific activities or symptoms. You need to provide privacy for any discussion of sexuality and maintain a nonjudgmental attitude. Sexuality and the need to express sexual feelings remain throughout the human life span. 12. A nurse is teaching a health promotion class for older adults. In which order will the nurse list the most common to least common conditions that can lead to death in older adults? 1. Chronic obstructive lung disease 2. Cerebrovascular accidents 3. Heart disease 4. Cancer ANS: B Heart disease is the leading cause of death in older adults followed by cancer, chronic lung disease, and stroke (cerebrovascular accidents). 13. A nurse is observing skin integrity of an older adult. Which finding will the nurse document as a normal finding? ANS: C Loss of skin elasticity is a common finding in the older adult. Other common findings include pigmentation changes, glandular atrophy (oil, moisture, and sweat glands), thinning hair (facial hair: decreased in men, increased in women), slower nail growth, and atrophy of epidermal arterioles. 14. An older-adult patient in no acute distress reports being less able to taste and smell. What is the nurse’s best response to this information? ANS: D Diminished taste and smell senses are common findings in older adults. Scheduling an appointment at a smell and taste disorders clinic, testing the vestibulocochlear nerve, or an attempt to rule out cranial nerve damage is unnecessary at this time as per the information provided. 15. A nurse is assessing an older adult for cognitive changes. Which symptom will the nurse report as normal? a. 4, 1, 2, 3 b. 3, 4, 1, 2 c. 2, 3, 4, 1 d. 1, 2, 3, 4 a. Oily skin b. Faster nail growth c. Decreased elasticity d. Increased facial hair in men a. Notify the health care provider immediately to rule out cranial nerve damage. b. Schedule the patient for an appointment at a smell and taste disorders clinic. c. Perform testing on the vestibulocochlear nerve and a hearing test. d. Explain to the patient that diminished senses are normal findings. 19 A&E I Comprehensive Testbank ANS: C Slower reaction time is a common change in the older adult. Symptoms of cognitive impairment, such as disorientation, loss of language skills, loss of the ability to calculate, and poor judgment are not normal aging changes and require further investigation of underlying causes. 16. An older patient with dementia and confusion is admitted to the nursing unit after hip replacement surgery. Which action will the nurse include in the plan of care? ANS: A Patients with dementia need a routine. Continuing to reorient a patient with dementia is nonproductive and not advised. Patients with dementia need limited choices. Social interaction based on the patient’s abilities is to be promoted. 17. A nurse is helping an older-adult patient with instrumental activities of daily living. The nurse will be assisting the patient with which activity? ANS: C Instrumental activities of daily living or IADLs (such as the ability to write a check, shop, prepare meals, or make phone calls) and activities of daily living or ADLs (such as bathing, dressing, and toileting) are essential to independent living. 18. A male older-adult patient expresses concern and anxiety about decreased penile firmness during an erection. What is the nurse’s best response? ANS: D Aging men typically experience an erection that is less firm and shorter acting and have a less forceful ejaculation. Testosterone lessens with age and sometimes (not always) leads to a loss of libido. However, for both men and women sexual desires, thoughts, and actions continue throughout all decades of life. Sexuality involves love, warmth, sharing, and touching, not just the act of intercourse. Touch complements traditional sexual methods or serves as an alternative sexual expression when physical intercourse is not desired or a. Disorientation b. Poor judgment c. Slower reaction time d. Loss of language skills a. Keep a routine. b. Continue to reorient. c. Allow several choices. d. Socially isolate patient. a. Taking a bath b. Getting dressed c. Making a phone call d. Going to the bathroom a. Tell the patient that libido will always decrease, as well as the sexual desires. b. Tell the patient that touching should be avoided unless intercourse is planned. c. Tell the patient that heterosexuality will help maintain stronger libido. d. Tell the patient that this change is expected in aging adults. 20 A&E I Comprehensive Testbank possible. Clearly not all older adults are heterosexual, and there is emerging research on older adult, lesbian, gay, bisexual, and transgender individuals and their health care needs. 19. A patient asks the nurse what the term polypharmacy means. Which information should the nurse share with the patient? ANS: D Polypharmacy refers to the concurrent use of many medications. It does not have anything to do with side effects, adverse drug effects, or risks of medication use due to aging. 20. An outcome for an older-adult patient living alone is to be free from falls. Which statement indicates the patient correctly understands the teaching on safety concerns? ANS: A Postural hypotension is an intrinsic factor that can cause falls. Changing positions slowly indicates a correct understanding of this concept. Environmental hazards outside and within the home such as poor lighting, slippery or wet flooring, and items on floor that are easy to trip over such as throw rugs are other factors that can lead to falls. Impaired vision and poor lighting are other risk factors for falls and should be avoided (dim lighting). Inappropriate footwear such as smooth bottom socks also contributes to falls. 21. A nurse’s goal for an older adult is to reduce the risk of adverse medication effects. Which action will the nurse take? ANS: A Strategies for reducing the risk for adverse medication effects include reviewing the medications with older adults at each visit; examining for potential interactions with food or other medications; simplifying and individualizing medication regimens; taking every opportunity to inform older adults and their families about all aspects of medication use; and encouraging older adults to question their health care providers about all prescribed and over-the-counter medications. Although polypharmacy often reflects inappropriate prescribing, the concurrent use of multiple medications is often necessary when an older adult has multiple acute and chronic conditions. Older adults are at risk for adverse drug effects because of age-related changes in the absorption, distribution, metabolism, and excretion of drugs. Work collaboratively with the older adult to ensure safe and appropriate use of all medications—both prescribed medications and over-the-counter medications and herbal options. 22. An older-adult patient has developed acute confusion. The patient has been on tranquilizers for the past week. The patient’s vital signs are normal. What should the nurse do? a. This is multiple side effects experienced when taking medications. b. This is many adverse drug effects reported to the pharmacy. c. This is the multiple risks of medication effects due to aging. d. This is concurrent use of many medications. a. “I’ll take my time getting up from the bed or chair.” b. “I should dim the lighting outside to decrease the glare in my eyes.” c. “I’ll leave my throw rugs in place so that my feet won’t touch the cold tile.” d. “I should wear my favorite smooth bottom socks to protect my feet when walking around.” a. Review the patient’s list of medications at each visit. b. Teach that polypharmacy is to be avoided at all cost. c. Avoid information about adverse effects. d. Focus only on prescribed medications. 21 A&E I Comprehensive Testbank ANS: A Some sedatives and tranquilizers prescribed for acutely confused older adults sometimes cause or exacerbate confusion. Carefully administer drugs used to manage confused behaviors, taking into account age-related changes in body systems that affect pharmacokinetic activity. When confusion has a physiological cause (such as an infection), specifically treat that cause, rather than the confused behavior. When confusion varies by time of day or is related to environmental factors, nonpharmacological measures such as making the environment more meaningful, providing adequate light, etc., should be used. Making phone calls to friends or family members allows older adults to hear reassuring voices, which may be beneficial. 23. Which assessment finding of an older adult, who has a urinary tract infection, requires an immediate nursing intervention? ANS: A Confusion is a common manifestation in older adults with urinary tract infection; however, the cause requires further assessment. There may be another reason for the confusion. Confusion is not a normal finding in the older adult, even though it is commonly seen with concurrent infections. Difficulty hearing, presbycusis, is an expected finding in an older adult. Older adults tend to have lower core temperatures. Coping with the death of a spouse is a psychosocial concern to be addressed after the acute physiological concern in this case. 24. Which patient statement is the most reliable indicator that an older adult has the correct understanding of health promotion activities? ANS: B General preventive measures for the nurse to recommend to older adults include keeping regular dental appointments to promote good oral hygiene, eating a low-fat, well-balanced diet, exercising regularly, and maintaining immunizations for seasonal influenza, tetanus, diphtheria and pertussis, shingles, and pneumococcal disease. 25. A 72-year-old woman was recently widowed. She worked as a teller at a bank for 40 years and has been retired for the past 5 years. She never learned how to drive. She lives in a rural area that does not have public transportation. Which psychosocial change does the nurse focus on as a priority? a. Take into account age-related changes in body systems that affect pharmacokinetic activity. b. Increase the dose of tranquilizer if the cause of the confusion is an infection. c. Note when the confusion occurs and medicate before that time. d. Restrict phone calls to prevent further confusion. a. Confusion b. Presbycusis c. Temperature of 97.9° F d. Death of a spouse 2 months ago a. “I need to increase my fat intake and limit protein.” b. “I still keep my dentist appointments even though I have partials now.” c. “I should discontinue my fitness club membership for safety reasons.” d. “I’m up-to-date on my immunizations, but at my age, I don’t need the influenza vaccine.” a. Sexuality b. Retirement 22 A&E I Comprehensive Testbank ANS: D The highest priority at this time is the potential for social isolation. This woman does not know how to drive and lives in a rural community that does not have public transportation. All of these factors contribute to her social isolation. Other possible changes she may be going through right now include sexuality related to her advanced age and recent death of her spouse; however, this is not the priority at this time. She has been retired for 5 years, so this is also not an immediate need. She may eventually experience needs related to environment, but the data do not support this as an issue at this time. MULTIPLE RESPONSE 1. A recently widowed older-adult patient is dehydrated and is admitted to the hospital for intravenous fluid replacement. During the evening shift, the patient becomes acutely confused. Which possible reversible causes will the nurse consider when assessing this patient? (Select all that apply.) ANS: A, B, C, D Delirium, or acute confusional state, is a potentially reversible cognitive impairment that is often due to a physiological event. Physiological causes include electrolyte imbalances, untreated pain, infection, cerebral anoxia, hypoglycemia, medication effects, tumors, subdural hematomas, and cerebrovascular infarction or hemorrhage. Sometimes it is also caused by environmental factors such as sensory deprivation or overstimulation, unfamiliar surroundings, or sleep deprivation or psychosocial factors such as emotional distress. Dementia is a gradual, progressive, and irreversible cerebral dysfunction. MATCHING A nurse is using different strategies to meet older patients’ psychosocial needs. Match the strategy the nurse is using to its description. 1. Body image 2. Validation therapy 3. Therapeutic communication 4. Reality orientation 5. Reminiscence 1.ANS:E2.ANS:D3.ANS:A4.ANS:B5.ANS:C Chapter 24: Communication Potter et al.: Fundamentals of Nursing, 9th Edition MULTIPLE CHOICE 1. Which types of nurses make the best communicators with patients? c. Environment d. Social isolation a. Electrolyte imbalance b. Sensory deprivation c. Hypoglycemia d. Drug effects e. Dementia a. Respecting the older adult’s uniqueness b. Improving level of awareness c. Listening to the patient’s past recollections d. Accepting describing of patient’s perspective e. Offering help with grooming and hygiene a. Those who learn effective psychomotor skills b. Those who develop critical thinking skills 23 A&E I Comprehensive Testbank ANS: B Nurses who develop critical thinking skills make the best communicators. Just liking people does not make an effective communicator because it is important to apply critical thinking standards to ensure sound effective communication. Just learning psychomotor skills does not ensure that the nurse will use those techniques, and communication involves more than psychomotor skills.Critical thinking helps the nurse overcome perceptual biases or human tendencies that interfere with accurately perceiving and interpreting messages from others. Nurses who maintain perceptual biases do not make good communicators. 2. A nurse believes that the nurse-patient relationship is a partnership and that both are equal participants. Which term should the nurse use to describe this belief? ANS: C Effective interpersonal communication requires a sense of mutuality, a belief that the nurse-patient relationship is a partnership and that both are equal participants. Critical thinking in nursing, based on established standards of nursing care and ethical standards, promotes effective communication and uses standards such as humility, self-confidence, independent attitude, and fairness. To be authentic (one’s self) and to respond appropriately to the other person are important for interpersonal relationships but do not mean mutuality. Attending is giving all of your attention to the patient. 3. A nurse wants to present information about flu immunizations to the older adults in the community. Which type of communication should the nurse use? ANS: A Public communication is interaction with an audience. Nurses have opportunities to speak with groups of consumers about health-related topics, present scholarly work to colleagues at conferences, or lead classroom discussions with peers or students. When nurses work on committees or participate in patient care conferences, they use a small group communication process. Interpersonal communication is one-on-one interaction between a nurse and another person that often occurs face to face. Intrapersonal communication is a powerful form of communication that you use as a professional nurse. This level of communication is also called selftalk. 4. A nurse is using therapeutic communication with a patient. Which technique will the nurse use to ensure effective communication? ANS: C c. Those who like different kinds of people d. Those who maintain perceptual biases a. Critical thinking b. Authentic c. Mutuality d. Attend a. Public b. Small group c. Interpersonal d. Intrapersonal a. Interpersonal communication to change negative self-talk to positive self-talk b. Small group communication to present information to an audience c. Electronic communication to assess a patient in another city d. Intrapersonal communication to build strong teams 24 A&E I Comprehensive Testbank Electronic communication is the use of technology to create ongoing relationships with patients and their health care team. Intrapersonal communication is self-talk. Interpersonal communication is one-on-one interaction between a nurse and another person that often occurs face to face. Public communication is used to present information to an audience. Small group communication is interaction that occurs when a small number of persons meet. When nurses work on committees or participate in patient care conferences, they use a small group communication process. 5. A nurse is standing beside the patient’s bed. Nurse: How are you doing? Patient: I don’t feel good. Which element will the nurse identify as feedback? ANS: D “I don’t feel good” is the feedback because the feedback is the message the receiver returns. The sender is the person who encodes and delivers the message, and the receiver is the person who receives and decodes the message. The nurse is the sender. The patient is the receiver. “How are you doing?” is the message. 6. A nurse is sitting at the patient’s bedside taking a nursing history. Which zone of personal space is the nurse using? ANS: B Personal space is 18 inches to 4 feet and involves things such as sitting at a patient’s bedside, taking a patient’s nursing history, or teaching an individual patient. Intimate space is 0 to 18 inches and involves things such as performing a physical assessment, bathing, grooming, dressing, feeding, and toileting a patient. The socioconsultative zone is 9 to 12 feet and involves things such as giving directions to visitors in the hallway and giving verbal report to a group of nurses. The public zone is 12 feet and greater and involves things such as speaking at a community forum, testifying at a legislative hearing, or lecturing. 7. A smiling patient angrily states, “I will not cough and deep breathe.” How will the nurse interpret this finding? ANS: C An inappropriate affect is a facial expression that does not match the content of a verbal message (e.g., smiling when describing a sad situation). The patient is smiling but is angry, which indicates an inappropriate affect. The patient’s personal space was not violated. The patient’s vocabulary is not poor. Individuals who use a common language share denotative meaning: baseball has the same meaning for everyone who speaks English, but code denotes cardiac arrest primarily to health care providers. The patient’s denotative meaning is correct for cough and deep breathe. a. Nurse b. Patient c. How are you doing? d. I don’t feel good. a. Socio-consultative b. Personal c. Intimate d. Public a. The patient’s denotative meaning is wrong. b. The patient’s personal space was violated. c. The patient’s affect is inappropriate. d. The patient’s vocabulary is poor. 25 A&E I Comprehensive Testbank 8. The nurse asks a patient where the pain is, and the patient responds by pointing to the area of pain. Which form of communication did the patient use? ANS: B The patient gestured (pointed), which is a type of nonverbal communication. Gestures emphasize, punctuate, and clarify the spoken word. Pointing to an area of pain is sometimes more accurate than describing its location. Verbal is the spoken word or message. Intonation or tone of voice dramatically affects the meaning of a message. Vocabulary consists of words used for verbal communication. 9. A patient has been admitted to the hospital numerous times. The nurse asks the patient to share a personal story about the care that has been received. Which interaction is the nurse using? ANS: C In a therapeutic relationship, nurses often encourage patients to share personal stories. Sharing stories is called narrative interaction. Socializing is an important initial component of interpersonal communication. It helps people get to know one another and relax. It is easy, superficial, and not deeply personal. Nonjudgmental acceptance of the patient is an important characteristic of the relationship. Acceptance conveys a willingness to hear a message or acknowledge feelings; it is not a technique that involves personal stories. SBAR is a popular communication tool that helps standardize communication among health care providers. SBAR stands for Situation, Background, Assessment, and Recommendation. 10. Before meeting the patient, a nurse talks to other caregivers about the patient. Which phase of the helping relationship is the nurse in with this patient? ANS: A The time before the nurse meets the patient is called the preinteraction phase. This phase can involve things such as reviewing available data, including the medical and nursing history, talking to other caregivers who have information about the patient, or anticipating health concerns or issues that can arise. The orientation phase occurs when the nurse and the patient meet and get to know one another. This phase can involve things such as setting the tone for the relationship by adopting a warm, empathetic, caring manner. The working phase occurs when the nurse and the patient work together to solve problems and accomplish goals. The termination phase occurs during the ending of the relationship. This phase can involve things such as reminding the patient that termination is near. 11. During the initial home visit, a home health nurse lets the patient know that the visits are expected to end in about a month. Which phase of the helping relationship is the nurse in with this patient? a. Verbal b. Nonverbal c. Intonation d. Vocabulary a. Nonjudgmental b. Socializing c. Narrative d. SBAR a. Preinteraction b. Orientation c. Working d. Termination a. Preinteraction b. Orientation 26 A&E I Comprehensive Testbank ANS: B Letting the patient know when to expect the relationship to be terminated occurs in the orientation phase. Preinteraction occurs before the nurse meets the patient. Working occurs when the nurse and the patient work together to solve problems and accomplish goals. Termination occurs during the ending of the relationship. 12. A nurse and a patient work on strategies to reduce weight. Which phase of the helping relationship is the nurse in with this patient? ANS: C The working phase occurs when the nurse and the patient work together to solve problems and accomplish goals. Preinteraction occurs before the nurse meets the patient. Orientation occurs when the nurse and the patient meet and get to know each other. Termination occurs during the ending of the relationship. 13. A nurse uses SBAR when providing a hands-off report to the oncoming shift. What is the rationale for the nurse’s action? ANS: D SBAR is a popular communication tool that helps standardize communication among health care providers. Common courtesy is part of professional communication but is not the purpose of SBAR. Being trustworthy means helping others without hesitation. Autonomy is being self-directed and independent in accomplishing goals and advocating for others. 14. A patient was admitted 2 days ago with pneumonia and a history of angina. The patient is now having chest pain with a pulse rate of 108. Which piece of data will the nurse use for “B” when using SBAR? ANS: C The B in SBAR stands for background information. The background information in this situation is the history of angina. Having chest pain is the Situation (S). Pulse rate of 108 is the Assessment (A). Oxygen is needed is the Recommendation (R). 15. A patient just received a diagnosis of cancer. Which statement by the nurse demonstrates empathy? c. Working d. Termination a. Preinteraction b. Orientation c. Working d. Termination a. To promote autonomy b. To use common courtesy c. To establish trustworthiness d. To standardize communication a. Having chest pain b. Pulse rate of 108 c. History of angina d. Oxygen is needed a. “Tomorrow will be better.” b. “This must be hard news to hear.” 27 A&E I Comprehensive Testbank ANS: B “This must be hard” is an example of empathy. Empathy is the ability to understand and accept another person’s reality, accurately perceive feelings, and communicate this understanding to the other. An example of false reassurance is “Tomorrow will be better.” “I believe you can overcome this” is an example of sharing hope. “What is your biggest fear?” is an open-ended question that allows patients to take the conversational lead and introduces pertinent information about a topic. 16. A nurse is taking a history on a patient who cannot speak English. Which action will the nurse take? ANS: A Interpreters are often necessary for patients who speak a foreign language. Using a family member can lead to legal issues, speech therapists help patients with aphasia, and mental health nurse specialists help angry or highly anxious patients to communicate more effectively. 17. A nurse is using SOLER to facilitate active listening. Which technique should the nurse use for R? ANS: A In SOLER, the R stands for relax. It is important to communicate a sense of being relaxed and comfortable with the patient. Active listening enhances trust because the nurse communicates acceptance and respect for the patient, but it is not the R in SOLER. Reminisce is a therapeutic communication technique, especially when used with the elderly. Reassuring can be therapeutic if the nurse reassures patients that there are many kinds of hope and that meaning and personal growth can come from illness experiences. However, false reassurance can block communication. 18. An older-adult patient is wearing a hearing aid. Which technique should the nurse use to facilitate communication? ANS: B Turning off the television will facilitate communication. Patients who are hearing impaired benefit when the following techniques are used: check for hearing aids and glasses, reduce environmental noise, get the patient’s attention before speaking, do not chew gum, and speak at normal volume—do not shout. Using at least 14- point print is for sight/visually impaired, not hearing impaired. 19. When making rounds, the nurse finds a patient who is not able to sleep because of surgery in the morning. Which therapeutic response is most appropriate? c. “What’s your biggest fear about this diagnosis?” d. “I believe you can overcome this because I’ve seen how strong you are.” a. Obtain an interpreter. b. Refer to a speech therapist. c. Let a close family member talk. d. Find a mental health nurse specialist. a. Relax b. Respect c. Reminisce d. Reassure a. Chew gum. b. Turn off the television. c. Speak clearly and loudly. d. Use at least 14-point print. 28 A&E I Comprehensive Testbank ANS: D “It must be difficult not to know what the surgeon will find. What can I do to help?” is using therapeutic communication techniques of empathy and asking relevant questions. False reassurances (“You will be okay” and “Don’t worry”) tend to block communication. Patients frequently interpret “why” questions as accusations or think the nurse knows the reason and is simply testing them. 20. Which situation will cause the nurse to intervene and follow up on the nursing assistive personnel’s (NAP) behavior? ANS: A The nurse needs to intervene to correct the use of “honey.” Avoid terms of endearment such as “honey,” “dear,” “grandma,” or “sweetheart.” Communicate with older adults on an adult level, and avoid patronizing or speaking in a condescending manner. Facing an older-adult patient, making sure the older adult has clean glasses, and allowing time to respond facilitate communication with older-adult patients and should be encouraged, not stopped. 21. A confused older-adult patient is wearing thick glasses and a hearing aid. Which intervention is the priorityto facilitate communication? ANS: B Allowing time for patients to respond will facilitate communication, especially for a confused, older patient. Focusing on tasks to be completed and limiting conversations do not facilitate communication; in fact, they block communication. Using gestures and other nonverbal cues is not effective for visually impaired (thick glasses) patients or for patients who are confused. 22. The staff is having a hard time getting an older-adult patient to communicate. Which technique should the nurse suggest the staff use? a. “You will be okay. Your surgeon will talk to you in the morning.” b. “Why can’t you sleep? You have the best surgeon in the hospital.” c. “Don’t worry. The surgeon ordered a sleeping pill to help you sleep.” d. “It must be difficult not to know what the surgeon will find. What can I do to help?” a. The nursing assistive personnel is calling the older-adult patient “honey.” b. The nursing assistive personnel is facing the older-adult patient when talking. c. The nursing assistive personnel cleans the older-adult patient’s glasses gently. d. The nursing assistive personnel allows time for the older-adult patient to respond. a. Focus on tasks to be completed. b. Allow time for the patient to respond. c. Limit conversations with the patient. d. Use gestures and other nonverbal cues. a. Try changing topics often. b. Allow the patient to reminisce. c. Ask the patient for explanations. d. Involve only the patient in conversations. 29 A&E I Comprehensive Testbank ANS: B Encouraging older adults to share life stories and reminisce about the past has a therapeutic effect and increases their sense of well-being. Avoid sudden shifts from subject to subject. It is helpful to include the patient’s family and friends and to become familiar with the patient’s favorite topics for conversation. Asking for explanations is a nontherapeutic technique. 23. A nurse is implementing nursing care measures for patients’ special communication needs. Which patient will need the most nursing care measures? ANS: D Facial trauma, laryngeal cancer, or endotracheal intubation often prevents movement of air past vocal cords or mobility of the tongue, resulting in inability to articulate words. An extremely breathless person needs to use oxygen to breathe rather than speak. Persons with high anxiety are sometimes unable to perceive environmental stimuli or hear explanations. People who are alert, have strong self-esteem, and are cooperative and pain free do not cause communication concerns. Although hunger, blindness, and difficulty hearing can cause communication concerns, dyspnea, tracheostomy, and anxiety all contribute to communication concerns. 24. A patient is aphasic, and the nurse notices that the patient’s hands shake intermittently. Which nursing action is most appropriate to facilitate communication? ANS: A Using a pen and paper can be frustrating for a nonverbal (aphasic) patient whose handwriting is shaky; the nurse can revise the care plan to include use of a picture board instead. An interpreter is used for a patient who speaks a foreign language. A hearing aid is used for the hard of hearing, not for an aphasic patient. 25. Which behavior indicates the nurse is using a process recording correctly to enhance communication with patients? ANS: D Analysis of a process recording enables a nurse to evaluate the following: examine whether nursing responses blocked or facilitated the patient’s efforts to communicate. Sympathy is concern, sorrow, or pity felt for the patient and is nontherapeutic. Clichés and stereotyped remarks are automatic responses that communicate the nurse is not taking concerns seriously or responding thoughtfully. Passive responses serve to avoid conflict or to sidestep issues. 26. A patient says, “You are the worst nurse I have ever had.” Which response by the nurse is most assertive? a. The patient who is oriented, pain free, and blind b. The patient who is alert, hungry, and has strong self-esteem c. The patient who is cooperative, depressed, and hard of hearing d. The patient who is dyspneic, anxious, and has a tracheostomy a. Use a picture board. b. Use pen and paper. c. Use an interpreter. d. Use a hearing aid. a. Shows sympathy appropriately b. Uses automatic responses fluently c. Demonstrates passive remarks accurately d. Self-examines personal communication skills a. “I think you’ve had a hard day.” b. “I feel uncomfortable hearing that statement.” 30 A&E I Comprehensive Testbank ANS: B Assertive responses contain “I” messages such as “I want,” “I need,” “I think,” or “I feel.” While all of these start with “I,” the only one that is the most assertive is “I feel uncomfortable hearing that statement.” An assertive nurse communicates self-assurance; communicates feelings; takes responsibility for choices; and is respectful of others’ feelings, ideas, and choices. “I think you’ve had a hard day” is not addressing the problem. Arguing (“How can you say that?”) is not assertive or therapeutic. Showing disapproval (using words like right) is not assertive or therapeutic. MULTIPLE RESPONSE 1. Which behaviors indicate the nurse is using critical thinking standards when communicating with patients? (Select all that apply.) ANS: B, C, E A self-confident attitude is important because the nurse who conveys confidence and comfort while communicating more readily establishes an interpersonal helping-trusting relationship. In addition, an independent attitude encourages the nurse to communicate with colleagues and share ideas about nursing interventions. An attitude of humility is necessary to recognize and communicate the need for more information before making a decision. Faith and supportiveness are attributes of caring, not critical thinking standards. 2. A nurse is implementing nursing care measures for patients with challenging communication issues. Which types of patients will need these nursing care measures? (Select all that apply.) ANS: A, B, C, F Challenging communication situations include patients who are flirtatious, demanding, frightened, or developmentally delayed. A child who has received little environmental stimulation possibly is behind in language development, thus making communication more challenging. Patients who are cooperative and have good eyesight (see small print) do not cause challenging communication situations. MATCHING A nurse is using AIDET to communicate with patients and families. Match the letters of the acronym to the behavior a nurse will use. c. “I don’t think you should say things like that. It is not right.” d. “I have been checking on you regularly. How can you say that?” a. Instills faith b. Uses humility c. Portrays self-confidence d. Exhibits supportiveness e. Demonstrates independent attitude a. A child who is developmentally delayed b. An older-adult patient who is demanding c. A female patient who is outgoing and flirty d. A male patient who is cooperative with treatments e. An older-adult patient who can clearly see small print f. A teenager frightened by the prospect of impending surgery a. Nurse describes procedures and tests. b. Nurse lets the patient know how long the procedure will last. c. Nurse recognizes the person with a positive attitude. 31 A&E I Comprehensive Testbank 1.A 2.I 3.D 4.E 5.T 1.ANS:C2.ANS:E3.ANS:B4.ANS:A5.ANS:D Chapter 25: Patient Education Potter et al.: Fundamentals of Nursing, 9th Edition MULTIPLE CHOICE 1. A nurse is teaching a patient’s family member about permanent tube feedings at home. Which purpose of patient education is the nurse meeting? ANS: D Teach family members to help the patient with health care management (e.g., giving medications through gastric tubes and doing passive range-of-motion exercises) when coping with impaired functions. Not all patients fully recover from illness or injury. Many have to learn to cope with permanent health alterations. Health promotion involves healthy people staying healthy, while illness prevention is prevention of diseases. Restoration of health occurs if the teaching is about a temporary tube feeding, not a permanent tube feeding. 2. A nurse is teaching a group of healthy adults about the benefits of flu immunizations. Which type of patient education is the nurse providing? ANS: D As a nurse, you are a visible, competent resource for patients who want to improve their physical and psychological well-being. In the school, home, clinic, or workplace, you promote health and prevent illness by providing information and skills that enable patients to assume healthier behaviors. Injured and ill patients need information and skills to help them regain or maintain their level of health; this is referred to as restoration of health. Not all patients fully recover from illness or injury. Many have to learn to cope with permanent health alterations; this is known as coping with impaired functions. Analogies supplement verbal instruction with familiar images that make complex information more real and understandable. For example, when explaining arterial blood pressure, use an analogy of the flow of water through a hose. 3. A nurse’s goal is to provide teaching for restoration of health. Which situation indicates the nurse is meeting this goal? d. Nurse thanks the patient. e. Nurse tells the patient “I am an RN and will be managing your care.” a. Health promotion b. Illness prevention c. Restoration of health d. Coping with impaired functions a. Health analogies b. Restoration of health c. Coping with impaired functions d. Promotion of health and illness prevention a. Teaching a family member to provide passive range of motion for a stroke patient 32 A&E I Comprehensive Testbank ANS: D Injured or ill patients need information and skills to help them regain or maintain their levels of health. An example includes teaching a teenager with a broken leg how to use crutches. Not all patients fully recover from illness or injury. Many have to learn to cope with permanent health alterations. New knowledge and skills are often necessary for patients and/or family members to continue activities of daily living. Teaching family members to help the patient with health care management (e.g., giving medications through gastric tubes, doing passive range-of-motion exercises) is an example of coping with long-term impaired functions. For a woman with a hysterectomy, teaching about adoption is not restoration of health; restoration of health in this situation would involve activity restrictions and incision care if needed. In childbearing classes, you teach expectant parents about physical and psychological changes in the woman and about fetal development; this is part of health maintenance. 4. A nurse attends a seminar on teaching/learning. Which statement indicates the nurse has a good understanding of teaching/learning? ANS: C Teaching is most effective when it responds to the learner’s needs. It is impossible to separate teaching from learning. Teaching is an interactive process that promotes learning. Teaching consists of a conscious, deliberate set of actions that help individuals gain new knowledge, change attitudes, adopt new behaviors, or perform new skills. 5. A nurse is determining if teaching is effective. Which finding best indicates learning has occurred? ANS: B Learning is the purposeful acquisition of new knowledge, attitudes, behaviors, and skills: patient demonstrates how to inject insulin. A new mother exhibits learning when she demonstrates how to bathe her newborn. A nurse presenting information and a primary care provider handing a pamphlet to a patient are examples of teaching. A family member listening to a lecture does not indicate that learning occurred; a change in knowledge, attitudes, behaviors, and/or skills must be evident. 6. A nurse is teaching a patient about the Speak Up Initiatives. Which information should the nurse include in the teaching session? b. Teaching a woman who recently had a hysterectomy about possible adoption c. Teaching expectant parents about changes in childbearing women d. Teaching a teenager with a broken leg how to use crutches a. “Teaching and learning can be separated.” b. “Learning is an interactive process that promotes teaching.” c. “Teaching is most effective when it responds to the learner’s needs.” d. “Learning consists of a conscious, deliberate set of actions designed to help the teacher.” a. A nurse presents information about diabetes. b. A patient demonstrates how to inject insulin. c. A family member listens to a lecture on diabetes. d. A primary care provider hands a diabetes pamphlet to the patient. a. If you still do not understand, ask again. b. Ask a nurse to be your advocate or supporter. c. The nurse is the center of the health care team. d. Inappropriate medical tests are the most common mistakes. 33 A&E I Comprehensive Testbank ANS: A If you still do not understand, ask again is part of the S portion of the Speak Up Initiatives. Speak up if you have questions or concerns. You (the patient) are the center of the health care team, not the nurse. Ask a trusted family member or friend to be your advocate (advisor or supporter), not a nurse. Medication errors are the most common health care mistakes, not inappropriate medical tests. 7. A nurse teaches a patient with heart failure healthy food choices. The patient states that eating yogurt is better than eating cake. Which element represents feedback? ANS: D Feedback needs to demonstrate the success of the learner in achieving objectives (i.e., the learner verbalizes information or provides a return demonstration of skills learned). The nurse is the sender. The patient (learner) is the receiver. The teaching is the message. 8. While preparing a teaching plan, the nurse describes what the learner will be able to accomplish after the teaching session about healthy eating. Which action is the nurse completing? ANS: A Learning objectives describe what the learner will exhibit as a result of successful instruction. Positive reinforcement follows feedback and reinforces good behavior and promotes continued compliance. Interpersonal communication is necessary for the teaching/learning process, but describing what the learner will be able to do after successful instruction constitutes learning objectives. Facts and knowledge will be presented in the teaching session. 9. A patient learns that a normal adult heartbeat is 60 to 100 beats/min after a teaching session with a nurse. In which domain did learning take place? ANS: B The patient acquired knowledge, which is cognitive. Cognitive learning includes all intellectual skills and requires thinking. In the hierarchy of cognitive behaviors, the simplest behavior is acquiring knowledge. Kinesthetic is a type of learner who learns best with a hands-on approach. Affective learning deals with expression of feelings and development of attitudes, beliefs, or values. Psychomotor learning involves acquiring skills that require integration of mental and physical activities, such as the ability to walk or use an eating utensil. 10. A nurse is trying to help a patient begin to accept the chronic nature of diabetes. Which teaching technique should the nurse use to enhance learning? a. The nurse b. The patient c. The nurse teaching about healthy food choices d. The patient stating that eating yogurt is better than eating cake a. Developing learning objectives b. Providing positive reinforcement c. Presenting facts and knowledge d. Implementing interpersonal communication a. Kinesthetic b. Cognitive c. Affective d. Psychomotor a. Lecture b. Role play 34 A&E I Comprehensive Testbank ANS: B Affective learning deals with expression of feelings and acceptance of attitudes, beliefs, or values. Role play and discussion (one-on-one and group) are effective teaching methods for the affective domain. Lecture and question and answer sessions are effective teaching methods for the cognitive domain. Demonstration is an effective teaching method for the psychomotor domain. 11. A nurse is describing a patient’s perceived ability to successfully complete a task. Which term should the nurse use to describe this attribute? ANS: A Self-efficacy, a concept included in social learning theory, refers to a person’s perceived ability to successfully complete a task. Motivation is a force that acts on or within a person (e.g., an idea, an emotion, a physical need) to cause the person to behave in a particular way. An attentional set is the mental state that allows the learner to focus on and comprehend a learning activity. Learning occurs when the patient is actively involved in the educational session. 12. A toddler is going to have surgery on the right ear. Which teaching method is most appropriate for this developmental stage? ANS: C Use play to teach a procedure or activity (e.g., handling examination equipment, applying a bandage to a doll) to toddlers. Encouraging independent learning is for the young or middle adult. Use of discussion is for older children, adolescents, and adults, not for toddlers. Use problem solving to help adolescents make choices. Problem solving is too advanced for a toddler. 13. A nurse is preparing to teach a patient about smoking cessation. Which factors should the nurse assess to determine a patient’s ability to learn? ANS: C Developmental and physical capabilities reflect one’s ability to learn. Sociocultural background and motivation are factors determining readiness to learn. Psychosocial adaptation to illness and active participation are factors in readiness to learn. Readiness to learn is related to the stage of grieving. Overall physical health does reflect ability to learn; however, because it is paired here with stage of grieving (which is a readiness to learn factor), this is incorrect. 14. A nurse is teaching a patient about heart failure. Which environment will the nurse use? c. Demonstration d. Question and answer sessions a. Self-efficacy b. Motivation c. Attentional set d. Active participation a. Encourage independent learning. b. Develop a problem-solving scenario. c. Wrap a bandage around a stuffed animal’s ear. d. Use discussion throughout the teaching session. a. Sociocultural background and motivation b. Stage of grieving and overall physical health c. Developmental capabilities and physical capabilities d. Psychosocial adaptation to illness and active participation 35 A&E I Comprehensive Testbank ANS: B The ideal environment for learning is a room that is well lit and has good ventilation, appropriate furniture, and a comfortable temperature. Although a quiet room is appropriate, a darkened room interferes with the patient’s ability to watch your actions, especially when demonstrating a skill or using visual aids such as posters or pamphlets. A room that is cold, hot, or stuffy makes the patient too uncomfortable to focus on the information being presented. Learning in a group of six or less is more effective and avoids distracting behaviors. 15. Which assessment finding will cause the nurse to begin teaching a patient because the patient is ready to learn? ANS: C Motivation underlies a person’s desire or willingness to learn. Motivation is a force that acts on or within a person (e.g., an idea, emotion, or a physical need) to cause the person to behave in a particular way. For example, a patient with a below-the-knee amputation is motivated to learn how to walk with assistive devices, indicating a readiness to learn. Do not confuse readiness to learn with ability to learn. All the other answers are examples of ability to learn because this often depends on the patient’s level of physical development and overall physical health. To learn psychomotor skills, a patient needs to possess a certain level of strength, coordination, and sensory acuity. For example, it is useless to teach a patient to transfer from a bed to a wheelchair if he or she has insufficient upper body strength. An older patient with poor eyesight or an inability to grasp objects tightly cannot learn to apply an elastic bandage or handle a syringe. 16. A nurse is teaching a patient with a risk for hypertension how to take a blood pressure. Which action by the nurse is the priority? ANS: D The teaching process focuses on the patient’s learning needs, motivation, and ability to learn; writing learning objectives and goals is also included. Nursing and teaching processes are not the same. Assessing laboratory results for high cholesterol and performing nursing care therapies are all components of the nursing process, not the teaching process. 17. A patient has heart failure and kidney failure. The patient needs teaching about dialysis. Which nursing action is most appropriate for assessing this patient’s learning needs? a. A darkened, quiet room b. A well-lit, ventilated room c. A private room at 85° F temperature d. A group room for 10 to 12 patients with heart failure a. A patient has the ability to grasp and apply the elastic bandage. b. A patient has sufficient upper body strength to move from a bed to a wheelchair. c. A patient with a below-the-knee amputation is motivated about how to walk with assistive devices. d. A patient has normal eyesight to identify the markings on a syringe and coordination to handle a syringe. a. Assess laboratory results for high cholesterol and other data. b. Identify that teaching is the same as the nursing process. c. Perform nursing care therapies to address hypertension. d. Focus on a patient’s learning needs and objectives. a. Assess the patient’s total health care needs. b. Assess the patient’s health literacy. 36 A&E I Comprehensive Testbank ANS: B Because health literacy influences how you deliver teaching strategies, it is critical for you to assess a patient’s health literacy before providing instruction. The nursing process requires assessment of all sources of data to determine a patient’s total health care needs. Evaluation of the teaching process involves determining outcomes of the teaching/learning process and the achievement of learning objectives; assessing the goals of patient care is the evaluation component of the nursing process. 18. A nurse is teaching a patient about hypertension. In which order from first to last will the nurse implement the steps of the teaching process? 1. Set mutual goals for knowledge of hypertension. 2. Teach what the patient wants to know about hypertension. 3. Assess what the patient already knows about hypertension. 4. Evaluate the outcomes of patient education for hypertension. ANS: C Assessment is the first step of any teaching session, then diagnosing, planning (goals), implementation, and evaluation. 19. A patient had a stroke and must use a cane for support. A nurse is preparing to teach the patient about the cane. Which learning objective/outcome is most appropriate for the nurse to include in the teaching plan? ANS: A Outcomes often describe a behavior that identifies the patient’s ability to do something on completion of teaching such as will empty a colostomy bag or will administer an injection. Understand, learn, and know are not behaviors that can be observed or evaluated. 20. Which learning objective/outcome has the highest priority for a patient with life-threatening, severe food allergies that require an EpiPen (epinephrine)? ANS: D Once you assist in meeting patient needs related to basic survival (how to give epinephrine), you can discuss other topics, such as nutritional needs and side effects of medications. For example, a patient recently diagnosed with coronary artery disease has deficient knowledge related to the illness and its implications. The c. Assess all sources of patient data. d. Assess the goals of patient care. a. 1, 3, 2, 4 b. 2, 3, 1, 4 c. 3, 1, 2, 4 d. 3, 2, 1, 4 a. The patient will walk to the bathroom and back to bed using a cane. b. The patient will understand the importance of using a cane. c. The patient will know the correct use of a cane. d. The patient will learn how to use a cane. a. The patient will identify the main ingredients in several foods. b. The patient will list the side effects of epinephrine. c. The patient will learn about food labels. d. The patient will administer epinephrine. 37 A&E I Comprehensive Testbank patient benefits most by first learning about the correct way to take nitroglycerin and how long to wait before calling for help when chest pain occurs. Thus, in this situation, the patient benefits most by first learning about the correct way to take epinephrine. “The patient will learn about food labels” is not objective and measurable and is not correctly written. 21. After a teaching session on taking blood pressures, the nurse tells the patient, “You took that blood pressure like an experienced nurse.” Which type of reinforcement did the nurse use? ANS: A Reinforcers come in the form of social acknowledgments (e.g., nods, smiles, words of encouragement), pleasurable activities (e.g., walks or play time), and tangible rewards (e.g., toys or food). The entrusting approach is a teaching approach that provides a patient the opportunity to manage self-care. It is not a type of reinforcement. 22. A patient with heart failure is learning to reduce salt in the diet. When will be the best time for the nurse to address this topic? ANS: C In this situation, because the teaching is about food, coordinating it with routine nursing care that involves food can be effective. Many nurses find that they are able to teach more effectively while delivering nursing care. For example, while hanging blood, you explain to the patient why the blood is necessary and the symptoms of a transfusion reaction that need to be reported immediately. At bedtime would be a good time to discuss routines that enhance sleep. At bath time would be a good time to describe skin care and how to prevent pressure ulcers. At medication time would be a good time to explain the purposes and side effects of the medication. 23. A nurse is teaching a patient who has low health literacy about chronic obstructive pulmonary disease (COPD) while giving COPD medications. Which technique is most appropriate for the nurse to use? ANS: D Include the most important information at the beginning of the session for patients with literacy or learning disabilities. Also, use visual cues and simple, not complex, analogies when appropriate. Another technique is to frequently ask patients for feedback to determine whether they comprehend the information. Additionally, provide teaching materials that reflect the reading level of the patient, with attention given to short words and sentences, large type, and simple format (generally, information written on a fifth grade reading level is recommended for adult learners). a. Social acknowledgment b. Pleasurable activity c. Tangible reward d. Entrusting a. At bedtime, while the patient is relaxed b. At bath time, when the nurse is cleaning the patient c. At lunchtime, while the nurse is preparing the food tray d. At medication time, when the nurse is administering patient medication a. Use complex analogies to describe COPD. b. Ask for feedback to assess understanding of COPD at the end of the session. c. Offer pamphlets about COPD written at the eighth grade level with large type. d. Include the most important information on COPD at the beginning of the session. 38 A&E I Comprehensive Testbank 24. A nurse is teaching a culturally diverse patient with a learning disability about nutritional needs. What must the nurse do first before starting the teaching session? ANS: C Establishing trust is important for all patients, especially culturally diverse and learning disabled patients, before starting teaching sessions. Obtaining pictures of food, getting an interpreter, and referring to a dietitian all occur after rapport/trust is established. 25. A nurse is teaching an older-adult patient about strokes. Which teaching technique is most appropriate for the nurse to use? ANS: C With older adults, keep the teaching session short with small amounts of information. Also, if using written material, assess the patient’s ability to read and use information that is printed in large type and in a color that contrasts highly with the background (e.g., black 14-point print on matte white paper). Avoid blues and greens because they are more difficult to see. Speak in a low tone of voice (lower tones are easier to hear than higher tones). Directly face the older-adult learner when speaking. 26. A patient who is going to surgery has been taught how to cough and deep breathe. Which evaluation method will the nurse use? ANS: A To demonstrate mastery of the skill, have the patient perform a return demonstration under the same conditions that will be experienced at home or in the place where the skill is to be performed. Computer instruction is use of a programmed instruction format in which computers store response patterns for learners and select further lessons on the basis of these patterns (programs can be individualized). Computer instruction is a teaching tool, rather than an evaluation tool. Verbalization of steps can be an evaluation tool, but it is not as effective as a return demonstration when evaluating a psychomotor skill. The Cloze test, a test of reading comprehension, asks patients to fill in the blanks that are in a written paragraph. 27. A patient has been taught how to change a colostomy bag but is having trouble measuring and manipulating the equipment and has many questions. What is the nurse’s next action? a. Obtain pictures of food. b. Get an interpreter. c. Establish a rapport. d. Refer to a dietitian. a. Speak in a high tone of voice to describe strokes. b. Use a pamphlet about strokes with large font in blues and greens. c. Provide specific information about strokes in short, small amounts. d. Begin the teaching session facing the teaching white board with stroke information. a. Return demonstration b. Computer instruction c. Verbalization of steps d. Cloze test a. Refer to a mental health specialist. b. Refer to a wound care specialist. c. Refer to an ostomy specialist. d. Refer to a dietitian. 39 A&E I Comprehensive Testbank ANS: C Resources that specialize in a particular health need (e.g., wound care or ostomy specialists) are integral to successful patient education. A mental health specialist is helpful for emotional issues rather than for physical problems. A dietitian is a resource for nutritional needs. A wound care specialist provides complex wound care. 28. A nurse is teaching a patient about healthy eating habits. Which learning objective/outcome for the affective domain will the nurse add to the teaching plan? ANS: C Affective learning deals with expression of feelings and acceptance of attitudes, beliefs, or values. Having the patient value healthy eating habits falls within the affective domain. Stating three facts or identifying two foods for a healthy snack falls within the cognitive domain. Cooking falls within the psychomotor domain. 29. A nurse is assessing the ability to learn of a patient who has recently experienced a stroke. Which question/ statement will best assess the patient’s ability to learn? ANS: B A patient’s reading level affects ability to learn. One way to assess a patient’s reading level and level of understanding is to ask the patient to read instructions from an educational handout and then explain their meaning. Reading level is often difficult to assess because patients who are functionally illiterate are often able to conceal it by using excuses such as not having the time or not being able to see. Asking patients what they want to know identifies previous learning and learning needs and preferences; it does not assess ability to learn. Motivation (desire to learn) is related to readiness to learn, not ability to learn. Just asking a patient if he or she feels strong is not as effective as actually assessing the patient’s strength. 30. A nurse is preparing to teach a kinesthetic learner about exercise. Which technique will the nurse use? ANS: A Kinesthetic learners process knowledge by moving and participating in hands-on activities. Return demonstrations and role playing work well with these learners. Patients who are visual-spatial learners enjoy learning through pictures and visual charts to explain concepts. The verbal/linguistic learner demonstrates strength in the language arts and therefore prefers learning by listening or reading information. Patients who learn through logical-mathematical reasoning think in terms of cause and effect, and respond best when required to predict logical outcomes. Specific teaching strategies could include open-ended questioning or problem solving exercises, like a case study. MULTIPLE RESPONSE 1. A nurse is asked by a co-worker why patient education/teaching is important. Which statements will the nurse share with the co-worker? (Select all that apply.) a. The patient will state three facts about healthy eating. b. The patient will identify two foods for a healthy snack. c. The patient will verbalize the value of eating healthy. d. The patient will cook a meal with low-fat oil. a. “What do you want to know about strokes?” b. “Please read this handout and tell me what it means.” c. “Do you feel strong enough to perform the tasks I will teach you?” d. “On a scale from 1 to 10, tell me where you rank your desire to learn.” a. Let the patient touch and use the exercise equipment. b. Provide the patient with pictures of the exercise equipment. c. Let the patient listen to a video about the exercise equipment. d. Provide the patient with a case study about the exercise equipment. 40 A&E I Comprehensive Testbank ANS: A, B, C, D Patient education has long been a standard for professional nursing practice. All state Nurse Practice Acts acknowledge that patient teaching falls within the scope of nursing practice. Patient education is an essential component of providing safe, patient-centered care. It is important to document evidence of successful patient education in patients’ medical records. Patient education is effective for children. Different techniques must be used with children. Creating a well-designed, comprehensive teaching plan that fits a patient’s unique learning needs reduces health care costs, improves quality of care, and ultimately changes behaviors to improve patient outcomes. 2. A nurse is preparing to teach patients. Which patient finding will cause the nurse to postpone a teaching session? (Select all that apply.) ANS: A, B, E Any condition (e.g., pain, fatigue) that depletes a person’s energy also impairs the ability to learn, so the session should be postponed until the pain is relieved and the patient is rested. Postpone teaching when an illness becomes aggravated by complications such as a high fever or respiratory difficulty. A mild level of anxiety motivates learning. When patients are ready to learn, they frequently ask questions. When the patient enters the stage of acceptance, the stage compatible with learning, introduce a teaching plan. Chapter 29: Infection Prevention and Control Potter et al.: Fundamentals of Nursing, 9th Edition MULTIPLE CHOICE 1. The nurse and a new nurse in orientation are caring for a patient with pneumonia. Which statement by the new nurse will indicate a correct understanding of this condition? ANS: A Infections are infectious and/or communicable. Infectious diseases may not pose a risk for transmission to others, although they are serious for the patient. Pneumonia is not a communicable disease—a disease that is transmitted directly from one individual to the next, so there is no need for isolation. A private negative–air a. “Patient education is an essential component of safe, patient-centered care.” b. “Patient education is a standard for professional nursing practice.” c. “Patient teaching falls within the scope of nursing practice.” d. “Patient teaching is documented and part of the chart.” e. “Patient education is not effective with children.” f. “Patient teaching can increase health care costs.” a. The patient is hurting. b. The patient is fatigued. c. The patient is mildly anxious. d. The patient is asking questions. e. The patient is febrile (high fever). f. The patient is in the acceptance phase. a. “An infectious disease like pneumonia may not pose a risk to others.” b. “We need to isolate the patient in a private negative-pressure room.” c. “Clinical signs and symptoms are not present in pneumonia.” d. “The patient will not be able to return home.” 41 A&E I Comprehensive Testbank pressure room is used for tuberculosis, not pneumonia. Clinical signs and symptoms are present in pneumonia. Frequently, patients with pneumonia do return home unless there are extenuating circumstances. 2. The patient and the nurse are discussing Rickettsia rickettsii—Rocky Mountain spotted fever. Which patient statement to the nurse indicates understanding regarding the mode of transmission for this disease? ANS: C Rocky Mountain spotted fever is caused by bacteria transmitted by the bite of ticks. Wearing a repellent that is designed for repelling ticks, mosquitoes, and other insects can help in preventing transmission of this disease. Drinking plenty of uncontaminated water, wearing sunscreen, and using alcohol-based hand gels for cleaning hands are all important activities to participate in while camping, but they do not contribute to or prevent transmission of this disease. 3. The nurse is providing an educational session for a group of preschool workers. The nurse reminds the group about the most important thing to do to prevent the spread of infection. Which information did the nurse share with the preschool workers? ANS: D The single most important thing that individuals can do to prevent the spread of infection is to wash their hands before and after eating, going to the bathroom, changing a diaper, and wiping a nose and between touching each individual child. It is important for preschool children to have a nutritious diet; a healthy individual can fight infection more effectively. A health care provider, along with the parent, makes decisions about dietary supplements. Cleaning the toys can decrease the number of pathogens but is not the most important thing to do in this scenario. 4. The nurse is admitting a patient with an infectious disease process. Which question will be most appropriate for a nurse to ask about the patient’s susceptibility to this infectious process? ANS: B Multiple factors influence a patient’s susceptibility to infection. Patients with chronic diseases such as diabetes mellitus and multiple sclerosis are also more susceptible to infection because of general debilitation and nutritional impairment. Other factors include age, nutritional status, trauma, and smoking. The other questions are part of an admission assessment process but are not pertinent to the infectious disease process. 5. The patient experienced a surgical procedure, and Betadine was utilized as the surgical prep. Two days postoperatively, the nurse’s assessment indicates that the incision is red and has a small amount of purulent drainage. The patient reports tenderness at the incision site. The patient’s temperature is 100.5° F, and the WBC is 10,500/mm3. Which action should the nurse take first? a. “When camping, I will use sunscreen.” b. “When camping, I will drink bottled water.” c. “When camping, I will wear insect repellent.” d. “When camping, I will wash my hands with hand gel.” a. Encourage preschool children to eat a nutritious diet. b. Suggest that parents provide a multivitamin to the children. c. Clean the toys every afternoon before putting them away. d. Wash their hands between each interaction with children. a. “Do you have a spouse?” b. “Do you have a chronic disease?” c. “Do you have any children living in the home?” d. “Do you have any religious beliefs that will influence your care?” a. Plan to change the surgical dressing during the shift. 42 A&E I Comprehensive Testbank ANS: B The nursing assessment indicates signs and symptoms of infection, requiring the primary health care provider to be notified of the patient’s needs. SBAR—Situation, Background, Assessment, and Recommendation—can be utilized to organize thoughts and data and to provide a thorough explanation of the patient’s current status. The reevaluation of temperature is a good choice, but it will take longer than 4 hours to make a change in the white blood cells. Changing the dressing may be a need during the shift but is not a first priority. Checking to see about the skin preparation used 2 days ago may or may not be useful information at this time. 6. The nurse is providing an education session to an adult community group about the effects of smoking on infection. Which information is most important for the nurse to include in the educational session? ANS: B A normal defense mechanism against infection in the respiratory tract is the cilia lining the upper airways of the lungs and normal mucus. When a patient inhales a microbe, the cilia and mucus trap the microbe and sweep them up and out to be expectorated or swallowed. Smoking may alter this defense mechanism and increase the patient’s potential for infection. Smoking can be expensive, the smell does cling to hair and clothing, and the tar within the smoke can alter the color of a patient’s nails. This information can be included in the education but does not constitute the most important point. 7. A female adult patient presents to the clinic with reports of a white discharge and itching in the vaginal area. A nurse is taking a health history. Which question is the priority? ANS: C Antibiotics and oral contraceptives can disrupt normal flora in the vagina, causing an overgrowth of Candida albicans in that area. It is important to ask the patient about current medications to obtain information that may assist with diagnosis. The body contains normal flora (microorganisms) that live on the surface of skin, saliva, oral mucosa, gastrointestinal tract, and genitourinary tract. The normal flora of the vagina causes vaginal secretions to achieve a low pH, inhibiting the growth of many microorganisms. Visiting the primary health care provider is important for the patient’s health maintenance but is not the priority. Learning about the patient’s eating and sleeping habits will assist in the plan of care but is not the priority. 8. The nurse is caring for a school-aged child who has injured the right leg after a bicycle accident. Which signs and symptoms will the nurse assess for to determine if the child is experiencing a localized inflammatory response? b. Utilize SBAR to notify the primary health care provider. c. Reevaluate the temperature and white blood cell count in 4 hours. d. Check to see what solution was used for skin preparation in surgery. a. Smoke from tobacco products clings to your clothing and hair. b. Smoking affects the cilia lining the upper airways in the lungs. c. Smoking can affect the color of the patient’s fingernails. d. Smoking tobacco products can be very expensive. a. “When was the last time you visited your primary health care provider?” b. “Has this condition affected your eating habits in any way?” c. “What medications are you currently taking?” d. “Are you able to sleep at night?” a. Malaise, anorexia, enlarged lymph nodes, and increased white blood cells b. Chest pain, shortness of breath, and nausea and vomiting c. Dizziness and disorientation to time, date, and place 43 A&E I Comprehensive Testbank ANS: D The body’s cellular response to an injury is seen as inflammation. Signs of localized inflammation include swelling, redness, heat, pain or tenderness, and loss of function in the affected body part. Systemic signs of inflammation include fever, malaise, and anorexia, as well as enlarged lymph nodes and increased white blood cells. Chest pain, shortness of breath, and nausea and vomiting are signs and symptoms of a cardiac alteration. Dizziness and disorientation to time, date, and place may indicate a neurologic alteration. 9. Which interventions utilized by the nurse will indicate the ability to recognize a localized inflammatory response? ANS: D Signs of localized inflammation include swelling, redness, heat, pain or tenderness, and loss of function in the affected body part. One sign of the inflammatory response, particularly after an injury, is swelling or edema. Resting the affected injured area, using ice as ordered, wrapping the area to provide support—particularly if it is an extremity—and elevating the injured area will help to decrease swelling or edema. Turning, coughing, and deep breathing are utilized for postoperative patients and for immobilized patients to help prevent an infectious process such as pneumonia. Orientation to date, time, and place is an intervention utilized with many different types of patients who may be confused. Vigorous range of motion would irritate the inflammatory process. Range of motion is utilized for individuals who need to improve movement of their extremities, including immobilized patients. 10. The nurse is caring for a group of medical-surgical patients. Which patient is most at risk for developing an infection? ANS: C The patient who is recovering from a right total hip surgery has a large incision from the surgery. This break in the skin increases the likelihood of infection. Any break in the integrity of the skin and mucous membranes allows pathogens to enter and exit the body. The patient has had anesthesia, which depresses the respiratory system and has the potential to decrease the expansion of alveoli and to increase the chance of infection in the respiratory system. A patient who is having chest pain, experiencing dehydration, or being admitted with heart problems does not have open incisions that break the skin; therefore, his or her infection risk is lower. 11. The nurse is caring for a patient with leukemia and is preparing to provide fluids through a vascular access (IV) device. Which nursing intervention is a priority in this procedure? ANS: C d. Edema, redness, tenderness, and loss of function a. Vigorous range-of-motion exercises b. Turn, cough, and deep breathe c. Orient to date, time, and place d. Rest, ice, and elevation a. A patient who is in observation for chest pain b. A patient who has been admitted with dehydration c. A patient who is recovering from a right total hip surgery d. A patient who has been admitted for stabilization of heart problems a. Review the procedure with the patient. b. Position the patient comfortably. c. Maintain surgical aseptic technique. d. Gather available supplies. 44 A&E I Comprehensive Testbank You maintain surgical aseptic technique at the patient’s bedside (e.g., when inserting IV or urinary catheters, suctioning the tracheobronchial airway, and sterile dressing changes) because patients with disease processes of the immune system are at particular risk for infection. These diseases include leukemia, AIDS, lymphoma, and aplastic anemia. These disease processes weaken the defenses against an infectious organism. Reviewing the procedure with the patient, positioning the patient, and gathering the supplies are all important steps in the procedure but are not the priority in the procedure since the patient already has a compromised immune response. 12. The nurse is caring for an adult patient in the clinic who has been evacuated and is a victim of flooding. The nurse teaches the patient about rest, exercise, and eating properly and how to utilize deep breathing and visualization. What is the primary rationale for the nurse’s actions related to the teaching? ANS: C The body responds to emotional or physical stress by the general adaptation syndrome. If stress extends for long periods of time, this can lead to exhaustion, whereby energy stores are depleted and the body has no defenses against invading organisms. Techniques of deep breathing and visualization may be helpful with pain, but they are not the primary reason. The teachings listed are not all standard interventions taught at every health care visit. There is no data to indicate the patient requested this information for the family. 13. The nurse is caring for a patient who is susceptible to infection. Which instruction will the nurse include in an educational session to decrease the risk of infection? ANS: C A patient’s nutritional health directly influences susceptibility to infection. A reduction in the intake of protein and other nutrients such as carbohydrates and fats reduces body defenses against infection and impairs wound healing. This is the only teaching point that directly influences risk. Teaching the patient how to take a temperature can help the patient assess if there is a fever, but it is not related to decreasing the individual’s risk for infection. Teaching the patient about fall prevention or about the effects of alcohol does not decrease the risk of infection. 14. A diabetic patient presents to the clinic for a dressing change. The wound is located on the right foot and has purulent yellow drainage. Which action will the nurse take to prevent the spread of infection? ANS: D Localized infections are most common in the skin or with mucous membrane breakdown. Wear gloves and other personal protective equipment as appropriate when examining or providing treatment to localized a. Topics taught are standard information taught during health care visits. b. The patient requested this information to teach the extended family members. c. Stress for long periods of time can lead to exhaustion and decreased resistance to infection. d. These techniques will help the patient manage the pain and loss of personal belongings. a. Teaching the patient about fall prevention b. Teaching the patient to take a temperature c. Teaching the patient to select nutritious foods d. Teaching the patient about the effects of alcohol a. Position the patient comfortably on the stretcher. b. Explain the procedure for dressing change to the patient. c. Review the medication list that the patient brought from home. d. Don gloves and other appropriate personal protective equipment. 45 A&E I Comprehensive Testbank infected areas to create a protective barrier. Positioning the patient, explaining the procedure, and reviewing the medication list are all tasks that need to be completed, but they do not prevent the spread of infection. 15. A patient presents with pneumonia. Which priorityintervention should be included in the plan of care for this patient? ANS: A Systemic infection, like pneumonia, causes more generalized symptoms than local infection. This type of infection can result in fever, fatigue, nausea and vomiting, and malaise; be alert for changes in the patient’s level of activity and responsiveness. Nurses do not assume but assess and communicate with the patient about pain. While providing the patient with ice chips may be appropriate, it is not a priority and there is no reason for the patient to be limited to ice. Maintaining the room temperature at 65° F is too cold. 16. The nurse is caring for a patient in an intensive care unit who needs a bath. Which priority action will the nurse take to decrease the potential for a health care–associated infection? ANS: C The Centers for Disease Control and Prevention (CDC) recommends the use of chlorhexidine (CHG) bathing for patients in intensive care units, patients who are scheduled for surgery, and all patients with invasive central line catheters as part of MRSA reduction efforts. Using local anesthetics, nonallergenic tape, and filtered water does not affect the cause of a health care–associated infection by, for example, decreasing microbial counts like a CHG bath. 17. The infection control nurse is reviewing data for the medical-surgical unit. The nurse notices an increase in postoperative infections from Aspergillus. Which type of health care–associated infection will the nurse report? ANS: B An exogenous infection comes from microorganisms found outside the individual such as Salmonella, Clostridium tetani, and Aspergillus. They do not exist as normal floras. A vector transmits microorganisms and is usually a type of insect or organism. Endogenous infection occurs when part of the patient’s flora becomes altered and an overgrowth results (e.g., staphylococci, enterococci, yeasts, and streptococci). This often happens when a patient receives broad-spectrum antibiotics that alter the normal floras. A suprainfection develops when broad-spectrum antibiotics eliminate a wide range of normal flora organisms, not just those causing infection. 18. The patient has contracted a urinary tract infection (UTI) while in the hospital. Which action will most likely increase the risk of a patient contracting a UTI? a. Observe the patient for decreased activity tolerance. b. Assume the patient is in pain and treat accordingly. c. Provide the patient ice chips as requested. d. Maintain the room temperature at 65° F. a. Use local anesthetic on reddened areas. b. Use nonallergenic tape on dressings. c. Use a chlorhexidine wash. d. Use filtered water. a. Vector b. Exogenous c. Endogenous d. Suprainfection a. Reusing the patient’s graduated receptacle to empty the drainage bag. b. Allowing the drainage bag port to touch the graduated receptacle. 46 A&E I Comprehensive Testbank ANS: B Allowing the urinary drainage bag port to touch contaminated items (graduated receptacle) may introduce bacteria into the urinary system and contribute to a urinary tract infection. The urinary drainage bag should be emptied at least once a shift. Patients should have their own receptacle for measurement to prevent crosscontamination. Repeated catheter irrigations increase the chance so irrigating infrequently will be beneficial in reducing the risk. 19. Which nursing action will most likely increase a patient’s risk for developing a health care–associated infection? ANS: B Using clean technique (medical asepsis) to insert a urinary catheter would place the patient at risk for a health care–associated infection. Urinary catheters need to be inserted using sterile technique, which is also referred to as surgical asepsis. Surgical aseptic technique (also called sterile technique) should be used when suctioning an airway because it is considered a sterile body cavity. Washing from clean to dirty (urinary meatus toward rectum) is correct for decreasing infection risk. Bottled solutions may be used repeatedly during a 24-hour period; however, special care is needed to ensure that the solution in the bottle remains sterile. After 24 hours, the solution should be discarded. 20. The nurse is caring for a patient in labor and delivery. When near completing an assessment of the patient’s cervix, the electronic infusion device being used on the intravenous (IV) infusion alarms. Which sequence of actions is most appropriate for the nurse to take? ANS: C Completing the assessment while wearing gloves, removing gloves, washing hands after contact with body fluids, and then assessing the intravenous infusion will assist in the prevention and transfer of any potential organisms to this intravenous line. Completing the assessment, removing gloves, and silencing the alarm leaves out the crucial step of decontaminating and washing the hands. Discontinuing the assessment and assessing the IV leaves out removing the gloves and decontamination, as well as completing the assessment for the patient. Discontinuing the assessment, removing gloves, using hand gel, and assessing the IV is incorrect because upon exposure to body fluids, washing hands is appropriate. 21. The nurse is dressed and is preparing to care for a patient in the perioperative area. The nurse has scrubbed hands and has donned a sterile gown and gloves. Which action will indicate a break in sterile technique? c. Emptying the urinary drainage bag at least once a shift. d. Irrigating the catheter infrequently. a. Uses surgical aseptic technique to suction an airway b. Uses a clean technique for inserting a urinary catheter c. Uses a cleaning stroke from the urinary meatus toward the rectum d. Uses a sterile bottled solution more than once within a 24-hour period a. Complete the assessment, remove gloves, and silence the alarm. b. Discontinue the assessment, silence the alarm, and assess the intravenous site. c. Complete the assessment, remove gloves, wash hands, and assess the intravenous infusion. d. Discontinue the assessment, remove gloves, use hand gel, and assess the intravenous infusion. a. Touching clean protective eyewear b. Standing with hands above waist area c. Accepting sterile supplies from the surgeon 47 A&E I Comprehensive Testbank ANS: A Touching nonsterile (clean) protective eyewear once gowned and gloved with sterile gown and gloves would indicate a break in sterile technique. Sterile objects remain sterile only when touched by another sterile object. Standing with hands folded on the chest is common practice and prevents arms and hands from touching unsterile objects. Accepting sterile supplies from the surgeon who has opened them with the appropriate technique is acceptable. Staying with a sterile table once opened is a common practice to ascertain that no one or nothing has contaminated the table. 22. The nurse is caring for a patient with an incision. Which actions will best indicate an understanding of medical and surgical asepsis for a sterile dressing change? ANS: C Utilize clean gloves (medical asepsis) to remove contaminated dressings and sterile supplies, including gloves and dressings (surgical asepsis–sterile technique) to reapply sterile dressings. Wearing sterile gowns and gloves is not necessary when removing soiled dressings. Donning clean gloves to dress a sterile wound would contaminate the sterile supplies. Utilizing clean supplies for a sterile dressing would not help in decreasing the number of microbes at the incision site. 23. The nurse is caring for a patient in the endoscopy area. The nurse observes the technician performing these tasks. Which observation will require the nurse to intervene? ANS: C Standard precautions are used to prevent and control the spread of infection. Transferring contaminated equipment without the protection of gloves can assist in the spread of microbes to inanimate objects and to the person doing the transfer; therefore, the nurse must intervene. Utilizing gloves, washing hands, covering contaminated supplies during transfer, and disinfecting equipment in the appropriate way in the appropriate places utilize principles of basic medical asepsis and standard precautions and can break the chain of infection. 24. The nurse is caring for a patient who is at risk for infection. Which action by the nurse indicates correct understanding about standard precautions? ANS: D Standard precautions include the wearing of eyewear whenever there is a possibility of a splash or splatter, like when emptying the urinary drainage bag. Teaching the patient about good nutrition is positive but does not apply to standard precautions. Standard precautions apply to contact with blood, body fluid (except sweat), d. Staying with the sterile table once it is open a. Donning clean goggles, gown, and gloves to dress the wound b. Donning sterile gown and gloves to remove the wound dressing c. Utilizing clean gloves to remove the dressing and sterile supplies for the new dressing d. Utilizing clean gloves to remove the dressing and clean supplies for the new dressing a. Washing hands after removing gloves b. Disinfecting endoscopes in the workroom c. Removing gloves to transfer the endoscope d. Placing the endoscope in a container for transfer a. Teaches the patient about good nutrition b. Dons gloves when wearing artificial nails c. Disposes an uncapped needle in the designated container d. Wears eyewear when emptying the urinary drainage bag 48 A&E I Comprehensive Testbank nonintact skin, and mucous membranes from all patients. Artificial nails are not worn when using standard precautions. Any needles should be disposed of uncapped, or a mechanical safety device is activated for recapping. 25. The nurse is caring for a patient who has just delivered a neonate. The nurse is checking the patient for excessive vaginal drainage. Which precaution will the nurse use? ANS: C Standard precautions apply to contact with blood, body fluid, nonintact skin, and mucous membranes of all patients. Contact precautions apply to individuals with infections that can be transmitted by direct or indirect contact. Protective environment precautions apply to individuals who have undergone transplantations and gene therapy. Droplet precautions focus on diseases that are transmitted by large droplets. 26. The nurse is caring for a patient in the hospital. The nurse observes the nursing assistive personnel (NAP) turning off the handle faucet with bare hands. Which professional practice principle supports the need for follow-up with the NAP? ANS: A The nurse is responsible for providing a safe environment for the patient. The effectiveness of infection control practices depends on conscientiousness and consistency in using effective aseptic technique by all health care providers. After washing hands, turn off a handle faucet with a dry paper towel, and avoid touching the handles with your hands to assist in preventing the transfer of microorganisms. Wet towels and hands allow the transfer of pathogens from faucet to hands. The principles and procedures for washing hands are universal and apply to all members of health care teams. Being resourceful and aware of the cost of health care is important, but taking shortcuts that may endanger an individual’s health is not a prudent practice. 27. The nurse is caring for a patient who becomes nauseated and vomits without warning. The nurse has contaminated hands. Which action is best for the nurse to take next? ANS: A The Centers for Disease Control and Prevention (CDC) recommends that when hands are visibly soiled, one should wash with a non-antimicrobial soap or with antimicrobial soap. Cleaning hands with wipes or using waterless hand gel does not meet this standard. If hands are not visibly soiled, use an alcohol-based waterless antiseptic agent for routinely decontaminating hands. Wiping hands with a dry paper towel will occur after the nurse has washed both hands. 28. The nurse is performing hand hygiene before assisting a health care provider with insertion of a chest tube. While washing hands, the nurse touches the sink. Which action will the nurse take next? a. Contact b. Droplet c. Standard d. Protective environment a. The nurse is responsible for providing a safe environment for the patient. b. Different scopes of practice allow modification of procedures. c. Allowing the water to run is a waste of resources and money. d. This is a key step in the procedure for washing hands. a. Wash hands with an antimicrobial soap and water. b. Clean hands with wipes from the bedside table. c. Use an alcohol-based waterless hand gel. d. Wipe hands with a dry paper towel. 49 A&E I Comprehensive Testbank ANS: D The inside of the sink and the edges of the sink, faucet, and handles are considered contaminated areas. If the hands touch any of these areas during handwashing, repeat the handwashing procedure utilizing antiseptic soap. There is no need to inform the health care provider or be relieved of this assignment. If the hands are contaminated when touching the sink, drying hands and proceeding with the procedure could possibly contaminate and contribute to increased microbial counts during the procedure, resulting in infection for the patient. Extending the time for washing the hands (although this is what will happen when the procedure is repeated) is not the focus. The focus is to repeat the whole hand hygiene procedure utilizing antiseptic soap. 29. The nurse on the surgical team and the surgeon have completed a surgery. After donning gloves, gathering instruments, and placing in the transport carrier, what is the next step in handling the instruments used during the procedure? ANS: A Surgical instruments need to be cleaned and sterilized. Disinfecting, boiling, or cleaning is not utilized on critical items that will be reused on patients in the hospital environment. Items that are used on sterile tissue or in the vascular system present a high risk of infection if they become contaminated with bacteria. 30. The nurse is observing a family member changing a dressing for a patient in the home health environment. Which observation indicates the family member has a correct understanding of how to manage contaminated dressings? ANS: A Contaminated dressings and other infectious, disposable items should be placed in impervious plastic or brown paper bags and then disposed of properly in garbage containers. Gloves should be worn during this process. Parts of the dressing should not be saved, even though they may seem clean, because microbes may be present. 31. The nurse is caring for a group of patients. Which patient will the nurse see first? ANS: A a. Inform the health care provider and recruit another nurse to assist. b. Rinse and dry hands, and begin assisting the health care provider. c. Extend the handwashing procedure to 5 minutes. d. Repeat handwashing using antiseptic soap. a. Sending to central sterile for cleaning and sterilization b. Sending to central sterile for cleaning and disinfection c. Sending to central sterile for cleaning and boiling d. Sending to central sterile for cleaning a. The family member places the used dressings in a plastic bag. b. The family member saves part of the dressing because it is clean. c. The family member removes gloves and gathers items for disposal. d. The family member wraps the used dressing in toilet tissue before placing in trash. a. A patient with Clostridium difficile in droplet precautions b. A patient with tuberculosis in airborne precautions c. A patient with MRSA infection in contact precautions d. A patient with a lung transplant in protective environment precautions 50 A&E I Comprehensive Testbank A patient with Clostridium difficile should be on contact precautions, not droplet; therefore, the nurse will see this patient first to correct the precautions. All the rest are on correct precautions. Patients with tuberculosis belong in airborne precautions; patients with MRSA infection belong in contact precautions; and patients with lung transplants belong in protective environment precautions. 32. The home health nurse is teaching a patient and family about hand hygiene in the home. Which situation will cause the nurse to emphasize washing hands before and after? ANS: B Patients and family members should perform hand hygiene before and after treatments and when coming in contact with body fluids. Shaking hands does not require washing of hands before and after. Washing hands before and after opening the refrigerator and using the computer is not required. 33. The surgical mask the perioperative nurse is wearing becomes moist. Which action will the perioperative nurse take next? ANS: A After the mask is worn for several hours, it can become moist. The mask should be changed as soon as possible because moisture does not provide a barrier to microorganisms and is ineffective. Waiting to change the mask, air-drying it, or wearing it because it is comfortable does not support the principles of infection control. 34. The nurse is caring for a patient on contact precautions. Which action will be most appropriate to prevent the spread of disease? ANS: D Contact precautions are a type of isolation precaution used for patients with illness that can be transmitted through direct or indirect contact. Patients who are on contact precautions should have dedicated equipment wherever possible. This would mean, for example, that one blood pressure cuff and one stethoscope would stay in the room with the patient and would be used for that patient only. A gown and gloves may be required for interactions with a patient who is on contact precautions. A face mask and goggles are not part of contact precautions. A room with negative airflow is needed for patients placed on airborne precautions; it is not necessary for a patient on contact precautions. When a patient on contact precautions needs to be transported, the patient should wear clean gown, and hands cleaned, and the infectious material is contained or covered. 35. The nurse is caring for a patient who has cultured positive for Clostridium difficile. Which action will the nurse take next? a. Shaking hands b. Performing treatments c. Opening the refrigerator d. Working on a computer a. Apply a new mask. b. Reapply the mask after it air-dries. c. Change the mask when relieved by next shift. d. Do not change the mask if the nurse is comfortable. a. Place the patient in a room with negative airflow. b. Wear a gown, gloves, face mask, and goggles for interactions with the patient. c. Transport the patient safely and quickly when going to the radiology department. d. Use a dedicated blood pressure cuff that stays in the room and is used for that patient only. 51 A&E I Comprehensive Testbank ANS: A Clostridium difficile is a spore-forming organism that can be transmitted through direct and indirect patient contact. Because Clostridium difficile is a spore-forming organism, hand sanitizer is not effective in preventing its transmission. Hands must be washed with soap and water to prevent transmission. This organism is not transmitted via the droplet route; therefore, droplet precautions are not needed. An N95 respirator is used primarily for patients with airborne illness, especially tuberculosis. While all patients should be taught cough etiquette, this action is not specifically related to the patient having Clostridium difficile. 36. The nurse is changing linens for a postoperative patient and feels a prick in the left hand. A nonactivated safe needle is noted in the linens. For which condition is the nurse most at risk? ANS: B Bloodborne pathogens such as those associated with hepatitis B and C are most commonly transmitted by contaminated needles. Clostridium difficile and MRSA are spread by contact. Diphtheria is spread by droplets when one is within 3 feet of the patient. 37. The nurse is caring for a patient who has a bloodborne pathogen. The nurse splashes blood above the glove to intact skin while discontinuing an intravenous (IV) infusion. Which step(s) will the nurse take next? ANS: B After getting splashed with blood from a patient who has a known bloodborne pathogen, it is important to cleanse the site immediately and thoroughly with soap and running water and notify the manager for guidance on next steps in the process. Removing the blood with an alcohol swab, delaying washing, and doing nothing because the splash was to intact skin could possibly spread the blood within the room and could spread the infection. Contain contamination immediately to prevent contact spread. 38. Which process will be required after exposure of a nurse to blood by a cut from a used scalpel in the operative area? a. Instruct assistive personnel to use soap and water rather than sanitizer. b. Wear an N95 respirator when entering the patient room. c. Place the patient on droplet precautions. d. Teach the patient cough etiquette. a. Diphtheria b. Hepatitis B c. Clostridium difficile d. Methicillin-resistant Staphylococcus aureus a. Obtain an alcohol swab, remove the blood with an alcohol swab, and continue care. b. Immediately wash the site with soap and running water, and seek guidance from the manager. c. Do nothing; accidentally getting splashed with blood happens frequently and is part of the job. d. Delay washing of the site until the nurse is finished providing care to the patient. a. Placing the scalpel in a needle safe container b. Testing the patient and offering treatment to the nurse c. Removing sterile gloves and disposing of in kick bucket d. Providing a medical evaluation of the nurse to the manager 52 A&E I Comprehensive Testbank ANS: B Follow-up for risk of infection begins with patient testing. Patients should be tested for HIV and hepatitis B and C. Testing of the nurse is dependent on the results of patient testing; if the patient is positive for one of these infections, the nurse will be started on testing and treatment. Removing sterile gloves and placing sharps in appropriate containers are always part of the perioperative process and are not the process for postexposure. A confidential medical evaluation is provided to the nurse, not the manager. 39. The nurse is caring for a patient who needs a protective environment. The nurse has provided the care needed and is now leaving the room. In which order will the nurse remove the personal protective equipment, beginning with the first step? 1. Remove eyewear/face shield and goggles. 2. Perform hand hygiene, leave room, and close door. 3. Remove gloves. 4. Untie gown, allow gown to fall from shoulders, and do not touch outside of gown; dispose of properly. 5. Remove mask by strings; do not touch outside of mask. 6. Dispose of all contaminated supplies and equipment in designated receptacles. ANS: D The correct order for removing personal protective equipment for a patient in a protective environment and for performing associated tasks is to remove gloves, remove eyewear, remove gown, remove mask, perform hand hygiene, leave room and close doors, and dispose of all contaminated supplies and equipment in a manner that prevents the spread of microorganisms. 40. The nurse manager is evaluating current infection control data for the intensive care unit. The nurse compares past patient data with current data to look for trends. The nurse manager examines the infection chain for possible solutions. In which order will the nurse arrange the items for the infection chain beginning with the first step? 1. A mode of transmission 2. An infectious agent or pathogen 3. A susceptible host 4. A reservoir or source for pathogen growth 5. A portal of entry to a host 6. A portal of exit from the reservoir ANS: D For spread of infection, the chain has to be uninterrupted with an infectious agent, a reservoir and portal of exit, a mode of transmission, a portal of entry, and a susceptible host. The nurse manager is evaluating the chain of infection to determine actions that could be implemented to influence the spread of infection in the a. 3, 1, 4, 5, 6, 2 b. 1, 4, 5, 3, 6, 2 c. 1, 4, 5, 3, 2, 6 d. 3, 1, 4, 5, 2, 6 a. 3, 2, 4, 1, 5, 6 b. 1, 3, 5, 4, 6, 2 c. 4, 2, 1, 6, 3, 5 d. 2, 4, 6, 1, 5, 3 53 A&E I Comprehensive Testbank intensive care unit. Understanding the spread of infection and directing actions toward those steps have the potential to decrease infection in the setting. MULTIPLE RESPONSE 1. The nurse is caring for a patient in protective environment. Which actions will the nurse take? (Select all that apply.) ANS: B, D, E This form of isolation requires a specialized room with positive airflow. The airflow rate is set at greater than 12 air exchanges/hr, and all air is filtered through a HEPA filter. Isolation disrupts normal social relationships with visitors and caregivers. Take the opportunity to listen to a patient’s concerns or interests. Open drapes or shades and remove excess supplies and equipment. Patients are not allowed to have dried or fresh flowers or potted plants in these rooms. All health care personnel wear an N95 respirator every time they enter the room for patients, and a private room with negative airflow is required for patients on airborne precautions. 2. The nurse is assessing a new patient admitted to home health. Which questions will be most appropriate for the nurse to ask to determine the risk of infection? (Select all that apply.) ANS: A, B, C, D In the home setting, the objective is that the patient and/or family will utilize proper infection control techniques. Asking the patient and family about handwashing, risk of infection, recent travel, and signs and symptoms of infection is important in evaluating the patient’s knowledge based on infection control strategies. Activity assessment is important for evaluation of the overall status of the patient, and knowing who runs errands gives you information on who is helping to meet the needs of the patient, but neither of these relates to decreasing the risk of infection. 3. The circulating nurse in the operating room is observing the surgical technologist while applying a sterile gown and gloves to care for a patient having an appendectomy. Which behaviors indicate to the nurse that the procedure by the surgical technologist is correct? (Select all that apply.) a. Wear an N95 respirator when entering the patient’s room. b. Maintain airflow rate greater than 12 air exchanges/hr. c. Place in special room with negative-pressure airflow. d. Open drapes during the daytime. e. Listen to the patient’s interests. f. Place dried flowers in a plastic vase. a. “Can you explain the risk for infection in your home?” b. “Have you traveled outside of the United States?” c. “Will you demonstrate how to wash your hands?” d. “What are the signs and symptoms of infection?” e. “Are you able to walk to the mailbox?” f. “Who runs errands for you?” a. Ties the back of own gown b. Touches only the inside of gown c. Slips arms into arm holes simultaneously d. Extended fingers fully into both of the gloves e. Uses hands covered by sleeves to open gloves f. Applies surgical cap and face mask in the operating suite 54 A&E I Comprehensive Testbank ANS: B, C, D, E To maintain sterility, the surgical technologist (ST) touches the inside of the gown that will be against the body. Arms are slipped simultaneously into the gown to prevent contamination. Using the sleeves covering the hands maintains the principle of sterile only touching sterile to open gloves. Extending the fingers fully into both gloves ensures that the ST has full dexterity while using the sterile gloved hand. Surgical cap, face mask, and eye wear are applied before entering the surgical area and completing the surgical scrub. Reaching behind to tie the back of the gown will contaminate the sterile area of the gown. 4. The nurse is preparing to insert a urinary catheter. The nurse is using open gloving to apply the sterile gloves. Which steps will the nurse take? (Select all that apply.) ANS: B, D, E, F Sterile objects held below the waist are considered contaminated. Gloving the dominant hand helps to improve dexterity. Slipping the fingers underneath the second glove cuff helps to keep the gloved fingers sterile. Interlocking fingers ensures a smooth fit over the fingers. Sterile supplies are opened by carefully separating and peeling apart the sides of the package. This prevents the sterile contents from accidentally opening and touching contaminated objects. While putting on the first glove, touching only the outside surface of the glove will contaminate the sterile item; touch only the inside of the glove—the piece that will be against the skin. 5. The nurse has received a report from the emergency department that a patient with tuberculosis will be coming to the unit. Which items will the nurse need to care for this patient? (Select all that apply.) ANS: A, B, D, F Caring for this patient requires a private room, negative-pressure airflow in room, and wearing an N95 respirator that has been fit-tested, gloves, gown, and eyewear. Tuberculosis is a disease that is transmitted by droplets that remain in the air for long periods of time, requiring airborne precautions. This patient will not be in droplet precautions and instead requires airborne precaution signs. This type of patient requires more than the average surgical mask for protection. 6. The nurse and the student nurse are caring for two different patients on the medical-surgical unit. One patient is in airborne precautions, and one is in contact precautions. The nurse explains to the student different interventions for care. Which information will the nurse include in the teaching session? (Select all that apply.) a. While putting on the first glove, touch only the outside surface of the glove. b. With gloved dominant hand, slip fingers underneath second glove cuff. c. Remove outer glove package by tearing the package open. d. Lay glove package on clean flat surface above waistline. e. Glove the dominant hand of the nurse first. f. After second glove is on, interlock hands. a. Private room b. Negative-pressure airflow in room c. Surgical mask, gown, gloves, eyewear d. N95 respirator, gown, gloves, eyewear e. Communication signs for droplet precautions f. Communication signs for airborne precautions a. Dispose of supplies to prevent the spread of microorganisms. b. Wash hands before entering and leaving both of the patients’ rooms. 55 A&E I Comprehensive Testbank ANS: A, B, D, E Washing hands, properly disposing of supplies, applying knowledge of the disease process, and having patients in airborne precautions wear a mask during transfer are all principles to follow when caring for patients in isolation. Multiple differences are evident among these types of isolation, including the type of room used for the patient and what the nurse wears while caring for the patient. It is important to check the working order of a negative-pressure room before admitting a patient to the room, each shift the patient is in the room, and if and when the device alarms. Checking the working order of the negative-pressure rooms at discharge is not necessary. Chapter 35: Sexuality Potter et al.: Fundamentals of Nursing, 9th Edition MULTIPLE CHOICE 1. A nurse is caring for a 15-year-old who in the past 6 months has had multiple male and female sexual partners. Which response by the nurse will be mosteffective? ANS: A Some adolescents participate in risky behaviors. The nurse should acknowledge this feeling to the patient and offer education and alternatives, while giving the patient the autonomy to make his or her own decisions. Adolescents who engage in sexual risk behaviors experience negative health outcomes such as STIs and unintended pregnancy. In addition, the pattern of risk-taking behavior tends to be established and continue throughout life. The nurse should not force the patient to make a choice of orientation and should not pass judgment on a patient’s sexual orientation or social network; this would make the patient feel defensive and would eliminate the trust in the relationship. Involving parents is not the first line of action; parents should be notified only if the child is in a life or death situation. 2. A nurse is caring for a patient who expresses a desire to have an elective abortion. The nurse’s religious and ethical values are strongly opposed. How should the nurse best handle the situation? c. Be consistent in nursing interventions since there is only one difference in the precautions. d. Apply the knowledge the nurse has of the disease process to prevent the spread of microorganisms. e. Have patients in airborne precautions wear a mask during transportation to other departments. f. Check the working order of the negative-pressure room for the airborne precaution patient on admission and at discharge. a. “Sexually transmitted infections and unwanted pregnancy are a real risk. Let’s discuss what you think is the best method for protecting yourself.” b. “Having sexual interaction with both males and females places you at higher risk for STIs. To protect yourself, you need to decide which orientation you are.” c. “Your current friends are leading you to make poor choices. You should find new friends to hang out with.” d. “I think it’s best to notify your parents. They know what’s best for you and can help make sure you practice safe sex.” a. Attempt to educate the patient about the consequences of abortion. b. Refer the patient to a family planning center or another health professional. c. Continue to care for the patient, and limit conversation as much as possible. 56 A&E I Comprehensive Testbank ANS: B The nurse must be aware of personal beliefs and values and is not required to participate in counseling or procedures that compromise those values. However, the patient is entitled to nonjudgmental care and should be referred to someone who can create a trusting environment. The nurse should not care for a patient if the quality of care could be jeopardized. The nurse should not attempt to push personal values onto a patient. The nurse also should not create tension by informing the patient that he or she does not have the same morals; this could cause the patient to feel guilty or defensive when receiving care from any health care professional. 3. Which patient is most in need of a nurse’s referral to adoption services? ANS: B Adoption is an option for someone with infertility, especially if infertility treatments are unavailable owing to religious or financial constraints. A patient who wishes to have an elective abortion may be educated about all the possibilities, but the nurse should approach the patient in a nonjudgmental manner and should accept the patient’s decision. When a patient has recently miscarried, the nurse should assess the patient’s feelings about the loss and should address any concerns the patient may have about fertility. Infertility is the inability to conceive after 1 year of unprotected intercourse; therefore, talking about adoption after one miscarriage or after only 3 months of attempting conception would be too soon. 4. The nurse is caring for a patient who recently had unprotected sex with a partner who has HIV. Which response by the nurse is best? ANS: C Highly active retroviral therapy increases the survival time of a person with HIV or AIDS. HIV antibodies will not show up in blood work for 6 weeks to 3 months. The infection stage of HIV lasts for about a month after the virus is contracted; during that time, the patient may experience flu-like symptoms. A support group may be beneficial for a patient who contracts HIV; however, it is unknown whether the patient has contracted HIV, and antiretroviral therapy has helped people live beyond the 10 years expected if HIV goes untreated. 5. An 18-year-old male patient informs the nurse that he isn’t sure if he is homosexual because he is attracted to both genders. Which response by the nurse will help establish a trusting relationship? ANS: C d. Inform the patient that, because of immoral issues, another nurse will have to care for her. a. A woman considering abortion for an unwanted pregnancy b. An infertile couple religiously opposed to artificial insemination c. A woman who suffered miscarriage during her first pregnancy d. An infertile couple who has been attempting conception for 3 months a. “You should have your blood drawn today to see if you were infected.” b. “If you have the virus, you will have flu-like symptoms in 6 months.” c. “Highly active antiretroviral therapy has been shown effective in slowing the disease process.” d. “I will set you up with a support group to help you cope with dying within the next 10 years.” a. “Don’t worry. It’s just a phase you will grow out of.” b. “Those are abnormal impulses. You should seek therapy.” c. “At your age, it is normal to be curious about both genders.” d. “Having questions about sexuality is normal but if these sexual activities make you feel bad you should stop.” 57 A&E I Comprehensive Testbank Adolescents have questions about sexuality. The patient will feel most comfortable discussing his sexual concerns further if the nurse establishes that it is normal to ask questions about sexuality. The nurse can then discuss in greater detail. Although it is normal for young adults to be curious about sexuality, the nurse should use caution in giving advice on taking sexual action. The nurse should promote safe sex practices. Telling the patient not to worry dismisses his concern. Telling the patient that he is abnormal might offend the patient and prevent him from establishing an open relationship. 6. A nurse is caring for a 35-year-old female patient who recently started taking antidepressants after repeated attempts at fertility treatment. The patient tells the nurse, “I feel happier, but my sex drive is gone.” Which nursing diagnosis has the highest priority? ANS: A Antidepressants have adverse effects on sexual desire and response. The nurse should be sure to educate the patient on the potential for these side effects and how to correct for them, for example, using lubricant to ease discomfort. The patient has taken steps toward effective coping by seeking therapy. The patient has not expressed a reason for the nurse to be concerned about contraceptives. The nurse should always assess for concerns about violence in a patient’s life. Although some antidepressants have been related to self-directed violence, this patient focus is on becoming pregnant (fertility treatments) but sex drive is gone. 7. A nurse is using the PLISSIT model when caring for a patient with dyspareunia from diminished vaginal secretions. The nurse suggests using water-soluble lubricants. Which component of PLISSIT is the nurse using? ANS: C The nurse is using the specific suggestions (SS). The PLISSIT model is as follows: Permission to discuss sexuality issues Limited Information related to sexual health problems being experienced Specific Suggestions—only when the nurse is clear about the problem Intensive Therapy—referral to professional with advanced training if necessary 8. A patient who has had several sexual partners in the past month expresses a desire to use a contraceptive. Which contraceptive method should the nurse recommend? ANS: A Condoms are both a contraceptive and a barrier against STIs and HIV; proper use will greatly reduce the risk. Spermicides, diaphragms, and oral contraceptives all protect against pregnancy; however, they are not a barrier and do not prevent bodily fluids from coming in contact with the patient during sexual intercourse. a. Sexual dysfunction b. Ineffective coping c. Risk for self-directed violence d. Deficient knowledge about contraception a. P b. LI c. SS d. IT a. Condom b. Diaphragm c. Spermicide d. Oral contraceptive 58 A&E I Comprehensive Testbank 9. A woman who has been in a monogamous relationship for the past 6 months presents to clinic with herpes on her labia. The patient is distraught because her partner must have cheated on her. Which response by the nurse is most effective in establishing an open rapport with a patient? ANS: B If open communication is to be established with the patient, the patient must know that she can trust health care team members. By telling the patient that all encounters are confidential, the nurse establishes trust. Sharing a story brings the focus to the nurse, inhibiting open rapport. The nurse does not tell the patient what to do, because that should be the patient’s decision. Forcing the patient to confide by sharing every sexual encounter may hinder a trusting relationship. 10. A nurse is preparing a community class about sexually transmitted infections. Which primary group will the nurse focus on for this class? ANS: B About 20 million people in the United States are diagnosed with an STI each year, with the highest incidence occurring in men who have sex with men, bisexual men, and youths between the ages of 15 and 24. While bisexual women, youths between the ages of 24 and 27, and pregnant women and their partners are important, they are not the primary groups affected by STIs. 11. The nurse is leading a seminar about menopause and age-related changes. Which response from a group member indicates the nurse needs to follow up? ANS: B Believing that orgasms are no long achievable requires follow-up to correct this misconception. Orgasms are achievable at any age; however, it may take longer with aging. All other statements indicate that the patient does have an understanding of age-related changes and needs no follow-up. Both genders experience a reduced availability of sex hormones. The excitement phase prolongs in both men and women. Men often have erections that are less firm and shorter acting. 12. A patient who had a colostomy placed 1 month ago is feeling depressed and does not want to participate in sexual activities anymore. The patient is afraid that the partner does not want sex. The patient is afraid the ostomy is physically unattractive. Which initial nursing intervention will be most effective in helping this patient resume sexual activity? a. Share a story. b. Inform the patient that all encounters are confidential. c. Encourage the patient to break up with her partner for cheating. d. Tell the patient that she must be honest about every sexual experience she has had. a. Bisexual women b. Men who have sex with men c. Youths between the ages of 24 and 27 d. Pregnant women and their partners a. “Hormones of sexual regulation decrease with aging.” b. “Orgasms are no longer achievable after menopause.” c. “The excitement phase is prolonged as we age.” d. “As men age, their erection may be less firm.” a. Inform the patient about a support group for people with colostomies. b. Reassure the patient that lots of people resume sex the same week the colostomy is placed. 59 A&E I Comprehensive Testbank ANS: D The nurse should first address the patient’s need to be comfortable with his or her own body image; once the patient’s issues related to body image are resolved, intimacy may follow. Reassuring the patient that others manage to have sexual intercourse with an ostomy may help to decrease anxiety but may have the unintended effect of making the patient feel abnormal because he or she has not yet resumed sexual activity. Support groups may be helpful for the patient, but this is not the most effective initial intervention a nurse can provide; this may be helpful later. The patient is worried about the ostomy; incorporating it into intimate activities is insensitive and can even be damaging to the stoma. 13. A mother brings her 12-year-old daughter into a clinic and inquires about getting a human papillomavirus (HPV) vaccine that day. Which information will the nurse share with the mother and daughter about the HPV vaccine? ANS: B The HPV vaccine is effective against the four most common types of HPVs that can cause cervical cancer. It is not effective against HIV, chlamydia, or pregnancy. 14. A parent asks about the human papillomavirus (HPV) vaccine. Which information will the nurse include in the teaching session? ANS: B The vaccine is safe for girls as young as 9 years old and is recommended for females ages 11 to 26 if they have not already completed the three required injections. Booster doses currently are not recommended. The vaccine is most effective if administered before sexual activity or exposure. 15. A nursing student is providing education to a group of older adults who are in an independent living retirement village. Which statement made by the nursing student requires the nurse to intervene? ANS: C c. Teach the patient about intimate activities that can be done to incorporate the ostomy. d. Discuss ways to adapt to new body image so the patient will be comfortable in resuming intimacy. a. Protects against human immunodeficiency virus (HIV) b. Protects against cervical cancer c. Protects against chlamydia d. Protects against pregnancy a. It is recommended for girls 6 to 9 years old. b. It is recommended for females ages 11 to 26. c. It is recommended that booster injections be given. d. It is recommended to receive four required injections. a. “Avoiding alcohol use will enhance your sexual functioning.” b. “You need to tell your partner how you feel about sex and any fears you may have.” c. “You do not need to worry about getting a sexually transmitted infection at this point in your life.” d. “Using pillows and taking pain medication if needed before having sexual intercourse often help alleviate pain and improve sexual functioning.” 60 A&E I Comprehensive Testbank Research indicates many older adults are more sexuality active than previously thought and engage in high-risk sexual encounters, resulting in a steady increase HIV and STI rates over the past 12 years. Therefore, the nurse needs to intervene when the student tells the older adults that they are not at risk for developing an STI. Avoiding the use of alcohol; using pillows; taking pain medications before having intercourse if needed; and communicating thoughts, fears, and feelings about sex all enhance sexual functioning. 16. A nurse is interviewing a woman who uses a diaphragm. Which information from the patient will require the nurse to follow up? ANS: A The woman needs to be refitted after a significant change in weight (10-pound gain or loss) or pregnancy. The diaphragm is a round, rubber dome that has a flexible spring around the edge. It is used with a contraceptive cream or jelly and is inserted in the vagina so it provides a contraceptive barrier over the cervical opening. 17. A nurse is conducting a sexual assessment. Which question is appropriate for the nurse to ask? ANS: A Asking about any changes in the way you feel about yourself is an appropriate question to ask during a sexual assessment. Asking about favorite sex position with men and/or women is inappropriate and invasive. The assessment needs to focus on the patient, not the partner. Asking “why” questions is nontherapeutic and is judgmental in this scenario. 18. A 15-year-old patient is concerned because her mother wants her to receive the human papillomavirus (HPV) vaccination, but the patient is unsure if she wants it. Which response by the nurse is most therapeutic? ANS: A The nurse should encourage health promotion behaviors but first must consider the autonomy of the patient and assess the patient for more data. The nurse should value the input of the patient in making a decision and assess what the patient is thinking to address any concerns the patient may have. The HPV vaccine is a preventative treatment; whether or not the patient is sexually active (asking about how many sexual partners) does not matter in this case. The nurse should not make assumptions about a patient’s home life (mother knows best); instead, the nurse should ask questions while establishing a therapeutic relationship. Recommending the patient get the vaccine as soon as possible is in violation of the patient’s rights. 19. A nurse is reviewing a patient’s history. Which priority finding will alert the nurse to assess the patient for possible sexual dysfunction? a. “I have lost 12 pounds on this diet.” b. “I use the diaphragm to prevent pregnancy.” c. “I use a contraceptive cream with my diaphragm.” d. “I know this provides a barrier over the cervical opening.” a. Have you noticed any changes in the way you feel about yourself? b. What is your favorite sex position with men and with women? c. Do you think your partner is attractive? d. Why do you like men over women? a. Ask the patient what concerns she may have about the vaccination. b. Inquire about how many sexual partners she has had in the past year. c. Remind her that her mother knows best and that she should respect her parents’ wishes. d. Promote the importance of the vaccine, and recommend that the patient get the vaccine as soon as possible. a. Takes vacations out of the country 61 A&E I Comprehensive Testbank ANS: B Medications that can affect sexual functioning include antihypertensive, antipsychotics, antidepressants, and antianxiety. Taking vacations out of the country, exercise classes, and afternoon naps are not as priority for sexual functioning as medications. 20. A nurse is assessing a child for sexual abuse. Which assessment findings will the nurse expect? ANS: A Behavioral signs of sexual abuse in a child include physical aggression, sleep disturbance, poor peer relationships, and substance abuse. Panic attacks, anorexia, anxiety, and depression are behavioral signs for adults. 21. The nurse is teaching a patient how to use a condom. Which instructions will the nurse provide? ANS: D Teach patients to pull out right after ejaculating and to hold onto the condom when pulling out. Store condoms in a cool, dry place away from sunlight. Instruct patient to never reuse a condom or use a damaged condom. Instruct patient to only use water-based lubricants (e.g., K-Y jelly) to prevent the condom from breaking; do not use petroleum jelly, massage oils, body lotions, or cooking oil. 22. A nurse is caring for a patient with dyspareunia. In which order will the nurse provide care, starting with the first step? 1. Determine which signs and symptoms of dyspareunia the patient has. 2. Mutually decide upon goals and objectives for dyspareunia. 3. Ask the patient if the dyspareunia is improving. 4. Develop a nursing diagnosis for the patient. 5. Use resources to help resolve the problem. ANS: B The nurse should use the nursing process when caring for patients with sexual dysfunction. Determine signs and symptoms (assessment); develop a nursing diagnosis (diagnosis); mutually decide upon goals (planning); b. Takes antianxiety medication c. Takes exercise classes d. Takes afternoon naps a. Physical aggression and sleep disturbances b. Many peers and no drug usage c. Panic attacks and anorexia d. Anxiety and depression a. Store in a warm lit space. b. Use massage oils for lubrication. c. Rinse and reuse the condom if needed. d. Hold onto the condom when pulling out. a. 5, 3, 1, 4, 2 b. 1, 4, 2, 5, 3 c. 3, 1, 4, 2, 5 d. 4, 2, 5, 3, 1 62 A&E I Comprehensive Testbank use resources to help resolve the problem (implementation); and ask if the dyspareunia is improving (evaluation). MULTIPLE RESPONSE 1. An older couple expresses concern because they are easily fatigued during sexual intercourse and cannot reach climax. Which strategies to increase sexual stamina will the nurse offer? (Select all that apply.) ANS: A, E, F Alcohol, tobacco, and certain medications (such as narcotics for pain) may cause drowsiness and fatigue and negatively affect sexual stamina. Eating well-balanced meals can help to increase energy levels. Planning sexual activity when the couple is well rested will help them not get fatigued as quickly. Encouraging intimate touching may help increase libido but not energy levels. Extra lubrication and taking pain medications may ease the discomfort of sexual intercourse but are not appropriate interventions for fatigue. Chapter 36: Spiritual Health Potter et al.: Fundamentals of Nursing, 9th Edition MULTIPLE CHOICE 1. A co-worker asks the nurse to explain spirituality. What is the nurse’s best response? ANS: B Spirituality is often defined as an awareness of one’s inner self and a sense of connection to a higher being, to nature, or to some purpose greater than oneself. Spirituality is an important factor that helps individuals achieve the balance needed to maintain health and well-being and to cope with illness. Florence Nightingale believed that spirituality was a force that provided energy needed to promote a healthy hospital environment and that caring for a person’s spiritual needs was just as essential as caring for his or her physical needs. The word spiritualitycomes from the Latin word spiritus, which refers to breath or wind. The spirit gives life to a person. 2. The nurse is caring for a patient who is an agnostic. Which information should the nurse consider when planning care for this patient? ANS: C Some people do not believe in the existence of God (atheist), or they believe that there is no known ultimate reality (agnostic). Nonetheless, spirituality is important regardless of a person’s religious beliefs. Agnostics discover meaning in what they do or how they live because they find no ultimate meaning for the way things are. They believe that people bring meaning to what they do. a. Plan sexual activity around a time when the couple feels rested. b. Encourage intimate touching, such as hugging and kissing. c. Use extra lubrication to decrease discomfort. d. Take pain medication before intercourse. e. Avoid alcohol and tobacco. f. Eat well-balanced meals. a. It has a minor effect on health. b. It is awareness of one’s inner self. c. It is not as essential as physical needs. d. It refers to fire or giving of life to a person. a. The patient is devoid of spirituality. b. The patient does not believe in God. c. The patient believes there is no known ultimate reality. d. The patient finds no meaning through relationship with others. 63 A&E I Comprehensive Testbank 3. The nurse is caring for an Islam patient who wants a snack. Which action by the nurse is most appropriate? ANS: B Islam religion does allow beef. Islam does not allow pork or alcohol. Ham and bacon are pork. Kosher is allowed for Judaism. 4. A nurse is teaching a patient how to meditate. Which information from the patient indicates effective learning? ANS: C The steps of meditation include sitting in a comfortable position with the back straight; breathe slowly; and focus on a sound, prayer, or image. Meditation should occur for 10 to 20 minutes twice a day. 5. The nurse is admitting a patient to the hospital. The patient is a very spiritual person but does not practice any specific religion. How will the nurse interpret this finding? ANS: C The patient’s statement is reasonable and is not contradictory. Many people tend to use the terms spiritualityand religion interchangeably. Although closely associated, these terms are not synonymous. Religious practices encompass spirituality, but spirituality does not need to include religious practice. When a person has the attitude of something to live for and look forward to, hope is present. 6. A nurse hears the following comments from different patients. Which patient comment does the nurse identify as faith? ANS: B Faith allows people to have firm beliefs despite lack of physical evidence (life after death). Religion refers to the system of organized beliefs and worship that a person practices to outwardly express spirituality (go to church). When a person has the attitude of something to live for and look forward to, hope is present (look forward to each day). Self-transcendence is the belief that there is a force outside of and greater than the person (awe when looking at a sunset). 7. A nurse is caring for a Hindu patient. Which action will the nurse take? a. Offers a ham sandwich b. Offers a beef sandwich c. Offers a kosher sandwich d. Offers a bacon sandwich a. I will lie on the floor. b. I will breathe quickly. c. I will focus on an image. d. I will do this for 10 minutes every day. a. This indicates a strong religious affiliation. b. This statement is contradictory. c. This statement is reasonable. d. This indicates a lack of hope. a. I go to church every Sunday. b. I believe there is life after death. c. I have something to look forward to each day. d. I get a feeling of awe when looking at the sunset. 64 A&E I Comprehensive Testbank ANS: D Hindus practice prayer and purity rituals. Blessingway is a practice of the Navajos that attempts to remove ill health by means of stories, songs, rituals, prayers, symbols, and sand paintings. Islams must be able to practice the Five Pillars of Islam. Holy Communion is practiced in the Christian religion. 8. The nurse is caring for a patient with a chronic illness who is having conflicts with beliefs. Which health care team member will the nurse ask to see this patient? ANS: A Other important resources to patients are spiritual advisors and members of the clergy. Spiritual care helps people identify meaning and purpose in life, look beyond the present, and maintain personal relationships, as well as a relationship with a higher being or life force. A psychiatrist is for emotional health. A social worker focuses on social, financial, and community resources. An occupational therapist provides care with vocational issues and functioning within physical limitations. 9. The nurse is caring for a patient with a terminal disease. The nurse sits down and lightly touches the patient’s hand. Which technique is the nurse using? ANS: B Establishing presence by sitting with a patient to attentively listen to his or her feelings and situation, talking with the patient, crying with the patient, and simply offering time are powerful spiritual care approaches. Benner explains that presence involves “being with” a patient versus “doing for” a patient. Transcendence is the belief that a force outside of and greater than the person exists beyond the material world. In settings where health promotion activities occur, patients often need information, counseling, and guidance to make the necessary choices to remain healthy. 10. The nurse and the patient have the same religious affiliation. Which action will the nurse take? ANS: C It is important not to impose personal value systems on the patient. This is particularly true when the patient’s values and beliefs are similar to those of the nurse because it then becomes very easy to make false assumptions. It is not a must to use a formal assessment tool when assessing a patient’s beliefs. It is important a. Allow time to practice the Five Pillars. b. Allow time to practice Blessingway. c. Allow time for Holy Communion. d. Allow time for purity rituals. a. The clergy b. A psychiatrist c. A social worker d. An occupational therapist a. “Doing for” b. Establishing presence c. Offering transcendence d. Providing health promotion a. Must use a formal assessment tool to determine patient’s beliefs. b. Assume that both have the same spiritual beliefs. c. Do not impose personal values on the patient. d. Skip the spiritual belief assessment. 65 A&E I Comprehensive Testbank to conduct the spiritual belief assessment; conducting an assessment is therapeutic because it expresses a level of caring and support. 11. A nurse makes a connection with the patient when providing spiritual care. Which type of connectedness did the nurse experience? ANS: B Interpersonal means connected with others and the environment. Intrapersonal means connected within oneself. Transpersonal means connected with God or an unseen higher power. There is no such term as multipersonal for connectedness. 12. The patient is admitted with chronic anxiety. Which action is most appropriate for the nurse to take? ANS: C Do not just look at the patient’s anxiety as a problem to solve with quick remedies, but rather look at how the anxiety influences the patient’s ability to function and achieve goals established in life (not just anxiety relief). Mobilizing the patient’s hope is central to a healing relationship. 13. In caring for the patient’s spiritual needs, the nurse asks 20 questions to assess the patient’s relationship with God and a sense of life purpose and satisfaction. Which method is the nurse using? ANS: A The spiritual well-being scale (SWB) has 20 questions that assess a patient’s relationship with God and his or her sense of life purpose and life satisfaction. The FICA assessment tool evaluates spirituality and is closely correlated to quality of life. This does not describe belief or hope. 14. A male patient in stable condition is in the intensive care unit (ICU) and is asking to see his spouse and two daughters. What should the nurse do? ANS: B a. Intrapersonal b. Interpersonal c. Transpersonal d. Multipersonal a. Focus on finding quick remedies for the anxiety. b. Realize that the patient’s only goal is relief of the anxiety. c. Look at how anxiety influences the patient’s ability to function. d. Help the patient realize that there is little hope of relief from anxiety. a. The spiritual well-being scale b. The FICA assessment tool c. Belief tool d. Hope scale a. Allow only 5 to 10 minutes with the family. b. Allow the wife and daughters to visit at the patient’s request. c. Allow the two daughters to visit, and let the wife visit when they leave. d. Allow the wife and one daughter to enter the ICU but not the other daughter. 66 A&E I Comprehensive Testbank Use of support systems is important in any health care setting. Allowing the family to visit is appropriate since the patient is in stable condition. When patients depend on family and friends for support, encourage them to visit the patient. As long as no interference with active patient care is involved, there is no reason to limit visitation. Limiting the visit is not necessary since the patient is stable. Breaking the family apart is not needed; the patient is stable and can see all three at once. 15. The nurse is caring for a patient who has been diagnosed with a terminal illness. The patient states, “I just don’t feel like going to work. I have no energy, and I can’t eat or sleep.” The patient shows no interest in taking part in the care by saying, “What’s the use?” Which response by the nurse is best? ANS: A All of the patient’s description are describing a loss of hope. While losses of energy, appetite, and sleep are indicated, they only address a part of patient’s problems. A loss of hope encompasses the holistic view of the patient. 16. The patient is having a difficult time dealing with an AIDS diagnosis. The patient states, “It’s not fair. I’m totally isolated from God and my family because of this. Even my father hates me for this. He won’t even speak to me.” What should the nurse do? ANS: B Application of therapeutic communication principles and caring helps you establish therapeutic trust with patients. The nurse should not offer false hope (father will accept the situation soon). The nurse should help the patient maintain feelings of control, not no control. The nurse should encourage renewing relationships if possible and establishing connections with self, significant others, and God. 17. The nurse is caring for a group of patients. Which patient will the nurse see first? ANS: A A patient saying that God has left and there is no reason for living must be seen first for safety reasons. It must be determined by the nurse if the patient is planning suicide or is just angry and frustrated. A patient refusing treatment on the Sabbath is within that patient’s right and doesn’t need to be seen first. A patient with a folk healer is within the patient’s right and does not need to be seen first. A patient praying to Allah is within the patient’s right and does not need to be seen first. 18. A nurse is providing spiritual care to patients. Which action is essential for the nurse to take? a. It sounds like you have lost hope. b. It sounds like you have lost energy. c. It sounds like you have lost your appetite. d. It sounds like you have lost the ability to sleep. a. Tell the patient to move on and focus on getting better. b. Use therapeutic communication to establish trust and caring. c. Assure the patient that the father will accept this situation soon. d. Point out that the patient has no control and that he or she must face the consequences. a. A patient saying that God has left and there is no reason for living. b. A patient refusing treatment on the Sabbath. c. A patient having a folk healer in the room. d. A patient praying to Allah. a. Know one’s own personal beliefs. b. Learn about other religions. 67 A&E I Comprehensive Testbank ANS: A Because each person has a unique spirituality, you need to know your own beliefs so you are able to care for each patient without bias. While learning about religions, visiting other religious areas of worship, and traveling to areas that do not have the same beliefs are beneficial, they are not essential. MULTIPLE RESPONSE 1. A nurse is evaluating a patient’s spiritual care. Which areas will the nurse include in the evaluation process? (Select all that apply.) ANS: A, B, C, D In evaluating care include a review of the patient’s self-perception regarding spiritual health, the patient’s view of his or her purpose in life, discussion with the family and friends about connectedness, and determining whether the patient’s expectations were met. Attainment of spiritual health is a lifelong goal; it is not permanent once obtained. 2. Spiritual distress has been identified in a patient who has been diagnosed with a chronic illness. Which interventions will the nurse add to the care plan? (Select all that apply.) ANS: A, D, E Interventions that are appropriate for spiritual distress include (1) helping the patient develop/identify activities to heal body, mind, and spirit; (2) offering to pray with the patient; and (3) teaching relaxation, guided imagery, and medication. Attendance at church should be encouraged as well as spending time with a support group. MATCHING A nurse is providing spiritual care to a group of patients. Match the group to their belief. 1. Hinduism 2. Buddhism 3. Islam 4. Judaism 5. Appalachians c. Visit churches, temples, mosques, or synagogues. d. Travel to other areas that do not have the same beliefs. a. Review the patient’s view of the purpose in life. b. Ask whether the patient’s expectations were met. c. Discuss with family and friends the patient’s connectedness. d. Review the patient’s self-perception regarding spiritual health. e. Impress on the patient that spiritual health is permanent once obtained. a. Offer to pray with the patient. b. Avoid time with the support group. c. Have the patient avoid church attendance. d. Develop activities to heal body, mind, and spirit. e. Teach relaxation, guided imagery, and meditation. a. Nature controls life and health. b. Organ transplantation or donation is not considered. c. Observance of the Sabbath is important. d. Past sins cause illness. e. Nonhuman spirits invading the body cause illness. 68 A&E I Comprehensive Testbank 1.ANS:D2.ANS:E3.ANS:B4.ANS:C5.ANS:A Week 2 Critical Thinking, The Nursing Process, Loss, Death, and Grief Chapter 15: Critical Thinking in Nursing Practice Potter et al.: Fundamentals of Nursing, 9th Edition MULTIPLE CHOICE 1. Which action should the nurse take when using critical thinking to make clinical decisions? ANS: C A critical thinker considers what is important in each clinical situation, imagines and explores alternatives, considers ethical principles, and makes informed decisions about the care of patients. Patient care can be provided in many ways. The use of evidence-based knowledge, or knowledge based on research or clinical expertise, makes you an informed critical thinker. Following health care provider’s orders is not considered a critical thinking skill. If your knowledge causes you to question a health care provider’s order, do so. 2. Which patient scenario of a surgical patient in pain is most indicative of critical thinking? ANS: C Asking the patient what pain-relief methods have worked in the past is an example of exploring many options for pain relief. Nonpharmacological pain-relief methods are available, as are medications for pain. Administering medication based on a previous assessment is not practicing according to standards of care. The nurse is to conduct an assessment each shift on assigned patients and intervene accordingly. Pain is subjective. The nurse should offer pain-relief methods based on the patient’s reports without being judgmental. 3. Which action indicates a registered nurse is being responsible for making clinical decisions? a. Make decisions based on intuition. b. Accept one established way to provide care. c. Consider what is important in a given situation. d. Read and follow the heath care provider’s orders. a. Administering pain-relief medication according to what was given last shift b. Offering pain-relief medication based on the health care provider’s orders c. Asking the patient what pain-relief methods, pharmacological and nonpharmacological, have worked in the past d. Explaining to the patient that self-reporting of severe pain is not consistent with the minor procedure that was performed a. Applies clear textbook solutions to patients’ problems 69 A&E I Comprehensive Testbank ANS: B Registered nurses are responsible for making clinical decisions to take immediate action when a patient’s condition worsens. Patient care should be based on evidence-based practice, not on tradition. Most patients have health care problems for which there are no clear textbook solutions. Care plans should be individualized for each patient, not just for groups. 4. A charge nurse is supervising the care of a new nurse. Which action by a new nurse indicates the charge nurse needs to intervene? ANS: D The charge nurse must intervene when the nurse is using previous shift assessments to make a decision; this is inappropriate. Nurses are responsible for assessing their own patients to make decisions. Making informed, ethical decisions in the patient’s best interest is practicing responsibly and does not need follow-up from the charge nurse. 5. Which action demonstrates a nurse utilizing reflection to improve clinical decision making? ANS: C Reflection utilizes critical thinking when thinking back on the effectiveness of interventions and how they were performed. It involves purposeful thinking back or recalling a situation to discover its purpose or meaning. The other options are not examples of reflection but do represent good nursing practice. Using an objective approach and obtaining data in an orderly fashion do not involve purposefully thinking back to discover the meaning or purpose of a situation. Providing evidence-based explanations for nursing interventions does not always involve thinking back to discover the meaning of a situation. 6. A nursing instructor needs to evaluate students’ abilities to synthesize data and identify relationships between nursing diagnoses. Which learning assignment is best suited for this instructor’s needs? ANS: A Concept mapping challenges the student to synthesize data and identify relationships between nursing diagnoses. The primary purpose of concept mapping is to better synthesize relevant data about a patient, including assessment data, nursing diagnoses, health needs, nursing interventions, and evaluation measures. b. Takes immediate action when a patient’s condition worsens c. Uses only traditional methods of providing care to patients d. Formulates standardized care plans solely for groups of patients a. Making an ethical clinical decision b. Making an informed clinical decision c. Making a clinical decision in the patient’s best interest d. Making a clinical decision based on previous shift assessments a. Obtains data in an orderly fashion b. Uses an objective approach in patient situations c. Improves a plan of care while thinking back on interventions effectiveness d. Provides evidence-based explanations and research for care of assigned patients a. Concept mapping b. Reflective journaling c. Lecture and discussion d. Reading assignment with a written summary 70 A&E I Comprehensive Testbank Reflective journaling involves thinking back to clarify concepts. Reading assignments and lecture do not best provide an instructor the ability to evaluate students’ abilities to synthesize data. 7. A nurse is using a critical thinking model to provide care. Which component is first that helps a nurse make clinical decisions? ANS: D The first component of the critical thinking model is a nurse’s specific knowledge base. After acquiring a sound knowledge base, the nurse can then apply knowledge to different clinical situations using the nursing process to gain valuable experience. Clinical learning experiences are necessary to acquire clinical decisionmaking skills. The nursing process competency is the third component of the critical thinking model. Eleven attitudes define the central features of a critical thinker and how a successful critical thinker approaches a problem. 8. Which action by a nurse indicates application of the critical thinking model to make the best clinical decisions? ANS: D The nursing process competency is the third component of the critical thinking model. In your practice, you will apply critical thinking components during each step of the nursing process. Care plans should be individualized, and recalling facts does not utilize critical thinking skills to make clinical decisions. The new nurse should not rely on the charge nurse to determine priorities of care. 9. A nurse is using the critical thinking skill of evaluation. Which action will the nurse take? ANS: C Reviewing the effectiveness of interventions best describes evaluation. Examining the meaning of data is inference. Supporting findings and conclusions provides explanations. Searching for links between the data and the nurse’s assumptions describes analysis. 10. The patient appears to be in no apparent distress, but vital signs taken by assistive personnel reveal an extremely low pulse. The nurse then auscultates an apical pulse and asks the patient whether there is any history of heart problems. The nurse is utilizing which critical thinking skill? a. Attitude b. Experience c. Nursing process d. Specific knowledge base a. Drawing on past clinical experiences to formulate standardized care plans b. Relying on recall of information from past lectures and textbooks c. Depending on the charge nurse to determine priorities of care d. Using the nursing process a. Examine the meaning of data. b. Support findings and conclusions. c. Review the effectiveness of nursing actions. d. Search for links between the data and the nurse’s assumptions. a. Evaluation b. Explanation c. Interpretation 71 A&E I Comprehensive Testbank ANS: C Interpretation involves being orderly in data collection, looking for patterns to categorize data, and clarifying uncertain data. This nurse is clarifying the data in this situation. Evaluation involves determining the effectiveness of interventions or care provided. The nurse in this scenario is assessing the patient, not evaluating interventions. Self-regulation is reflecting on experiences. Explanation is supporting findings and conclusions. The nurse in this question is clarifying uncertain data (determining cause of the low pulse), not supporting the finding of a low pulse. 11. A patient continues to report postsurgical incision pain at a level of 9 out of 10 after pain medicine is given. The next dose of pain medicine is not due for another hour. What should the critically thinking nurse do first? ANS: A The critically thinking nurse should explore all options for pain relief first. The nurse should use critical thinking to determine the cause of the pain and determine various options for pain, not just ordered pain medications. The nurse can act independently to determine all options for pain relief and does not have to wait until after the health care provider rounds are completed. Explaining the cause of the pain does not address options for pain relief. 12. Which action should the nurse take to best develop critical thinking skills? ANS: C Nursing is a practice discipline. Clinical learning experiences are necessary to acquire clinical decision-making skills. Studying for longer hours, interviewing nurses, and attending inservices do not provide opportunities for clinical decision making, as do actual clinical experiences. 13. While caring for a hospitalized older-adult female post hip surgery, the nurse is faced with the task of inserting an indwelling urinary catheter, which involves rotating the hip into a contraindicated position. Which action should the nurse take? ANS: B The nurse must use critical thinking skills in this situation to adapt positioning technique. In practice, patient procedures are not always presented as in a textbook, but they are individualized. A urologist consult is not warranted for positioning problems. Postponing insertion of the catheter is not an appropriate action. 14. The nurse enters a room to find the patient sitting up in bed crying. How will the nurse display a critical thinking attitude in this situation? d. Self-regulation a. Explore other options for pain relief. b. Discuss the surgical procedure and reason for the pain. c. Explain to the patient that nothing else has been ordered. d. Offer to notify the health care provider after morning rounds are completed. a. Study 3 hours more each night. b. Attend all inservice opportunities. c. Actively participate in clinical experiences. d. Interview staff nurses about their nursing experiences. a. Postpone catheter insertion until the next shift. b. Adapt the positioning technique to the situation. c. Notify the health care provider for a urologist consult. d. Follow textbook procedure with contraindicated position. 72 A&E I Comprehensive Testbank ANS: D A clinical sign or symptom (crying) often indicates a variety of problems. Explore and learn more about the patient so as to make appropriate clinical judgments. This is demonstrating curiosity, which is an attitude of critical thinking. Checking on the patient 30 minutes later, providing tissues, and limiting visitors may be appropriate actions but these actions do not address critical thinking. 15. A patient is having trouble reaching the water fountain while holding on to crutches. The nurse suggests that the patient place the crutches against the wall while stabilizing him or herself with two hands on the water fountain. Which critical thinking attitude did the nurse use in this situation? ANS: B The nurse uses creativity in this situation to figure out how the patient can safely get a drink of water. Humility is recognizing when more information is needed to make a decision. Confidence is being well prepared to perform nursing care safely. This question best illustrates the attitude of creativity. Risk taking is demonstrating the courage to speak out or to question orders based on the nurse’s own knowledge base. 16. A nurse is pulled from the surgical unit to work on the oncology unit. Which action by the nurse displays humility and responsibility? ANS: B Humility and responsibility are displayed when the nurse realizes lack of knowledge and requests an orientation to the unit. The other answer choices represent inappropriate actions in this situation and are not examples of humility and responsibility. The nurse should explore all options before refusing an assignment. The nurse should not make assumptions. Assuming is not an example of critical thinking. Admitting lack of knowledge is an example of humility, but going home does not illustrate an example of responsibility. 17. A nurse is using professional standards to influence clinical decisions. What is the rationale for the nurse’s actions? ANS: D Professional standards promote the highest level of quality nursing care. Application of professional standards requires you to use critical thinking for the good of individuals or groups. Bypassing the patient’s feelings is a. Provide privacy and check on the patient 30 minutes later. b. Set a box of tissues at the patient’s bedside before leaving the room. c. Limit visitors while the patient is upset. d. Ask the patient about the crying. a. Humility b. Creativity c. Risk taking d. Confidence a. Refusing the assignment b. Asking for an orientation to the unit c. Admitting lack of knowledge and going home d. Assuming that patient care will be the same as on the other units a. Establishes minimal passing standards for testing b. Utilizes evidence-based practice based on nurses’ needs c. Bypasses the patient’s feelings to promote ethical standards d. Uses critical thinking for the highest level of quality nursing care 73 A&E I Comprehensive Testbank not practicing according to professional standards. The primary purpose of professional standards is not to establish minimal passing standards for testing. Patient care should be based on patient needs, not on nurses’ needs. 18. A nurse who is caring for a patient with a pressure ulcer applies the recommended dressing according to hospital policy. Which standard is the nurse following? ANS: D The standards of professional responsibility that a nurse tries to achieve are the standards cited in Nurse Practice Acts, institutional practice guidelines (hospital/facility policy), and professional organizations’ standards of practice (e.g., The American Nurses Association Standards of Professional Performance). Intellectual standards are guidelines or principles for rational thought. Fairness and independent reasoning are two examples of critical thinking attitudes that are designed to help nurses make clinical decisions. 19. A nurse is reviewing care plans. Which finding, if identified in a plan of care, should the registered nurse revise? ANS: B The nurse should not assume when a patient is going to be discharged and document this information in a plan of care. Making assumptions is not an example of a critical thinking skill. The purpose of the nursing process is to diagnose and treat human responses (e.g., patient symptoms, need for knowledge) to actual or potential health problems. Use of the process allows nurses to help patients meet agreed-on outcomes for better health. The patient’s outcomes, having several actual health problems, and a description of the patient’s abilities to meet the goal are all appropriate to document in the nursing plan of care. 20. In which order will the nurse use the nursing process steps during the clinical decision-making process? 1. Evaluating goals 2. Assessing patient needs 3. Planning priorities of care 4. Determining nursing diagnoses 5. Implementing nursing interventions ANS: A The American Nurses Association developed standards that set forth the framework necessary for critical thinking in the application of the five-step nursing process: assessment, diagnosis, planning, implementation, and evaluation. a. Fairness b. Intellectual standards c. Independent reasoning d. Institutional practice guidelines a. Patient’s outcomes for learning b. Nurse’s assumptions about hospital discharge c. Identification of several actual health problems d. Documentation of patient’s ability to meet the goal a. 2, 4, 3, 5, 1 b. 4, 3, 2, 1, 5 c. 1, 2, 4, 5, 3 d. 5, 1, 2, 3, 4 74 A&E I Comprehensive Testbank MULTIPLE RESPONSE 1. Which findings will alert the nurse that stress is present when making a clinical decision? (Select all that apply.) ANS: A, B, C, D Learn to recognize when you are feeling stressed—your muscles will tense, you become reactive when others communicate with you, you have trouble concentrating, and you feel very tired. Emotions are not managed when stressed. Chapter 16: Nursing Assessment Potter et al.: Fundamentals of Nursing, 9th Edition MULTIPLE CHOICE 1. The nurse is using critical thinking skills during the first phase of the nursing process. Which action indicates the nurse is in the first phase? ANS: A The assessment phase of the nursing process involves data collection to complete a thorough patient database and is the first phase. Identifying nursing diagnoses occurs during the diagnosis phase or second phase. The nurse carries out interventions during the implementation phase (fourth phase), and determining whether outcomes have been achieved takes place during the evaluation phase (fifth phase) of the nursing process. 2. A nurse is using the problem-oriented approach to data collection. Which action will the nurse take first? ANS: B A problem-oriented approach focuses on the patient’s current problem or presenting situation rather than on an observational overview. The database is not always completed using a chronological approach if focusing on the current problem. Making interpretations of the data is not data collection. Data interpretation occurs while appropriate nursing diagnoses are assigned. The question is asking about data collection. 3. After reviewing the database, the nurse discovers that the patient’s vital signs have not been recorded by the nursing assistive personnel (NAP). Which clinical decision should the nurse make? a. Tense muscles b. Reactive responses c. Trouble concentrating d. Very tired feelings e. Managed emotions a. Completes a comprehensive database b. Identifies pertinent nursing diagnoses c. Intervenes based on priorities of patient care d. Determines whether outcomes have been achieved a. Complete the questions in chronological order. b. Focus on the patient’s presenting situation. c. Make accurate interpretations of the data. d. Conduct an observational overview. 75 A&E I Comprehensive Testbank ANS: C The nurse should ask the nursing assistive personnel to record the vital signs for review before administering medicines or transporting the patient to another department. The nurse should not make assumptions when providing high-quality patient care, and omitting the vital signs is not an appropriate action. 4. The nurse is gathering data on a patient. Which data will the nurse report as objective data? ANS: C Objective data are observations or measurements of a patient’s health status, like respirations. Inspecting the condition of a surgical incision or wound, describing an observed behavior, and measuring blood pressure are examples of objective data. States “doesn’t feel good,” reports a headache, and nausea are all subjective data. Subjective data include the patient’s feelings, perceptions, and reported symptoms. Only patients provide subjective data relevant to their health condition. 5. A patient expresses fear of going home and being alone. Vital signs are stable and the incision is nearly completely healed. What can the nurse infer from the subjective data? ANS: C Subjective data include expressions of fear of going home and being alone. These data indicate (use inference) that the patient is apprehensive about discharge. Expressing fear is not an appropriate sign that a patient is able to perform dressing changes independently. An order from a health care provider is required before a patient is taught to resume previous medications. The nurse cannot infer that surgery was not successful if the incision is nearly completely healed. 6. Which method of data collection will the nurse use to establish a patient’s database? ANS: C a. Administer scheduled medications assuming that the NAP would have reported abnormal vital signs. b. Have the patient transported to the radiology department for a scheduled x-ray, and review vital signs upon return. c. Ask the NAP to record the patient’s vital signs before administering medications. d. Omit the vital signs because the patient is presently in no distress. a. States “doesn’t feel good” b. Reports a headache c. Respirations 16 d. Nauseated a. The patient can now perform the dressing changes without help. b. The patient can begin retaking all of the previous medications. c. The patient is apprehensive about discharge. d. The patient’s surgery was not successful. a. Reviewing the current literature to determine evidence-based nursing actions b. Checking orders for diagnostic and laboratory tests c. Performing a physical examination d. Ordering medications 76 A&E I Comprehensive Testbank You will learn to conduct different types of assessments: the patient-centered interview during a nursing health history, a physical examination, and the periodic assessments you make during rounding or administering care. A nursing database includes a physical examination. The nurse reviews the current literature in the implementation phase of the nursing process to determine evidence-based actions, and the health care provider is responsible for ordering medications. The nurse uses results from the diagnostic and laboratory tests to establish a patient database, not checking orders for tests. 7. A nurse is gathering information about a patient’s habits and lifestyle patterns. Which method of data collection will the nurse use that will best obtain this information? ANS: C The nursing health history also includes a description of a patient’s habits and lifestyle patterns. Lab results and physical assessment will not reveal as much about the patient’s habits and lifestyle patterns as the nursing health history. Collecting data is part of the working phase of the interview. 8. While interviewing an older female patient of Asian descent, the nurse notices that the patient looks at the ground when answering questions. What should the nurse do? ANS: A To conduct an accurate and complete assessment, consider a patient’s cultural background. This nurse needs to practice culturally competent care and appreciate the cultural differences. Assuming that the patient is depressed or in need of a psychological evaluation or to force eye contact is inappropriate. 9. A nurse has already set the agenda during a patient-centered interview. What will the nurse do next? ANS: B After setting the agenda, the nurse should conduct the actual interview and proceed with data collection, such as asking about the patient’s current chief concerns or problems. Introductions occur before setting the agenda. Begin an interview by introducing yourself and your position and explaining the purpose of the interview. Your aim is to set an agenda for how you will gather information about a patient’s current chief concerns or problems. The termination phase includes telling the patient when the interview is nearing an end. Telling the patient that medications will be given later when the nurse returns would typically take place during the termination phase of the interview. 10. The nurse is attempting to prompt the patient to elaborate on the reports of daytime fatigue. Which question should the nurse ask? a. Carefully review lab results. b. Conduct the physical assessment. c. Perform a thorough nursing health history. d. Prolong the termination phase of the interview. a. Consider cultural differences during this assessment. b. Ask the patient to make eye contact to determine her affect. c. Continue with the interview and document that the patient is depressed. d. Notify the health care provider to recommend a psychological evaluation. a. Begin with introductions. b. Ask about the chief concerns or problems. c. Explain that the interview will be over in a few minutes. d. Tell the patient “I will be back to administer medications in 1 hour.” 77 A&E I Comprehensive Testbank ANS: B The question asking the patient what factors might be contributing to the fatigue will elicit the best open-ended response. Asking whether the patient is stressed and asking if the patient is sleeping 8 hours a night are closedended questions eliciting simple yes or no responses. Asking about normal work hours will elicit a matter-offact response and does not prompt the patient to elaborate on the daytime fatigue or ask about the contributing reasons. 11. A nurse is conducting a nursing health history. Which component will the nurse address? ANS: B Some components of a nursing health history include chief concern, patient expectations, spiritual health, and review of systems. Current treatment orders are located under the Orders section in the patient’s chart and are not a part of the nursing health history. Patient concerns, not nurse’s concerns, are included in the database. Goals that are mutually established, not nurse’s goals, are part of the nursing care plan. 12. While the patient’s lower extremity, which is in a cast, is assessed, the patient tells the nurse about an inability to rest at night. The nurse disregards this information, thinking that no correlation has been noted between having a leg cast and developing restless sleep. Which action would have been best for the nurse to take? ANS: D The nurse must use critical thinking skills in this situation to assess first in this situation. The best response is to gather more assessment data by asking the patient about usual sleep patterns and the onset of having difficulty resting. The nurse should assess before documenting and should not ignore the patient’s report of a problem or postpone it till the next shift. 13. The nurse begins a shift assessment by examining a surgical dressing that is saturated with serosanguineous drainage on a patient who had open abdominal surgery yesterday (or 1 day ago). Which type of assessment approach is the nurse using? ANS: D a. “Is there anything that you are stressed about right now that I should know?” b. “What reasons do you think are contributing to your fatigue?” c. “What are your normal work hours?” d. “Are you sleeping 8 hours a night?” a. Nurse’s concerns b. Patient expectations c. Current treatment orders d. Nurse’s goals for the patient a. Tell the patient to just focus on the leg and cast right now. b. Document the sleep patterns and information in the patient’s chart. c. Explain that a more thorough assessment will be needed next shift. d. Ask the patient about usual sleep patterns and the onset of having difficulty resting. a. Gordon’s Functional Health Patterns b. Activity-exercise pattern assessment c. General to specific assessment d. Problem-oriented assessment 78 A&E I Comprehensive Testbank The nurse is not doing a complete, general assessment and then focusing on specific problem areas. Instead, the nurse focuses immediately on the problem at hand (dressing and drainage from surgery) and performs a problem-oriented assessment. Utilizing Gordon’s Functional Health Patterns is an example of a structured database-type assessment technique that includes 11 patterns to assess. The nurse in this question is performing a specific problem-oriented assessment approach, not a general approach. The nurse is not performing an activity-exercise pattern assessment in this question. 14. Which statement by a nurse indicates a good understanding about the differences between data validation and data interpretation? ANS: C Validation, by definition, involves comparing data with other sources for accuracy. Data interpretation involves identifying abnormal findings, clarifying information, and identifying patient problems. The nurse should validate data before interpreting the data and making inferences. The nurse is interpreting and validating patient data, not professional standards. 15. Which scenario best illustrates the nurse using data validation when making a nursing clinical decision for a patient? ANS: A The only scenario that validates a patient’s report with a nurse’s observation is changing the wound dressing. The nurse validates what the patient says by observing the dressing. The rest of the examples have the nurse acting only from a patient and/or family reports, not the nurse’s assessment. 16. While completing an admission database, the nurse is interviewing a patient who states “I am allergic to latex.” Which action will the nurse take first? ANS: B The nurse should further assess and ask the patient to describe the type of reaction. The patient will not need to be placed in isolation; before terminating the interview or documenting the allergy, health care personnel need to be aware of what type of response the patient suffered. 17. A patient verbalizes a low pain level of 2 out of 10 but exhibits extreme facial grimacing while moving around in bed. What is the nurse’s initial action in response to these observations? a. “Data interpretation occurs before data validation.” b. “Validation involves looking for patterns in professional standards.” c. “Validation involves comparing data with other sources for accuracy.” d. “Data interpretation involves discovering patterns in professional standards.” a. The nurse determines to remove a wound dressing when the patient reveals the time of the last dressing change and notices old and new drainage. b. The nurse administers pain medicine due at 1700 at 1600 because the patient reports increased pain and the family wants something done. c. The nurse immediately asks the health care provider for an order of potassium when a patient reports leg cramps. d. The nurse elevates a leg cast when the patient reports decreased mobility. a. Immediately place the patient in isolation. b. Ask the patient to describe the type of reaction. c. Proceed to the termination phase of the interview. d. Document the latex allergy on the medication administration record. 79 A&E I Comprehensive Testbank ANS: C First, the nurse needs to clarify/verify what was observed with what the patient states. Proceeding to the next room is ignoring this visual cue. The nurse cannot assume the patient does not want pain medicine just because he reports a 2 out of 10 on the pain scale. The nurse should not administer medication for moderate to severe pain if it is not necessary. 18. The nurse is interviewing a patient with a hearing deficit. Which area should the nurse use to conduct this interview? ANS: A Distractions should be eliminated as much as possible when interviewing a patient with a hearing deficit. The best place to conduct this interview is in the patient’s room with the door closed. The waiting area does not provide privacy. Pain can sometimes inhibit someone’s ability to concentrate, so before pain medication is administered is not advisable. It is best for the patient to be as comfortable as possible when conducting an interview. Assessing a patient while another member of the health care team is working would be distracting and is not the best time for an interview to take place. 19. A new nurse is completing an assessment on an 80-year-old patient who is alert and oriented. The patient’s daughter is present in the room. Which action by the nurse will require follow-up by the charge nurse? ANS: B Gathering data from family members is acceptable, but when a patient is able to interact, nurses need to include information from the older adult to complete the assessment. Therefore, the charge nurse must correct this misconception. When assessing an older adult, nurses need to listen carefully and allow the patient to speak. Positive nonverbal communication, such as making eye contact, nodding, and leaning forward, shows interest in the patient. Thus, the charge nurse does not need to intervene or follow up. MULTIPLE RESPONSE 1. A nurse is completing an assessment. Which findings will the nurse report as subjective data? (Select all that apply.) a. Proceed to the next patient’s room to make rounds. b. Determine the patient does not want any pain medicine. c. Ask the patient about the facial grimacing with movement. d. Administer the pain medication ordered for moderate to severe pain. a. The patient’s room with the door closed b. The waiting area with the television turned off c. The patient’s room before administration of pain medication d. The waiting room while the occupational therapist is working on leg exercises a. The nurse makes eye contact with the patient. b. The nurse speaks only to the patient’s daughter. c. The nurse leans forward while talking with the patient. d. The nurse nods periodically while the patient is speaking. a. Patient’s temperature b. Patient’s wound appearance c. Patient describing excitement about discharge d. Patient pacing the floor while awaiting test results 80 A&E I Comprehensive Testbank ANS: C, E Subjective data include patient’s feelings, perceptions, and reported symptoms. Expressing feelings such as excitement or fear is an example of subjective data. Objective data are observations or measurements of a patient’s health status. In this question, the appearance of the wound and the patient’s temperature are objective data. Pacing is an observable patient behavior and is also considered objective data. MATCHING A nurse is completing an assessment using the PQRST to obtain data about the patient’s chest pain. Match the questions to the components of the PQRST that the nurse will be using. 1. Provokes 2. Quality 3. Radiate 4. Severity 5.Time 1.ANS:B2.ANS:D3.ANS:A4.ANS:E5.ANS:C Chapter 17: Nursing Diagnosis Potter et al.: Fundamentals of Nursing, 9th Edition MULTIPLE CHOICE 1. After assessing a patient, a nurse develops a standard formal nursing diagnosis. What is the rationale for the nurse’s actions? ANS: B The standard formal nursing diagnosis serves several purposes. Nursing diagnoses distinguish the nurse’s role from that of the physician/health care provider and help nurses focus on the scope of nursing practice (not medical) while fostering the development of nursing knowledge. A nursing diagnosis provides the precise definition that gives all members of the health care team a common language for understanding the patient’s needs. A diagnosis is a clinical judgment based on information. 2. Which diagnosis will the nurse document in a patient’s care plan that is NANDA-I approved? e. Patient’s expression of fear regarding upcoming surgery a. Where is the pain located? b. What causes the pain? c. Does it come and go? d. What does the pain feel like? e. What is the rating on a scale of 0 to 10? a. To form a language that can be encoded only by nurses b. To distinguish the nurse’s role from the physician’s role c. To develop clinical judgment based on other’s intuition d. To help nurses focus on the scope of medical practice a. Sore throat b. Acute pain 81 A&E I Comprehensive Testbank ANS: B Acute pain is the only NANDA-I approved diagnosis listed. Sleep apnea and heart failure are medical diagnoses, and sore throat is subjective data. 3. A nurse develops a nursing diagnostic statement for a patient with a medical diagnosis of pneumonia with chest x-ray results of lower lobe infiltrates. Which nursing diagnosis did the nurse write? ANS: D The related to factor of alveolar-capillary membrane changes is accurately written because it is a patient response to the disease process of pneumonia that the nurse can treat. The related to factor should be the cause of the problem (nursing diagnosis) that a nurse can address. The related to factors of dehydration and pneumonia are all medical diagnoses that the nurse cannot change. A diagnostic test or a chronic dysfunction is not an etiology or a condition that a nursing intervention is able to treat. 4. The nurse is reviewing a patient’s plan of care, which includes the nursing diagnostic statement, Impaired physical mobility related to tibial fracture as evidenced by patient’s inability to ambulate. Which part of the diagnostic statement does the nurse need to revise? ANS: A The etiology, or related to factor, of tibial fracture is a medical diagnosis and needs to be revised. The nursing diagnosis is appropriate because the patient is unable to ambulate. A collaborative problem is an actual or potential physiological complication that nurses monitor to detect the onset of changes in a patient’s health status; there is no collaborative problem listed. The defining characteristic (subjective and objective data that support the diagnosis) is appropriate for Impaired physical mobility. 5. A nurse is using assessment data gathered about a patient and combining critical thinking to develop a nursing diagnosis. What is the nurse doing? ANS: C Diagnostic reasoning is defined as a process of using the assessment data gathered about a patient to logically explain a clinical judgment, in this case a nursing diagnosis. Defining characteristics are assessment findings that support the nursing diagnosis. Defining characteristics are the subjective and objective clinical cues, which a nurse gathers intentionally and unintentionally. The nurse organizes all of the patient’s data into meaningful and usable data clusters, which lead to a diagnostic conclusion. Diagnostic labeling is simply the name of the diagnosis. c. Sleep apnea d. Heart failure a. Ineffective breathing pattern related to pneumonia b. Risk for infection related to chest x-ray procedure c. Risk for deficient fluid volume related to dehydration d. Impaired gas exchange related to alveolar-capillary membrane changes a. Etiology b. Nursing diagnosis c. Collaborative problem d. Defining characteristic a. Assigning clinical cues b. Defining characteristics c. Diagnostic reasoning d. Diagnostic labeling 82 A&E I Comprehensive Testbank 6. A patient presents to the emergency department following a motor vehicle crash and suffers a right femur fracture. The leg is stabilized in a full leg cast. Otherwise, the patient has no other major injuries, is in good health, and reports only moderate discomfort. Which is the most pertinent nursing diagnosis the nurse will include in the plan of care? ANS: C Based on the assessment data provided, the only supportive evidence for one of the diagnosis options is “Reports only moderate discomfort,” which would support Acute pain. No supportive evidence is provided for any of the other diagnoses. The patient may indeed develop signs or symptoms of the other problems, but supportive data are presently lacking in the provided information. 7. The nurse is reviewing a patient’s database for significant changes and discovers that the patient has not voided in over 8 hours. The patient’s kidney function lab results are abnormal, and the patient’s oral intake has significantly decreased since previous shifts. Which step of the nursing process should the nurse proceed to after this review? ANS: A After a thorough assessment, the nurse should proceed to analyzing the data and formulating a nursing diagnosis before proceeding with developing the plan of care and determining appropriate interventions; this is the diagnosis phase. The evaluation phase involves determining whether the goals were met and interventions were effective. 8. A patient with a spinal cord injury is seeking to enhance urinary elimination abilities by learning selfcatheterization versus assisted catheterization by home health nurses and family members. The nurse adds Readiness for enhanced urinary elimination in the care plan. Which type of diagnosis did the nurse write? ANS: C A health promotion nursing diagnosis is a clinical judgment concerning motivation and desire to increase wellbeing and actualize human health potential. A problem-focused nursing diagnosis describes a clinical judgment concerning an undesirable human response to a health condition/life process that exists in an individual, family, or community. A risk nursing diagnosis is a clinical judgment concerning the vulnerability of an individual, family, group or community for developing an undesirable human response to health conditions/life processes. A collaborative problem is an actual or potential physiological complication that nurses monitor to detect the onset of changes in a patient’s health status. 9. A nurse administers an antihypertensive medication to a patient at the scheduled time of 0900. The nursing assistive personnel (NAP) then reports to the nurse that the patient’s blood pressure was low when it was taken at 0830. The NAP states that was busy and had not had a chance to tell the nurse yet. The patient begins to complain of feeling dizzy and light-headed. The blood pressure is rechecked and it has dropped even lower. In which phase of the nursing process did the nurse firstmake an error? a. Posttrauma syndrome b. Constipation c. Acute pain d. Anxiety a. Diagnosis b. Planning c. Implementation d. Evaluation a. Risk b. Problem focused c. Health promotion d. Collaborative problem 83 A&E I Comprehensive Testbank ANS: A The diagnostic process should flow from the assessment. In this case, the nurse should have assessed the patient’s blood pressure before giving the medication. The nurse could have prevented the patient’s untoward reaction if the low blood pressure was assessed first. Diagnosis follows assessment. Administering the medication occurs in implementation, but this is not the first error. There are no errors in evaluation. 10. A nurse adds the following diagnosis to a patient’s care plan: Constipation related to decreased gastrointestinal motility secondary to pain medication administration as evidenced by the patient reporting no bowel movement in seven days, abdominal distention, and abdominal pain. Which element did the nurse write as the defining characteristic? ANS: C Abdominal distention, no reported bowel movement, and abdominal pain are the defining characteristics. Decreased gastrointestinal motility secondary to pain medication is an etiology or related to factor. Constipation (problem or NANDA-1 diagnosis) is the identified problem derived from the defining characteristics. 11. The patient database reveals that a patient has decreased oral intake, decreased oxygen saturation when ambulating, reports of shortness of breath when getting out of bed, and a productive cough. Which elements will the nurse identify as defining characteristics for the diagnostic label of Activity intolerance? ANS: B There are defining characteristics (observable assessment cues such as patient behavior, physical signs) that support each problem-focused diagnostic judgment. The signs and symptoms, or defining characteristics, for the diagnosis Activity intolerance include decreased oxygen saturation when ambulating and reports of shortness of breath when getting out of bed. The key to supporting the diagnosis of Activity intolerance is that only these two characteristics involve how the patient tolerates activity. Decreased oral intake and productive cough do not define activity intolerance. 12. A nurse performs an assessment on a patient. Which assessment data will the nurse use as an etiology for Acute pain? a. Assessment b. Diagnosis c. Implementation d. Evaluation a. Decreased gastrointestinal motility b. Pain medication c. Abdominal distention d. Constipation a. Decreased oral intake and decreased oxygen saturation when ambulating b. Decreased oxygen saturation when ambulating and reports of shortness of breath when getting out of bed c. Reports of shortness of breath when getting out of bed and a productive cough d. Productive cough and decreased oral intake a. Discomfort while changing position b. Reports pain as a 7 on a 0 to 10 scale c. Disruption of tissue integrity 84 A&E I Comprehensive Testbank ANS: C Disruption of tissue integrity is a possible cause or etiology of pain. A report of pain, headache, and discomfort are examples of things a patient might say (subjective data or defining characteristics) that may lead a nurse to select Acute pain as a nursing diagnosis. 13. A new nurse writes the following nursing diagnoses on a patient’s care plan. Which nursing diagnosis will cause the nurse manager to intervene? ANS: B Hemorrhage is a collaborative problem, not a nursing diagnosis; the nurse manager will need to correct this misunderstanding with the new nurse. Nurses manage collaborative problems such as hemorrhage, infection, and paralysis using medical, nursing, and allied health (e.g., physical therapy) interventions. Wandering, urinary retention, and impaired swallowing are all examples of nursing diagnoses. 14. A patient has a bacterial infection in left lower leg. Which nursing diagnosis will the nurse add to the patient’s care plan? ANS: C Impaired skin integrity is the only nursing diagnosis listed that will correlate to the patient information. While risk for infection is a nursing diagnosis, the patient is not at risk; the patient has an actual infection. Infection can be a medical diagnosis as well as a collaborative problem. Staphylococcal leg infection is a medical diagnosis. 15. A nurse adds a nursing diagnosis to a patient’s care plan. Which information did the nurse document? ANS: A Decreased cardiac output related to altered myocardial contractility is a correctly written nursing diagnosis. Patient needs a low-fat diet related to inadequate heart perfusion is a goal phrased statement, not a nursing diagnosis. Offer a low-fat diet is an intervention, not a diagnosis. Acute pain related to discomfort is a circular diagnosis and gives no direction to nursing care. 16. A charge nurse is evaluating a new nurse’s plan of care. Which finding will cause the charge nurse to follow up? d. Dull headache a. Wandering b. Hemorrhage c. Urinary retention d. Impaired swallowing a. Infection b. Risk for infection c. Impaired skin integrity d. Staphylococcal leg infection a. Decreased cardiac output related to altered myocardial contractility. b. Patient needs a low-fat diet related to inadequate heart perfusion. c. Offer a low-fat diet because of heart problems. d. Acute heart pain related to discomfort. a. Assigning a documented nursing diagnosis of Risk for infection for a patient on intravenous (IV) antibiotics 85 A&E I Comprehensive Testbank ANS: C Developing nursing diagnoses before completion of the database needs to be corrected by the charge nurse. Always identify a nursing diagnosis from the data, not the reverse. The data should be clustered and reviewed to see if any patterns are present before a nursing diagnosis is assigned. Risk for infection is an appropriate diagnosis for a patient with an intravenous (IV) site in place. The IV site involves a break in skin integrity and is a potential source of infection. The diagnostic process should proceed in steps. Completing the interview and physical examination before adding a nursing diagnosis is appropriate. The patient’s cultural background and developmental stage are important to include in a patient database. 17. A patient exhibits the following symptoms: tachycardia, increased thirst, headache, decreased urine output, and increased body temperature. The nurse analyzes the data. Which nursing diagnosis will the nurse assign to the patient? ANS: C The signs the patient is exhibiting are consistent with deficient fluid volume (dehydration). Even without knowing the clinical manifestations of dehydration, the question can be answered by the process of elimination. Adult failure to thrive, hypothermia, and nausea are not appropriate diagnoses because data are insufficient to support these diagnoses. 18. Which question would be most appropriate for a nurse to ask a patient to assist in establishing a nursing diagnosis of Diarrhea? ANS: B The nurse needs to first ensure that the symptoms support the diagnosis. By definition, diarrhea means that a patient is having frequent stools; therefore, asking about the number of bowel movements is most appropriate. Asking about irritating foods and medications may help the nurse determine the cause of the diarrhea, but first the nurse needs to make sure the diagnosis is appropriate. Asking the patient if he can make it to the bathroom will help to establish a diagnosis of incontinence, not diarrhea. The question is asking for the most appropriate statement to establish the diagnosis of Diarrhea. 19. A nurse assesses that a patient has not voided in 6 hours. Which question should the nurse ask to assist in establishing a nursing diagnosis of Urinary retention? ANS: A b. Completing an interview and physical examination before adding a nursing diagnosis c. Developing nursing diagnoses before completing the database d. Including cultural and religious preferences in the database a. Adult failure to thrive b. Hypothermia c. Deficient fluid volume d. Nausea a. “What types of foods do you think caused your upset stomach?” b. “How many bowel movements a day have you had?” c. “Are you able to get to the bathroom in time?” d. “What medications are you currently taking?” a. “Do you feel like you need to go to the bathroom?” b. “Are you able to walk to the bathroom by yourself?” c. “When was the last time you took your medicine?” d. “Do you have a safety rail in your bathroom at home?” 86 A&E I Comprehensive Testbank The nurse must establish that the patient feels the urge and is unable to void. The question “Do you feel like you need to go to the bathroom?” is the most appropriate to ask. This question can be answered without knowledge of the diagnosis of Urinary retention. Discussing the ability to walk to the bathroom and asking about safety rails pertain to mobility and safety issues, not to retention of urine. Taking certain medications may lead to urinary retention, but that information would establish the etiology. The question is asking for the nurse to first establish the correct diagnosis. 20. A nurse is developing nursing diagnoses for a patient. Beginning with the first step, place in order the steps the nurse will use. 1. Observes the patient having dyspnea (shortness of breath) and a diagnosis of asthma 2. Writes a diagnostic label of impaired gas exchange 3. Organizes data into meaningful clusters 4. Interprets information from patient 5. Writes an etiology ANS: A The diagnostic process flows from the assessment process (observing and gathering data) and includes decision-making steps. These steps include data clustering, identifying patient health problems, and formulating the diagnosis (diagnosis is written as problem or NANDA-I approved diagnosis then etiology or cause). MULTIPLE RESPONSE 1. A nurse is developing nursing diagnoses for a group of patients. Which nursing diagnoses will the nurse use? (Select all that apply.) ANS: C, D Impaired physical mobility and Nausea are the only correctly written nursing diagnoses. All the rest are incorrectly written. Anxiety lists a diagnostic test as the etiology. Impaired gas exchange lists a medical diagnosis as the etiology. Risk for falls has a legally inadvisable statement for an etiology. Chapter 18: Planning Nursing Care Potter et al.: Fundamentals of Nursing, 9th Edition MULTIPLE CHOICE 1. The nurse completes a thorough assessment of a patient and analyzes the data to identify nursing diagnoses. Which step will the nurse take next in the nursing process? a. 1, 3, 4, 2, 5 b. 1, 3, 4, 5, 2 c. 1, 4, 3, 5, 2 d. 1, 4, 3, 2, 5 a. Anxiety related to barium enema b. Impaired gas exchange related to asthma c. Impaired physical mobility related to incisional pain d. Nausea related to adverse effect of cancer medication e. Risk for falls related to nursing assistive personnel leaving bedrail down a. Assessment b. Diagnosis c. Planning d. Implementation 87 A&E I Comprehensive Testbank ANS: C After identifying a patient’s nursing diagnoses and collaborative problems, a nurse prioritizes the diagnoses, sets patient-centered goals and expected outcomes, and chooses nursing interventions appropriate for each diagnosis. This is the third step of the nursing process, planning. The assessment phase of the nursing process involves gathering data. The implementation phase involves carrying out appropriate nursing interventions. During the evaluation phase, the nurse assesses the achievement of goals and effectiveness of interventions. 2. A patient’s plan of care includes the goal of increasing mobility this shift. As the patient is ambulating to the bathroom at the beginning of the shift, the patient suffers a fall. Which initial action will the nurse take next to revise the plan of care? ANS: D Nurses revise a plan when a patient’s status changes; assessment is the first step. Know also that a plan of care is dynamic and changes as the patient’s needs change. Asking physical therapy to assist the patient is premature before assessing the patient and awaiting the health care provider’s orders. The nurse may not need to disregard all previous diagnoses. Some diagnoses may still apply, but the patient needs to be assessed first. Setting new priorities is not recommended before assessment and establishing diagnoses. 3. Which information indicates a nurse has a good understanding of a goal? ANS: C A goal is a broad statement that describes a desired change in a patient’s condition or behavior. A goal is mutually set with the patient. An expected outcome is the measurable changes (patient behavior, physical state, or perception) that must be achieved to reach a goal. Expected outcomes are time limited, measurable ways of determining if a goal is met. 4. A nurse is developing a care plan for a patient with a pelvic fracture on bed rest. Which goal statement is realistic for the nurse to assign to this patient? ANS: A A goal is a broad statement of desired change; the patient will increase activity level is a broad statement. Turning is the expected outcome. When determining goals, the nurse needs to ensure that the goal is individualized and realistic for the patient. Since the patient is on bed rest, using a walker and bedside commode is contraindicated. a. Consult physical therapy. b. Establish a new plan of care. c. Set new priorities for the patient. d. Assess the patient. a. It is a statement describing the patient’s accomplishments without a time restriction. b. It is a realistic statement predicting any negative responses to treatments. c. It is a broad statement describing a desired change in a patient’s behavior. d. It is a measurable change in a patient’s physical state. a. Patient will increase activity level this shift. b. Patient will turn side to back to side with assistance every 2 hours. c. Patient will use the walker correctly to ambulate to the bathroom as needed. d. Patient will use a sliding board correctly to transfer to the bedside commode as needed. 88 A&E I Comprehensive Testbank 5. The following statements are on a patient’s nursing care plan. Which statement will the nurse use as an outcome for a goal of care? ANS: A An expected outcome is a specific and measurable change that is expected as a result of nursing care. Verbalizing decreased pain on a 0 to 10 scale is an outcome. The other three options in this question are goals. Demonstrating increased mobility in 2 days and understanding necessary dietary changes by discharge are short-term goals because they are expected to occur in less than a week. Demonstrating increased tolerance to activity over a month-long period is a long-term goal because it is expected to occur over a longer period of time. 6. A charge nurse is reviewing outcome statements using the SMART approach. Which patient outcome statement will the charge nurse praise to the new nurse? ANS: C An expected outcome should be patient centered; should address one patient response; should be specific, measurable, attainable, realistic, and timed (SMART approach). The statement “The patient will feed self at all mealtimes today without reports of shortness of breath” includes all SMART criteria for goal writing. “The patient will ambulate in hallways” is missing a time limit. Administering pain medication and monitoring the patient’s heart rhythm are nursing interventions; they do not reflect patient behaviors or actions. 7. A nursing assessment for a patient with a spinal cord injury leads to several pertinent nursing diagnoses. Which nursing diagnosis is the highest priority for this patient? ANS: D Reflex urinary incontinence is highest priority. If a patient’s incontinence is not addressed, then the patient is at higher risk of impaired skin integrity and infection. Remember that the Risk for diagnoses are potential problems. They may be prioritized higher in some cases but not in this situation. Spiritual distress is an actual diagnosis, but the adverse effects that could result from not assisting the patient with urinary elimination take priority in this case. 8. The new nurse is caring for six patients in this shift. After completing their assessments, the nurse asks where to begin in developing care plans for these patients. Which statement is an appropriate suggestion by another nurse? a. The patient will verbalize a decreased pain level less than 3 on a 0 to 10 scale by the end of this shift. b. The patient will demonstrate increased tolerance to activity over the next month. c. The patient will understand needed dietary changes by discharge. d. The patient will demonstrate increased mobility in 2 days. a. The patient will ambulate in hallways. b. The nurse will monitor the patient’s heart rhythm continuously this shift. c. The patient will feed self at all mealtimes today without reports of shortness of breath. d. The nurse will administer pain medication every 4 hours to keep the patient free from discomfort. a. Risk for impaired skin integrity b. Risk for infection c. Spiritual distress d. Reflex urinary incontinence 89 A&E I Comprehensive Testbank ANS: D Work from your plan of care and use patients’ priorities to organize the order for delivering interventions and organizing documentation of care. When developing a plan of care, the nurse needs to rank the nursing diagnoses in order of priority, then select appropriate interventions. Choosing all the interventions should take place after ranking of the diagnoses, and interventions should be prioritized by patient needs, not just by time. The chosen interventions should be evidence based with scientific rationales, but the diagnoses need to be prioritized first to prioritize interventions. Goals for a patient should be mutually set, not just chosen by the nurse. 9. A patient’s son decides to stay at the bedside while his father is confused. When developing the plan of care for this patient, what should the nurse do? ANS: D The family is often a resource to help the patient meet health care goals. Family should be included in the plan of care as much as possible. Meeting some of the family’s needs as well as the patient’s needs will possibly improve the patient’s level of wellness. The son should not be asked to leave if at all possible. In some situations, it may be best that family members not remain in the room, but no evidence in the question stem suggests that this is the case in this situation. The suggestion of asking a female member to stay is not a justified action without a legitimate reason. No reason is given in this question stem for such a suggestion. 10. A nurse is caring for a patient with a nursing diagnosis of Constipation related to slowed gastrointestinal motility secondary to pain medications. Which outcome is mostappropriate for the nurse to include in the plan of care? ANS: A The identified problem, or nursing diagnosis, is constipation. Therefore, the outcome should be that the constipation is relieved. To measure constipation relief, the nurse will be observing for the patient to have a bowel movement. During planning, you select goals and expected outcomes for each nursing diagnosis or problem to provide clear direction for the type of interventions needed to care for your patient and to then evaluate the effectiveness of these interventions. Not taking pain medications may or may not relieve the constipation. Although not taking pain medicines might be an intervention, the nurse doesn’t want the patient to be in pain to relieve constipation. Other measures, such as administering laxatives or stool softeners, might be appropriate interventions but they are not outcomes. The patient walking unassisted to the bathroom addresses mobility, not constipation. The patient may need to walk to the bathroom to have a bowel movement, a. “Choose all the interventions and perform them in order of time needed for each one.” b. “Make sure you identify the scientific rationale for each intervention first.” c. “Decide on goals and outcomes you have chosen for the patients.” d. “Begin with the highest priority diagnoses, then select appropriate interventions.” a. Individualize the care plan only according to the patient’s needs. b. Request that the son leave at bedtime, so the patient can rest. c. Suggest that a female member of the family stay with the patient. d. Involve the son in the plan of care as much as possible. a. Patient will have one soft, formed bowel movement by end of shift. b. Patient will walk unassisted to bathroom by the end of shift. c. Patient will be offered laxatives or stool softeners this shift. d. Patient will not take any pain medications this shift. 90 A&E I Comprehensive Testbank but the appropriate outcome for constipation is that the constipation is relieved as evidenced by a bowel movement—something that the nurse can observe. 11. The nurse performs an intervention for a collaborative problem. Which type of intervention did the nurse perform? ANS: C Collaborative interventions, or interdependent interventions, are therapies that require the combined knowledge, skill, and expertise of multiple health care professionals. Health care provider-initiated (HCP) interventions are dependent nursing interventions, or actions that require an order from the HCP. Nurseinitiated interventions are the independent nursing interventions, or actions that a nurse initiates without supervision or direction from others. 12. A registered nurse administers pain medication to a patient suffering from fractured ribs. Which type of nursing intervention is this nurse implementing? ANS: D The nurse does not have prescriptive authority to order pain medications, unless the nurse is an advanced practice nurse. The intervention is therefore dependent. Administering a medication, implementing an invasive procedure (e.g., inserting a Foley catheter, starting an intravenous [IV] infusion), and preparing a patient for diagnostic tests are examples of health care provider-initiated interventions. A collaborative, or an interdependent, intervention involves therapies that require combined knowledge, skill, and expertise from multiple health care professionals. Nurse-initiated interventions are the independent nursing interventions, or actions that a nurse initiates without supervision or direction from others. 13. Which action indicates the nurse is using a PICOT question to improve care for a patient? ANS: B The best answer is implementing interventions based on scientific research. Using results of a literature search to a PICOT question can help a nurse decide which interventions to use. Practicing based on evidence presented in court is incorrect. Practice is based on current research. Using standardized care plans may be one example of evidence-based practice, but it is not used on all patients. The nurse must be careful in using standardized care plans to ensure that each patient’s plan of care is still individualized. Planning care based on tradition is incorrect because nursing care should be based on current research. 14. A nurse is developing a care plan. Which intervention is most appropriate for the nursing diagnostic statement Risk for loneliness related to impaired verbal communication? a. Dependent b. Independent c. Interdependent d. Physician-initiated a. Collaborative b. Independent c. Interdependent d. Dependent a. Practices nursing based on the evidence presented in court b. Implements interventions based on scientific research c. Uses standardized care plans for all patients. d. Plans care based on tradition a. Provide the patient with a writing board each shift. b. Obtain an interpreter for the patient as soon as possible. 91 A&E I Comprehensive Testbank ANS: A Choose interventions to alter the etiological (related to) factor or causes of the diagnosis. If the etiology is impaired verbal communication, then the nurse should choose an intervention that will address the problem. Providing the patient with a writing board will allow the patient to communicate by writing because the patient is unable to communicate verbally at this time. Obtaining an interpreter might be an appropriate intervention if the patient spoke a foreign language. Assisting with swallowing exercises will help the patient with swallowing, which is a different etiology than impaired verbal communication. Asking the family to provide a sitter at all times is many times unrealistic and does not relate to the impaired verbal communication; the goal would relate to the loneliness. 15. A nurse is completing a care plan. Which intervention is most appropriate for the nursing diagnostic statement Impaired skin integrity related to shearing forces? ANS: B The most appropriate intervention for the diagnosis of Impaired skin integrity is to turn the patient. Choose interventions to alter the etiological (related to) factor or causes of the diagnosis. The other options do not directly address the shearing forces. The patient may need pain medication, but Acute pain would be another nursing diagnosis. Monitoring vital signs does not have when or how often these should be done. Keeping the side rails up addresses safety, not skin integrity. 16. A patient has reduced muscle strength following a left-sided stroke and is at risk for falling. Which intervention is most appropriate for the nursing diagnostic statement Risk for falls? ANS: D Risk for falls is a risk (potential) nursing diagnosis; therefore, the nurse needs to implement actions that will prevent a fall. Assisting the patient into and out of bed is the most appropriate intervention to prevent the patient from falling. Encouraging activity builds muscle strength, and helping the patient with transfers ensures patient safety. Encouraging the patient to stay in bed will not promote muscle strength. Decreased muscle strength is the risk factor placing the patient in jeopardy of falling. The side rails should be up, not down, according to agency policy. This will remind the patient to ask for help to get up and will keep the patient from rolling out of bed. The patient should be placed near the nurses’ station, so a staff member can quickly get to the room and assist the patient if necessary. 17. Which action will the nurse take after the plan of care for a patient is developed? c. Assist the patient in performing swallowing exercises each shift. d. Ask the family to provide a sitter to remain with the patient at all times. a. Administer pain medication every 4 hours as needed. b. Turn the patient every 2 hours, even hours. c. Monitor vital signs, especially rhythm. d. Keep the bed side rails up at all times. a. Keep all side rails down at all times. b. Encourage patient to remain in bed most of the shift. c. Place patient in room away from the nurses’ station if possible. d. Assist patient into and out of bed every 4 hours or as tolerated. a. Place the original copy in the chart, so it cannot be tampered with or revised. b. Communicate the plan to all health care professionals involved in the patient’s care. c. File the plan of care in the administration office for legal examination. 92 A&E I Comprehensive Testbank ANS: B Setting realistic goals and outcomes often means you must communicate these goals and outcomes to caregivers in other settings who will assume responsibility for patient care. The plan of care communicates nursing care priorities to nurses and other health care professionals. Know also that a plan of care is dynamic and changes as the patient’s needs change. All health care professionals involved in the patient’s care need to be informed of the plan of care. The plan of care is not sent to the administrative office or quality assurance office. 18. A nurse is preparing to make a consult. In which order, beginning with the first step, will the nurse take? 1. Identify the problem. 2. Discuss the findings and recommendation. 3. Provide the consultant with relevant information about the problem. 4. Contact the right professional, with the appropriate knowledge and expertise. 5. Avoid bias by not providing a lot of information based on opinion to the consultant. ANS: A The first step in making a consultation is to assess the situation and identify the general problem area. Second, direct the consultation to the right professional such as another nurse or social worker. Third, provide a consultant with relevant information about the problem area and seek a solution. Fourth, do not prejudice or influence consultants. Fifth, be available to discuss a consultant’s findings and recommendations. 19. A hospital’s wound nurse consultant made a recommendation for nurses on the unit about how to care for the patient’s dressing changes. Which action should the nurses take next? ANS: A Incorporate the consultant’s recommendations into the care plan. The wound nurse clearly recommends that nurses on the unit, not the patient, should continue dressing changes. The nurses should not make a wrong assumption that the wound nurse is doing all the dressing changes. The recommendation states for the nurses to do the dressing changes. If the nurses feel strongly about obtaining another opinion, then the health care provider should be contacted. No evidence in the question suggests that the patient needs a second opinion. MULTIPLE RESPONSE 1. A nurse is planning care for a patient with a nursing diagnosis of Impaired skin integrity. The patient needs many nursing interventions, including a dressing change, several intravenous antibiotics, and a walk. Which factors does the nurse consider when prioritizing interventions? (Select all that apply.) d. Send the plan of care to quality assurance for review. a. 1, 4, 3, 5, 2 b. 4, 1, 3, 2, 5 c. 1, 4, 5, 3, 2 d. 4, 3, 1, 5, 2 a. Include dressing change instructions and frequency in the care plan. b. Assume that the wound nurse will perform all dressing changes. c. Request that the health care provider look at the wound. d. Encourage the patient to perform the dressing changes. a. Rank all the patient’s nursing diagnoses in order of priority. b. Do not change priorities once they’ve been established. c. Set priorities based solely on physiological factors. 93 A&E I Comprehensive Testbank ANS: A, D, E By ranking a patient’s nursing diagnoses in order of importance and always monitoring changing signs and symptoms (defining characteristics) of patient problems, you attend to each patient’s most important needs and better organize ongoing care activities. Prioritizing the problems, or nursing diagnoses, will help the nurse decide which problem to address first. Symptom pattern recognition from your assessment database and certain knowledge triggers help you understand which diagnoses require intervention and the associated time frame to intervene effectively. Planning requires critical thinking applied through deliberate decision making and problem solving. The nurse avoids setting priorities based solely on physiological factors; other factors should be considered as well. The order of priorities changes as a patient’s condition and needs change, sometimes within a matter of minutes. 2. A nurse is teaching the staff about the benefits of Nursing Outcomes Classification. Which information should the nurse include in the teaching session? (Select all that apply.) ANS: C, E Nursing Outcomes Classification (NOC) links outcomes to NANDA International nursing diagnoses. Such a rating system adds objectivity to judging a patient’s progress. Using standardized nursing terminologies such as NOC makes it more possible to measure aspects of nursing care on a national and international level. The indicators for each NOC outcome allow measurement of the outcomes at any point on a 5-point Likert scale from most negative to most positive. This resource is an option you can use in selecting goals and outcomes (not interventions) for your patients. The Nursing Interventions Classification model includes three levels: domains, classes, and interventions for ease of use. The seven domains are the highest level (level 1) of the model, using broad terms (e.g., safety and basic physiological) to organize the more specific classes and interventions. Chapter 19: Implementing Nursing Care Potter et al.: Fundamentals of Nursing, 9th Edition MULTIPLE CHOICE 1. A nurse is providing nursing care to patients after completing a care plan from nursing diagnoses. In which step of the nursing process is the nurse? d. Consider time as an influencing factor. e. Utilize critical thinking. a. Includes seven domains for level 1 b. Uses an easy 3-point Likert scale c. Adds objectivity to judging a patient’s progress d. Allows choice in which interventions to choose e. Measures nursing care on a national and international level a. Assessment b. Planning c. Implementation 94 A&E I Comprehensive Testbank ANS: C Implementation, the fourth step of the nursing process, formally begins after a nurse develops a plan of care. With a care plan based on clear and relevant nursing diagnoses, a nurse initiates interventions that are designed to assist the patient in achieving the goals and expected outcomes needed to support or improve the patient’s health status. The nurse gathers data during the assessment phase and mutually sets goals and prioritizes care during the planning phase. During the evaluation phase, the nurse determines the achievement of goals and effectiveness of interventions. 2. The nurse is teaching a new nurse about protocols. Which information from the new nurse indicates a correct understanding of the teaching? ANS: B A clinical practice guideline or protocol is a systematically developed set of statements that helps nurses, physicians, and other health care providers make decisions about appropriate health care for specific clinical situations. This guideline establishes interventions for specific health care problems or conditions. The protocol does not replace the nursing care plan. Evidence-based guidelines from protocols can be incorporated into an individualized plan of care. A clinical guideline is not the same as a hospital policy. Standing orders contain orders for the care of a specific group of patients. A protocol is not a prescriptive order form like a standing order. 3. The standing orders for a patient include acetaminophen 650 mg every 4 hours prn for headache. After assessing the patient, the nurse identifies the need for headache relief and determines that the patient has not had acetaminophen in the past 4 hours. Which action will the nurse take next? ANS: A A standing order is a preprinted document containing orders for the conduct of routine therapies, monitoring guidelines, and/or diagnostic procedures for specific patients with identified clinical problems. The nurse will administer the medication. Notifying the health care provider is not necessary if a standing order exists. The nursing assistive personnel are not licensed to administer medications; therefore, medication administration should not be delegated to this person. A pain assessment should be performed before and after pain medication administration to assess the need for and effectiveness of the medication. 4. Which action indicates a nurse is using critical thinking for implementation of nursing care to patients? d. Evaluation a. Protocols are guidelines to follow that replace the nursing care plan. b. Protocols assist the clinician in making decisions and choosing interventions for specific health care problems or conditions. c. Protocols are policies designating each nurse’s duty according to standards of care and a code of ethics. d. Protocols are prescriptive order forms that help individualize the plan of care. a. Administer the acetaminophen. b. Notify the health care provider to obtain a verbal order. c. Direct the nursing assistive personnel to give the acetaminophen. d. Perform a pain assessment only after administering the acetaminophen. a. Determines whether an intervention is correct and appropriate for the given situation b. Reads over the steps and performs a procedure despite lack of clinical competency c. Establishes goals for a particular patient without assessment 95 A&E I Comprehensive Testbank ANS: A As you implement interventions, use critical thinking to confirm whether the interventions are correct and still appropriate for a patient’s clinical situation. You are responsible for having the necessary knowledge and clinical competency to perform interventions for your patients safely and effectively. The nurse needs to recognize the safety hazards of performing an intervention without clinical competency and seek assistance from another nurse. The nurse cannot evaluate interventions until they are implemented. Patients need ongoing assessment before establishing goals because patient conditions can change very rapidly. 5. A nurse is reviewing a patient’s care plan. Which information will the nurse identify as a nursing intervention? ANS: C Providing assistance to a patient who is ambulating is a nursing intervention. The statement, “The patient will ambulate in the hallway twice this shift using crutches correctly” is a patient outcome. Impaired physical mobilityis a nursing diagnosis. The statement that the patient is unable to bear weight and ambulate can be included with assessment data and is a defining characteristic for the diagnosis of Impaired physical mobility. 6. A patient recovering from a leg fracture after a fall reports having dull pain in the affected leg and rates it as a 7 on a 0 to 10 scale. The patient is not able to walk around in the room with crutches because of leg discomfort. Which nursing intervention is priority? ANS: D The patient’s pain is a 7, indicating the priority is pain relief (administer pain medication). Acute pain is the priority because the nurse can address the problem of immobility after the patient receives adequate pain relief. Assisting the patient to walk or obtaining a walker will not address the pain the patient is experiencing. 7. The nurse is caring for a patient who requires a complex dressing change. While in the patient’s room, the nurse decides to change the dressing. Which action will the nurse take just before changing the dressing? ANS: C Always be sure a patient is physically and psychologically ready for any interventions or procedures. After determining the patient’s readiness for the dressing change, the nurse gathers needed supplies. The nurse establishes goals and outcomes before intervening. The nurse needs to ask another staff member to help if necessary after determining readiness of the patient. d. Evaluates the effectiveness of interventions a. The patient will ambulate in the hallway twice this shift using crutches correctly. b. Impaired physical mobility related to inability to bear weight on right leg. c. Provide assistance while the patient walks in the hallway twice this shift with crutches. d. The patient is unable to bear weight on right lower extremity. a. Assist the patient to walk in the room with crutches. b. Obtain a walker for the patient. c. Consult physical therapy. d. Administer pain medication. a. Gathers and organizes needed supplies b. Decides on goals and outcomes for the patient c. Assesses the patient’s readiness for the procedure d. Calls for assistance from another nursing staff member 96 A&E I Comprehensive Testbank 8. A patient visiting with family members in the waiting area tells the nurse “I don’t feel good, especially in the stomach.” What should the nurse do? ANS: B In this case, the environment needs to be conducive to completing a thorough assessment. A patient’s care environment needs to be safe and conducive to implementing therapies. When you need to expose a patient’s body parts, do so privately by closing room doors or curtains because the patient will then be more relaxed; the patient needs to return to the room for an abdominal assessment for privacy and comfort. The family can remain in the waiting area while the nurse assists the patient back to the room. Beginning the assessment in the waiting area (lie down on the sofa) in the presence of family and other visitors does not promote privacy and patient comfort. Telling the patient that the dinner tray is almost ready is making an assumption that the abdominal discomfort is due to not eating. The nurse needs to perform an assessment first. 9. A newly admitted patient who is morbidly obese asks the nurse for assistance to the bathroom for the first time. Which action should the nurse take initially? ANS: C Before beginning care, review the plan to determine the need for assistance and the type required. Before intervening, the nurse must check the patient’s orders. For example, if the patient is on bed rest, the nurse will need to explain the use of a bedpan rather than helping the patient get out of bed to go to the bathroom. Asking for assistive personnel is appropriate after making sure the patient can get out of bed. If the patient is obese, the nurse will likely need assistance in getting the patient to the bathroom. Medicating the patient before checking the orders is not advised in this situation. Before medicating for pain, the nurse needs to perform a pain assessment. Offering the patient a walker is a premature intervention until the orders are verified. 10. A new nurse is working in a unit that uses interdisciplinary collaboration. Which action will the nurse take? ANS: B Good communication between other health care providers builds trust and is related to the acceptance of your role in the health care team. As a beginning nurse, you will not be considered a leader of the health care team, but your input as an interdisciplinary team member is critical. Interdisciplinary involves other health care providers, not just nurses. Organizational culture includes leadership, communication processes, shared beliefs about the quality of clinical guidelines, and conflict resolution. 11. Which action should the nurse take first during the initial phase of implementation? a. Request that the family leave, so the patient can rest. b. Ask the patient to return to the room, so the nurse can inspect the abdomen. c. Ask the patient when the last bowel movement was and to lie down on the sofa. d. Tell the patient that the dinner tray will be ready in 15 minutes and that may help the stomach feel better. a. Ask for at least two other assistive personnel to come to the room. b. Medicate the patient to alleviate discomfort while ambulating. c. Review the patient’s activity orders. d. Offer the patient a walker. a. Act as a leader of the health care team. b. Develop good communication skills. c. Work solely with nurses. d. Avoid conflict. a. Determine patient outcomes and goals. 97 A&E I Comprehensive Testbank ANS: D Assessment is a continuous process that occurs each time the nurse interacts with a patient. During the initial phase of implementation, reassess the patient. Determining the patient’s goals and prioritizing diagnoses take place in the planning phase before choosing interventions. Evaluation is the last step of the nursing process. 12. Vital signs for a patient reveal a high blood pressure of 187/100. Orders state to notify the health care provider for diastolic blood pressure greater than 90. What is the nurse’s first action? ANS: C Communication to other health care professionals must be timely, accurate, and relevant to a patient’s clinical situation. The best answer is to reassess the patient for other symptoms or problems, and then notify the health care provider according to the orders. Reviewing the potassium level does not address the problem of high blood pressure. The nurse does not follow the protocol since the order says to notify the health care provider. The orders read to notify the health care provider, not administer medications. 13. Which initial intervention is most appropriate for a patient who has a new onset of chest pain? ANS: A Preparation for implementation ensures efficient, safe, and effective nursing care; the first activity is reassessment. The cause of the patient’s chest pain is unknown, so the patient needs to be reassessed before pain medication is administered or a chest x-ray is obtained. The nurse then notifies the patient’s health care provider of the patient’s current condition in anticipation of receiving further orders. The patient’s chest pain could be due to muscular injury or a pulmonary issue. The nurse needs to reassess first. 14. A nurse is making initial rounds on patients. Which intervention for a patient with poor wound healing should the nurse perform first? ANS: C The most appropriate initial intervention is to assess the wound (observe wound appearance and edges). The nurse must assess the wound first before the findings can be documented, reinforcement of the dressing, and the actual skill of dressing changes. b. Prioritize patient’s nursing diagnoses. c. Evaluate interventions. d. Reassess the patient. a. Follow the clinical protocol for a stroke. b. Review the most recent lab results for the patient’s potassium level. c. Assess the patient for other symptoms or problems, and then notify the health care provider. d. Administer an antihypertensive medication from the stock supply, and then notify the health care provider. a. Reassess the patient. b. Notify the health care provider. c. Administer a prn medication for pain. d. Call radiology for a portable chest x-ray. a. Reinforce the wound dressing as needed with 4 × 4 gauze. b. Perform the ordered dressing change twice daily. c. Observe wound appearance and edges. d. Document wound characteristics. 98 A&E I Comprehensive Testbank 15. The nurse establishes trust and talks with a school-aged patient before administering an injection. Which type of implementation skill is the nurse using? ANS: B Nursing practice includes cognitive, interpersonal, and psychomotor skills. Interpersonal skills involve developing trusting relationships with patients, conveying caring and compassion, and communicating clearly. Cognitive skills include critical thinking and decision-making skills. Psychomotor skill requires the integration of cognitive and motor abilities, such as administering the injection. Being judgmental is not appropriate in nursing; nurses are nonjudgmental. 16. The nurse inserts an intravenous (IV) catheter using the correct technique and following the recommended steps according to standards of care and hospital policy. Which type of implementation skill is the nurse using? ANS: C Nursing practice includes cognitive, interpersonal, and psychomotor skills. Psychomotor skill requires the integration of cognitive and motor abilities. The nurse in this example displayed the psychomotor skill of inserting an intravenous catheter while following standards of care and integrating knowledge of anatomy and physiology. Cognitive involves the application of critical thinking and use of good judgment in making sound clinical decisions. Interpersonal skills involve developing trusting relationships with patients, conveying caring and compassion, and communicating clearly. 17. A staff development nurse is providing an inservice for other nurses to educate them about the Nursing Interventions Classification (NIC) system. During the inservice, which statement made by one of the nurses in the room requires the staff development nurse to clarify the information provided? ANS: A NIC does not help determine the cost of services provided by nurses. The staff development nurse would need to correct this misconception. Because this system is specific to nursing practice, it would not help medical students determine the costs of care. The NIC system developed by the University of Iowa differentiates nursing practice from that of other health care disciplines. All the other statements are true. Benefits of using NIC include enhancing communication among nursing staff and documentation, especially within health information systems such as an electronic documentation system. NIC also helps nurses identify the nursing interventions they implement most frequently. Units that identify routine nursing interventions can use this information to develop checklists for orientation. a. Cognitive b. Interpersonal c. Psychomotor d. Judgmental a. Cognitive b. Interpersonal c. Psychomotor d. Judgmental a. “This system can help medical students determine the cost of the care they provide to patients.” b. “If the nursing department uses this system, communication among nurses who work throughout the hospital may be enhanced.” c. “We could use this system to help organize orientation for new nursing employees because we can better explain the nursing interventions we use most frequently on our unit.” d. “The NIC system provides one way to improve safe and effective documentation in the hospital’s electronic health record.” 99 A&E I Comprehensive Testbank 18. The nurse is intervening for a family member with role strain. Which direct care nursing intervention is most appropriate? ANS: B Family caregivers need assistance in adjusting to the physical and emotional demands of caregiving. Sometimes they need respite (i.e., a break from providing care). Counseling is an example of a direct care nursing intervention. The other options do not address the identified problem of role strain (activities of daily living and range-of-motion exercises). Consulting is an indirect care nursing intervention. 19. The nurse is intervening for a patient that has a risk for a urinary infection. Which direct care nursing intervention is most appropriate? ANS: A Teaching proper handwashing technique is a direct care nursing intervention. All the rest are indirect nursing care: cleaning the toilet, transporting specimens, and performing hand-off reports. 20. The nurse is revising the care plan. In which order will the nurse perform the tasks, beginning with the first step? 1. Revise specific interventions. 2. Revise the assessment column. 3. Choose the evaluation method. 4. Delete irrelevant nursing diagnoses. ANS: A Modification of an existing written care plan includes four steps: 1. Revise data in the assessment column to reflect the patient’s current status. Date any new data to inform other members of the health care team of the time that the change occurred. 2. Revise the nursing diagnoses. Delete nursing diagnoses that are no longer relevant and add and date any new diagnoses. Revise related factors and the patient’s goals, outcomes, and priorities. Date any revisions. 3. Revise specific interventions that correspond to the new nursing diagnoses and goals. Be sure that revisions reflect the patient’s present status. 4. Choose the method of evaluation for determining whether you achieved patient outcomes. MULTIPLE RESPONSE a. Assisting with activities of daily living b. Counseling about respite care options c. Teaching range-of-motion exercises d. Consulting with a social worker a. Teaches proper handwashing technique b. Properly cleans the patient’s toilet c. Transports urine specimen to the lab d. Informs the oncoming nurse during hand-off a. 2, 4, 1, 3 b. 4, 2, 1, 3 c. 3, 4, 2, 1 d. 4, 2, 3, 1 100 A&E I Comprehensive Testbank 1. A nurse is implementing interventions for a group of patients. Which actions are nursing interventions? (Select all that apply.) ANS: C, D, E A nursing intervention is any treatment based on clinical judgment and knowledge that a nurse performs to enhance patient outcomes. Repositioning, teaching, and transferring a patient are examples of nursing interventions. Ordering a chest x-ray and prescribing antibiotics are examples of medical interventions performed by a health care provider. 2. A nurse is providing nursing care to a group of patients. Which actions are direct care interventions? (Select all that apply.) ANS: A, B, C, E All of the interventions listed (ambulating, inserting a feeding tube, performing resuscitation, and teaching) are direct care interventions involving patient and nurse interaction, except documenting wound care. Documenting wound care is an example of an indirect intervention. 3. A nurse is preparing to carry out interventions. Which resources will the nurse make sure are available? (Select all that apply.) ANS: A, B, D A nurse will organize time and resources in preparation for implementing nursing care. Most nursing procedures require some equipment or supplies. Before performing an intervention, decide which supplies you need and determine their availability. Patient care staff (assistive personnel) work together as patients’ needs demand it. A patient’s care environment needs to be safe and conducive to implementing therapies. Confidence and creativity are needed to provide safe and effective patient care; however, these are critical thinking attitudes, not resources. 4. Which interventions are appropriate for a patient with diabetes and poor wound healing? (Select all that apply.) a. Order chest x-ray for suspected arm fracture. b. Prescribe antibiotics for a wound infection. c. Reposition a patient who is on bed rest. d. Teach a patient preoperative exercises. e. Transfer a patient to another hospital unit. a. Ambulating a patient b. Inserting a feeding tube c. Performing resuscitation d. Documenting wound care e. Teaching about medications a. Equipment b. Safe environment c. Confidence d. Assistive personnel e. Creativity a. Perform dressing changes twice a day as ordered. b. Teach the patient about signs and symptoms of infection. 101 A&E I Comprehensive Testbank ANS: A, B, C, E Nursing priorities include interventions directed at enhancing wound healing. Teaching the patient about signs and symptoms of infection will help the patient identify signs of appropriate wound healing and know when the need for calling the health care provider arises. Performing dressing changes, controlling blood sugars through administration of medications, and instructing the family in dressing changes all contribute to wound healing. As long as a patient is stable and alert, it is appropriate to allow family to assist with care. The patient should be allowed to discuss body image changes. Chapter 20: Evaluation Potter et al.: Fundamentals of Nursing, 9th Edition MULTIPLE CHOICE 1. A nurse determines that the patient’s condition has improved and has met expected outcomes. Which step of the nursing process is the nurse exhibiting? ANS: D Evaluation, the final step of the nursing process, is crucial to determine whether, after application of the first four steps of the nursing process, a patient’s condition or well-being improves and if goals have been met. Assessment, the first step of the process, includes data collection. Planning, the third step of the process, involves setting priorities, identifying patient goals and outcomes, and selecting nursing interventions. During implementation, nurses carry out nursing care, which is necessary to help patients achieve their goals. 2. A nurse completes a thorough database and carries out nursing interventions based on priority diagnoses. Which action will the nurse take next? ANS: D Evaluation, the final step of the nursing process, is crucial to determine whether, after application of the first four steps of the nursing process, a patient’s condition or well-being improves. Assessment involves gathering information about the patient. During the planning phase, patient outcomes are determined. Implementation involves carrying out appropriate nursing interventions. 3. A new nurse asks the preceptor to describe the primary purpose of evaluation. Which statement made by the nursing preceptor is most accurate? c. Instruct the family about how to perform dressing changes. d. Gently refocus patient from discussing body image changes. e. Administer medications to control the patient’s blood sugar as ordered. a. Assessment b. Planning c. Implementation d. Evaluation a. Assessment b. Planning c. Implementation d. Evaluation a. “An evaluation helps you determine whether all nursing interventions were completed.” b. “During evaluation, you determine when to downsize staffing on nursing units.” 102 A&E I Comprehensive Testbank ANS: C Evaluation is a methodical approach for determining if nursing implementation was effective in influencing a patient’s progress or condition in a favorable way. During evaluation, you do not simply determine whether nursing interventions were completed. The evaluation process is not used to determine when to downsize staffing or how to eliminate paperwork and care planning. 4. After assessing the patient and identifying the need for headache relief, the nurse administers acetaminophen for the patient’s headache. Which action by the nurse is priority for this patient? ANS: C The nurse’s priority action for this patient is to evaluate whether the nursing intervention of administering acetaminophen was effective. The nurse does not have enough evaluative data at this point to determine whether headache needs to be discontinued. Assessment is the nurse’s responsibility and is not to be delegated to nursing assistive personnel. The nurse does not have enough evaluative data to determine whether the patient’s plan of care needs to be revised. 5. A nurse is getting ready to discharge a patient who has a problem with physical mobility. What does the nurse need to do before discontinuing the patient’s plan of care? ANS: B You evaluate whether the results of care match the expected outcomes and goals set for a patient before discontinuing a patient’s plan of care. The patient needs transportation, but that does not address the patient’s mobility status. Whether the patient has a follow-up appointment and ensuring that prescriptions are filled do not evaluate the problem of mobility. 6. The nurse is evaluating whether patient goals and outcomes have been met for a patient with physical mobility problems due to a fractured leg. Which finding indicates the patient has met an expected outcome? ANS: B The patient’s being able to ambulate in the hallway with crutches is an expected outcome of nursing care. The outcomes of nursing practice are the measurable conditions of patient, family or community status, behavior, or perception. These outcomes are the criteria used to judge success in delivering nursing care. The option c. “Nurses use evaluation to determine the effectiveness of nursing care.” d. “Evaluation eliminates unnecessary paperwork and care planning.” a. Eliminate headache from the nursing care plan. b. Direct the nursing assistive personnel to ask if the headache is relieved. c. Reassess the patient’s pain level in 30 minutes. d. Revise the plan of care. a. Determine whether the patient has transportation to get home. b. Evaluate whether patient goals and outcomes have been met. c. Establish whether the patient has a follow-up appointment scheduled. d. Ensure that the patient’s prescriptions have been filled to take home. a. The nurse provides assistance while the patient is walking in the hallways. b. The patient is able to ambulate in the hallway with crutches. c. The patient will deny pain while walking in the hallway. d. The patient’s level of mobility will improve. 103 A&E I Comprehensive Testbank stating, “The patient’s level of mobility will improve” is a broader goal statement. The nurse’s assisting a patient to ambulate is an intervention. The patient’s denying pain is an expected outcome for pain, not for physical mobility problems. 7. The nurse is evaluating whether a patient’s turning schedule was effective in preventing the formation of pressure ulcers. Which finding indicates success of the turning schedule? ANS: D To determine whether a turning schedule is successful, the nurse needs to assess for the presence of skin breakdown. Redness on any part of the body, including only the patient’s heels, indicates that the turning schedule was not successful. Documentation of interventions does not evaluate whether patient outcomes were met. Eating 100% of meals does not evaluate the effectiveness of a turning schedule. 8. A nurse has instituted a turn schedule for a patient to prevent skin breakdown. Upon evaluation, the nurse finds that the patient has a stage II pressure ulcer on the buttocks. Which action will the nurse take next? ANS: A If a nursing diagnosis is unresolved or if you determine that a new problem has perhaps developed, reassessment is necessary. A complete reassessment of patient factors relating to an existing nursing diagnosis and etiology is necessary when modifying a plan. The nurse must assess before revising, delegating and applying medication. The breakdown may be a result of inadequate nutritional intake and medication cannot be applied unless there is an order. 9. A new nurse is confused about using evaluative measures when caring for patients and asks the charge nurse for an explanation. Which response by the charge nurse is most accurate? ANS: B You conduct evaluative measures to determine if your patients met expected outcomes, not if nursing interventions were completed. Evaluative measures are assessment skills and techniques. Evaluative measures are not multiple-page documents, and they are used to assess the patient’s status, not the nurse’s performance or progress from novice to expert. 10. The nurse is caring for a patient who has an open wound and is evaluating the progress of wound healing. Which priority action will the nurse take? a. Staff documentation of turning the patient every 2 hours b. Presence of redness only on the heels of the patient c. Patient’s eating 100% of all meals d. Absence of skin breakdown a. Reassess the patient and situation. b. Revise the turning schedule to increase the frequency. c. Delegate turning to the nursing assistive personnel. d. Apply medication to the area of skin that is broken down. a. “Evaluative measures are multiple-page documents used to evaluate nurse performance.” b. “Evaluative measures include assessment data used to determine whether patients have met their expected outcomes and goals.” c. “Evaluative measures are used by quality assurance nurses to determine the progress a nurse is making from novice to expert nurse.” d. “Evaluative measures are objective views for completion of nursing interventions.” a. Ask the nursing assistive personnel if the wound looks better. 104 A&E I Comprehensive Testbank ANS: C You examine the results of care by using evaluative measures, which are assessment skills and techniques (e.g., observations, physiological measurements, use of measurement scales, and patient interview). The nurse performs evaluative measures, such as completing a wound assessment, to evaluate wound healing. Nurses do not delegate assessment to nursing assistive personnel. Documenting “better” is subjective and does not objectively describe the wound. Leaving the dressing off for the nurse’s benefit of easier access is not a part of the evaluation process. 11. The nurse is caring for a patient who has an order to change a dressing twice a day, at 0600 and 1800. At 1400, the nurse notices that the dressing is saturated and leaking. What is the nurse’s next action? ANS: B Because the dressing is saturated and leaking, the nurse needs to revise the plan of care and change the dressing now. Reflection-in-action involves a nurse’s ability to recognize how a patient is responding and then adjusting interventions as a result. A nurse will either change the frequency of an intervention, change how the intervention is delivered, or select a new intervention. Waiting until 1800 or for another 2 hours is not appropriate because assessment data reflect that the dressing is saturated and needs to be changed now. Data are insufficient to support discontinuing the plan of care. Instead, data at this time indicate the need for revision of the plan of care. 12. A goal for a patient with diabetes is to demonstrate effective coping skills. Which patient behavior will indicate to the nurse achievement of this outcome? ANS: A Evaluative data that show signs of effective coping will help the nurse determine whether the patient has met the outcome. Talking to family and friends is the only positive option. During evaluation, you perform evaluative measures that allow you to compare clinical data, patient behavior measures, and patient self-report measures collected before implementation with the evaluation findings gathered after administering nursing care. Next, you evaluate whether the results of care match the expected outcomes and goals set for a patient. Consuming high-carbohydrate foods (patient is a diabetic), disliking support group, and spending the day in bed indicate unsuccessful progress toward meeting the patient’s goal. 13. A nurse is providing education to a patient about self-administering subcutaneous injections. The patient demonstrates the self-injection. Which type of indicator did the nurse evaluate? b. Document the progress of wound healing as “better” in the chart. c. Measure the wound and observe for redness, swelling, or drainage. d. Leave the dressing off the wound for easier access and more frequent assessments. a. Wait and change the dressing at 1800 as ordered. b. Revise the plan of care and change the dressing now. c. Reassess the dressing and the wound in 2 hours. d. Discontinue the plan of care for wound care. a. States feels better after talking with family and friends b. Consumes high-carbohydrate foods when stressed c. Dislikes the support group meetings d. Spends most of the day in bed a. Health status b. Health behavior c. Psychological self-control 105 A&E I Comprehensive Testbank ANS: B Health behavior involves demonstrating a psychomotor skill such as self-injection. Health status is a clinical indicator such as exercise tolerance or blood pressure control. The skill is psychomotor, not psychological selfcontrol. Health service utilization is readmission within 30 days or emergency department use. 14. A nurse is evaluating the goal of acceptance of body image in a young teenage girl. Which statement made by the patient is the best indicator of progress toward the goal? ANS: C The nurse is evaluating the improvement in body image. The only positive comment made is that the patient is wearing the blue dress to match her eyes. Worrying about others, making my hips stick out, and going to the pool next summer do not reflect positive changes in body image. 15. A nurse is evaluating goals and expected outcomes for a confused patient. Which finding indicates positive progress toward resolving the confusion? ANS: D The goal for this patient would address a decrease or absence of confusion. Thus, one possible sign that a patient’s confusion is improving is seen when a patient can correctly state the names of family members in the room. You examine the results of care by using evaluative measures that relate to goals and expected outcomes. Keeping the side rails up and using a bed alarm are interventions to promote patient safety and prevent falls. The patient’s denying pain indicates positive progress toward resolving pain. The patient’s wandering the halls is a sign of confusion. 16. A nurse identifies a fall risk when assessing a patient upon admission. The nurse and the patient agree that the goal is for the patient to remain free from falls. However, the patient fell just before shift change. Which action is the nurse’s priority when evaluating the patient? ANS: A When goals and outcomes are not met, you identify the factors that interfere with their achievement. The nurse identifies factors that interfered with goal achievement to determine the cause of the fall. The fall may not have been due to an error by the nursing assistive personnel; therefore, counseling should be reserved until after the cause has been determined. The patient remains a fall risk, so the fall risk sign should remain on the door. The nurse witnessing the fall or the nurse assigned to the patient needs to complete the documentation. The charge nurse can be consulted to review the documentation. d. Health service utilization a. “I’m worried about what those other girls will think of me.” b. “I can’t wear that color. It makes my hips stick out.” c. “I’ll wear the blue dress. It matches my eyes.” d. “I will go to the pool next summer.” a. Patient wanders halls at night. b. Patient’s side rails are up with bed alarm activated. c. Patient denies pain while ambulating with assistance. d. Patient correctly states names of family members in the room. a. Identify factors interfering with goal achievement. b. Counsel the nursing assistive personnel on duty when the patient fell. c. Remove the fall risk sign from the patient’s door because the patient has suffered a fall. d. Request that the more experienced charge nurse complete the documentation about the fall. 106 A&E I Comprehensive Testbank 17. A patient was recently diagnosed with pneumonia. The nurse and the patient have established a goal that the patient will not experience shortness of breath with activity in 3 days with an expected outcome of having no secretions present in the lungs in 48 hours. Which evaluative measure will the nurse use to demonstrate progress toward this goal? ANS: D In this case, the patient’s goal is to not experience shortness of breath with activity in 3 days. If the lung sounds are clear following use of inhaler, the nurse can determine that the patient is making progress toward achieving the expected outcome. One way for the nurse to evaluate the expected outcome is to assess the patient’s lung sounds. Goals are broad statements that describe changes in a patient’s condition or behavior. Expected outcomes are measurable criteria used to evaluate goal achievement. When an outcome is met, you know that the patient is making progress toward goal achievement. The time frame of 4 days in the first option is not appropriate because this time frame exceeds the time frame stated in the goal. Congestion indicates fluid in the lungs, and a respiratory rate of 30 breaths per minute is elevated/abnormal. This indicates that the patient is still probably experiencing shortness of breath and secretions in the lungs. 18. A nurse is evaluating an expected outcome for a patient that states heart rate will be less than 80 beats/min by 12/3. Which finding will alert the nurse that the goal has been met? ANS: A Heart rate 78 beats/min on 12/3 indicates the goal has been met. Comparing expected and actual findings allows you to interpret and judge a patient’s condition and whether predicted changes have occurred. Expected outcome states less than 80, not 80. The date is by 12/3, not 12/4. 19. A nurse is modifying a patient’s care plan after evaluation of patient care. In which order, starting with the first step, will the nurse perform the tasks? 1. Revise nursing diagnosis. 2. Reassess blood pressure reading. 3. Retake blood pressure after medication. 4. Administer new blood pressure medication. 5. Change goal to blood pressure less than 140/90. ANS: B If a nursing diagnosis is unresolved or if you determine that a new problem has perhaps developed, reassessment is necessary. A complete reassessment of patient factors relating to an existing nursing diagnosis and etiology is necessary when modifying a plan. After reassessment, determine which nursing diagnoses are a. No sputum or cough present in 4 days b. Congestion throughout all lung fields in 2 days c. Shallow, fast respirations 30 breaths per minute in 1 day d. Lungs clear to auscultation following use of inhaler a. Heart rate 78 beats/min on 12/3 b. Heart rate 78 beats/min on 12/4 c. Heart rate 80 beats/min on 12/3 d. Heart rate 80 beats/min on 12/4 a. 1, 5, 2, 4, 3 b. 2, 1, 5, 4, 3 c. 4, 3, 1, 5, 2 d. 5, 4, 5, 1, 2 107 A&E I Comprehensive Testbank accurate for the situation; revise as needed. When revising a care plan, review the goals and expected outcomes for necessary changes after the diagnosis. Then evaluate and revise interventions as needed. MULTIPLE RESPONSE 1. A nurse is caring for a group of patients. Which evaluative measures will the nurse use to determine a patient’s responses to nursing care? (Select all that apply.) ANS: A, B, C, E You examine the results of care by using evaluative measures, which are assessment skills and techniques (e.g., observations, physiological measurements, use of measurement scales, and patient interview). Examples of evaluative measures include assessment of wound healing and respiratory status, blood pressure measurement, and assessment of patient feelings. You conduct evaluative measures to determine if your patients met expected outcomes, not if nursing interventions were completed. 2. Which nursing actions will the nurse perform in the evaluation phase of the nursing process? (Select all that apply.) ANS: B, D, E The expected outcomes established during planning are the standards against which you judge whether goals have been met and if care is successful. You evaluate whether the results of care match the expected outcomes and goals set for a patient. Documentation and reporting are important parts of evaluation because it is crucial to share information about a patient’s progress and current status. Using self-reflection and correcting errors are indicators a nurse is performing evaluation. Setting priorities is part of planning, and ambulating with a patient in the hallway is an intervention, so it is included in the implementation step of the nursing process. Chapter 37: The Experience of Loss, Death, and Grief Potter et al.: Fundamentals of Nursing, 9th Edition MULTIPLE CHOICE 1. A nurse encounters a family who experienced the death of their adult child last year. The parents are talking about the upcoming anniversary of their child’s death. The nurse spends time with them discussing their child’s life and death. Which nursing principle does the nurse’s action best demonstrate? ANS: A Anniversary reactions can reopen grief processes. A nurse should openly acknowledge the loss and talk about the common renewal of grief feeling around the anniversary of the individual’s death; this facilitates normal a. Observations of wound healing b. Daily blood pressure measurements c. Findings of respiratory rate and depth d. Completion of nursing interventions e. Patient’s subjective report of feelings about a new diagnosis of cancer a. Set priorities for patient care. b. Determine whether outcomes or standards are met. c. Ambulate patient 25 feet in the hallway. d. Document results of goal achievement. e. Use self-reflection and correct errors. a. Facilitation of normal mourning b. Pain-management technique c. Grief evaluation d. Palliative care 108 A&E I Comprehensive Testbank mourning. The nurse is not attempting to alleviate a physical pain. The actions are of open communication, not evaluation. Palliative care refers to comfort measures for symptom relief. 2. A cancer patient asks the nurse what the criteria are for hospice care. Which information should the nurse share with the patient? ANS: D Patients accepted into a hospice program usually have less than 6 months to live. Patients with a terminal illness are not eligible until that point. Palliative care provides assistance with pain management when a patient is not eligible for hospice care. An advance directive can be completed by any person, even those who are healthy. 3. A terminally ill patient is experiencing constipation secondary to pain medication. Which is the best method for the nurse to improve the patient’s constipation problem? ANS: D Opioid medication is known to slow peristalsis, which places the patient at high risk for constipation. Laxatives are indicated for opioid-induced constipation. Massaging the patient’s abdomen may cause further discomfort. Discontinuing pain medication is inappropriate for a terminally ill patient. Enema administration twice a day is not the best step in the treatment of opioid-induced constipation. 4. A severely depressed patient cannot state any positive attributes to life. The nurse patiently sits with this patient and assists the patient to identify several activities the patient is actually looking forward to in life. Which spiritual concept is the nurse trying to promote? ANS: C Hope gives a person the ability to see life as enduring or having meaning or purpose. The nurse’s actions do not address time management, reminiscence, or faith. Time management is organizing and prioritizing activities to be completed in a timely manner. Reminiscence is the relationship by mentally or verbally anecdotally relieving and remembering the person and past experiences. Faith is belief in a higher power. 5. In preparation for the eventual death of a female hospice patient of the Muslim faith, the nurse organizes a meeting of all hospice caregivers. A plan of care to be followed when this patient dies is prepared. Which information will be included in the plan? a. It is for those needing assistance with pain management. b. It is for those having a terminal illness, such as cancer. c. It is for those with completion of an advance directive. d. It is for those expected to live less than 6 months. a. Contact the health care provider to discontinue pain medication. b. Administer enemas twice daily for 7 days. c. Massage the patient’s abdomen. d. Use a laxative. a. Time management b. Reminiscence c. Hope d. Faith a. Prepare the body for autopsy. b. Prepare the body for cremation. c. Allow male Muslims to care for the body after death has occurred. 109 A&E I Comprehensive Testbank ANS: D Muslims of the same gender prepare the body for burial. Muslim faith discourages cremation and autopsy to preserve the sanctity of the soul of the deceased and promote burial as soon as possible after death. 6. Family members gather in the emergency department after learning that a family member was involved in a motor vehicle accident. After learning of the family member’s unexpected death, the surviving family members begin to cry and scream in despair. Which phase does the nurse determine the family is in according to the Attachment Theory? ANS: C Yearning and searching characterize the second bereavement phase in the Attachment Theory. Emotional outbursts of tearful sobbing and acute distress are common in this phase. During the numbing phase, the family is protected from the full impact of the loss. During disorganization and despair, the reason why the loss occurred is constantly examined. Reorganization is the last stage of the Attachment Theory in which the person accepts the change and builds new relationships. 7. A nursing assistive personnel (NAP) is caring for a dying patient. Which action by the NAP will cause the nurse to intervene? ANS: B Patients should never be forced to eat so the nurse will intervene to correct this inappropriate behavior. Eating in the last days of life often causes the patient pain and discomfort. Equally, as the body is shutting down the nutrients in food are not able to be absorbed. Therefore, forcing patients to eat serves no beneficial purpose for the patient. All the rest are correct and do not need the nurse to intervene. Elevating the head of the bed is appropriate and will promote ease of breathing and lung expansion and facilitate postural drainage. Giving mouth care will protect membranes if dehydrated, nauseated, or vomiting. Keeping skin clean, dry, and moisturized will decrease skin discomfort and prevent skin breakdown. 8. An Orthodox Jewish rabbi has been pronounced dead. The nursing assistive personnel respectfully ask family members to leave the room and go home as postmortem care is provided. Which statement from the supervising nurse is best? ANS: B Jewish culture calls for family members or religious officials to stay with the body until the time of burial. A male provider is unnecessary. Requesting or expecting the family to go home is not providing culturally d. Allow female Muslims to care for the body after death has occurred. a. Numbing b. Reorganization c. Yearning and searching d. Disorganization and despair a. Elevating head of bed b. Making the patient eat c. Giving mouth care every 2 to 4 hours d. Keeping skin clean, dry, and moisturized a. “I should have called a male colleague to handle the body.” b. “Family members stay with the body until burial the next day.” c. “I wish they would go home because we have work to do here.” d. “Family will quietly leave after praying and touching the rabbi’s head.” 110 A&E I Comprehensive Testbank sensitive care. Hindus and Muslims usually have persons of the same gender handle the body after death. Buddhists often say prayers while touching and standing at the head of the deceased. 9. A palliative team is caring for a dying patient in severe pain. Which action is the priority? ANS: D The primary goal of palliative care is to help patients and families achieve the best quality of life. Providing support for the patient’s nurse is not the primary obligation when the patient is experiencing severe pain. Not all collaborative team members on the palliative team would be able to provide postmortem care, as is the case for nutritionists, social workers, and pharmacists. Teaching about stages of grief should not be the focus when severe pain is present. 10. A veteran is hospitalized after surgical amputation of both lower extremities owing to injuries sustained during military service. Which type of loss will the nurse focus the plan of care on for this patient? ANS: B Loss of a body part from injury is a situational loss. Maturational losses occur as part of normal life transitions across the life span. A perceived loss is uniquely defined by the person experiencing the loss and is less obvious to other people. Uncomplicated loss is not a type of loss; it is a description of normal grief. 11. “I know it seems strange, but I feel guilty being pregnant after the death of my son last year,” said a woman during her routine obstetrical examination. The nurse spends extra time with this woman, helping her realize bonding with this unborn child will not mean she is replacing the one who died. Which nursing technique does this demonstrate? ANS: C The nurse facilitates mourning in family members who are still surviving. By acknowledging the pregnant woman’s emotions, the nurse helps the mother bond with her fetus and recognizes the emotions that still exist for the deceased child. The nurse is not attempting to help the patient eradicate grief, which would be unrealistic. Curative therapy (curing a disease) and spiritual promotion (belief in a higher power or in the meaning of life) are not addressed by the nurse’s statement. 12. A patient has had two family members die during the past 2 days. Which coping strategy is most appropriate for the nurse to suggest to the patient? a. Provide postmortem care for the patient. b. Support the patient’s nurse in grieving. c. Teach the patient the stages of grief. d. Enhance the patient’s quality of life. a. Perceived loss b. Situational loss c. Maturational loss d. Uncomplicated loss a. Providing curative therapy b. Promoting spirituality c. Facilitating mourning d. Eradicating grief a. Writing in a journal b. Drinking alcohol to go to sleep c. Exercising vigorously rather than sleeping 111 A&E I Comprehensive Testbank ANS: A Coping strategies may be healthy and effective like talking, journaling, and sharing emotions with others. They may also be unhealthy and ineffective like increased use of alcohol, drugs, and violence. Although exercise is important for self-care, sleep is also important. Shutting oneself away from friends and family by not talking about the sadness is not effective; the patient should spend time with people who are nurturing. 13. A female nurse is called into the supervisor’s office regarding her deteriorating work performance since the loss of her spouse 2 years ago. The woman begins sobbing and says that she is “falling apart” at home as well. Which type of grief is the female nurse experiencing? ANS: C In complicated grief, a person has a prolonged or significantly difficult time moving forward after a loss. Normal grief is the most common reaction to death; it involves a complex range of normal coping strategies. Disenfranchised grief involves a relationship that is not socially sanctioned. Perceived grief is not a type of grief; perceived loss is a loss that is less obvious to other people. 14. A nurse is caring for a patient in the last stages of dying. Which finding indicates the nurse needs to prepare the family for death? ANS: D Altered breathing such as Cheyne-Stokes pattern, apnea, labored, or irregular breathing is a sign of impending death. Cyanotic, pallor, or mottling of skin occurs. Urine is decreased and a dark color. Decreased muscle tone, relaxed jaw muscles, and sagging mouth also occur. 15. The mother of a child who died recently keeps the child’s room intact. Family members are encouraging her to redecorate and move forward in life. Which type of grief will the home health nurse recognize the mother is experiencing? ANS: A Family members will grieve differently. One sign of normal grief is keeping the deceased individual’s room intact as a way to keep that person alive in the minds of survivors. This is happening after the family member is deceased, so it is not end-of-life grief. It is not abnormal or complicated grief; the child died recently. 16. A nurse is caring for a dying patient. One of the nurse’s goals is to promote dignity and validation of the dying person’s life. Which action will the nurse take to best achieve this goal? d. Avoiding talking with friends and family members a. Normal grief b. Perceived grief c. Complicated grief d. Disenfranchised grief a. Redness of skin b. Clear-colored urine c. Tense muscles tone d. Cheyne-Stokes breathing a. Normal b. End-of-life c. Abnormal d. Complicated a. Take pictures of visitors. 112 A&E I Comprehensive Testbank ANS: D Listening to family members’ stories validates the importance of the dying individual’s life and reinforces the dignity of the person’s life. Taking pictures of visitors does not address the value of a person’s life. Calling organ donation and providing private visiting time are components of the dying process, but they do not validate a dying person’s life. 17. A nurse is caring for a dying patient. When is the best time for the nurse to discuss end-of-life care? ANS: A Because most deaths are now “negotiated” among patients, family members, and the health care team, discuss end-of-life care preferences early in the assessment phase of the nursing process. Doing so during the planning, implementation, and evaluation phases is too late. 18. A nurse is providing postmortem care. Which action will the nurse take? ANS: A Leave dentures in the mouth to maintain facial shape. Raise the head of the bed as soon as possible after death to prevent discoloration of the face. Cover the body with a clean sheet. Autopsy often does not allow removal of tubes, equipment, and indwelling lines. 19. A nurse lets the transplant coordinator make a request for organ and tissue donation from the patient’s family. What is the primary rationale for the nurse’s action? ANS: D In accordance with federal law, a specially trained professional (e.g., transplant coordinator or social worker) makes requests for organ and tissue donation at the time of every death. Given the complex and sensitive nature of such requests, only specially trained personnel make the requests. Although the nurse may be less knowledgeable than the coordinator, uncomfortable asking the family, or not wanting to upset the family, the primary rationale is to be in accordance with federal law. 20. A patient cancels a scheduled appointment because the patient will be attending a Shivah for a family member. Which response by the nurse is best? b. Provide quiet visiting time. c. Call the organ donation coordinator. d. Listen to family stories about the person. a. During assessment b. During planning c. During implementation d. During evaluation a. Leave dentures in the mouth. b. Lower the head of the bed. c. Cover the body with a sterile sheet. d. Remove all tubes for an autopsy. a. The nurse is not as knowledgeable as the coordinator. b. The nurse is uncomfortable asking the family. c. The nurse does not want to upset the family. d. The nurse is following a federal law. 113 A&E I Comprehensive Testbank ANS: D A death has occurred and saying that you are sorry for their loss is appropriate. The Jewish mourning ritual of Shivah is a time period when normal life activities come to a stop. Those mourning welcome friends into the home as a way of honoring the dead and receive support during the mourning period. Cultural variables can influence a person’s response to grief. It is not when families come together for end-of-life decisions. It is not because the appointment fell on the Sabbath. It is not about missionary outreach. 21. During a follow-up visit, a female patient is describing new onset of marital discord with her terminally ill spouse to the hospice nurse. Which Kübler-Ross stage of dying is the patient experiencing? ANS: B Kübler-Ross’s traditional theory involves five stages of dying. The anger stage of adjustment to an impending death can involve resistance, anger at God, anger at people, and anger at the situation. Denial would involve failure to accept a death. Bargaining is an action to delay acceptance of death by bartering. Depression would present as withdrawal from others. 22. A previously toilet trained toddler has started wetting again. A nurse is gathering a health history from the grandparent. Which health history finding will the nurse most likely consider as the cause of the wetting? ANS: B A child’s stage of development and chronological age will influence grieving. Toddlers can show grief from a loss of parent(s) through changes in their eating patterns, changes in their sleeping patterns, fussiness, and changes in their bowel and bladder habits. It is common for younger children to regress when under increased stress. Siblings being sick, dietary changes, and playmates moving away are unlikely to cause wetting. 23. A patient’s father died a week ago. Both the patient and the patient’s spouse talk about the death. The patient’s spouse is experiencing headaches and fatigue. The patient is having trouble sleeping, has no appetite, and gets choked up most of the time. How should the nurse interpret these findings as the basis for a follow-up assessment? ANS: D a. “When families come together for end-of-life decisions, it provides connections.” b. “We will reschedule so the appointment does not fall on the Sabbath.” c. “Missionary outreach is so important for spiritual comfort.” d. “I’m so sorry for your loss.” a. Denial b. Anger c. Bargaining d. Depression a. Dietary changes b. Recent parental death c. Playmate moved away d. Sibling was sick 2 days a. The patient is dying and the spouse is angry. b. The patient is ill and the spouse is malingering. c. Both the patient and the spouse are likely in denial. d. Both the patient and the spouse are likely grieving. 114 A&E I Comprehensive Testbank Both are likely grieving from the loss of the patient’s father. Symptoms of normal grief include headache, fatigue, insomnia, appetite disturbance, and choking sensation. Different people manifest different symptoms. There is no data to support the spouse is angry or malingering. There is no data to support the patient is dying or ill. Denial is assessed when the person cannot accept the loss; both talked about the loss. MULTIPLE RESPONSE 1. A nurse is documenting end-of-life care. Which information will the nurse include in the patient’s electronic medical record? (Select all that apply.) ANS: B, D, E Documentation of end-of-life care includes the following: time and date of death, location of body identification tags, time of body transfer and destination and personal articles left on and secured to the body. Reason for the death is not appropriate; this is a medical judgment and not a nursing judgment. How ethically the family grieved is judgmental and does not belong in the chart. We must remain open to the varying views and beliefs of grieving that are in contrast to our own in order to best support and care for our patients and their families. a. Reason for the death b. Time and date of death c. How ethically the family grieved d. Location of body identification tags e. Time of body transfer and destination 115 A&E I Comprehensive Testbank Week 3 Safety and Fall Prevention among Older Adults, Preventing Complications of Immobility Chapter 27: Patient Safety and Quality Potter et al.: Fundamentals of Nursing, 9th Edition MULTIPLE CHOICE 1. A home health nurse is performing a home assessment for safety. Which comment by the patient will cause the nurse to follow up? ANS: D Using a nonvented heater introduces carbon monoxide into the environment and decreases the available oxygen for human consumption and the nurse should follow up to correct this behavior. Checking the chimney and heater, changing the batteries on the detector, and following up on symptoms such as dizziness, nausea, and fatigue are all statements that are safe and appropriate and need no follow-up. 2. The nurse is caring for an older-adult patient admitted with nausea, vomiting, and diarrhea due to food poisoning. The nurse completes the health history. Which priority concern will require collaboration with social services to address the patient’s health care needs? ANS: A Electricity is needed for refrigeration of food, and lack of electricity could have contributed to the nausea, vomiting, and diarrhea due to food poisoning. This discussion about the patient’s electrical needs can be referred to social services. Foods that are inadequately prepared or stored or subject to unsanitary conditions increase the patient’s risk for infections and food poisoning, and an assessment should include storage practices. The water supply, the increased number of individuals in the home, and not having a microwave may or may not be concerns but do not pertain to the current health care needs of this patient. 3. The patient has been diagnosed with a respiratory illness and reports shortness of breath. The nurse adjusts the temperature to facilitate the comfort of the patient. At which temperature range will the nurse set the thermostat? ANS: B a. “Every December is the time to change batteries on the carbon monoxide detector.” b. “I will schedule an appointment with a chimney inspector next week.” c. “If I feel dizzy when using the heater, I need to have it inspected.” d. “When it is cold outside in the winter, I will use a nonvented furnace.” a. The electricity was turned off 3 days ago. b. The water comes from the county water supply. c. A son and family recently moved into the home. d. This home is not furnished with a microwave oven. a. 60° to 64° F b. 65° to 75° F c. 15° to 17° C d. 25° to 28° C 116 A&E I Comprehensive Testbank A person’s comfort zone is usually between 18.3° and 23.9° C (65° and 75° F). The other ranges are too low or too high and do not reflect the average person’s comfort zone. 4. A homeless adult patient presents to the emergency department. The nurse obtains the following vital signs: temperature 94.8° F, blood pressure 106/56, apical pulse 58, and respiratory rate 12. Which vital sign should the nurse address immediately? ANS: B The temperature indicates the patient is experiencing hypothermia. Homeless individuals are more at risk for hypothermia. While all the vital signs are low, the most critical vital sign at this time is the temperature. 5. A nurse is teaching the patient and family about wound care. Which technique will the nurse teach to best prevent transmission of pathogens? ANS: A One of the most effective methods for limiting the transmission of pathogens is the medically aseptic practice of hand hygiene. The most common means of transmission of pathogens is by the hands. While washing the wound is needed, the best method to prevent transmission is hand hygiene. Wearing gloves and possibly eye protection help protect the nurse, but handwashing is best for limiting the transmission of pathogens. 6. The nurse is monitoring for Never Events. Which finding indicates the nurse will report a Never Event? ANS: B The Centers for Medicare and Medicaid Services names select serious reportable events as Never Events (i.e., adverse events that should never occur in a health care setting). A surgical sponge left in a patient’s incision is a Never Event. No blood incompatibility reaction is safe practice. Pulmonary embolism after certain orthopedic procedures is like a total knee and hip replacement. Stage III and IV pressure ulcers are Never Events. 7. The nurse discovers a patient on the floor. The patient states that he fell out of bed. The nurse assesses the patient and places the patient back in bed. Which action should the nurse take next? ANS: B a. Respiratory rate b. Temperature c. Apical pulse d. Blood pressure a. Wash hands b. Wash wound c. Wear gloves d. Wear eye protection a. No blood incompatibility occurs with a blood transfusion. b. A surgical sponge is left in the patient’s incision. c. Pulmonary embolism after lung surgery d. Stage II pressure ulcer a. Do nothing, no harm has occurred. b. Notify the health care provider. c. Complete an incident report. d. Assess the patient. 117 A&E I Comprehensive Testbank Report immediately to physician or health care provider if the patient sustains a fall or an injury. The nurse must provide safe care, and doing nothing is not safe care. The scenario indicates the nurse has already assessed the patient. After the patient has stabilized, completing an incident report would be the last step in the process. 8. When making rounds the nurse observes a purple wristband on a patient’s wrist. How will the nurse interpret this finding? ANS: B In 2008 the American Hospital Association issued an advisory recommending that hospitals standardize wristband colors: red for patient allergies, yellow for fall risk, and purple for do not resuscitate preferences. Purple does not indicate seizures. 9. A nurse reviews the history of a newly admitted patient. Which finding will alert the nurse that the patient is at risk for falls? ANS: D Numerous factors increase the risk of falls, including a history of falling, being age 65 or over, reduced vision, orthostatic hypotension, lower extremity weakness, gait and balance problems, urinary incontinence, improper use of walking aids, and the effects of various medications (e.g., anticonvulsants, hypnotics, sedatives, certain analgesics). 10. The nurse is assessing a patient for lead poisoning. Which patient is the nurse most likely assessing? ANS: B The incidence of lead poisoning is highest in late infancy and toddlerhood. Children at this stage explore the environment and, because of their increased level of oral activity, put objects in their mouths. Young infant is too young. A preschooler and an adolescent are too old. 11. A nurse is teaching a community group of school-aged parents about safety. Which safety item is mostimportant for the nurse to include in the teaching session? ANS: A a. The patient is allergic to certain medications or foods. b. The patient has do not resuscitate preferences. c. The patient has a high risk for falls. d. The patient is at risk for seizures. a. 55 years old b. 20/20 vision c. Urinary continence d. Orthostatic hypotension a. Young infant b. Toddler c. Preschooler d. Adolescent a. Proper fit of a bicycle helmet b. Proper fit of soccer shin guards c. Proper fit of swimming goggles d. Proper fit of baseball sliding shorts 118 A&E I Comprehensive Testbank Head injuries are a major cause of death, with bicycle accidents being one of the major causes of such injuries. Proper fit of the helmet helps to decrease head injuries resulting from these bicycle accidents. Goggles, shin guards, and sliding shorts are important sports safety equipment and should fit properly, but they do not protect from this leading cause of death. 12. The nurse is presenting an educational session on safety for parents of adolescents. Which information will the nurse include in the teaching session? ANS: A Increased aggressiveness (psychosocial clue) and blood spots on clothing (environmental clue) may indicate substance abuse. School-age children are often uncoordinated. Seat belts should be used all the time. In fact, teens have the lowest rate of seat belt use. 13. The nurse is discussing about threats to adult safety with a college group. Which statement by a group member indicates understanding of the topic? ANS: A Lifestyle choices frequently affect adult safety. Smoking conveys great risk for pulmonary and cardiovascular disease. It is prudent to secure belongings. When an individual has been determined to be the designated driver, that individual does not consume alcohol, beer, or wine. Sleep is important no matter the age of the individual and is important for rest and integration of learning. 14. The nurse is teaching a group of older adults at an assisted-living facility about age-related physiological changes affecting safety. Which question would be mostimportant for the nurse to ask this group? ANS: A The ability to hear safety alerts and seek shelter is imperative to life safety. Decreased hearing acuity alters the ability to hear emergency vehicle sirens. Natural disasters such as floods, tsunamis, hurricanes, tornadoes, and wildfires are major causes of death and injury. Although age-related changes may cause a decrease in sight that affects reading, and although tasting is impaired and opening jars as arthritis sets in are important to patients and to those caring for them, being able to hear safety alerts is the most important. 15. The nurse is caring for a hospitalized patient. Which behavior alerts the nurse to consider the need for a restraint? a. Increased aggressiveness and blood spots on clothing may indicate substance abuse. b. Increased aggressiveness is an environmental clue that may indicate an adolescent is abusing. c. Adolescents need information about the effects of uncoordination on accidents. d. Adolescents need to be reminded to use seat belts primarily on long trips. a. “Smoking even at parties is not good for my body.” b. “Our campus is safe; we leave our dorms unlocked all the time.” c. “As long as I have only two drinks, I can still be the designated driver.” d. “I am young, so I can work nights and go to school with 2 hours’ sleep.” a. “Are you able to hear the tornado sirens in your area?” b. “Are you able to read your favorite book?” c. “Are you able to taste spices like before?” d. “Are you able to open a jar of pickles?” 119 A&E I Comprehensive Testbank ANS: B Patients who are confused, disoriented, and wander or repeatedly fall or try to remove medical devices (e.g., oxygen equipment, IV lines, or dressings) often require the temporary use of restraints to keep them safe. Restraints can be used to prevent interruption of therapy such as traction, IV infusions, NG tube feeding, or Foley catheterization. Refusing to call for help, although unsafe, is not a reason for restraint. Getting confused at night regarding the time or not sleeping and bothering the staff to ask for items is not a reason for restraint. 16. The nurse is trying to use alternatives rather than restrain a patient. Which finding will cause the nurse to determine the alternative is working? ANS: C Restraint alternatives include more frequent observations, social interaction such as involvement of family during visitation, frequent reorientation, regular exercise, and the introduction of familiar and meaningful stimuli (e.g., involve in hobbies such as knitting or crocheting or looking at family photos) within the environment or folding washcloths. Getting up constantly can be cause for concern. Apologizing is not an alternative to restraints. Getting restless when the sitter leaves indicates the alternative is not working. 17. The nurse is caring for a patient who suddenly becomes confused and tries to remove an intravenous (IV) infusion. Which priority action will the nurse take? ANS: A When a patient becomes suddenly confused, the priority is to assess the patient, to identify the reason for change in behavior, and to try to eliminate the cause. If interventions and alternatives are exhausted, the nurse working with the health care provider may determine the need for restraints. 18. The nurse is monitoring for the four categories of risk that have been identified in the health care environment. Which examples will alert the nurse that these safety risks are occurring? ANS: D a. The patient refuses to call for help to go to the bathroom. b. The patient continues to remove the nasogastric tube. c. The patient gets confused regarding the time at night. d. The patient does not sleep and continues to ask for items. a. The patient continues to get up from the chair at the nurses’ station. b. The patient gets restless when the sitter leaves for lunch. c. The patient folds three washcloths over and over. d. The patient apologizes for being “such a bother.” a. Assess the patient. b. Gather restraint supplies. c. Try alternatives to restraint. d. Call the health care provider for a restraint order. a. Tile floors, cold food, scratchy linen, and noisy alarms b. Dirty floors, hallways blocked, medication room locked, and alarms set c. Carpeted floors, ice machine empty, unlocked supply cabinet, and call light in reach d. Wet floors unmarked, patient pinching fingers in door, failure to use lift for patient, and alarms not functioning properly 120 A&E I Comprehensive Testbank Specific risks to a patient’s safety within the health care environment include falls, patient-inherent accidents, procedure-related accidents, and equipment-related accidents. Wet floors contribute to falls, pinching finger in door is patient inherent, failure to use the lift is procedure related, and an alarm not functioning properly is equipment related. Tile floors and carpeted or dirty floors do not necessarily contribute to falls. Cold food, ice machine empty, and hallways blocked are not patient-inherent issues in the hospital setting but are more of patient satisfaction, infection control, or fire safety issues. Scratchy linen, unlocked supply cabinet, and medication room locked are not procedure-related accidents. These are patient satisfaction issues and control of supply issues and are examples of actually following a procedure correctly. Noisy alarms, call light within reach, and alarms set are not equipment-related accidents but are examples of following a procedure correctly. 19. Which activity will cause the nurse to monitor for equipment-related accidents? ANS: A Accidents that are equipment related result from the malfunction, disrepair, or misuse of equipment or from an electrical hazard. To avoid rapid infusion of IV fluids, all general-use and patient-controlled analgesic pumps need to have free-flow protection devices. Measuring devices used by the nurse to measure urine, computer documentation, and manual dispensing devices can break or malfunction but are not used directly on a patient and are considered procedure-related accidents. 20. A patient is admitted and is placed on fall precautions. The nurse teaches the patient and family about fall precautions. Which action will the nurse take? ANS: D A fall-reduction program includes a fall risk assessment of every patient, conducted on admission and routinely (see hospital policy) until a patient’s discharge. The timing of visitors would not affect falls. All four side rails are considered a restraint and can contribute to falling. Individuals on high risk for fall alerts should be checked frequently, at least every hour. 21. A nurse is inserting a urinary catheter. Which technique will the nurse use to prevent a procedure-related accident? ANS: C The potential for infection is reduced when surgical asepsis is used for sterile dressing changes or any invasive procedure such as insertion of a urinary catheter. Pathogenic and clean asepsis are not types of asepsis. Medical asepsis is not sterile. 22. A nurse is providing care to a patient. Which action indicates the nurse is following the National Patient Safety Goals? a. Uses a patient-controlled analgesic pump b. Uses a computer-based documentation record c. Uses a measuring device that measures urine d. Uses a manual medication-dispensing device a. Check on the patient once a shift. b. Encourage visitors in the early evening. c. Place all four side rails in the “up” position. d. Keep the patient on fall risk until discharge. a. Pathogenic asepsis b. Medical asepsis c. Surgical asepsis d. Clean asepsis 121 A&E I Comprehensive Testbank ANS: C One of the National Patient Safety Goals is to use medicines safely. For example, proper preparation and administration of medications, use of patient and medication bar coding, and “smart” intravenous (IV) pumps reduce medication errors. Identifying patients correctly is a national patient safety goal, and two identifiers are needed, not one. Another goal is to prevent infection; starting an IV should be a sterile technique, not a clean technique. While obtaining vital signs is a component of safe care, it does not meet a national patient safety goal. 23. During the admission assessment, the nurse assesses the patient for fall risk. Which finding will alert the nurse to an increased risk for falls? ANS: B Numerous factors increase the risk of falls, including a history of falling and the effects of various medications such as anticonvulsants, hypnotics, sedatives, and certain analgesics. Being oriented will decrease risk for falls while disorientation will increase the risk of falling. Walking has many benefits, including increasing strength, which would be beneficial in decreasing risk. Becoming widowed would increase stress and may affect concentration but is not a great risk. 24. A patient may need restraints. Which task can the nurse delegate to a nursing assistive personnel? ANS: D The application and routine checking of a restraint can be delegated to nursing assistive personnel. The skill of assessing a patient’s behavior, orientation to the environment, need for restraints, and appropriate use cannot be delegated. A nurse must obtain an order from a health care provider. 25. A patient has an ankle restraint applied. Upon assessment the nurse finds the toes a light blue color. Which action will the nurse take next? ANS: A a. Identifies patient with one identifier before transporting to x-ray department b. Initiates an intravenous (IV) catheter using clean technique on the first try c. Uses medication bar coding when administering medications d. Obtains vital signs to place on a surgical patient’s chart a. The patient is oriented. b. The patient takes a hypnotic. c. The patient walks 2 miles a day. d. The patient recently became widowed. a. Determining the need for restraints b. Assessing the patient’s orientation c. Obtaining an order for a restraint d. Applying the restraint a. Remove the restraint. b. Place a blanket over the feet. c. Immediately do a complete head-to-toe neurologic assessment. d. Take the patient’s blood pressure, pulse, temperature, and respiratory rate. 122 A&E I Comprehensive Testbank If the patient has altered neurovascular status of an extremity such as cyanosis, pallor, and coldness of skin or complains of tingling, pain, or numbness, remove the restraint immediately and notify the health care provider. Light blue is cyanosis, indicating the restraints are too tight, not that the patient is cold and needs a blanket. A complete head-to-toe neurological assessment is not needed at this time. The nurse can take vital signs after the restraint is removed. 26. The emergency department has been notified of a potential bioterrorism attack. Which action by the nurse is priority? ANS: B Manage all patients with suspected or confirmed bioterrorism-related illnesses using standard precautions. For certain diseases, additional precautions may be necessary. The early signs of a bioterrorism-related illness often include nonspecific symptoms (e.g., nausea, vomiting, diarrhea, skin rash, fever, confusion) that may persist for several days before the onset of more severe disease. Limit the transport and movement of patients to movement that is essential for treatment and care. Psychosocial concerns (post-traumatic stress) are important but are not the first priority at this moment. 27. The patient is confused, is trying to get out of bed, and is pulling at the intravenous infusion tubing. Which nursing diagnosis will the nurse add to the care plan? ANS: D The patient’s behaviors support the nursing diagnosis of Risk for injury. The patient is confused, is pulling at the intravenous line, and is trying to climb out of bed. Injury could result if the patient falls out of bed or begins to bleed from a pulled line. Nothing in the scenario indicates that this patient lacks knowledge or is at risk for poisoning. Nothing in the scenario refers to the patient’s home maintenance. 28. A confused patient is restless and continues to try to remove the oxygen cannula and urinary catheter. What is the priority nursing diagnosis and intervention to implement for this patient? ANS: A The priority nursing diagnosis is Risk for injury. This patient could cause harm to self by interrupting the oxygen therapy or by damaging the urethra by pulling the urinary catheter out. Before restraining a patient, it is important to implement and exhaust alternatives to restraint. Alternatives can include more frequent observations. This patient may have deficient knowledge; educating the patient about treatments could be considered as an alternative to restraints. However, the nursing diagnosis of highest priority is risk for injury. This scenario does not indicate that the patient has a disturbed body image or that the patient is at risk for suffocation. a. Monitor for specific symptoms. b. Manage all patients using standard precautions. c. Transport patients quickly and efficiently through the elevators. d. Prepare for post-traumatic stress associated with this bioterrorism attack. a. Impaired home maintenance b. Deficient knowledge c. Risk for poisoning d. Risk for injury a. Risk for injury: Check on patient every 15 minutes. b. Risk for suffocation: Place “Oxygen in Use” sign on door. c. Disturbed body image: Encourage patient to express concerns about body. d. Deficient knowledge: Explain the purpose of oxygen therapy and the urinary catheter. 123 A&E I Comprehensive Testbank 29. The patient applies sequential compression devices after going to the bathroom. The nurse checks the patient’s application of the devices and finds that they have been put on upside down. Which nursing diagnosis will the nurse add to the patient’s plan of care? ANS: B The patient has a knowledge need and requires instruction regarding the device and its purpose and procedure. The nurse will intervene by teaching the patient about the sequential compression device and instructing the patient to call for assistance when getting up to go to the bathroom in the future, so that the nurse may assist with removal and proper reapplication. No data support a risk for falls, impaired physical mobility, or suffocation. 30. The nurse enters the patient’s room and notices a small fire in the headlight above the patient’s bed. In which order will the nurse perform the steps, beginning with the first one? 1. Pull the alarm. 2. Remove the patient. 3. Use the fire extinguisher. 4. Close doors and windows. ANS: A Nurses use the mnemonic RACE to set priorities in case of fire. The steps are as follows: Rescue and remove all patients in immediate danger; Activate the alarm; Confine the fire by closing doors and windows; and Extinguish the fire using an appropriate extinguisher. 31. The nurse is providing information regarding safety and accidental poisoning to a grandparent who will be taking custody of a 1-year-old grandchild. Which comment by the grandparent will cause the nurse to intervene? ANS: D The administration of ipecac syrup or induction of vomiting is no longer recommended for routine home treatment of poisoning. The nurse must intervene to provide additional teaching. All the rest are correct and do not require follow up. The poison control number is 800-222-1222. After a caustic substance such as bleach has been drunk, do not induce vomiting. This can cause further burning and injury as the substance is eliminated. Loss of consciousness associated with poisoning requires calling 911. 32. A home health nurse is assessing a family’s home after the birth of an infant. A toddler also lives in the home. Which finding will cause the nurse to follow up? a. Risk for falls b. Deficient knowledge c. Risk for suffocation d. Impaired physical mobility a. 2, 1, 4, 3 b. 1, 2, 4, 3 c. 1, 2, 3, 4 d. 2, 1, 3, 4 a. “The number for poison control is 800-222-1222.” b. “Never induce vomiting if my grandchild drinks bleach.” c. “I should call 911 if my grandchild loses consciousness.” d. “If my grandchild eats a plant, I should provide syrup of ipecac.” 124 A&E I Comprehensive Testbank ANS: A Plastic grocery bags increase the risk for suffocation. The nurse will follow up with instructions to remove or keep locked or out of reach. All the rest are correct and do not require follow-up. Electrical outlets should be covered to reduce electrical shock. Bumper pads are not used in the crib to prevent suffocation, strangulation, or entrapment. Crib slats should be less than 6 cm apart. 33. Which patient will the nurse see first? ANS: B The nurse will see the patient shaving with an electric razor first as this is an actual problem. Do not use oxygen around electrical equipment or flammable products. A lighter on the bedside table and a shiny, round battery are potential problems, not actual. Plus, it would be hard, almost impossible, for a 1 month old to actually grab the battery when it is out of arm’s reach. A baby should use a pacifier without strings. 34. A home health nurse is teaching a family to prevent electrical shock. Which information will the nurse include in the teaching session? ANS: B A guideline to prevent electrical shock is to disconnect items before cleaning. Do not run wires under carpeting. Grasp the plug, not the cord, when unplugging items. Use electrical tape to secure the cord to the floor, preferably against baseboards. 35. The nurse has placed a yellow armband on a 70-year-old patient. Which observation by the nurse will indicate the patient has an understanding of this action? ANS: B A yellow armband is an alert for high risk of falls. Red nonslip footwear helps to grip the floor and decreases the chance of falling. The communication armband should stay in place and should not be removed, so that all a. Plastic grocery bags are neatly stored under the counter. b. Electric outlets are covered in all rooms. c. No bumper pads are in the crib. d. Crib slats are 5 cm apart. a. A 56-year-old patient with oxygen with a lighter on the bedside table b. A 56-year-old patient with oxygen using an electric razor for grooming c. A 1-month-old infant looking at a shiny, round battery just out of arm’s reach d. A 1-month-old infant with a pacifier that has no string around the baby’s neck a. Run wires under the carpet. b. Disconnect items before cleaning. c. Grasp the cord when unplugging items. d. Use masking tape to secure cords to the floor. a. The patient removes the armband to bathe. b. The patient wears the red nonslip footwear. c. The patient insists on taking a “water” pill in the evening. d. The patient who is allergic to penicillin asks the name of a new medicine. 125 A&E I Comprehensive Testbank members of the interdisciplinary team have the information about the high risk for falls. A red armband indicates an allergy. Give diuretics (“water” pill) in the morning to decrease risk of falls during the night— when most falls occur. 36. An older-adult patient is using a wheelchair to attend a physical therapy session. Which action by the nurse indicates safe transport of the patient? ANS: B A correct action when using a wheelchair is to back wheelchair into an elevator, leading with large rear wheels first. A patient’s buttocks should be well back into the seat. A locked wheelchair should be placed on a patient’s strong or unaffected side. Brakes should be securely locked when a patient is transferring. MULTIPLE RESPONSE 1. A home health nurse is assessing the home for fire safety. Which information from the family will cause the nurse to intervene? (Select all that apply.) ANS: A, C, D Incorrect information will cause the nurse to intervene. Accidental home fires typically result from smoking in bed. Advise families to only purchase newer model space heaters that have all of the current safety features. The PASS method is used for fire extinguishers. All the rest are correct and do not require follow-up. Candles should not be left burning when no one is home. Keep a fire extinguisher in the kitchen, near the furnace, and in the garage. 2. The nurse is caring for an older adult who presents to the clinic after a fall. The nurse reviews fall prevention in the home. Which information will the nurse include in the teaching session? (Select all that apply.) ANS: B, C, E Walking to the mailbox in summer provides exercise when pathways are not icy and slick. Encourage annual vision and hearing examinations. Pathways that are clutter free reduce fall risk. Using a hose to water plants and using tubs without safety strips are all items the patient should avoid to help in the prevention of falls in the home. 3. A patient requires restraints after alternatives are not successful. The nurse is reviewing the orders. Which findings indicate to the nurse the order is legal and appropriate for safe care? (Select all that apply.) a. Positions patient’s buttocks close to the front of wheelchair seat b. Backs wheelchair into elevator, leading with large rear wheels first c. Places locked wheelchair on same side of bed as patient’s weaker side d. Unlocks wheelchair for easy maneuverability when patient is transferring a. Smoking in bed helps me relax and fall asleep. b. We never leave candles burning when we are gone. c. We use the same space heater my grandparents used. d. We use the RACE method when using the fire extinguisher. e. There is a fire extinguisher in the kitchen and garage workshop. a. Water outdoor plants with a nozzle and hose. b. Walk to the mailbox in the summer. c. Encourage yearly eye examinations. d. Use bathtubs without safety strips. e. Keep pathways clutter free. a. Health care provider orders restraints prn (as needed). 126 A&E I Comprehensive Testbank ANS: B, D, E A physician’s/health care provider’s order is required, based on a face-to-face assessment of the patient. The order must be current, state the type and location of restraint, and specify the duration and circumstances under which it will be used. These orders need to be renewed within a specific time frame according to the policy of the agency. In hospital settings each original restraint order and renewal is limited to 8 hours for adults, 2 hours for ages 9 through 17, and 1 hour for children under age 9. Restraints are not to be ordered prn (as needed). 4. The nurse is performing the “Timed Get Up and Go (TUG)” assessment. Which actions will the nurse take? (Select all that apply.) ANS: C, D, F The nurse instructs the patient to walk 10 feet (3 m) as quickly and safely as possible and observes for unsteadiness in the patient’s gait. For accuracy, a patient should have one practice trial that is not included in the score. Patient taking less than 20 seconds to complete TUG is adequate for independent mobility. Score over 30 seconds is dependent and at risk for fall. Counting does not begin after instructions. The patient rises from a straight back chair without using arms for support. 5. The nurse is completing an admission history on a new home health patient. The patient has been experiencing seizures as the result of a recent brain injury. Which interventions should the nurse utilize for this patient and family? (Select all that apply.) ANS: D, E Prolonged or repeated seizures indicate status epilepticus, a medical emergency that requires intensive monitoring and treatment. Family should know what to do. Family should reorient and reassure the patient after consciousness is regained. Never force apart a patient’s clenched teeth. Do not place any objects into patient’s mouth such as fingers, medicine, tongue depressor, or airway when teeth are clenched. Do not lift patient from floor to bed while seizure is in progress. Do not restrain patient; hold limbs loosely if they are flailing. Loosen clothing. b. Health care provider writes the type and location of the restraint. c. Health care provider renews orders for restraints every 24 hours. d. Health care provider performs a face-to-face assessment prior to the order. e. Health care provider specifies the duration and circumstances under which the restraint will be used. a. Ranks a patient as high risk for falls after patients takes 18 seconds to complete b. Teaches patient to rise from straight back chair using arms for support c. Instructs the patient to walk 10 feet as quickly and safely as possible d. Observes for unsteadiness in patient’s gait e. Begins counting after the instructions f. Allows the patient a practice trial a. Demonstrate how to restrain the patient in the event of a seizure. b. Instruct the family to move the patient to a bed during a seizure. c. Teach the family how to insert a tongue depressor during the seizure. d. Discuss with the family steps to take if the seizure does not discontinue. e. Instruct the family to reorient and reassure the patient after consciousness is regained. 127 A&E I Comprehensive Testbank 6. The nurse is assessing a patient who reports a previous fall and is using the SPLATT acronym. Which questions will the nurse ask the patient? (Select all that apply.) ANS: A, B, C, D Assess previous falls; using the acronym SPLATT: Symptoms at time of fall Previous fall Location of fall Activity at time of fall Time of fall Trauma after fall Medical diagnoses and an alert device are not components of SPLATT. 7. The nurse is caring for a group of medical-surgical patients. The unit has been notified of a fire on an adjacent wing of the hospital. The nurse quickly formulates a plan to keep the patients safe. Which actions will the nurse take? (Select all that apply.) ANS: A, B, C, D Closing all doors helps to contain smoke and fire. Noting the evacuation routes and oxygen shut-offs is important in case evacuation is needed. You will move bedridden patients from the scene of a fire by a stretcher, bed, or wheelchair. The nurse cannot wait until the fire department arrives to act. Type C fire extinguishers are used for electrical fires; type B is used for flammable liquids. 8. The nurse is caring for a patient in restraints. Which essential information will the nurse document in the patient’s medical record to provide safe care? (Select all that apply.) ANS: B, C, E, F a. Where did you fall? b. What time did the fall occur? c. What were you doing when you fell? d. What types of injuries occurred after the fall? e. Did you obtain an electronic safety alert device after the fall? f. What are your medical problems that may have caused the fall? a. Close all doors. b. Note evacuation routes. c. Note oxygen shut-offs. d. Move bedridden patients in their bed. e. Wait until the fire department arrives to act. f. Use type B fire extinguishers for electrical fires. a. One family member has gone to lunch. b. Patient is placed in bilateral wrist restraints at 0815. c. Bilateral radial pulses present, 2+, hands warm to touch d. Straps with quick-release buckle attached to bed side rails e. Attempts to distract the patient with television are unsuccessful. f. Released from restraints, active range-of-motion exercises completed 128 A&E I Comprehensive Testbank Proper documentation, including the behaviors that necessitated the application of restraints, the procedure used in restraining, the condition of the body part restrained (e.g., circulation to hand), and the evaluation of the patient response, is essential. Record nursing interventions, including restraint alternatives tried, in nurses’ notes. Record purpose for restraint, type and location of restraint used, time applied and discontinued, times restraint was released, and routine observations (e.g., skin color, pulses, sensation, vital signs, and behavior) in nurses’ notes and flow sheets. Straps are not attached to side rails. Comments about the activities of one family member are not necessarily required in nursing documentation of restraints. Chapter 28: Immobility Potter et al.: Fundamentals of Nursing, 9th Edition MULTIPLE CHOICE 1. A nurse is assessing body alignment. What is the nurse monitoring? ANS: A The terms body alignment and posture are similar and refer to the positioning of the joints, tendons, ligaments, and muscles while standing, sitting, and lying. Body alignment means that the individual’s center of gravity is stable. Body mechanics is a term used to describe the coordinated efforts of the musculoskeletal and nervous systems. Friction is a force that occurs in a direction to oppose movement. Immobility is the inability to move about freely. 2. A nurse is providing range of motion to the shoulder and must perform external rotation. Which action will the nurse take? ANS: D External rotation: With elbow flexed, move arm until thumb is upward and lateral to head. Circumduction: Move arm in full circle (Circumduction is combination of all movements of ball-and-socket joint.) Adduction: Lower arm sideways and across body as far as possible. Hyperextension: Move arm behind body, keeping elbow straight. 3. A nurse is providing passive range of motion (ROM) for a patient with impaired mobility. Which technique will the nurse use for each movement? ANS: D a. The relationship of one body part to another while in different positions b. The coordinated efforts of the musculoskeletal and nervous systems c. The force that occurs in a direction to oppose movement d. The inability to move about freely a. Moves patient’s arm in a full circle b. Moves patient’s arm cross the body as far as possible c. Moves patient’s arm behind body, keeping elbow straight d. Moves patient’s arm until thumb is upward and lateral to head with elbow flexed a. Each movement is repeated 5 times by the patient. b. Each movement is performed until the patient experiences pain. c. Each movement is completed quickly and smoothly by the nurse. d. Each movement is moved just to the point of resistance by the nurse. 129 A&E I Comprehensive Testbank Passive ROM exercises are performed by the nurse. Carry out movements slowly and smoothly, just to the point of resistance; ROM should not cause pain. Never force a joint beyond its capacity. Each movement needs to be repeated 5 times during the session. The patient moves all joints through ROM unassisted in active ROM. 4. A nurse is performing passive range of motion (ROM) and splinting on an at-risk patient. Which finding will indicate goal achievement for the nurse’s action? ANS: D Goal achievement for passive ROM is prevention of joint contractures. Contractures develop in joints not moved periodically through their full ROM. ROM exercises reduce the risk of contractures. Researchers noted that prompt use of splinting with prescribed ROM exercises reduced contractures and improved active range of joint motion in affected lower extremities. Deep breathing and coughing and using an incentive spirometer will help prevent atelectasis. Adequate hydration helps prevent renal calculi and urinary tract infections. Interventions aimed at prevention of pressure ulcers include positioning, skin care, and the use of therapeutic devices to relieve pressure. 5. A nurse is preparing to reposition a patient. Which task can the nurse delegate to the nursing assistive personnel? ANS: B The skill of moving and positioning patients in bed can be delegated to nursing assistive personnel (NAP). The nurse is responsible for assessing the patient’s level of comfort and for any hazards of immobility and assessing circulation. 6. A nurse is preparing to assess a patient for orthostatic hypotension. Which piece of equipment will the nurse obtain to assess for this condition? ANS: C A blood pressure cuff is needed. Orthostatic hypotension is a drop of blood pressure greater than 20 mm Hg in systolic pressure or 10 mm Hg in diastolic pressure and symptoms of dizziness, light-headedness, nausea, tachycardia, pallor, or fainting when the patient changes from the supine to standing position. A thermometer is used to assess for fever. Elastic stockings and sequential compression devices are used to prevent thrombus. 7. The patient has been in bed for several days and needs to be ambulated. Which action will the nurse take first? a. Prevention of atelectasis b. Prevention of renal calculi c. Prevention of pressure ulcers d. Prevention of joint contractures a. Determining the level of comfort b. Changing the patient’s position c. Identifying immobility hazards d. Assessing circulation a. Thermometer b. Elastic stockings c. Blood pressure cuff d. Sequential compression devices a. Maintain a narrow base of support. b. Dangle the patient at the bedside. c. Encourage isometric exercises. 130 A&E I Comprehensive Testbank ANS: B To prevent injury, nurses implement interventions that reduce or eliminate the effects of orthostatic hypotension. Mobilize the patient as soon as the physical condition allows, even if this only involves dangling at the bedside or moving to a chair. A wide base of support increases balance. Isometric exercises (i.e., activities that involve muscle tension without muscle shortening) have no beneficial effect on preventing orthostatic hypotension, but they improve activity tolerance. A high-calcium diet can help with osteoporosis but can be detrimental in an immobile patient. 8. A nurse reviews an immobilized patient’s laboratory results and discovers hypercalcemia. Which condition will the nurse monitor for most closely in this patient? ANS: B Renal calculi are calcium stones that lodge in the renal pelvis or pass through the ureters. Immobilized patients are at risk for calculi because they frequently have hypercalcemia. Hypercalcemia does not lead to hypostatic pneumonia, pressure ulcers, or thrombus formation. Immobility is one cause of hypostatic pneumonia, which is inflammation of the lung from stasis or pooling of secretions. A pressure ulcer is an impairment of the skin that results from prolonged ischemia (decreased blood supply) within tissues. A thrombus is an accumulation of platelets, fibrin, clotting factors, and cellular elements of the blood attached to the interior wall of a vein or artery, which sometimes occludes the lumen of the vessel. 9. A nurse is caring for an immobile patient. Which metabolic alteration will the nurse monitor for in this patient? ANS: D Immobility disrupts normal metabolic functioning: decreasing the metabolic rate, altering the metabolism of carbohydrates, fats, and proteins; causing fluid, electrolyte, and calcium imbalances; and causing gastrointestinal disturbances such as decreased appetite and slowing of peristalsis, leading to constipation. 10. A nurse is preparing a care plan for a patient who is immobile. Which psychosocial aspect will the nurse consider? ANS: D Loss of hope is a psychosocial aspect. Patients with restricted mobility may have some depression. Depression is an affective disorder characterized by exaggerated feelings of sadness, melancholy, dejection, worthlessness, emptiness, and hopelessness out of proportion to reality. All the rest are physiological aspects: bone mass, strength, and weight. d. Suggest a high-calcium diet. a. Hypostatic pneumonia b. Renal calculi c. Pressure ulcers d. Thrombus formation a. Increased appetite b. Increased diarrhea c. Increased metabolic rate d. Altered nutrient metabolism a. Loss of bone mass b. Loss of strength c. Loss of weight d. Loss of hope 131 A&E I Comprehensive Testbank 11. The nurse is preparing to lift a patient. Which action will the nurse take first? ANS: B When lifting, assess the weight you will lift, and determine the assistance you will need. The nurse has to assess before positioning a drawsheet or delegating the task. Manual lifting is the last resort, and it is used when the task at hand does not involve lifting most or all of the patient’s weight; most facilities have a no-lift policy. 12. The nurse is caring for an older-adult patient who has been diagnosed with a stroke. Which intervention will the nurse add to the care plan? ANS: A Nurses should encourage the older-adult patient to perform as many self-care activities as possible, thereby maintaining the highest level of mobility. Sometimes nurses inadvertently contribute to a patient’s immobility by providing unnecessary help with activities such as bathing and transferring. Placing the patient on bed rest without sufficient ambulation leads to loss of mobility and functional decline, resulting in weakness, fatigue, and increased risk for falls. After a stroke or brain attack, a patient likely receives gait training from a physical therapist; speech rehabilitation from a speech therapist; and help from an occupational therapist for ADLs such as dressing, bathing and toileting, or household chores. 13. The nurse is observing the way a patient walks. Which aspect is the nurse assessing? ANS: D Gait describes a particular manner or style of walking. Activity tolerance is the type and amount of exercise or work that a person is able to perform. Body alignment refers to the position of the joints, tendons, ligaments, and muscles while standing, sitting, and lying. Range of motion is the maximum amount of movement available at a joint in one of the three planes of the body: sagittal, frontal, or transverse. 14. A nurse is assessing the body alignment of a standing patient. Which finding will the nurse report as normal? a. Position a drawsheet under the patient. b. Assess weight and determine assistance needs. c. Delegate the task to a nursing assistive personnel. d. Attempt to manually lift the patient alone before asking for assistance. a. Encourage the patient to perform as many self-care activities as possible. b. Provide a complete bed bath to promote patient comfort. c. Coordinate with occupational therapy for gait training. d. Place the patient on bed rest to prevent fatigue. a. Activity tolerance b. Body alignment c. Range of motion d. Gait a. When observed laterally, the spinal curves align in a reversed “S” pattern. b. When observed posteriorly, the hips and shoulders form an “S” pattern. c. The arms should be crossed over the chest or in the lap. d. The feet should be close together with toes pointed out. 132 A&E I Comprehensive Testbank ANS: A When the patient is observed laterally, the head is erect and the spinal curves are aligned in a reversed “S” pattern. When observed posteriorly, the shoulders and hips are straight and parallel. The arms hang comfortably at the sides. The feet are slightly apart to achieve a base of support, and the toes are pointed forward. 15. The nurse is evaluating the body alignment of a patient in the sitting position. Which observation by the nurse will indicate a normal finding? ANS: B Both feet are supported on the floor, and the ankles are comfortably flexed. Body weight is evenly distributed on the buttocks and thighs. A 1- to 2-inch space is maintained between the edge of the seat and the popliteal space on the posterior surface of the knee to ensure that no pressure is placed on the popliteal artery or nerve. The patient’s forearms are supported on the armrest, in the lap, or on a table in front of the chair. 16. The nurse is assessing body alignment for a patient who is immobilized. Which patient position will the nurse use? ANS: B Assess body alignment for a patient who is immobilized or bedridden with the patient in the lateral position, not supine. Remove all positioning supports from the bed except for the pillow under the head, and support the body with an adequate mattress. 17. The nurse is assessing the patient for respiratory complications of immobility. Which action will the nurse take when assessing the respiratory system? ANS: B Auscultate the entire lung region to identify diminished breath sounds, crackles, or wheezes. Perform a respiratory assessment at least every 2 hours for patients with restricted activity. Inspect chest wall movements during the full inspiratory-expiratory cycle. Focus auscultation on the dependent lung fields because pulmonary secretions tend to collect in these lower regions. 18. The nurse is assessing an immobile patient for deep vein thromboses (DVTs). Which action will the nurse take? a. The edge of the seat is in contact with the popliteal space. b. Both feet are supported on the floor with ankles flexed. c. The body weight is directly on the buttocks only. d. The arms hang comfortably at the sides. a. Supine position b. Lateral position c. Lateral position with positioning supports d. Supine position with no pillow under the patient’s head a. Inspect chest wall movements primarily during the expiratory cycle. b. Auscultate the entire lung region to assess lung sounds. c. Focus auscultation on the upper lung fields. d. Assess the patient at least every 4 hours. a. Remove elastic stockings every 4 hours. b. Measure the calf circumference of both legs. c. Lightly rub the lower leg for redness and tenderness. 133 A&E I Comprehensive Testbank ANS: B Measure bilateral calf circumference and record it daily as an assessment for DVT. Unilateral increases in calf circumference are an early indication of thrombosis. Homan’s sign, or calf pain on dorsiflexion of the foot, is no longer a reliable indicator in assessing for DVT, and it is present in other conditions. Remove the patient’s elastic stockings and/or sequential compression devices (SCDs) every 8 hours, and observe the calves for redness, warmth, and tenderness. Instruct the family, patient, and all health care personnel not to massage the area because of the danger of dislodging the thrombus. 19. A nurse is assessing the skin of an immobilized patient. What will the nurse do? ANS: C Consistently use a standardized tool, such as the Braden Scale. This identifies patients with a high risk for impaired skin integrity. Skin assessment can be as often as every hour. Limiting fluids can lead to dehydration, increasing skin breakdown. Observe the skin often during routine care. 20. The nurse is caring for an older-adult patient with a diagnosis of urinary tract infection (UTI). Upon assessment the nurse finds the patient confused and agitated. How will the nurse interpret these assessment findings? ANS: D The primary symptom of compromised older patients with an acute urinary tract infection or fever is confusion. Acute confusion in older adults is not normal; a thorough nursing assessment is the priority. With the diagnosis of urinary tract infection, these are not early signs of dementia and they are not purely psychological. 21. A patient has damage to the cerebellum. Which disorder is most important for the nurse to assess? ANS: A Damage to the cerebellum causes problems with balance, and motor impairment is directly related to the amount of destruction of the motor strip. A stroke can lead to hemiplegia. Direct trauma to the musculoskeletal system results in bruises, contusions, sprains, and fractures. A complete transection of the spinal cord can lead to lower extremity paralysis. 22. Which patient will cause the nurse to select a nursing diagnosis of Impaired physical mobility for a care plan? d. Dorsiflex the foot while assessing for patient discomfort. a. Assess the skin every 4 hours. b. Limit the amount of fluid intake. c. Use a standardized tool such as the Braden Scale. d. Have special times for inspection so as to not interrupt routine care. a. These are normal signs of aging. b. These are early signs of dementia. c. These are purely psychological in origin. d. These are common manifestation with UTIs. a. Imbalance b. Hemiplegia c. Muscle sprain d. Lower extremity paralysis a. A patient who is completely immobile 134 A&E I Comprehensive Testbank ANS: B The diagnosis of Impaired physical mobility applies to the patient who has some limitation but is not completely immobile. The diagnosis of Risk for disuse syndrome applies to the patient who is immobile and at risk for multisystem problems because of inactivity. Beyond these diagnoses, the list of potential diagnoses is extensive because immobility affects multiple body systems. 23. The patient has the nursing diagnosis of Impaired physical mobility related to pain in the left shoulder. Which priority action will the nurse take? ANS: D The diagnosis related to pain requires the nurse to assist the patient with comfort measures so that the patient is then willing and more able to move. Pain must be controlled so the patient will not be reluctant to initiate movement. The diagnosis related to reluctance to initiate movement requires interventions aimed at keeping the patient as mobile as possible and encouraging the patient to perform self-care and ROM. 24. A nurse is developing an individualized plan of care for a patient. Which action is important for the nurse to take? ANS: A The nurse must develop an individualized plan of care for each nursing diagnosis and must set goals that are individualized, realistic, and measurable. The nurse should set realistic expectations for care and should include the patient and family when possible. The goals focus on preventing problems or risks to body alignment and mobility. 25. Which behavior indicates the nurse is using a team approach when caring for a patient who is experiencing alterations in mobility? ANS: C The nurse should collaborate with other health care team members such as physical or occupational therapists when considering mobility needs. For example, physical therapists are a resource for planning ROM or strengthening exercises. Nurses often delegate some interventions to nursing assistive personnel, but assessment of lung sounds is the nurse’s responsibility. Nursing assistive personnel may turn and position b. A patient who is not completely immobile c. A patient at risk for single-system involvement d. A patient who is at risk for multisystem problems a. Encourage the patient to do self-care. b. Keep the patient as mobile as possible. c. Encourage the patient to perform ROM. d. Assist the patient with comfort measures. a. Establish goals that are measurable and realistic. b. Set goals that are a little beyond the capabilities of the patient. c. Use the nurse’s own judgment and not be swayed by family desires. d. Explain that without taking alignment risks, there can be no progress. a. Delegates assessment of lung sounds to nursing assistive personnel b. Becomes solely responsible for modifying activities of daily living c. Consults physical therapy for strengthening exercises in the extremities d. Involves respiratory therapy for altered breathing from severe anxiety levels 135 A&E I Comprehensive Testbank patients, apply elastic stockings, help patients use the incentive spirometer, etc. Occupational therapists are a resource for planning activities of daily living that patients need to modify or relearn. A mental health advanced practice nurse or psychologist should be used for severe anxiety. 26. The patient is being admitted to the neurological unit with a diagnosis of stroke. When will the nurse begin discharge planning? ANS: A Discharge planning begins when a patient enters the health care system. In anticipation of the patient’s discharge from an institution, the nurse makes appropriate referrals or consults a case manager or a discharge planner to ensure that the patient’s needs are met at home. Referrals to home care or outpatient therapy are often needed. Planning the day before discharge, when outpatient therapy is no longer needed, and as soon as the discharge destination is known is too late. 27. Which goal is most appropriate for a patient who has had a total hip replacement? ANS: B “The patient will walk 100 feet using a walker by the time of discharge” is individualized, realistic, and measurable. “Ambulating briskly on a treadmill” is not realistic for this patient. The option that focuses on the nurse, not the patient, is not a measurable goal; this is an intervention. “The patient will ambulate by the time of discharge” is not measurable because it does not specify the distance. Even though we can see that the patient will ambulate, this does not quantify how far. 28. The nurse is working on an orthopedic rehabilitation unit that requires lifting and positioning of patients. Which personal injury will the nurse most likely try to prevent? ANS: C Back injuries are often the direct result of improper lifting and bending. The most common back injury is strain on the lumbar muscle group. While arm, hip, and ankle can occur, they are not as common as back. 29. A nurse is caring for a patient with osteoporosis and lactose intolerance. What will the nurse do? ANS: B a. At the time of admission b. The day before the patient is to be discharged c. When outpatient therapy will no longer be needed d. As soon as the patient’s discharge destination is known a. The patient will ambulate briskly on the treadmill by the time of discharge. b. The patient will walk 100 feet using a walker by the time of discharge. c. The nurse will assist the patient to ambulate in the hall 2 times a day. d. The patient will ambulate by the time of discharge. a. Arm b. Hip c. Back d. Ankle a. Encourage dairy products. b. Monitor intake of vitamin D. c. Increase intake of caffeinated drinks. d. Try to do as much as possible for the patient. 136 A&E I Comprehensive Testbank Encourage patients at risk to be screened for osteoporosis and assess their diets for calcium and vitamin D intake. Patients who have lactose intolerance need dietary teaching about alternative sources of calcium. Caffeine should be decreased. The goal of the patient with osteoporosis is to maintain independence with ADLs. Assistive ambulatory devices, adaptive clothing, and safety bars help the patient maintain independence. 30. A nurse is providing care to a group of patients. Which patient will the nurse see first? ANS: A A patient on prolonged bed rest will be prone to deep vein thrombosis, which can lead to an embolus. An embolus can travel through the circulatory system to the lungs and impair circulation and oxygenation, resulting in tachycardia and shortness of breath. Venous emboli that travel to the lungs are sometimes life threatening. While the patient with a reddened area needs to be turned, a patient with renal calculi needing the restroom, and a patient needing range of motion, these are not as life threatening as the chest pain and dyspnea. 31. The patient is immobilized after undergoing hip replacement surgery. Which finding will alert the nurse to monitor for hemorrhage in this patient? ANS: B Heparin and low-molecular-weight heparin are the most widely used drugs in the prophylaxis of deep vein thrombosis. Because bleeding is a potential side effect of these medications, continually assess the patient for signs of bleeding. Pulmonary secretions that become thick and tenacious are difficult to remove and are a sign of inadequate hydration or developing pneumonia but not of bleeding. SCDs consist of sleeves or stockings made of fabric or plastic that are wrapped around the leg and are secured with Velcro. They decrease venous stasis by increasing venous return through the deep veins of the legs. They do not usually cause bleeding. Elastic stockings also aid in maintaining external pressure on the muscles of the lower extremities and in promoting venous return. They do not usually cause bleeding. 32. The nurse needs to move a patient up in bed using a drawsheet. The nurse has another nurse helping. In which order will the nurses perform the steps, beginning with the first one? 1. Grasp the drawsheet firmly near the patient. 2. Move the patient and drawsheet to the desired position. 3. Position one nurse at each side of the bed. 4. Place the drawsheet under the patient from shoulder to thigh. 5. Place your feet apart with a forward-backward stance. 6. Flex knees and hips and on count of three shift weight from the front to back leg. a. A patient with a hip replacement on prolonged bed rest reporting chest pain and dyspnea b. A bedridden patient who has a reddened area on the buttocks who needs to be turned c. A patient on bed rest who has renal calculi and needs to go to the bathroom d. A patient after knee surgery who needs range of motion exercises a. Thick, tenacious pulmonary secretions b. Low-molecular-weight heparin doses c. SCDs wrapped around the legs d. Elastic stockings (TED hose) a. 1, 4, 5, 6, 3, 2 b. 4, 1, 3, 5, 6, 2 137 A&E I Comprehensive Testbank ANS: C Assisting a patient up in bed with a drawsheet (two or three nurses): (1) Place the patient supine with the head of the bed flat. A nurse stands on each side of the bed. (2) Remove the pillow from under the patient’s head and shoulders and place it at the head of the bed. (3) Turn the patient side to side to place the drawsheet under the patient, extending it from shoulders to thighs. (4) Return the patient to the supine position. (5) Fanfold the drawsheet on both sides, with each nurse grasping firmly near the patient. (6) Nurses place their feet apart with a forward-backward stance. Nurses should flex knees and hips. On the count of three, nurses should shift their weight from front to back leg and move the patient and drawsheet to the desired position in the bed. 33. The nurse is caring for a patient who needs to be placed in the prone position. Which action will the nurse take? ANS: A Placing a pillow under the patient’s abdomen after turning decreases hyperextension of lumbar vertebrae and strain on lower back; breathing may also be enhanced. Head is turned toward one side with a small pillow to reduce flexion or hyperextension of cervical vertebrae. Legs should be supported with pillows to elevate toes and prevent footdrop. Forty-five degrees is the position for Fowler’s position; prone is on the stomach. 34. The nurse is caring for a patient with a spinal cord injury and notices that the patient’s hips have a tendency to rotate externally when the patient is supine. Which device will the nurse use to help prevent injury secondary to this rotation? ANS: C A trochanter roll prevents external rotation of the hips when the patient is in a supine position. Hand rolls maintain the thumb in slight adduction and in opposition to the fingers. Hand-wrist splints are individually molded for the patient to maintain proper alignment of the thumb and the wrist. The trapeze bar is a triangular device that hangs down from a securely fastened overhead bar that is attached to the bedframe. It allows the patient to pull with the upper extremities to raise the trunk off the bed, to assist in transfer from bed to wheelchair, or to perform upper arm exercises. 35. The patient is unable to move self and needs to be pulled up in bed. What will the nurse do to make this procedure safe? ANS: D c. 3, 4, 1, 5, 6, 2 d. 5, 6, 3, 1, 4, 2 a. Place pillow under the patient’s abdomen after turning. b. Turn head toward one side with large, soft pillow. c. Position legs flat against bed. d. Raise head of bed to 45 degrees. a. Hand rolls b. A trapeze bar c. A trochanter roll d. Hand-wrist splints a. Place the pillow under the patient’s head and shoulders. b. Do by self if the bed is in the flat position. c. Place the side rails in the up position. d. Use a friction-reducing device. 138 A&E I Comprehensive Testbank This is not a one-person task. Helping a patient move up in bed without help from other co-workers or without the aid of an assistive device (e.g., friction-reducing pad) is not recommended and is not considered safe for the patient or the nurse. Remove the pillow from under head and shoulders and place it at the head of the bed to prevent striking the patient’s head against the head of the bed. When pulling a patient up in bed, the bed should be flat to gain gravity assistance, and the side rails should be down. 36. The nurse is caring for a patient who is immobile and needs to be turned every 2 hours. The patient has poor lower extremity circulation, and the nurse is concerned about irritation of the patient’s toes. Which device will the nurse use? ANS: B A foot cradle may be used in patients with poor peripheral circulation as a means of reducing pressure on the tips of a patient’s toes. A trochanter roll prevents external rotation of the hips when the patient is in a supine position. Hand rolls maintain the thumb in slight adduction and in opposition to the fingers. The trapeze bar is a triangular device that hangs down from a securely fastened overhead bar that is attached to the bedframe. It allows the patient to pull with the upper extremities to raise the trunk off the bed, to assist in transfer from bed to wheelchair, or to perform upper arm exercises. 37. A nurse delegates a position change to a nursing assistive personnel. The nurse instructs the NAP to place the patient in the lateral position. Which finding by the nurse indicates a correct outcome? ANS: A In the side-lying (or lateral) position the patient rests on the side with the major portion of body weight on the dependent hip and shoulder. Patients in the supine position rest on their backs. Sims’ position is semiprone. The patient in the prone position lies face or chest down on the abdomen. 38. A nurse is evaluating care of an immobilized patient. Which action will the nurse take? ANS: B From your perspective as the nurse, you are to evaluate outcomes and response to nursing care and compare the patient’s actual outcomes with the outcomes selected during planning. Ask if the patient’s expectations (subjective data) of care are being met, and use objective data to determine the success of interventions. Just as it was important to include the patient during the assessment and planning phase of the care plan, it is essential to have the patient’s evaluation of the plan of care, not just the patient’s family and health care team. 39. A nurse is supervising the logrolling of a patient. To which patient is the nurse most likely providing care? a. Hand rolls b. A foot cradle c. A trapeze bar d. A trochanter roll a. Patient is lying on side. b. Patient is lying on back. c. Patient is lying semiprone. d. Patient is lying on abdomen. a. Focus on whether the interdisciplinary team is satisfied with the care. b. Compare the patient’s actual outcomes with the outcomes in the care plan. c. Involve primarily the patient’s family and health care team to determine goal achievement. d. Use objective data solely in determining whether interventions have been successful. 139 A&E I Comprehensive Testbank ANS: A A nurse supervises and aids personnel when there is a health care provider’s order to logroll a patient. Patients who have suffered from spinal cord injury or are recovering from neck, back, or spinal surgery often need to keep the spinal column in straight alignment to prevent further injury. Hypostatic pneumonia, total knee replacement, and Stage IV ulcers do not have to be logrolled. 40. The nurse is providing teaching to an immobilized patient with impaired skin integrity about diet. Which diet will the nurse recommend? ANS: A Because the body needs protein to repair injured tissue and rebuild depleted protein stores, give the immobilized patient a high-protein, high-calorie diet. A high-carbohydrate, low-fat diet is not beneficial for an immobilized patient. Vitamins B and C are needed rather than A and E. Fluid restriction can be detrimental to the immobilized patient; this can lead to dehydration. A bland diet is not necessary for immobilized patients. 41. The nurse is caring for a patient who has had a stroke causing total paralysis of the right side. To help maintain joint function and minimize the disability from contractures, passive ROM will be initiated. When should the nurse begin this therapy? ANS: B Passive ROM exercises should begin as soon as the patient’s ability to move the extremity or joint is lost. The nurse should not wait for the acute phase to end. It may be some time before the patient enters the rehab unit or the patient requests it, and contractures could form by then. 42. The nurse is admitting a patient who has been diagnosed as having had a stroke. The health care provider writes orders for “ROM as needed.” What should the nurse do next? ANS: D Further assessment of the patient is needed to determine what the patient is able to perform. Some patients are able to move some joints actively, whereas the nurse passively moves others. With a weak patient, the nurse may have to support an extremity while the patient performs the movement. In general, exercises need to be as active as health and mobility allow. a. A patient with neck surgery b. A patient with hypostatic pneumonia c. A patient with a total knee replacement d. A patient with a Stage IV pressure ulcer a. High protein, high calorie b. High carbohydrate, low fat c. High vitamin A, high vitamin E d. Fluid restricted, bland a. After the acute phase of the disease has passed b. As soon as the ability to move is lost c. Once the patient enters the rehab unit d. When the patient requests it a. Restrict patient’s mobility as much as possible. b. Realize the patient is unable to move extremities. c. Move all the patient’s extremities. d. Further assess the patient. 140 A&E I Comprehensive Testbank 43. A nurse is assessing pressure points in a patient placed in the Sims’ position. Which areas will the nurse observe? ANS: B In the Sims’ position pressure points include the ileum, humerus, clavicle, knees, and ankles. The lateral position pressure points include the ear, shoulder, anterior iliac spine, and ankles. The prone position pressure points include the chin, elbows, female breasts, hips, knees, and toes. Supine position pressure points include the occipital region of the head, vertebrae, coccyx, elbows, and heels. 44. The patient is admitted to a skilled care unit for rehabilitation after the surgical procedure of fixation of a fractured left hip. The patient’s nursing diagnosis is Impaired physical mobility related to musculoskeletal impairment from surgery and pain with movement. The patient is able to use a walker but needs assistance ambulating and transferring from the bed to the chair. Which nursing intervention is most appropriate for this patient? ANS: B Assist with walking and measure how far the patient walks to quantify progress. The nurse should allow the patient to do as much for self as possible. Therefore, the nurse should observe the patient transferring from the bed to the chair using the walker and should provide assistance as needed. The patient should be encouraged to use adequate pain medication to decrease the effects of pain and to increase mobility. The patient should be instructed on safe transfer and ambulation techniques in an environment with few distractions, not in the cafeteria. 45. The patient has been diagnosed with a spinal cord injury and needs to be repositioned using the logrolling technique. Which technique will the nurse use for logrolling? ANS: A At least three to four people are needed to perform this skill safely. Have the patient cross the arms on the chest to prevent injury to the arms. Move the patient as one unit in a smooth, continuous motion on the count of three. Gently lean the patient as a unit back toward pillows for support. MULTIPLE RESPONSE 1. Upon assessment a nurse discovers that a patient has erythema. Which actions will the nurse take? (Select all that apply.) a. Chin, elbow, hips b. Ileum, clavicle, knees c. Shoulder, anterior iliac spine, ankles d. Occipital region of the head, coccyx, heels a. Obtain assistance and physically transfer the patient to the chair. b. Assist with ambulation and measure how far the patient walks. c. Give pain medication after ambulation so the patient will have a clear mind. d. Bring the patient to the cafeteria for group instruction on ambulation. a. Use at least three people. b. Have the patient reach for the opposite side rail when turning. c. Move the top part of the patient’s torso and then the bottom part. d. Do not use pillows after turning. a. Consult a dietitian. b. Increase fiber in the diet. 141 A&E I Comprehensive Testbank ANS: A, D, E If skin shows areas of erythema and breakdown, increase the frequency of turning and repositioning; place the turning schedule above the patient’s bed; implement other activities per agency skin care policy or protocol (e.g., assess more frequently, consult dietitian, place patient on pressure-relieving mattress). Increased fiber will help constipation. Chest physiotherapy is for respiratory complications. 2. The nurse is caring for a patient with impaired physical mobility. Which potential complications will the nurse monitor for in this patient? (Select all that apply.) ANS: A, C, D Immobility leads to complications such as hypostatic pneumonia. Other possible complications include footdrop and impaired skin integrity. Interruptions in the sleep-wake cycle and social isolation are more common complications than somnolence or increased socialization. 3. The nurse is caring for a patient who has had a recent stroke and is paralyzed on the left side. The patient has no respiratory or cardiac issues but cannot walk. The patient cannot button a shirt and cannot feed self due to being left-handed and becomes frustrated very easily. The patient has been eating very little and has lost 2 lbs. The patient asks the nurse, “How can I go home like this? I’m not getting better.” Which health care team members will the nurse need to consult? (Select all that apply.) ANS: A, B, E, F Physical therapists are a resource for planning ROM or strengthening exercises, and occupational therapists are a resource for planning ADLs that patients need to modify or relearn. Because of the loss of 2 lbs and eating very little, a dietitian will also be helpful. Referral to a mental health advanced practice nurse, a licensed social worker, or a physiologist to assist with coping or other psychosocial issues is also wise. Because the patient exhibits good cardiac and respiratory function, respiratory therapy and cardiac rehabilitation probably are not needed at this time. MATCHING Upon assessment a nurse discovers postural abnormalities on several patients. Match the abnormalities to the findings the nurse observed. c. Place on chest physiotherapy. d. Increase frequency of turning. e. Place on pressure-relieving mattress. a. Footdrop b. Somnolence c. Hypostatic pneumonia d. Impaired skin integrity e. Increased socialization a. Dietitian b. Physical therapist c. Respiratory therapist d. Cardiac rehabilitation therapist e. Occupational therapist f. Psychologist a. Lateral-S- or C-shaped spinal column with vertebral rotation b. Legs curved inward so knees come together as person walks 142 A&E I Comprehensive Testbank 1. Lordosis 2. Kyphosis 3. Scoliosis 4. Genu valgum 5. Genu varum 6. Torticollis 1.ANS:E2.ANS:F3.ANS:A4.ANS:B5.ANS:C6.ANS:D Chapter 39: Activity and Exercise Potter et al.: Fundamentals of Nursing, 9th Edition MULTIPLE CHOICE 1. A nurse observes a patient rising from a chair slowly by pushing on the chair arms. Which type of tension and contraction did the nurse observe? ANS: A This movement causes eccentric tension and isotonic contraction. Eccentric tension helps control the speed and direction of movement. For example, when using an overhead trapeze, the patient slowly lowers himself to the bed. The lowering is controlled when the antagonistic muscles lengthen. By pushing on the chair arms and rising eccentric tension and isotonic contraction occurred. In concentric tension, increased muscle contraction causes muscle shortening, resulting in movement such as when a patient uses an overhead trapeze to pull up in bed. Concentric and eccentric muscle actions are necessary for active movement and therefore are referred to as dynamic or isotonic contraction. Isometric contraction (static contraction) causes an increase in muscle tension or muscle work but no shortening or active movement of the muscle (e.g., instructing the patient to tighten and relax a muscle group, as in quadriceps set exercises or pelvic floor exercises). 2. A nurse notices that a patient has a structural curvature of the spine associated with vertebral rotation. Which condition will the nurse most likely find documented in the patient’s medical record? ANS: A Scoliosis is a structural curvature of the spine associated with vertebral rotation. Osteogenesis imperfecta is an inherited disorder that makes bones porous, short, bowed, and deformed. Osteomalacia is an uncommon metabolic disease characterized by inadequate and delayed mineralization, resulting in compact and spongy bone. Arthritis is an inflammatory joint disease characterized by inflammation or destruction of the synovial membrane and articular cartilage and by systemic signs of inflammation. 3. A nurse is caring for a patient who has some immobility from noninflammatory joint degeneration. The nurse is teaching the patient about this process. Which information will the nurse include in the teaching session? c. One or both legs bent outward at knee d. Inclining of head to affected side e. Exaggeration of anterior convex curve of lumbar spine f. Increased convexity in curvature of thoracic spine a. Eccentric tension and isotonic contraction b. Eccentric tension and isometric contraction c. Concentric tension and isotonic contraction d. Concentric tension and isometric contraction a. Scoliosis b. Arthritis c. Osteomalacia d. Osteogenesis a. This will affect synovial fluid. 143 A&E I Comprehensive Testbank ANS: D Joint degeneration, which can occur with inflammatory and noninflammatory disease, is marked by changes in articular cartilage combined with overgrowth of bone at the articular ends. Degenerative changes commonly affect weight-bearing joints. Synovial fluid is normal in noninflammatory diseases. Inflammatory joint disease (e.g., arthritis) is characterized by inflammation or destruction of the synovial membrane and articular cartilage and by systemic signs of inflammation. 4. The nurse is providing care to a patient who is bedridden. The nurse raises the height of the bed. What is the rationale for the nurse’s action? ANS: C Raising the height of the bed when performing a procedure prevents bending too far at the waist and shifting the base of support. Balance is maintained by proper body alignment and posture through two simple techniques. First, widen the base of support by separating the feet to a comfortable distance. Second, increase balance by bringing the center of gravity closer to the base of support. 5. A nurse is following the no-lift policy when working to prevent personal injury. Which type of personal back injury is the nurse most likely trying to prevent? ANS: C The most common back injury for nurses is strain on the lumbar muscle group, which includes the muscles around the lumbar vertebrae. While cervical, thoracic, and sacral can occur, lumbar is the most common. 6. The nurse is caring for a patient in the emergency department with an injured shoulder. Which type of joint will the nurse assess? ANS: B Synovial joints, or true joints, such as the hinge type at the elbow, are freely movable and the most mobile, numerous, and anatomically complex body joints. Fibrous joints fit closely together and are fixed, permitting little, if any, movement such as the syndesmosis between the tibia and the fibula. Synergistic is a type of muscle, not joint. Cartilaginous joints have little movement but are elastic and use cartilage to unite separate bony surfaces such as the synchondrosis that attaches the ribs to the costal cartilage. b. This will affect the body systemically. c. This involves mostly non–weight-bearing joints. d. This involves overgrowth of bone at the articular ends. a. Narrows the nurse’s base of support. b. Allows the nurse to bring feet closer together. c. Prevents a shift in the nurse’s base of support. d. Shifts the nurse’s center of gravity farther away from the base of support. a. Thoracic b. Cervical c. Lumbar d. Sacral a. Fibrous b. Synovial c. Synergistic d. Cartilaginous 144 A&E I Comprehensive Testbank 7. The nurse is caring for a patient with inner ear problems. Which goal is the priority? ANS: A Within the inner ear are the semicircular canals, three fluid-filled structures that help maintain balance. Proprioception is the awareness of the position of the body and its parts, and proprioceptors are located on nerve endings, not the inner ear. Muscle strength is maintained with activity and exercise. Although body alignment is important, it is not maintained by the inner ear. 8. A nurse is teaching a health promotion class about isotonic exercises. Which types of exercises will the nurse give as examples? ANS: A Examples of isotonic exercises are walking, swimming, dance aerobics, jogging, bicycling, and moving arms and legs with light resistance. Isometric exercises involve tightening or tensing of muscles without moving body parts. Examples include quadriceps set exercises and contraction of the gluteal muscles. Examples of resistive isometric exercises are push-ups and hip lifting, as well as placing a footboard on the foot of the bed for patients to push against with their feet. 9. An adolescent tells the nurse that a health professional said the fibrous tissue that connects bone and cartilage was strained in a sporting accident. On which structure will the nurse focus an assessment? ANS: B Ligaments are white, shiny, and flexible bands of fibrous tissue that bind joints and connect bones and cartilage. Tendons are strong, flexible, and inelastic as they serve to connect muscle to bone. Muscles attach bone to bone. Synergistic muscles contract to accomplish the same movement. Antagonistic muscles cause movement at the joint. 10. A nurse is developing an exercise plan for a middle-aged patient. In which order will the nurse instruct the patient to execute the plan, beginning with the first step? 1. Design the fitness program. 2. Assemble equipment. 3. Assess fitness level. 4. Monitor progress. 5. Get started. a. Maintain balance. b. Maintain proprioception. c. Maintain muscle strength. d. Maintain body alignment. a. Swimming, jogging, and bicycling b. Tightening or tensing of muscles without moving body parts c. Quadriceps set exercises and contraction of the gluteal muscles d. Push-ups, hip lifting, pushing feet against a footboard on the bed a. Tendon b. Ligament c. Synergistic muscle d. Antagonistic muscle a. 5, 1, 3, 2, 4 145 A&E I Comprehensive Testbank ANS: D Five steps to beginning an exercise program are Step 1: Assess fitness level; Step 2: Design the fitness program; Step 3: Assemble equipment; Step 4: Get started; and Step 5: Monitor progress. 11. The nurse gives instructions to a nursing assistive personnel (NAP) regarding exercise for a patient. Which action by the NAP indicates a correct understanding of the directions? ANS: C The NAP notifies the nurse if a patient reports increased fatigue, dizziness, or light-headedness when obtaining preexercise and/or postexercise vital signs. The nurse first must assess the patient’s ability and tolerance to exercise. The nurse also teaches patients and their families how to implement exercise programs. The NAP can prepare patients for exercise (e.g., putting on shoes and clothing, providing hygiene needs, and obtaining preexercise and postexercise vital signs). The NAP can help the patient exercise. 12. The nurse is starting an exercise program in a local community as a health promotion project. Which information will the nurse include in the teaching session? ANS: A The cool-down period follows the exercise routine and usually lasts about 5 to 10 minutes. The purposes of weight training from a health perspective are to develop tone and strength and to simulate and maintain healthy bone. Stretching and flexibility exercises are ideal for warm-up and cool-down periods. The recommended frequency of aerobic exercise is 3 to 5 times per week or every other day for approximately 30 minutes. 13. The patient is eager to begin an exercise program with a 2-mile jog. The nurse instructs the patient to warm up. The patient does not want to waste time with a “warm-up.” Which information will the nurse share with the patient? ANS: B The warm-up activity prepares the body for activity and decreases the potential for injury and should not be omitted. It usually lasts about 5 to 10 minutes and may include stretching, calisthenics, and/or aerobic activity performed at a lower intensity. The warm-up is before the exercise, while the cool-down period is after the b. 1, 2, 3, 5, 4 c. 2, 5, 3, 1, 4 d. 3, 1, 2, 5, 4 a. Determines the patient’s ability to exercise b. Teaches the patient how to do the exercises c. Reports the patient got dizzy after exercising d. Advises the patient to work through the pain a. A cool-down period lasts about 5 to 10 minutes. b. The purpose of weight training is to bulk up muscles. c. Resistance training is appropriate for warm-up and cool-down periods. d. Aerobic exercise should be done 3 to 5 times per week for about 20 minutes. a. The warm-up in this case can be done after the 2-mile jog. b. The warm-up prepares the body and decreases the potential for injury. c. The warm-up allows the body to readjust gradually to baseline functioning. d. The warm-up should be performed with high intensity to prepare for the coming challenge. 146 A&E I Comprehensive Testbank exercise. The cool-down, not the warm-up, allows the body to readjust gradually to baseline functioning and provides an opportunity to combine movement such as stretching with relaxation-enhancing mind-body awareness. The warm-up should not be a high-intensity workout. 14. The nurse is caring for a patient who cannot bear weight but needs to be transferred from the bed to a chair. The nurse decides to use a transportable hydraulic lift. What will the nurse do? ANS: C The nurse should attach the hooks on the strap to the holes in the sling. Short straps hook to top holes of the sling; longer straps hook to the bottom of the sling. The horseshoe-shaped base goes under the side of the bed on the side with the chair. Position the patient and lower slowly into the chair in accordance with manufacturer guidelines to safely guide the patient into the back of the chair as the seat descends; then remove the straps and the mechanical/hydraulic lift. 15. The nurse is preparing to move a patient to a wheelchair. Which action indicates the nurse is following recommendations for safe patient handling? ANS: A Safe patient handling includes mentally reviewing the transfer steps before beginning the procedure to ensure both the patient’s and your safety. Use the patient’s strength when lifting, transferring, or moving when possible. Body mechanics alone do not protect the nurse from injury to the musculoskeletal system when moving, lifting, or transferring patients. After completing the assessment, nurses use an algorithm to guide decisions about safe patient handling. 16. A nurse is working in a facility that follows a comprehensive safe patient-handling program. Which finding will alert the nurse to intervene? ANS: A The nurse will follow up when lifts are not kept in convenient locations. Comprehensive safe patient-handling programs include the following elements: an ergonomics assessment protocol for health care environments, patient assessment criteria, algorithms for patient handling and movement, special equipment kept in convenient locations to help transfer patients, back injury resource nurses, an “after-action review” that allows the health care team to apply knowledge about moving patients safely in different settings, and a no-lift policy. 17. The patient is brought to the emergency department with possible injury to the left shoulder. Which area will the nurse assess to best determine joint mobility? a. Place a horseshoe-shaped base on the opposite side from the chair. b. Remove straps before lowering the patient to the chair. c. Hook longer straps to the bottom of the sling. d. Attach short straps to the bottom of the sling. a. Mentally reviews the transfer steps before beginning b. Uses own strength to transfer the patient c. Focuses solely on body mechanics d. Bases decisions on intuition a. Mechanical lifts are in a locked closet. b. Algorithms for patient handling are available. c. Ergonomic assessment protocols are being followed. d. A no-lift policy is in place with adherence by all staff. a. The patient’s gait b. The patient’s range of motion 147 A&E I Comprehensive Testbank ANS: B Assessing range of motion is one assessment technique used to determine the degree of joint mobility and injury to a joint. Gait is the manner or style of walking. It has little bearing on the shoulder damage. Assessing fine-motor coordination would be beneficial in helping to assess the patient’s ability to perform tasks such as feeding and dressing but would not help in evaluating the shoulder. Ethnic influences would not have a direct bearing on the amount of mobility in the joint. 18. The nurse is evaluating care of a patient for crutches. Which finding indicates a successful outcome? ANS: C When crutches are fitted, the tip of the crutch is 4 to 6 inches anterior to the front of the patient’s shoes, and the length of the crutch is two to three finger widths from the axilla. Position the handgrips so the axillae are not supporting the patient’s body weight. Pressure on the axillae increases risk to underlying nerves, which sometimes results in partial paralysis of the arm. Determine correct position of the handgrips with the patient upright, supporting weight by the handgrips with the elbows slightly flexed at 20 to 25 degrees. 19. The patient reports being tired and weak and lacks energy. Upon assessment, the nurse finds that patient has gained weight, and blood pressure and pulse are elevated after climbing stairs. Which nursing diagnosis will the nurse add to the care plan? ANS: C You consider nursing diagnoses of Activity intolerance or Fatigue in a patient who reports being tired and weak. Further review of assessed defining characteristics (e.g., abnormal heart rate and verbal report of weakness and the assessment findings occurring during the activity of climbing the stairs) leads to the definitive diagnosis (Activity intolerance). There is no data to support ineffective coping or decreased cardiac output. 20. The patient weighs 450 lbs (204.5 kg) and reports shortness of breath with any exertion. The health care provider has recommended beginning an exercise program. The patient states that she can hardly get out of bed and just cannot do anything around the house. Which nursing diagnosis will the nurse add to the care plan? ANS: A c. The patient’s ethnic influences d. The patient’s fine-motor coordination a. The top of the crutch is three to four finger widths from the armpit. b. The elbows are slightly flexed at 30 to 35 degrees when the patient is standing. c. The tip of the crutch is 4 to 6 inches anterior to the front of the patient’s shoes. d. The position of the handgrips allows the axilla to support the patient’s body weight. a. Fatigue b. Ineffective coping c. Activity intolerance d. Decreased cardiac output a. Activity intolerance related to excessive weight b. Impaired physical mobility related to bed rest c. Imbalanced nutrition: less than body requirements d. Impaired gas exchange related to shortness of breath 148 A&E I Comprehensive Testbank In this case, activity intolerance is related to the patient’s excessive weight. The patient is not on bed rest although claims that it is difficult to get out of bed, making this diagnosis inappropriate. Shortness of breath is a symptom, not a cause, of Impaired gas exchange, making this nursing diagnosis ineffective. The patient certainly has an imbalance of nutrition, but it is more than body requirements (obesity). 21. A patient with diabetes mellitus is starting an exercise program. Which types of exercises will the nurse suggest? ANS: B Instruct patients diagnosed with diabetes mellitus to perform low- to moderate-intensity exercises, carry a concentrated form of carbohydrates (sugar packets or hard candy), and wear a medical alert bracelet. Low intensity is not beneficial. Moderate to high and high intensity are not recommended for a beginner exercise program. 22. A patient is admitted with a stroke. The outcome of this disorder is uncertain, but the patient is unable to move the right arm and leg. The nurse starts passive range-of-motion (ROM) exercises. Which finding indicates successful goal achievement? ANS: D When patients cannot participate in active ROM, maintain joint mobility and prevent contractures by implementing passive ROM into the plan of care. Exercise and active ROM can improve muscle strength. ROM is not performed for the heart but for the joints. 23. A nurse is assessing a patient with activity intolerance for possible orthostatic hypotension. Which finding will help confirm orthostatic hypotension? ANS: C Orthostatic hypotension results in a drop of 20 mm Hg systolic or more in blood pressure when rising from sitting position (110/60). 120 to 140 means the blood pressure increased rather than dropped. 126 to 120 is only a six points’ difference. 140 to 130 is only a 10 points’ difference. 24. The nurse is teaching a patient how to use a cane. Which information will the nurse include in the teaching session? a. Low intensity b. Low to moderate intensity c. Moderate to high intensity d. High intensity a. Heart rate decreased. b. Contractures developed. c. Muscle strength improved. d. Joint mobility maintained. a. Blood pressure sitting 120/64; blood pressure 140/70 standing b. Blood pressure sitting 126/64; blood pressure 120/58 standing c. Blood pressure sitting 130/60; blood pressure 110/60 standing d. Blood pressure sitting 140/60; blood pressure 130/54 standing a. Place the cane at the top of the hip bone. b. Place the cane on the stronger side of the body. c. Place the cane in front of the body and then move the good leg. d. Place the cane 10 to 15 inches in front of the body when walking. 149 A&E I Comprehensive Testbank ANS: B Have the patient keep the cane on the stronger side of the body. A person’s cane length is equal to the distance between the greater trochanter and the floor. The cane should be moved first and then the weaker leg. For maximum support when walking, the patient places the cane forward 15 to 25 cm (6 to 10 inches), keeping body weight on both legs. The weaker leg is then moved forward to the cane, so body weight is divided between the cane and the stronger leg. 25. A nurse is assisting the patient to perform isometric exercises. Which action will the nurse take? ANS: C Instruct the patient to stop the activity if pain, fatigue, or discomfort is experienced. Assess for pain, shortness of breath, or a change in vital signs; if present, stop the exercise. Let each patient move at his or her own pace. Assess for joint limitations, and do not force a muscle or a joint during exercise. Teach patient to wear comfortable shoes and clothing for exercise. 26. The nurse is developing a plan of care for a patient diagnosed with activity intolerance. Which strategy will the nurse use to provide the best chance of maintaining patient compliance? ANS: B Keeping a log may increase adherence to an exercise prescription. Recommended frequency of aerobic exercise is 3 to 5 times per week or every other day for approximately 30 minutes. Focusing on the harm of not exercising is usually counterproductive. Instead, the nurse should instruct the patient about the physiological benefits of a regular exercise program. Developing a plan of exercise that the patient may perform at home may improve compliance. 27. The nurse is preparing to transfer an uncooperative patient who does not have upper body strength. Which piece of equipment will be best for the nurses to obtain? ANS: B Using a mechanical lift and full body sling to transfer an uncooperative patient who can bear partial weight or a patient who cannot bear weight and is either uncooperative or does not have upper body strength to move from bed to chair prevents musculoskeletal injuries to health care workers. The nurse should not attempt to move the patient with a drawsheet. The patient does not have upper body strength so an overhead trapeze is not appropriate. A friction-reducing slide sheet that minimizes shearing forces is not as effective as a full body sling. a. Encourage wearing tight shoes. b. Set the pace for the exercise session. c. Stop the exercise if pain is experienced. d. Force muscles or joints to go just beyond resistance. a. Performing 20 minutes of aerobic exercise 7 days a week with 10- minute warm-up and cool-down periods b. Instructing the patient to use an exercise log to record day, time, duration, and responses to exercise activity c. Stressing the harm of not exercising by getting the patient to take responsibility for current health status d. Arranging for the patient to join a gym that takes self-pay rather than insurance a. Drawsheet b. Full body sling c. Overhead trapeze d. Friction-reducing slide sheet 150 A&E I Comprehensive Testbank 28. The nurse is teaching a patient how to sit with crutches. In which order will the nurse present the instructions starting with the first step? 1. Place both crutches in one hand. 2. Grasp arm of chair with free hand. 3. Completely lower self into chair. 4. Transfer weight to crutches and unaffected leg. ANS: B A patient is sitting in a chair with crutches. Both crutches are held in one hand. The patient then transfers weight to the crutches and the unaffected leg. Next, the patient grasps the arm of the chair with the free hand and begins to lower self into chair. Finally, the patient completely lowers self into chair. 29. The nurse is caring for a group of patients. Which patient will the nurse see first? ANS: C The nurse must see the myocardial infarction patient first to stop this type of exercise. It is important to understand the energy expenditure (increased respiratory rate and increased work on the heart) associated with isometric exercises because the exercises are sometimes contraindicated in certain patients’ illnesses (e.g., myocardial infarction or chronic obstructive pulmonary disease). All the rest are appropriate. Stretching exercises are beneficial for patients with chronic obstructive pulmonary disease. Also instruct patients to perform low- to moderate-intensity exercises, carry a concentrated form of carbohydrates (sugar packets or hard candy), and wear a medical alert bracelet. The effect of a Tai Chi exercise program has demonstrated a significant reduction in systolic and diastolic blood pressures. MULTIPLE RESPONSE 1. A nurse is preparing to move a patient who is able to assist. Which principles will the nurse consider when planning for safe patient handling? (Select all that apply.) ANS: B, E, F When a patient is able to assist, remember the following principles: The wider the base of support, the greater the stability of the nurse; the lower the center of gravity, the greater the stability of the nurse; facing the direction of movement prevents abnormal twisting of the spine. The use of assistive equipment and continued a. 4, 1, 2, 3 b. 1, 4, 2, 3 c. 1, 2, 4, 3 d. 4, 2, 1, 3 a. A patient with chronic obstructive pulmonary disease doing stretching exercises b. A patient with diabetes mellitus carrying hard candy while doing exercises c. A patient with a heart attack doing isometric exercises d. A patient with hypertension doing Tai Chi exercises a. Keep the body’s center of gravity high. b. Face the direction of the movement. c. Keep the base of support narrow. d. Use the under-axilla technique. e. Use proper body mechanics. f. Use arms and legs. 151 A&E I Comprehensive Testbank use of proper body mechanics significantly reduces the risk of musculoskeletal injuries. Use arms and legs (not back) because the leg muscles are stronger, larger muscles capable of greater work without injury. The underaxilla technique is physically stressful for nurses and uncomfortable for patients. 2. A nurse is assessing activity tolerance of a patient. Which areas will the nurse assess? (Select all that apply.) ANS: A, B, C, E Physiological, emotional, and developmental factors (age) influence the patient’s activity tolerance. Factors influencing activity tolerance include physiological factors such as skeletal abnormalities, emotional factors such as anxiety/depression, developmental factors such as age and gender, and pregnancy status. Race is not a factor because people of all races are faced with similar factors that affect their activity tolerance. 3. A nurse is working in a facility that uses no-lift policies. Which benefits will the nurse observe in the facility? (Select all that apply.) ANS: A, D Implementing evidence-based interventions and programs (e.g., lift teams) reduces the number of work-related injuries, which improves the health of the nurse and reduces indirect costs to the health care facility (e.g., workers’ compensation and replacing injured workers). Knowing the movements and functions of muscles in maintaining posture and movement and implementing evidence-based knowledge about safe patient handling are essential to protecting the safety of both the patient and the nurse. 4. A nurse writes the following outcomes for a patient who has chronic obstructive pulmonary disease to improve activity level: Diastolic blood pressure will remain below 70 mm Hg with systolic below 130 mm Hg. Resting heart rate will range between 65 and 75. The last goal is that the patient will exercise 3 times a week. Which evaluative findings indicate successful goal achievement? (Select all that apply.) ANS: A, B, E Compare actual outcomes with expected outcomes to determine the patient’s health status and progression. Heart rate of 70 is between 65 and 75. Blood pressure 126/64 meets the goal. Did resistive training 1 time and aerobics 2 times equals exercising 3 times a week. Did stretching and flexibility exercises 2 times is below the 3 times a week. Blood pressure 140/90 is too high and does not meet the goal. COMPLETION a. Skeletal abnormalities b. Emotional factors c. Pregnancy status d. Race e. Age a. Reduced number of work-related injuries b. Increased musculoskeletal accidents c. Reduced safety of patients d. Improved health of nurses e. Increased indirect costs a. Resting heart rate 70 b. Blood pressure 126/64 c. Blood pressure 140/90 d. Reports doing stretching and flexibility exercises 2 times this week e. Reports doing resistive training 1 time and aerobics 2 times this week 152 A&E I Comprehensive Testbank 1. A 55-year-old patient is preparing to start an exercise program. The health care provider wants 60% of maximum target heart rate. Calculate the heart rate that the nurse will add to the care plan as the target heart rate. Record answer as a whole number. _________ maximum heart rate ANS: 99 Teach patients to calculate their maximum heart rate by subtracting their current age in years from 220 and then obtaining their target heart rate by taking 60% to 90% of the maximum, depending on their health care provider’s recommendation. 220 − 55 = 165 × 0.6 = 99. Week 4 Skin and Wound Care Hygiene, Introduction to Pharmacology and Medication Administration Chapter 32: Medication Administration Potter et al.: Fundamentals of Nursing, 9th Edition MULTIPLE CHOICE 1. A nurse is teaching a patient about medications. Which statement from the patient indicates teaching is effective? ANS: B For daily insulin, rotate site within anatomical area. Rotating injections within the same body part (instrasite rotation) provides greater consistency in absorption of medication. Parenteral medication absorption is not affected by the timing of meals. Taking a medication 30 minutes late is within the 60-minute window of the time medications should be taken. Medications are usually stopped based on the provider’s orders except in extenuating circumstances. With some medications, such as antibiotics, it is crucial that the full course of medication is taken to avoid relapse of infection. 2. A nurse is preparing to administer an injection to a patient. Which statement made by the patient is an indication for the nurse to use the Z-track method? ANS: C a. “My parenteral medication must be taken with food.” b. “I will rotate the sites in my left leg when I give my insulin.” c. “Once I start feeling better, I will stop taking my antibiotic.” d. “If I am 30 minutes late taking my medication, I should skip that dose.” a. “I am allergic to many medications.” b. “I’m really afraid that a big needle will hurt.” c. “The last shot like that turned my skin colors.” d. “My legs are too obese for the needle to go through.” 153 A&E I Comprehensive Testbank The Z-track is indicated when the medication being administered has the potential to irritate sensitive tissues. It is recommended that, when administering IM injections, the Z-track method be used to minimize local skin irritation by sealing the medication in muscle tissue. The Z-track method is not meant to reduce discomfort from the procedure. If a patient is allergic to a medication, it should not be administered. If a patient has additional subcutaneous tissue to go through, a needle of a different size may be selected. 3. A 2-year-old child is ordered to have eardrops daily. Which action will the nurse take? ANS: A Children up to 3 years of age should have the auricle pulled down and back, children 3 years of age to adults should have the auricle pulled upward and outward. Solution should be instilled 1 cm (1/2 in) above the opening of the ear canal. The patient should remain in the side-lying position 2 to 3 minutes. If a cotton ball is needed, place it into the outermost part of the ear canal. 4. A patient has an order to receive 0.3 mL of U-500 insulin. Which syringe will the nurse obtain to administer the medication? ANS: D Because there is no syringe currently designed to prepare U-500 insulin, many medication errors result with this kind of insulin. To prevent errors, ensure that the order for U-500 specifies units and volume (e.g., 150 units, 0.3 mL of U-500 insulin), and use tuberculin syringes to draw up the doses. A 3 mL and U-100 can result in inaccurate dosing. A needleless syringe will not be acceptable in this situation. 5. A patient has an order to receive 12.5 mg of hydrochlorothiazide. The nurse has on hand a 25 mg tablet of hydrochlorothiazide. How many tablet(s) will the nurse administer? ANS: A 1/2 tablet will be given. The nurse is careful to perform nursing calculations to ensure proper medication administration. The dose ordered is 12.5. The dose on hand is 25. 12.5/25 = 1/2 tablet. 6. The patient is to receive phenytoin (Dilantin) at 0900. When will be the ideal time for the nurse to schedule a trough level? ANS: B a. Pull the auricle down and back to straighten the ear canal. b. Pull the auricle upward and outward to straighten the ear canal. c. Sit the child up for 2 to 3 minutes after instilling drops in ear canal. d. Sit the child up to insert the cotton ball into the innermost ear canal. a. 3-mL syringe b. U-100 syringe c. Needleless syringe d. Tuberculin syringe a. 1/2 tablet b. 1 tablet c. 1 1/2 tablets d. 2 tablets a. 0800 b. 0830 c. 0900 d. 0930 154 A&E I Comprehensive Testbank Trough levels are generally drawn 30 minutes before the drug is administered. If the medication is administered at 0900, the trough should be drawn at 0830. 7. A patient is receiving vancomycin. Which function is the priority for the nurses to assess? ANS: B A side effect of vancomycin is ototoxicity—hearing. It does not affect vision, heart tones, or bowel sounds. 8. A health care provider orders lorazepam (Ativan) 1 mg orally 2 times a day. The dose available is 0.5 mg per tablet. How many tablet(s) will the nurse administer for each dose? ANS: B The nurse will give 2 tablets. It will take 2 tablets (0.5) to equal 1 mg OR ordered dose (1) over dose on hand (0.5). 1/0.5 = 2 tablets. 9. The nurse is preparing to administer an injection into the deltoid muscle of an adult patient. Which needle size and length will the nurse choose? ANS: C For an intramuscular injection into an adult deltoid muscle, a 25-gauge, 1-inch needle is recommended. An 18- gauge needle is too big. While a 23-gauge needle can be used, a 1/2-inch needle is too small. A 27-gauge, 5/8 -inch needle is used for intradermal. 10. When the nurse administers an IM corticosteroid injection, the nurse aspirates. What is the rationale for the nurse aspirating? ANS: C The purpose of aspiration is to ensure that the needle is in the muscle and not in the vascular system. Blood return upon aspiration indicates improper placement, and the injection should not be given. While a patient can aspirate fluid and food into the lungs, this is not related to the reason for why a nurse pulls back the syringe plunger after inserting the needle (aspirates) before injecting the medication. Reducing discomfort and prolonging absorption time are not reasons for aspirating medications. a. Vision b. Hearing c. Heart tones d. Bowel sounds a. 1 b. 2 c. 3 d. 4 a. 18 gauge × 1 1/2 inch b. 23 gauge × 1/2 inch c. 25 gauge × 1 inch d. 27 gauge × 5/8 inch a. Prevent the patient from choking. b. Increase the force of the injection. c. Ensure proper placement of the needle. d. Reduce the discomfort of the injection. 155 A&E I Comprehensive Testbank 11. The nurse is giving an IM injection. Upon aspiration, the nurse notices blood return in the syringe. What should the nurse do? ANS: B Blood return upon aspiration indicates improper placement, and the injection should not be given. Instead withdraw the needle, dispose of the syringe and needle properly, and prepare the medication again. Administering the medication into a blood vessel could have dangerous adverse effects, and the medication will be absorbed faster than intended owing to increased blood flow. Holding pressure is not an appropriate intervention. Pulling back the needle slightly does not guarantee proper placement of the needle and medication administration. 12. The nurse is planning to administer a tuberculin test with a 27-gauge, -inch needle. At which angle will the nurse insert the needle? ANS: A A 27-gauge, -inch needle is used for intradermal injections such as a tuberculin test, which should be inserted at a 5- to 15-degree angle, just under the dermis of the skin. Placing the needle at 30 degrees, 45 degrees, or 90 degrees will place the medication too deep. 13. The nurse closely monitors an older adult for signs of medication toxicity. Which physiological change is the reason for the nurse’s action? ANS: A The reduced glomerular filtration rate delays excretion, increasing chance for toxicity. In older adults, gastric motility and liver mass decrease. Esophageal stricture is not a physiological change associated with normal aging. 14. A registered nurse interprets that a scribbled medication order reads 25 mg. The nurse administers 25 mg of the medication to a patient and then discovers that the dose was incorrectly interpreted and should have been 15 mg. Who is ultimately responsible for the error? ANS: D a. Administer the injection at a slower rate. b. Withdraw the needle and prepare the injection again. c. Pull the needle back slightly and inject the medication. d. Give the injection and hold pressure over the site for 3 minutes. a. 15 degree b. 30 degree c. 45 degree d. 90 degree a. Reduced glomerular filtration b. Reduced esophageal stricture c. Increased gastric motility d. Increased liver mass a. Health care provider b. Pharmacist c. Hospital d. Nurse 156 A&E I Comprehensive Testbank Ultimately, the person administering the medication is responsible for ensuring that it is correct. The nurse administered the medication, so in this case it is the nurse. Accept full accountability and responsibility for all actions surrounding the administration of medications. Do not assume that a medication that is ordered for a patient is the correct medication or the correct dose. This is the importance of verifying the six rights of medication administration. The ultimate responsibility and accountability are with the nurse, not the health care provider, pharmacist, or hospital. 15. A patient is to receive a proton pump inhibitor through a nasogastric (NG) feeding tube. Which is the most important nursing action to ensure effective absorption? ANS: C If a medication needs to be given on an empty stomach or is not compatible with the feeding (e.g., phenytoin, carbamazepine [Tegretol], warfarin [Coumadin], fluoroquinolones, proton pump inhibitors), hold the feeding for at least 30 minutes before or 30 minutes after medication administration. Thoroughly shaking the medication mixes the medication before administration but does not affect absorption. Flushing the tube after all medications should be 30 to 60 mL of water; 15 to 30 mL of water is used for flushing between medications. Patients with NG tubes should never be positioned supine but instead should be positioned at least to a 30-degree angle to prevent aspiration, provided no contraindication condition is known. 16. A health care provider prescribes aspirin 650 mg every 4 hours PO when febrile. For which patient will this order be appropriate? ANS: D Aspirin is an analgesic, an antipyretic, and an anti-inflammatory medication. The provider wrote the medication to be given for a fever (febrile). Fevers are common in infections. If a child is bleeding, aspirin would be contraindicated; aspirin increases the likelihood of bleeding. Although it can be used for inflammatory problems (sprained ankle) and pain/analgesia (severe headache), this is not how the order was written. 17. A patient is in need of immediate pain relief for a severe headache. Which medication will the nurse administer to be absorbed the quickest? ANS: B IV is the fastest route for absorption owing to the increase in blood flow. The richer the blood supply to the site of administration, the faster a medication is absorbed. Medications administered intravenously enter the bloodstream and act immediately, whereas those given in other routes take time to enter the bloodstream and have an effect. Oral, subcutaneous (SQ), and intramuscular (IM) are others ways to deliver medication but with less blood flow, slowing absorption. a. Thoroughly shake the medication before administering. b. Position patient in the supine position for 30 minutes to 1 hour. c. Hold feeding for at least 30 minutes after medication administration. d. Flush tube with 10 to 15 mL of water, after all medications are administered. a. 7 year old with a bleeding disorder b. 21 year old with a sprained ankle c. 35 year old with a severe headache from hypertension d. 62 year old with a high fever from an infection a. Acetaminophen 650 mg PO b. Hydromorphone 4 mg IV c. Ketorolac 8 mg IM d. Morphine 6 mg SQ 157 A&E I Comprehensive Testbank 18. While preparing medications, the nurse knows one of the drug is an acidic medication. In which area does the nurse anticipate the drug will be absorbed? ANS: A Acidic medications pass through the gastric mucosa rapidly. Medications that are basic are not absorbed before reaching the small intestine. 19. The nurse administers a central nervous system stimulant to a patient. Which assessment finding indicates to the nurse that an idiosyncratic event is occurring? ANS: A An idiosyncratic event is a reaction opposite to what the effects of the medication normally are, or the patient overreacts or underreacts to the medication. Falls asleep is an opposite effect of what a central nervous system stimulant should do. A stimulant should make a patient restless and alert. A pruritic (itch) rash could indicate an allergic reaction. 20. An order is written for phenytoin 500 mg IM q3-4h prn for pain. The nurse recognizes that treatment of pain is not a standard therapeutic indication for this drug. The nurse believes that the health care provider meant to write hydromorphone. What should the nurse do? ANS: A If there is any question about a medication order because it is incomplete, illegible, vague, or not understood, contact the health care provider before administering the medication. The nurse cannot change the order without the prescriber’s consent; this is out of the nurse’s scope of practice. Ultimately, the nurse can be held responsible for administering an incorrect medication. If the prescriber is unwilling to change the order and does not justify the order in a reasonable and evidence-based manner, the nurse may refuse to give the medication and notify the supervisor. 21. A patient needs assistance in eliminating an anesthetic gaseous medication (nitrous oxide). Which action will the nurse take? ANS: A a. Stomach b. Mouth c. Small intestine d. Large intestine a. Falls asleep during daily activities b. Presents with a pruritic rash c. Develops restlessness d. Experiences alertness a. Call the health care provider to clarify the order. b. Give the patient hydromorphone, as it was meant to be written. c. Administer the medication and monitor the patient frequently. d. Refuse to give the medication and notify the nurse supervisor. a. Encourage the patient to cough and deep-breathe. b. Suction the patient’s respiratory secretions. c. Suggest voiding every 2 hours. d. Increase fluid intake. 158 A&E I Comprehensive Testbank Gaseous and volatile medications are excreted through gas exchange (lungs). Deep breathing and coughing will assist in clearing the medication more quickly. It is a gaseous medication and cannot be suctioned out of the lungs. It is not excreted through the kidneys so fluids and voiding will not help. 22. A nurse has withdrawn a narcotic from the medication dispenser and must waste a portion of the medication. What should the nurse do? ANS: A The nurse should follow Nurse Practice Acts and safe narcotic administration guidelines by having a nurse witness the “wasted” medication. The nurse cannot return the wasted medication to the medication dispenser. Wasted portions of medications are not placed in sharps containers. The nurse should not leave the narcotic unattended and call the health care provider to obtain matching dosages; the nurse is expected to obtain the correct dose. 23. A nurse teaches the patient about the prescribed buccal medication. Which statement by the patient indicates teaching by the nurse is successful? ANS: A Buccal medications should be placed in the side of the cheek and allowed to dissolve completely. Buccal medications act with the patient’s saliva and mucosa. The patient should not chew or swallow the medication or take any liquids with it. The patient should rotate sides of the cheek to avoid irritating the mucosal lining. 24. What is the nurse’s priority action to protect a patient from medication error? ANS: A One step to take to prevent medication errors is to read labels at least 3 times before administering the medication. The nurse should address the family’s concerns about medications before administering them. Do not discount their anxieties. The medication administration record should be checked against the patient’s hospital identification band; a room number is not an acceptable identifier. Medications should be given when scheduled, and medications with special assessment indications should be separated. Giving medications at one time can cause the patient to aspirate. 25. The nurse prepares a pain injection for a patient but had to check on another patient and asks a new nurse to give the medication. Which action by the new nurse is best? a. Have another nurse witness the wasted medication. b. Return the wasted medication to the medication dispenser. c. Place the wasted portion of the medication in the sharps container. d. Exit the medication room to call the health care provider to request an order that matches the dosages. a. “I should let the medication dissolve completely.” b. “I will place the medication in the same location.” c. “I can only drink water, not juice, with this medication.” d. “I better chew my medication first for faster distribution.” a. Reading medication labels at least 3 times before administering b. Administering as many of the medications as possible at one time c. Asking anxious family members to leave the room before giving a medication d. Checking the patient’s room number against the medication administration record a. Do not give the medication. b. Administer the medication just this once. 159 A&E I Comprehensive Testbank ANS: A Because the nurse who administers the medication is responsible for any errors related to it, nurses administer only the medications they prepare. You cannot delegate preparation of medication to another person and then administer the medication to the patient. The right medication cannot be verified by the new nurse; do not violate the six rights. Do not administer the medication even one time. Do not administer the medication regardless of the pain rating. Avoiding the issue is not appropriate or safe. 26. A patient is at risk for aspiration. Which nursing action is most appropriate? ANS: B Aspiration occurs when food, fluid, or medication intended for GI administration inadvertently enters the respiratory tract. To minimize aspiration risk, allow the patient, if capable, to self-administer medication. Patients should also hold their own cup to control how quickly they take in fluid. Some patients at risk for aspiration may require thickened liquids; thinning liquids does not decrease aspiration risk. Patients at risk for aspiration should not be given straws because use of a straw decreases the control the patient has over volume intake. Turning the head toward the weaker side helps the medication move down the stronger side of the esophagus. 27. A patient refuses medication. Which is the nurse’s first action? ANS: D The first response is to explore reasons the patient does not want the medication. After the assessment, the nurse can decide what to do next. Educating is important, but it is not the first action. Ultimately, the patient does have the right to refuse the medication; however, the nurse should first try to find out reasons for the refusal and provide education if needed based upon the assessment findings. Hiding medication and deceiving a patient into taking a medication is unethical and violates right to autonomy. 28. A patient who is being discharged today is going home with an inhaler. The patient is to administer 2 puffs of the inhaler twice daily. The inhaler contains 200 puffs. When should the nurse appropriately advise the patient to refill the medication? ANS: A Six weeks will be about the time the inhaler will need to be refilled. The inhaler should last the patient 50 days (2 puffs × 2/twice daily = 4; 200/4 = 50); the nurse should advise the patient to refill the prescription when c. Give the medication for any pain score greater than 8. d. Avoid the issue and pretend to not hear the request. a. Give the patient a straw to control the flow of liquids. b. Have the patient self-administer the medication. c. Thin out liquids so they are easier to swallow. d. Turn the head toward the stronger side. a. Educate the patient about the importance of the medication. b. Discreetly hide the medication in the patient’s favorite gelatin. c. Agree with the patient’s decision and document it in the chart. d. Explore with the patient reasons for not wanting to take the medication. a. 6 weeks from the start of using the inhaler b. As soon as the patient leaves the hospital c. When the inhaler is half empty. d. 50 days after discharge 160 A&E I Comprehensive Testbank there are 7 to 10 days of medication remaining. Refilling it as soon as the patient leaves the hospital or when the inhaler is half empty is too early. If the patient waits 50 days, the patient will run out of medication before it can be refilled. 29. The supervising nurse is watching nurses prepare medications. Which action by one of the nurses will the supervising nurse stop immediately? ANS: D The only insulin that can be given IV is regular. NPH cannot be given IV and must be stopped. All the rest demonstrate correct practice. Insulin is supposed to be rolled, not shaken. Glargine is supposed to be given by itself; it cannot be mixed with another medication. Correction insulin, also known as sliding-scale insulin, provides a dose of insulin based on the patient’s blood glucose level. The term correction insulin is preferred because it indicates that small doses of rapid- or short-acting insulins are needed to correct a patient’s elevated blood sugar. 30. Which patient does the nurse most closely monitor for an unintended synergistic effect? ANS: D The 72 year old seeing four different providers is likely to experience polypharmacy. Polypharmacy places the patient at risk for unintended mixing of medications that potentiate each other. When two medications have a synergistic effect, their combined effect is greater than the effect of the medications when given separately. The child taking too much of a medication by mistake could experience overdose or toxicity. The 50 year old is prescribed two different blood pressure medications for their synergistic effect, but this is a desired, intended event. A patient taking meth and mixing chemicals can be toxic. 31. Which patient using an inhaler would benefit mostfrom using a spacer? ANS: B A spacer is indicated for a patient who has difficulty coordinating the steps, like patients with limited mobility/ coordination. An alert adolescent with a repaired cleft palate would not need a spacer. Hearing impairment may make teaching the patient to use the inhaler difficult, but it does not indicate the need for a spacer. Although a patient with left-sided hemiparesis could have coordination problems, a patient using a dry powder inhaler does not require the use of spacers. 32. The prescriber wrote for a 40-kg child to receive 25 mg of medication 4 times a day. The therapeutic range is 5 to 10 mg/kg/day. What is the nurse’s priority? a. Rolls insulin vial between hands b. Administers a dose of correction insulin c. Draws up glargine (Lantus) in a syringe by itself d. Prepares NPH insulin to be given intravenously (IV) a. The 4 year old who has mistakenly taken a half bottle of vitamins b. The 35 year old who has ingested meth mixed with several household chemicals c. The 50 year old who is prescribed a second blood pressure medication d. The 72 year old who is seeing four different specialists a. A 15 year old with a repaired cleft palate who is alert b. A 25 year old with limited coordination of the extremities c. A 50 year old with hearing impairment who uses a hearing aid d. A 72 year old with left-sided hemiparesis using a dry powder inhaler a. Change the dose to one that is within range. b. Administer the medication because it is within the therapeutic range. 161 A&E I Comprehensive Testbank ANS: D The dosage range is 200 to 400 mg a day (5 × 40 = 200 and 10 × 40 = 400). The prescribed dose is 100 mg/day (4 × 25 = 100), which is below therapeutic range. The nurse should notify the health care provider first and ask for clarification on the order. The dose is not above the therapeutic range and is not at a toxic level. The nurse should never alter an order without the prescriber’s approval and consent. 33. The supervising nurse is observing several different nurses. Which action will cause the supervising nurse to intervene? ANS: C The supervising nurse must intervene with the nurse who is drawing up the NPH insulin first; if regular and intermediate-acting (NPH) insulin is ordered, prepare the regular insulin first to prevent the regular insulin from becoming contaminated with the intermediate-acting insulin. All the other actions are appropriate and do not need follow-up. The CDC no longer recommends aspiration when administering immunizations to reduce discomfort. In some cases, especially with a smaller gauge (22) IV needle, blood return is not aspirated, even if the IV is patent. If the IV site shows no signs of infiltration and IV fluid is infusing without difficulty, proceed with IV push slowly. Oral meds are contraindicated in patients with nasogastric suction. 34. A nurse is caring for a patient who is receiving pain medication through a saline lock. After obtaining a good blood return when the nurse is flushing the patient’s peripheral IV, the patient reports pain. Upon assessment, the nurse notices a red streak that is warm and tender to the touch. What is the nurse’s initial action? ANS: A The patient has phlebitis; the initial nursing action is do not administer the medication. The medication should not be given slowly. A cool or warm compress may be used later depending upon protocol, but it is not the first action. 35. The nurse is preparing to administer medications to two patients with the same last name. After the administration, the nurse realizes that did not check the identification of the patient before administering medication. Which action should the nurse complete first? c. Notify the health care provider that the prescribed dose is in the toxic range. d. Notify the health care provider that the prescribed dose is below the therapeutic range. a. A nurse administers a vaccine without aspirating. b. A nurse gives an IV medication through a 22-gauge IV needle without blood return. c. A nurse draws up the NPH insulin first when mixing a short-acting and intermediate-acting insulin. d. A nurse calls the health care provider for a patient with nasogastric suction and orders for oral meds. a. Do not administer the pain medication. b. Administer the pain medication slowly. c. Apply a warm compress to the site. d. Apply a cool compress to the site. a. Return to the room to check and assess the patient. b. Administer the antidote to the patient immediately. c. Alert the charge nurse that a medication error has occurred. 162 A&E I Comprehensive Testbank ANS: A When an error occurs, the patient’s safety and well-being are the top priorities. You first assess and examine the patient’s condition and notify the health care provider of the incident as soon as possible. The nurse’s first priority is to establish the safety of the patient by assessing the patient. Second, notify the charge nurse and the health care provider. Administer antidote if required. Finally, the nurse needs to complete proper documentation. 36. The nurse is caring for two patients with the same last name. In this situation which right of medication administration is the priority to reduce the chance of an error? ANS: B The nurse should ask the patient to verify identity and should check the patient’s ID bracelet against the medication record to ensure right patient. Acceptable patient identifiers include the patient’s name, an identification number assigned by a health care facility, or a telephone number. Do not use the patient’s room number as an identifier. To identify a patient correctly in an acute care setting, compare the patient identifiers on the MAR with the patient’s identification bracelet while at the patient’s bedside. Right medication, right dose, and right route are equally as important, but in this situation, right patient is the priority (two patients with the same last name). 37. A patient prefers not to take the daily allergy pill this morning because it causes drowsiness throughout the day. Which response by the nurse is best? ANS: B The nurse should use knowledge about the medication to educate the patient about potential response to medications. Then the medication schedule can be altered based on that knowledge. It is the patient’s right to refuse medication; however, the nurse should educate the patient on the importance and effects of the medication. Asking a patient to fit a schedule around a medication is unreasonable and will decrease compliance. The nurse should be supportive and should offer solutions to manage medication effects. 38. A nurse is preparing to administer a medication from a vial. In which order will the nurse perform the steps, starting with the first step? 1. Invert the vial. 2. Fill the syringe with medication. 3. Inject air into the airspace of the vial. 4. Clean with alcohol swab and allow to dry. 5. Pull back on the plunger the amount to be drawn up. d. Complete proper documentation of the medication error in the patient’s chart. a. Right medication b. Right patient c. Right dose d. Right route a. “The physician ordered it; therefore, you must take your medication every morning at the same time whether you’re drowsy or not.” b. “Let’s see if we can change the time you take your pill to 9 PM, so the drowsiness occurs when you would normally be sleeping.” c. “You can skip this medication on days when you need to be awake and alert.” d. “Try to get as much done as you can before you take your pill, so you can sleep in the afternoon.” 163 A&E I Comprehensive Testbank 6. Tap the side of the syringe barrel to remove air bubbles. ANS: C When preparing medication from a vial, the steps are as follows: Firmly and briskly wipe the surface of the rubber seal with an alcohol swab and allow to dry; pull back on the plunger to draw an amount of air into the syringe equal to the volume of medication to be aspirated from the vial; inject air into the airspace of the vial; invert the vial while keeping firm hold on the syringe and plunger; fill the syringe with medication; and tap the side of the syringe barrel carefully to dislodge any air bubbles. 39. A nurse is attempting to administer an oral medication to a child, but the child refuses to take the medication. A parent is in the room. Which statement by the nurse to the parent is best? ANS: B Children often have difficulties taking medication, but it can be less traumatic for the child if the parent administers the medication and the nurse supervises. Another nurse should help restrain a child if needed; the parent acts as a comforter, not a restrainer. Holding down the child is not the best option because it may further upset the child. Never administer an oral medication to a sleeping child. Don’t mix medications into the child’s favorite foods, because the child might start to refuse the food. 40. An older-adult patient needs an IM injection of antibiotic. Which site is best for the nurse to use? ANS: C The ventrogluteal site is the preferred and safest site for all adults, children, and infants. While the vastus lateralis is a large muscle that could be used it is not the preferred and safest. The dorsal gluteal site is a location for a subcutaneous injection, and this patient requires an IM injection. The deltoid is easily accessible, but this muscle is not well developed and is not the preferred site. 41. A nurse is preparing an intravenous IV piggyback infusion. In which order will the nurse perform the steps, starting with the first one? 1. Compare the label of the medication with the medication administration record at the patient’s bedside. 2. Connect the tubing of the piggyback infusion to the appropriate connector on the upper Y-port. 3. Hang the piggyback medication bag above the level of the primary fluid bag. 4. Clean the main IV line port with an antiseptic swab. 5. Connect the infusion tubing to the medication bag. a. 4, 1, 5, 3, 6, 2 b. 1, 4, 5, 3, 2, 6 c. 4, 5, 3, 1, 2, 6 d. 1, 4, 5, 3, 6, 2 a. “Please hold your child’s arms down, so I can give the full dose.” b. “I will prepare the medication for you and observe if you would like to try to administer the medication.” c. “Let’s turn the lights off and give your child a moment to fall asleep before administering the medication.” d. “Since your child loves applesauce, let’s add the medication to it, so your child doesn’t resist.” a. Deltoid b. Dorsal gluteal c. Ventrogluteal d. Vastus lateralis 164 A&E I Comprehensive Testbank 6. Regulate flow. ANS: D When preparing and administering IV piggybacks, use the following steps: Compare the label of medication with the medication administration record at the patient’s bedside; connect the infusion tubing to the medication bag; hang the piggyback medication bag above the level of the primary fluid bag; clean the main IV line port with an antiseptic swab; connect the tubing of the piggyback infusion to the appropriate connector on the upper Y-port; and regulate flow. 42. A nurse is administering oral medications to patients. Which action will the nurse take? ANS: B Placing medications that require preadministration assessment in a separate cup serves as a reminder to check before the medication is given, making it easier for the nurse to withhold medication if necessary. Medications should not be removed from their package until they are in the patient’s room because this makes identification of the pill easier and reduces contamination. When measuring a liquid, the nurse should use the meniscus level to measure; make sure it is at eye level on a hard surface like a countertop. Enteric coated medications should not be crushed. 43. A nurse is performing the three accuracy checks before administering an oral liquid medication to a patient. When will the nurse perform the second accuracy check? ANS: B Before going to the patient’s room, compare the patient’s name and name of medication on the label of prepared drugs with MAR for the second accuracy check. Selecting the correct medication from the stock supply, unit-dose drawer, or automated dispensing system (ADS) is the first check. The third accuracy check is comparing names of medications on labels with MAR at the patient’s bedside. Checking the orders is not one of the three accuracy checks but should be done if there is any confusion about an order. 44. A nurse is preparing to administer an antibiotic medication at 1000 to a patient but gets busy in another room. When should the nurse give the antibiotic medication? a. 5, 2, 1, 4, 3, 6 b. 5, 2, 1, 3, 4, 6 c. 1, 5, 4, 3, 2, 6 d. 1, 5, 3, 4, 2, 6 a. Remove the medication from the wrapper, and place it in a cup labeled with the patient’s information. b. Place all of the patient’s medications in the same cup, except medications with assessments. c. Crush enteric-coated medication, and place it in a medication cup with water. d. Measure liquid medication by bringing liquid medication cup to eye level. a. At the patient’s bedside b. Before going to the patient’s room c. When checking the medication order d. When selecting medication from the unit-dose drawer a. By 1030 b. By 1100 165 A&E I Comprehensive Testbank ANS: A Give time-critical scheduled medications (e.g., antibiotics, anticoagulants, insulin, anticonvulsants, and immunosuppressive agents) at the exact time ordered (within 30 minutes before or after scheduled dose). Give non–time critical scheduled medications within a range of either 1 or 2 hours of scheduled dose. 1100, 1130, and 1200 are too late. 45. The nurse is administering medications to several patients. Which action should the nurse take? ANS: A If the patient uses a corticosteroid, have him or her rinse the mouth out with water or salt water or brush teeth after inhalation to reduce risk of fungal infection. Piercing a blood vessel during a subcutaneous injection is very rare. Therefore, aspiration is not necessary when administering subcutaneous injections. When giving immunizations to adults: to avoid injection into subcutaneous tissue, spread the skin of the selected vaccine administration site taut between the thumb and forefinger, isolating the muscle. Never administer IV medications through tubing that is infusing blood, blood products, or parenteral nutrition solutions. MULTIPLE RESPONSE 1. A nurse is following safety principles to reduce the risk of needlestick injury. Which actions will the nurse take? (Select all that apply.) ANS: C, D, E Needles should not be forced into the box. Clearly mark receptacles to warn of danger. Using needleless systems when possible will further reduce the risk of needlestick injury. To prevent the risk of needlesticks, the nurse should never recap needles. The syringe and sheath are disposed of together in a receptacle. 2. Which methods will the nurse use to administer an intravenous (IV) medication that is incompatible with the patient’s IV fluid? (Select all that apply.) ANS: A, D, E c. By 1130 d. By 1200 a. Advise a patient after a corticosteroid inhaler treatment to rinse mouth with water. b. Administer an intravenous medication through tubing that is infusing blood. c. Pinch up the deltoid muscle of an adult patient receiving a vaccination. d. Aspirate before administering a subcutaneous injection in the abdomen. a. Recap the needle after giving an injection. b. Remove needle and dispose in sharps box. c. Never force needles into the sharps disposal. d. Use clearly marked sharps disposal containers. e. Use needleless devices whenever possible. a. Start another IV site. b. Administer slowly with the IV fluid. c. Do not give the medication and chart. d. Flush with 10 mL of sterile water before and after administration. e. Flush with 10 mL of normal saline before and after administration. 166 A&E I Comprehensive Testbank When IV medication is incompatible with IV fluids, stop the IV fluids, clamp the IV line above the injection site, flush with 10 mL of normal saline or sterile water, give the IV bolus over the appropriate amount of time, flush with another 10 mL of normal saline or sterile water at the same rate as the medication was administered, and restart the IV fluids at the prescribed rate. Do not administer the drug slowly with the IV; this is contraindicated when incompatibility exist. Not giving the medication and charting is inappropriate; this is not a prudent or safe action by the nurse. COMPLETION 1. A patient is taking 1 tablet of hydrocodone bitartrate 5 mg and acetaminophen 500 mg (Vicodin) every 4 hours. The patient is also taking 2 tablets of acetaminophen (Tylenol) 325 mg every 12 hours. How many grams of acetaminophen is the patient taking daily? Record your answer using one decimal place. ______ g ANS: 3.3 The nurse should calculate the dosage taken via the first medication and add it to the daily intake of the second medication. Then, convert milligrams to grams. 500 mg × 6 doses a day = 3000 mg/day + (2 tablets × 325 mg) × 2 doses a day = 1300 mg/day = 3300 mg/day total of acetaminophen; 3300 mg converted to grams = 3.3 grams. 2. The nurse is administering 250 mg of a medication elixir to the patient. The medication comes in a dose of 1000 mg/5 mL. How many milliliters should the nurse administer? Record your answer using two decimal places. ____ mL ANS: 1.25 The nurse needs to first determine how many milligrams are in each milliliter of the elixir. Then the nurse calculates how many milliliters would contain 250 mg. 1000 mg/5 mL = 200 mg/1 mL 250 mg/(X mL) = 200 mg/mL = 1.25 mL. OR Dose ordered over dose on hand (250/1000) × volume or amount on hand (5). 250/1000 = 0.25 × 5 = 1.25 mL. 3. The patient is to receive amoxicillin 500 mg q8h; the medication is dispensed at 250 mg/5 mL. How many milliliters will the nurse administer for one dose? Record your answer using a whole number. ___ mL ANS: 10 The drug is dispensed at 250 mg/5 mL. The nurse is to give 500 mg, which is 10 mL. OR Dose ordered over dose on hand (500/250) × volume or amount on hand (5). 500/250 × 5 = 10 mL. Chapter 40: Hygiene Potter et al.: Fundamentals of Nursing, 9th Edition MULTIPLE CHOICE 1. A nurse is preparing to provide hygiene care. Which principle should the nurse consider when planning hygiene care? ANS: B No two individuals perform hygiene in the same manner; it is important to individualize the patient’s care based on knowing about the patient’s unique hygiene practices and preferences. Hygiene care is never routine; this care requires intimate contact with the patient and communication skills to promote the therapeutic relationship. In addition, during hygiene, the nurse should take time to learn about the patient’s health promotion practices and needs, emotional needs, and health care education needs. a. Hygiene care is always routine and expected. b. No two individuals perform hygiene in the same manner. c. It is important to standardize a patient’s hygienic practices. d. During hygiene care do not take the time to learn about patient needs. 167 A&E I Comprehensive Testbank 2. A patient’s hygiene schedule of bathing and brushing teeth is largely influenced by family customs. For which age group is the nurse most likely providing care? ANS: B Family customs play a major role during childhood in determining hygiene practices such as the frequency of bathing, the time of day bathing is performed, and even whether certain hygiene practices such as brushing of the teeth or flossing are performed. As children enter adolescence, peer groups and media often influence hygiene practices. During the adult years involvement with friends and work groups shape the expectations that people have about personal appearance. Some older adults’ hygiene practices change because of changes in living conditions and available resources. 3. The patient has been diagnosed with diabetes. When admitted, the patient is unkempt and is in need of a bath and foot care. When questioned about hygiene habits, the nurse learns the patient takes a bath once a week and a sponge bath every other day. To provide ultimate care for this patient, which principle should the nurse keep in mind? ANS: C The nurse must assist the patient in developing new hygiene practices when indicated by an illness or condition. For example, the nurse will need to teach a patient with diabetes proper foot hygiene. Patients who appear unkempt often need further assessment regarding their ability to participate in daily hygiene. Patients with certain types of physical limitations or disabilities often lack the physical energy and dexterity to perform hygienic care. Culturally, maintaining cleanliness does not hold the same importance for some ethnic groups as it does for others. 4. The nurse is caring for a patient who refuses to bathe in the morning. When asked why, the patient says “I always bathe in the evening.” Which action by the nurse is best? ANS: A Allow the patient to follow normal hygiene practices; change the bath to evening. Patients have individual preferences about when to perform hygiene and grooming care. Knowing the patient’s personal preferences promotes individualized care for the patient. Hygiene care is never routine. Maintaining individual personal preferences is important unless new hygiene practices are indicated by an illness or condition. Cancelling hygiene and trying again is not an option since the nurse already knows the reason for refusal. Adapting practices to meet individual needs is required. 5. A nurse is completing an assessment of the patient. Which principle is a priority? a. Adolescent b. Preschooler c. Older adult d. Adult a. Patients who appear unkempt place little importance on hygiene practices. b. Personal preferences determine hygiene practices and are unchangeable. c. The patient’s illness may require teaching of new hygiene practices. d. All cultures value cleanliness with the same degree of importance. a. Defer the bath until evening and pass on the information to the next shift. b. Tell the patient that daily morning baths are the “normal” routine. c. Explain the importance of maintaining morning hygiene practices. d. Cancel hygiene for the day and attempt again in the morning. 168 A&E I Comprehensive Testbank ANS: D A patient’s condition is always changing, requiring ongoing critical thinking and changing of nursing diagnoses. Apply the elements of critical thinking as you use the nursing process to meet patients’ hygiene needs. Critical thinking will help you determine when foot care, daily bathing, and hygiene needs are important and when they are not. 6. When providing hygiene for an older-adult patient, the nurse closely assesses the skin. What is the rationale for the nurse’s action? ANS: B In older adults, daily bathing as well as bathing with water that is too hot or soap that is harsh causes the skin to become excessively dry. As the patient ages, the skin thins and loses its resiliency and moisture, and lubricating skin glands become less active, making the skin fragile and prone to bruising and breaking. 7. The nurse is bathing a patient and notices movement in the patient’s hair. Which action will the nurse take? ANS: A In community health and home care settings, it is particularly important to inspect the hair for lice so appropriate hygienic treatment can be provided. If pediculosis capitis (head lice) is suspected, the nurse must protect self against self-infestations by handwashing and by using gloves or tongue blades to inspect the patient’s hair. Suspicions cannot be ignored. Shaving hair off affected areas is the treatment for pediculosis pubis (crab lice) and is rarely used for head lice. Caution against use of products containing lindane because the ingredient is toxic and known to cause adverse reactions. 8. The patient has been brought to the emergency department following a motor vehicle accident. The patient is unresponsive. The driver’s license states that glasses are needed to operate a motor vehicle, but no glasses were brought in with the patient. Which action should the nurse take next? ANS: A An important aspect of an eye examination is to determine if the patient wears contact lenses, especially in patients who are unresponsive. To determine whether a contact lens is present, stand to the side of the patient’s eye and observe the cornea for the presence of a soft or rigid lens. It is also important to observe the sclera to a. Foot care will always be important. b. Daily bathing will always be important. c. Hygiene needs will always be important. d. Critical thinking will always be important. a. Outer skin layer becomes more resilient. b. Less frequent bathing may be required. c. Skin becomes less subject to bruising. d. Sweat glands become more active. a. Use gloves to inspect the hair. b. Apply a lindane-based shampoo immediately. c. Shave the hair off of the patient’s head. d. Ignore the movement and continue. a. Stand to the side of the patient’s eye and observe the cornea. b. Conclude that the glasses were lost during the accident. c. Notify the ambulance personnel for missing glasses. d. Ask the patient where the glasses are. 169 A&E I Comprehensive Testbank detect the presence of a lens that has shifted off the cornea. An undetected lens causes severe corneal injury when left in place too long. Never assume that glasses were lost or were not worn. Contacting ambulance personnel takes time and cannot assume the glasses are missing. Asking the patient where the glasses are is inappropriate since the patient is unresponsive. 9. A nurse is assessing a patient’s skin. Which patient is most at risk for impaired skin integrity? ANS: B Excessive moisture (diaphoretic) on the surface of the skin serves as a medium for bacterial growth and causes irritation, softens epidermal cells, and leads to skin maceration. A patient who is afebrile is not a high risk; however, a patient who is febrile (fever) is prone to skin breakdown. A patient with strong pedal pulses is not a high risk; however, a patient with vascular insufficiency is. A patient with adequate skin turgor is not a high risk; however, a patient with poor skin turgor is. 10. The nurse is caring for a patient who is immobile. The nurse frequently checks the patient for impaired skin integrity. What is the rationale for the nurse’s action? ANS: C Body parts exposed to pressure have reduced circulation to affected tissue. Patients with limited caloric and protein intake develop thinner, less elastic skin with loss of subcutaneous tissue. Inadequate blood flow causes ischemia and breakdown. Verbalization is affected when altered cognition occurs from dementia, psychological disorders, or temporary delirium, not from immobility. 11. The nurse is caring for a patient who has diabetes mellitus and circulatory insufficiency, with peripheral neuropathy and urinary incontinence. On which areas does the nurse focus care? ANS: A Patients with paralysis, circulatory insufficiency, or peripheral neuropathy (nerve damage) are unable to sense an injury to the skin (decreased pain sensation). The presence of urinary incontinence, circulatory insufficiency, and neuropathy can combine to result in breakdown, so the patient has an increased risk of skin impairment. While the patient may have decreased caloric intake, the patient will not have accelerated wound healing with circulatory insufficiency, neuropathy, and incontinence. While the patient is at high risk for skin infection, the low salivary pH level is not an issue. While the patient may have a high risk for impaired venous return from the circulatory insufficiency, there is no indication the patient has dementia. 12. The nurse is caring for a patient who has undergone surgery for a broken leg and has a cast in place. What should the nurse do to prevent skin impairment? a. A patient who is afebrile b. A patient who is diaphoretic c. A patient with strong pedal pulses d. A patient with adequate skin turgor a. Inadequate blood flow leads to decreased tissue ischemia. b. Patients with limited caloric intake develop thicker skin. c. Pressure reduces circulation to affected tissue. d. Verbalization of skin care needs is decreased. a. Decreased pain sensation and increased risk of skin impairment b. Decreased caloric intake and accelerated wound healing c. High risk for skin infection and low saliva pH level d. High risk for impaired venous return and dementia a. Assess surfaces exposed to the edges of the cast for pressure areas. 170 A&E I Comprehensive Testbank ANS: A Assess surfaces exposed to casts, cloth restraints, bandages and dressings, tubing, or orthopedic devices. An external device applied to or around the skin exerts pressure or friction on the skin, leading to skin impairment. When restricted from moving, dependent body parts are exposed to pressure that reduces circulation to affected tissues, promoting pressure ulcers. Patients with limited caloric and protein intake develop impaired or delayed wound healing. Keeping the blood pressure artificially low may decrease arterial blood supply, leading to ischemia and breakdown. 13. Which action by the nurse will be the most important for preventing skin impairment in a mobile patient with local nerve damage? ANS: D During a bath, assess the status of sensory nerve function by checking for touch, pain, heat, cold, and pressure. When restricted from moving freely, dependent body parts are exposed to pressure that reduces circulation. However, this patient is mobile and therefore is able to change positions. Limiting caloric and protein intake may result in impaired or delayed wound healing. A mobile patient can use bathroom facilities or a urinal and does not need a urinary catheter. 14. After performing foot care, the nurse checks the medical record and discovers that the patient has a foot disorder caused by a virus. Which condition did the nurse most likely observe? ANS: C Plantar warts appear on the sole of the foot and are caused by the papillomavirus. Corns are caused by friction and pressure from ill-fitting or loose shoes. Athlete’s foot (tinea pedis) is a fungal infection and can spread to other body parts. A callus is caused by local friction or pressure. 15. The nurse is caring for a patient who is reporting severe foot pain due to corns. The patient has been using oval corn pads to self-treat the corns, but they seem to be getting worse. Which information will the nurse share with the patient? b. Keep the patient’s blood pressure low to prevent overperfusion of tissue. c. Do not allow turning in bed because that may lead to redislocation of the leg. d. Restrict the patient’s dietary intake to reduce the number of times on the bedpan. a. Insert an indwelling urinary catheter. b. Limit caloric and protein intake. c. Turn the patient every 2 hours. d. Assess for pain during a bath. a. Corns b. A callus c. Plantar warts d. Athlete’s foot a. Corn pads are an adequate treatment and should be continued. b. The patient should avoid soaking the feet before using a pumice stone. c. Depending on severity, surgery may be needed to remove the corns. d. Tighter shoes would help to compress the corns and make them smaller. 171 A&E I Comprehensive Testbank ANS: C Surgical removal is necessary, depending on severity of pain and the size of the corn. Oval corn pads should be avoided because they increase pressure on the toes and reduce circulation. Warm water soaks soften corns before gentle rubbing with a callus file or pumice stone. Wider and softer shoes, especially shoes with a wider toe box, are helpful. 16. The patient is diagnosed with athlete’s foot (tinea pedis). The patient says that he is relieved because it is only athlete’s foot, and it can be treated easily. Which information should the nurse consider when formulating a response to the patient? ANS: A Athlete’s foot spreads to other body parts, especially the hands. It is contagious and frequently recurs. Drying feet well after bathing and applying powder help prevent infection. It is caused by a fungus, not lice, and is treated with applications of griseofulvin, miconazole, or tolnaftate. Plantar wars are treated with salicylic acid or electrodesiccation. 17. When assessing a patient’s feet, the nurse notices that the toenails are thick and separated from the nail bed. What does the nurse most likely suspect is the cause of this condition? ANS: A Inflammatory lesions and fungus of the nail bed cause thickened, horny nails that separate from the nail bed. Ask women whether they frequently polish their nails and use polish remover because chemicals in these products cause excessive nail dryness. Friction and pressure from ill-fitting or loose shoes causes keratosis (corns). It is seen mainly on or between toes, over bony prominences. 18. The nurse is providing education about the importance of proper foot care to a patient who has diabetes mellitus. Which primary goal is the nurse trying to achieve? ANS: D Foot ulceration is the most common single precursor to lower extremity amputations among persons with diabetes. Prevention of plantar warts and foot fungus are important but not the primary goal. Neuropathy is a degeneration of the peripheral nerves usually due to poor control of blood glucose levels; it is not a direct result of foot care. 19. The nurse is providing oral care to an unconscious patient and notes that the patient has extremely bad breath. Which term will the nurse use when reporting to the oncoming shift? a. Contagious with frequent recurrences b. Helpful to air-dry feet after bathing c. Treated with salicylic acid d. Caused by lice a. Fungi b. Friction c. Nail polish d. Nail polish remover a. Prevention of plantar warts b. Prevention of foot fungus c. Prevention of neuropathy d. Prevention of amputation a. Cheilitis b. Halitosis 172 A&E I Comprehensive Testbank ANS: B Halitosis is the term for “bad breath.” Cheilitis is the term for cracked lips. Dental caries are cavities in the teeth and could be a cause of the halitosis. Glossitis is the term for inflamed tongue. 20. The nurse is caring for a patient with diabetes. Which task will the nurse assign to the nursing assistive personnel? ANS: C The skill of making an occupied bed can be delegated to nursing assistive personnel. Nail care, teaching foot care, and assessing aspiration risk of a patient with diabetes must be performed by the RN; these skills cannot be delegated. 21. The patient is being treated for cancer with weekly radiation therapy to the head and chemotherapy treatments. Which assessment is the priority? ANS: D The oral cavity is the priority. Radiation to the head reduces salivary flow and lowers pH of saliva, leading to stomatitis and tooth decay, while chemotherapy drugs kill the normal cells lining the oral cavity, leading to ulcers and inflammation. While the feet, nail beds, and perineum are important, they are not as affected as the oral cavity with head or neck radiation and chemotherapy. 22. The nurse is providing oral care to an unconscious patient. Which action should the nurse take? ANS: D When providing oral hygiene to an unconscious patient, the nurse needs to protect him or her from choking and aspiration. Have two nurses provide care; one nurse does the actual cleaning, and the other caregiver removes secretions with suction equipment. The nurse can delegate nursing assistive personnel to participate. Some agencies use equipment that combines a mouth swab with the suction device. This device can be used safely by one nurse to provide oral care. Commercially made foam swabs are ineffective in removing plaque. Do not use lemon-glycerin sponges because they dry mucous membranes and erode tooth enamel. While cleansing the oral cavity, use a small oral airway or a padded tongue blade to hold the mouth open. Never use your fingers to hold the patient’s mouth open. A human bite contains multiple pathogenic microorganisms. 23. The nurse is teaching the patient about flossing and oral hygiene. Which instruction will the nurse include in the teaching session? c. Glossitis d. Dental caries a. Providing nail care b. Teaching foot care c. Making an occupied bed d. Determining aspiration risk a. Feet b. Nail beds c. Perineum d. Oral cavity a. Moisten the mouth using lemon-glycerin sponges. b. Hold the patient’s mouth open with gloved fingers. c. Use foam swabs to help remove plaque. d. Suction the oral cavity. 173 A&E I Comprehensive Testbank ANS: B Dental flossing removes plaque and tartar between teeth. To prevent bleeding, the patient should use unwaxed floss. Flossing once a day is sufficient. If toothpaste is applied to the teeth before flossing, fluoride will come in direct contact with tooth surfaces, aiding in cavity prevention. 24. The nurse is teaching the parents of a child who has head lice (pediculosis capitis). Which information will the nurse include in the teaching session? ANS: C Head lice are difficult to remove and spread to furniture and other people if not treated. Caution against use of products containing lindane because the ingredient is toxic and is known to cause adverse reactions. Treatments use medicated shampoo for eliminating lice. Manual removal is the best option when treatment has failed. 25. A patient has scaling of the scalp. Which term will the nurse use to report this finding to the oncoming staff? ANS: A Dandruff is scaling of the scalp that is accompanied by itching. Pediculosis (lice infestation) resides on scalp attached to hair strands; eggs look like oval particles, similar to dandruff. Alopecia is hair loss or balding. Xerostomia is dry mouth. 26. A nurse is providing a bath. In which order will the nurse clean the body, beginning with the first area? 1. Face 2. Eyes 3. Perineum 4. Arm and chest 5. Hands and nails 6. Back and buttocks 7. Abdomen and legs a. Using waxed floss prevents bleeding. b. Flossing removes plaque and tartar from the teeth. c. Performing flossing at least 3 times a day is beneficial. d. Applying toothpaste to the teeth before flossing is harmful. a. Treatment is use of regular shampoo. b. Products containing lindane are most effective. c. Head lice may spread to furniture and other people. d. Manual removal is not a realistic option as treatment. a. Dandruff b. Alopecia c. Pediculosis d. Xerostomia a. 1, 2, 5, 4, 7, 6, 3 b. 2, 1, 4, 5, 7, 3, 6 c. 2, 1, 5, 4, 6, 7, 3 174 A&E I Comprehensive Testbank ANS: B The sequence for giving a bath is as follows: eyes, face, both arms, chest, hands/nails, abdomen, both legs, perineal hygiene, back, and buttocks/anus. 27. The nurse is caring for a patient who has multiple ticks on lower legs and body. What should the nurse do to rid the patient of ticks? ANS: A Using blunt tweezers, grasp the tick as close to the head as possible and pull upward with even, steady pressure. Hold until the tick pulls out, usually for about 3 to 4 minutes. Save the tick in a plastic bag, and put it in the freezer if necessary to identify the type of tick. Because ticks transmit several diseases to people, they must be removed. Allowing them to drop off by themselves is not an option. Do not burn ticks off with a match or lighter. Miconazole is used to treat athlete’s foot; it is a fungal medication. Covering ticks with plastic does not remove ticks. 28. The nurse is providing oral care to a patient. In which order will the nurse clean the oral cavity, starting with the first area? 1. Roof of mouth, gums, and inside cheek 2. Chewing and inner tooth surfaces 3. Outer tooth surfaces 4. Tongue ANS: C Oral care is provided in the following sequence: Clean chewing and inner tooth surfaces first. Clean outer tooth surfaces. Moisten brush with chlorhexidine rinse to rinse. Use toothette to clean roof of mouth, gums, and inside cheeks. Gently brush tongue but avoid stimulating gag reflex. Rinse. 29. The nurse is caring for an older-adult patient with Alzheimer’s disease who is ambulatory but requires total assistance with activities of daily living (ADLs). The nurse notices that the patient is edentulous. Which area should the nurse assess? ANS: A Edentulous means without teeth; therefore, the nurse needs to assess the oral cavity. While older adults may want the room warmer and drafts should be avoided, this does not help with being edentulous. Edentulous does d. 1, 2, 4, 5, 3, 7, 6 a. Use blunt tweezers and pull upward with steady pressure. b. Burn the ticks with a match or small lighter. c. Allow the ticks to drop off by themselves. d. Apply miconazole and cover with plastic. a. 4, 1, 3, 2 b. 3, 2, 4, 1 c. 2, 3, 1, 4 d. 1, 4, 2, 3 a. Assess oral cavity. b. Assess room for drafts. c. Assess ankles for edema. d. Assess for reduced sensations. 175 A&E I Comprehensive Testbank not mean the patient has edema. While older-adult patients can have reduced sensations, this is not the meaning of edentulous. 30. A self-sufficient bedridden patient is unable to reach all body parts. Which type of bath will the nurse assign to the nursing assistive personnel? ANS: C A partial bath consists of washing body parts that the patient cannot reach, including the back, and providing a backrub. Dependent patients in need of partial hygiene or self-sufficient bedridden patients who are unable to reach all body parts receive a partial bed bath. Complete bed baths are administered to totally dependent patients in bed. The bag bath contains several soft, nonwoven cotton cloths that are premoistened in a solution of no-rinse surfactant cleanser and emollient. The sponge bath involves bathing from a bath basin or a sink with the patient sitting in a chair. 31. The nurse is preparing to provide a complete bed bath to an unconscious patient. The nurse decides to use a bag bath. In which order will the nurse clean the body, starting with the first area? 1. Neck, shoulders, and chest 2. Abdomen and groin/perineum 3. Legs, feet, and web spaces 4. Back of neck, back, and then buttocks 5. Both arms, both hands, web spaces, and axilla ANS: B Use all six chlorhexidene gluconate (CHG) cloths in the following order: 1. Cloth 1: Neck, shoulders, and chest 2. Cloth 2: Both arms, both hands, web spaces, and axilla 3. Cloth 3: Abdomen and then groin/perineum 4. Cloth 4: Right leg, right foot, and web spaces 5. Cloth 5: Left leg, left foot, and web spaces 6. Cloth 6: Back of neck, back, and then buttocks 32. The female nurse is caring for a male patient who is uncircumcised but not ambulatory and has full function of all extremities. The nurse is providing the patient with a partial bed bath. How should perineal care be performed for this patient? a. Bag bath b. Sponge bath c. Partial bed bath d. Complete bed bath a. 5, 1, 2, 3, 4 b. 1, 5, 2, 3, 4 c. 1, 5, 2, 4, 3 d. 5, 1, 2, 4, 3 a. Should be postponed because it may cause embarrassment b. Should be unnecessary because the patient is uncircumcised c. Should be done by the patient d. Should be done by the nurse 176 A&E I Comprehensive Testbank ANS: C If a patient is able to perform perineal self-care, encourage this independence. Patients most in need of perineal care are those at greatest risk for acquiring an infection such as uncircumcised males; perineal care is necessary. Embarrassment should not cause the nurse to overlook the patient’s hygiene needs. The nurse should provide this care only if the patient is unable to do so. 33. A nursing assistive personnel (NAP) is providing AM care to patients. Which action by the NAP will require the nurse to intervene? ANS: D The nurse must intervene if the NAP turns the patient’s head with a neck injury; this is contraindicated and must be stopped to prevent further injury. All the other actions are appropriate and do not need follow-up. Consult the medical record for any contraindications to a massage (e.g., fractured ribs, burns, and heart surgery). Before washing a patient’s hair, determine that there are no contraindications to procedure (e.g., neck injury). When providing a backrub, enhance relaxation by reducing noise (turning off the television) and ensuring that the patient is comfortable. 34. A nurse is providing AM care to patients. Which action will the nurse take? ANS: B CHG is safe to use on the perineum and external mucosa. If patient has diabetes or peripheral vascular disease with impaired circulation and/or sensation, do not soak feet. Maceration of skin may predispose to infection. Do not use long, firm strokes to wash the lower extremities of patients with history of deep vein thrombosis or blood-clotting disorders. Use short, light strokes instead. Eye should be cleansed from the inner to outer canthus on all patients. 35. The nurse is providing a complete bed bath to a patient using a commercial bath cleansing pack (bag bath). What should the nurse do? ANS: B The nurse should allow the skin to air-dry for 30 seconds. Drying the skin with a towel removes the emollient that is left behind after the water/cleanser solution evaporates. It is permissible to lightly cover the patient with a bath blanket or towel to prevent chilling. Do not rinse when using a bag bath. 36. A nurse is providing perineal care to a female patient. Which washing technique will the nurse use? a. Not offering a backrub to a patient with fractured ribs b. Not offering to wash the hair of a patient with neck trauma c. Turning off the television while giving a backrub to the patient d. Turning patient’s head with neck injury to side when giving oral care a. Soaks feet of patient with peripheral vascular disease b. Applies CHG solution to wash perineum of patient with a stroke c. Cleanses eye from outer canthus to inner canthus of patient with diabetes d. Uses long, firm stroke to wash legs of patient with blood-clotting disorder a. Rinse thoroughly. b. Allow the skin to air-dry. c. Do not use a bath towel. d. Dry the skin with a towel. a. Back to front 177 A&E I Comprehensive Testbank ANS: C Cleansing from pubic area to rectum (front to back) reduces the transfer of microorganisms to the urinary meatus and decreases the risk of urinary tract infection. Cleansing from rectum to pubic area or back to front increases the risk of urinary tract infection. Circular motions are used in male perineal care. 37. The nurse is providing perineal care to an uncircumcised male patient. Which action will the nurse take? ANS: C Return the foreskin to its natural position. Keeping the foreskin retracted leads to tightening of the foreskin around the shaft of the penis, causing local edema and discomfort. The foreskin may not return to its natural position on its own. Patients at greatest risk for infection are uncircumcised males. 38. Which instruction will the nurse provide to the nursing assistive personnel when providing foot care for a patient with diabetes? ANS: C Report any changes that may indicate inflammation or injury to tissue. Do not allow the diabetic patient to go barefoot; injury can lead to amputations. Clipping toenails is not allowed. Patients with peripheral vascular disease or diabetes mellitus often require nail care from a specialist to reduce the risk of infection. When assessing the patient’s feet, the nurse palpates the dorsalis pedis of the foot, not the brachial artery. 39. The debilitated patient is resisting attempts by the nurse to provide oral hygiene. Which action will the nurse take next? ANS: A If the patient is uncooperative, or is having difficulty keeping the mouth open, insert an oral airway. Insert it upside down, and then turn the airway sideways and over the tongue to keep the teeth apart. Do not use force. Position the patient on his or her side or turn the head to allow for drainage. Placing the patient in a flat, supine position could lead to aspiration. Hydrogen peroxide is irritating to mucosa. Even though the patient is debilitated, explain the steps of mouth care and the sensations that he or she will feel. Also tell the patient when the procedure is completed. 40. A nurse is providing oral care to a patient with stomatitis. Which technique will the nurse use? b. In a circular motion c. From pubic area to rectum d. Upward from rectum to pubic area a. Leave the foreskin alone because there is little chance of infection. b. Retract the foreskin for cleansing and allow it to return on its own. c. Retract the foreskin and return it to its natural position when done. d. Leave the foreskin retracted. a. Do not place slippers on the patient’s feet. b. Trim the patient’s toenails daily. c. Report sores on the patient’s toes. d. Check the brachial artery. a. Insert an oral airway. b. Place the patient in a flat, supine position. c. Use undiluted hydrogen peroxide as a cleaner. d. Quickly proceed while not talking to the patient. 178 A&E I Comprehensive Testbank ANS: A Stomatitis causes burning, pain, and change in food and fluid tolerance. Advise patients to avoid alcohol and commercial mouthwash and stop smoking. When caring for patients with stomatitis, brush with a soft toothbrush and floss gently to prevent bleeding of the gums. In some cases, flossing needs to be temporarily omitted from oral care. Normal saline rinses (approximately 30 mL) on awaking in the morning, after each meal, and at bedtime help clean the oral cavity. 41. The nurse is teaching a patient about contact lens care. Which instructions will the nurse include in the teaching session? ANS: B Thoroughly wash and rinse lens storage case on a daily basis. Clean periodically with soap or liquid detergent, rinse thoroughly with warm water, and air-dry. Do not use tap water to clean soft lenses. Lenses should be kept moist or wet when not worn. Use fresh solution daily when storing and disinfecting lenses. 42. The patient reports to the nurse about a perceived decrease in hearing. When the nurse examines the patient’s ear, a large amount of cerumen buildup at the entrance to the ear canal is observed. Which action will the nurse take next? ANS: C When cerumen is visible, gentle, downward retraction at the entrance to the ear canal causes the wax to loosen and slip out. Instruct the patient never to use sharp objects such as bobby pins or paper clips to remove earwax. Use of such objects can traumatize the ear canal and ruptures the tympanic membrane. Avoid the use of cottontipped applicators as well because they cause earwax to become impacted within the canal. Instilling cold or hot water causes nausea or vomiting. 43. The patient is being fitted with a hearing aid. In teaching the patient how to care for the hearing aid, which instructions will the nurse provide? ANS: B Adjust volume to a comfortable level for talking at a distance of 1 yard. Initially, wear a hearing aid for 15 to 20 minutes; then gradually increase wear time to 10 to 12 hours per day. Batteries last 1 week with daily a. Avoid commercial mouthwashes. b. Avoid normal saline rinses. c. Brush with a hard toothbrush. d. Brush with an alcohol-based toothpaste. a. Use tap water to clean soft lenses. b. Wash and rinse lens storage case daily. c. Reuse storage solution for up to a week. d. Keep the lenses is a cool dry place when not being used. a. Teach the patient how to use cotton-tipped applicators. b. Tell the patient to use a bobby pin to extract earwax. c. Apply gentle, downward retraction of the ear canal. d. Instill hot water into the ear canal to melt the wax. a. Change the battery every day or as needed. b. Adjust the volume for a talking distance of 1 yard. c. Wear the hearing aid 24 hours per day except when sleeping. d. Avoid the use of hairspray, but aerosol perfumes are allowed. 179 A&E I Comprehensive Testbank wearing of 10 to 12 hours. Avoid the use of hairspray and perfume while wearing hearing aids. Residue from the spray can cause the aid to become oily and greasy. 44. The patient is reporting an inability to clear nasal passages. Which action will the nurse take? ANS: A Excessive nasal secretions can be removed using gentle suctioning. However, patients usually remove secretions from the nose by gentle blowing into a soft tissue. Caution the patient against harsh blowing that creates pressure capable of injuring the eardrum, the nasal mucosa, and even sensitive eye structures. If the patient is unable to remove nasal secretions, assist by using a wet washcloth or a cotton-tipped applicator moistened in water or saline. Never insert the applicator beyond the length of the cotton tip. 45. A patient uses an in-the-canal hearing aid. Which assessment is a priority? ANS: B With this type of model (in-the-canal), cerumen tends to plug this model more than others. There are three popular types of hearing aids. An in-the-canal (ITC) aid is the newest, smallest, and least visible and fits entirely in the ear canal. It has cosmetic appeal, is easy to manipulate and place in the ear, and does not interfere with wearing eyeglasses or using the telephone, and the patient can wear it during most physical exercise. An in-the-ear aid (ITE, or intra-aural) is more noticeable than the ITC aid and is not for people with moisture or skin problems in the ear canal. The larger size of this type of aid (behind-the-ear, BTE, or postaural) can make use of eyeglasses and phones difficult; it is more difficult to keep in place during physical exercise. MULTIPLE RESPONSE 1. The nurse is caring for a patient with cognitive impairments. Which actions will the nurse take during AM care? (Select all that apply.) ANS: B, C If a patient is physically dependent or cognitively impaired, increase the frequency of skin assessment. Adapt your bathing procedures and the environment to reduce the triggers. For example, administer any ordered analgesic 30 minutes before a bath and be gentle in your approach. Keep the patient’s body as warm as possible with warm towels and be sure the room temperature is comfortable. 2. The nurse is caring for a patient who has peripheral neuropathy. Which clinical manifestations does the nurse expect to find upon assessment? (Select all that apply.) a. Use gentle suction to prevent tissue damage. b. Instruct patient to blow nose forcefully to clear the passage. c. Place a dry washcloth under the nose to absorb secretions. d. Insert a cotton-tipped applicator to the back of the nose. a. Eyeglass usage b. Cerumen buildup c. Type of physical exercise d. Excessive moisture problems a. Administer ordered analgesic 1 hour before bath time. b. Increase the frequency of skin assessment. c. Reduce triggers in the environment. d. Keep the room temperature cool. e. Be as quick as possible. a. Abnormal gait 180 A&E I Comprehensive Testbank ANS: A, B, D A patient with peripheral neuropathy has muscle wasting of lower extremities, foot deformities, and abnormal gait. A patient with vascular insufficiency will have decreased hair growth on legs and feet, absent or decreased pulses, and thickened nails. 3. A nurse is providing hygiene care to a bariatric patient using chlorhexidine gluconate (CHG) wipes. Which actions will the nurse take? (Select all that apply.) ANS: A, B, C CHG wipes are easy to use and accessible for older patients and bariatric patients, offering a no-rinse or -drying procedure. For a bariatric patient or a patient who is diaphoretic, provide special attention to body areas such as beneath the woman’s breasts, in the groin, skin folds, and perineal area, where moisture collects and irritates skin surfaces. Use wipes as directed on package—one wipe per each area of the body. CHG can leave the skin feeling sticky. If patients complain about its use, you need to explain their vulnerability to infection and how CHG helps reduce occurrence of health care–associated infection. 4. Which patients will the nurse determine are in need of perineal care? (Select all that apply.) ANS: A, B, D, E Patients most in need of perineal care include those at greatest risk for acquiring an infection (e.g., uncircumcised males, patients who have indwelling urinary catheters, or those who are recovering from rectal or genital surgery or childbirth). A patient with urinary and bowel incontinence needs perineal cleaning with each episode of soiling. Bariatric patients need special attention to body areas such as skin folds and the perineal area. In addition, women who are having a menstrual period require perineal care. Circumcised males are not at high risk for acquiring infection, and ambulatory patients can usually provide perineal self-care. 5. The patient must stay in bed for a bed change. Which actions will the nurse implement? (Select all that apply.) b. Foot deformities c. Absent or decreased pedal pulses d. Muscle wasting of lower extremities e. Decreased hair growth on legs and feet a. Do not rinse. b. Clean under breasts. c. Inform that the skin will feel sticky. d. Dry thoroughly between skin folds. e. Use two wipes for each area of the body. a. A patient with rectal and genital surgical dressings b. A patient with urinary and fecal incontinence c. A circumcised male who is ambulatory d. A patient who has an indwelling catheter e. A bariatric patient a. Apply sterile gloves. b. Keep soiled linen close to uniform. c. Advise patient will feel a lump when rolling over. d. Turn clean pillowcase inside out over the hand holding it. 181 A&E I Comprehensive Testbank ANS: C, D, E When making an occupied bed, advise patients they will feel a lump when turning, turn clean pillowcase inside out, and make a modified mitered corner. Clean gloves are used. Keep soiled linen away from uniform. Chapter 48: Skin Integrity and Wound Care Potter et al.: Fundamentals of Nursing, 9th Edition MULTIPLE CHOICE 1. The nurse is working on a medical-surgical unit that has been participating in a research project associated with pressure ulcers. Which risk factor will the nurse assess for that predisposes a patient to pressure ulcer development? ANS: A Patients who are confused or disoriented or who have changing levels of consciousness are unable to protect themselves. The patient may feel the pressure but may not understand what to do to relieve the discomfort or to communicate that he or she is feeling discomfort. Impaired sensory perception, impaired mobility, shear, friction, and moisture are other predisposing factors. Shortness of breath, muscular pain, and an adequate dietary intake are not included among the predisposing factors. 2. The nurse is caring for a patient who was involved in an automobile accident 2 weeks ago. The patient sustained a head injury and is unconscious. Which priorityelement will the nurse consider when planning care to decrease the development of a decubitus ulcer? ANS: B Pressure is the main element that causes pressure ulcers. Three pressure-related factors contribute to pressure ulcer development: pressure intensity, pressure duration, and tissue tolerance. When the intensity of the pressure exerted on the capillary exceeds 15 to 32 mm Hg, this occludes the vessel, causing ischemic injury to the tissues it normally feeds. High pressure over a short time and low pressure over a long time cause skin breakdown. Resistance, stress, and weight are not the priority causes of pressure ulcers. 3. Which nursing observation will indicate the patient is at risk for pressure ulcer formation? e. Make a modified mitered corner with sheet, blanket, and spread. a. Decreased level of consciousness b. Adequate dietary intake c. Shortness of breath d. Muscular pain a. Resistance b. Pressure c. Weight d. Stress a. The patient has fecal incontinence. b. The patient ate two thirds of breakfast. c. The patient has a raised red rash on the right shin. d. The patient’s capillary refill is less than 2 seconds. 182 A&E I Comprehensive Testbank ANS: A The presence and duration of moisture on the skin increase the risk of ulcer formation by making it susceptible to injury. Moisture can originate from wound drainage, excessive perspiration, and fecal or urinary incontinence. Bacteria and enzymes in the stool can enhance the opportunity for skin breakdown because the skin is moistened and softened, causing maceration. Eating a balanced diet is important for nutrition, but eating just two thirds of the meal does not indicate that the individual is at risk. A raised red rash on the leg again is a concern and can affect the integrity of the skin, but it is located on the shin, which is not a high-risk area for skin breakdown. Pressure can influence capillary refill, leading to skin breakdown, but this capillary response is within normal limits. 4. The wound care nurse visits a patient in the long-term care unit. The nurse is monitoring a patient with a Stage III pressure ulcer. The wound seems to be healing, and healthy tissue is observed. How should the nurse document this ulcer in the patient’s medical record? ANS: C When a pressure ulcer has been staged and is beginning to heal, the ulcer keeps the same stage and is labeled with the words “healing stage” or healing Stage III pressure ulcer. Once an ulcer has been staged, the stage endures even as the ulcer heals. This ulcer was labeled a Stage III, and it cannot return to a previous stage such as Stage I or II. This ulcer is healing, so it is no longer labeled a Stage III. 5. The nurse is admitting an older patient from a nursing home. During the assessment, the nurse notes a shallow open reddish, pink ulcer without slough on the right heel of the patient. How will the nurse stage this pressure ulcer? ANS: B This would be a Stage II pressure ulcer because it presents as partial-thickness skin loss involving epidermis and dermis. The ulcer presents clinically as an abrasion, blister, or shallow crater. Stage I is intact skin with nonblanchable redness over a bony prominence. With a Stage III pressure ulcer, subcutaneous fat may be visible, but bone, tendon, and muscles are not exposed. Stage IV involves full-thickness tissue loss with exposed bone, tendon, or muscle. 6. The nurse is completing a skin assessment on a patient with darkly pigmented skin. Which item should the nurse use first to assist in staging an ulcer on this patient? ANS: D When assessing a patient with darkly pigmented skin, proper lighting is essential to accurately complete the first step in assessment—inspection—and the entire assessment process. Natural light or a halogen light is recommended. Fluorescent light sources can produce blue tones on darkly pigmented skin and can interfere with an accurate assessment. Other items that could possibly be used during the assessment include gloves for a. Stage I pressure ulcer b. Healing Stage II pressure ulcer c. Healing Stage III pressure ulcer d. Stage III pressure ulcer a. Stage I b. Stage II c. Stage III d. Stage IV a. Disposable measuring tape b. Cotton-tipped applicator c. Sterile gloves d. Halogen light 183 A&E I Comprehensive Testbank infection control, a disposable measuring device to measure the size of the wound, and a cotton-tipped applicator to measure the depth of the wound, but these items are not the first items used. 7. The nurse is caring for a patient with a Stage IV pressure ulcer. Which type of healing will the nurse consider when planning care for this patient? ANS: B Stage IV pressure ulcers are full-thickness wounds that extend into the dermis and heal by scar formation because the deeper structures do not regenerate, hence the need for full-thickness repair. The full-thickness repair has four phases: hemostasis, inflammatory, proliferative, and maturation. A wound heals by primary intention when wounds such as surgical wounds have little tissue loss; the skin edges are approximated or closed, and the risk for infection is low. Partial-thickness repairs are done on partial-thickness wounds that are shallow, involving loss of the epidermis and maybe partial loss of the dermis. These wounds heal by regeneration because the epidermis regenerates. Tertiary intention is seen when a wound is left open for several days, and then the wound edges are approximated. Wound closure is delayed until risk of infection is resolved. 8. The nurse is caring for a group of patients. Which patient will the nurse see first? ANS: C The nurse should see the patient with an appendicitis first. Warm applications are contraindicated when the patient has an acute, localized inflammation such as appendicitis because the heat could cause the appendix to rupture. Although a Stage IV pressure ulcer is deep, it is not as critical as the appendicitis patient. The total Braden score ranges from 6 to 23; a lower total score indicates a higher risk for pressure ulcer development. A score of 18 can be assessed later. A healing incision is approximated (closed); this is a normal finding and does not need to be seen first. 9. The nurse is caring for a patient who is experiencing a full-thickness repair. Which type of tissue will the nurse expect to observe when the wound is healing? ANS: C Granulation tissue is red, moist tissue composed of new blood vessels, the presence of which indicates progression toward healing. Soft yellow or white tissue is characteristic of slough—a substance that needs to be removed for the wound to heal. Black or brown necrotic tissue is called eschar, which also needs to be removed for a wound to heal. Purulent drainage is indicative of an infection and will need to be resolved for the wound to heal. 10. The nurse is caring for a patient who has experienced a laparoscopic appendectomy. For which type of healing will the nurse focus the care plan? a. Partial-thickness wound repair b. Full-thickness wound repair c. Primary intention d. Tertiary intention a. A patient with a Stage IV pressure ulcer b. A patient with a Braden Scale score of 18 c. A patient with appendicitis using a heating pad d. A patient with an incision that is approximated a. Eschar b. Slough c. Granulation d. Purulent drainage a. Partial-thickness repair 184 A&E I Comprehensive Testbank ANS: D A clean surgical incision is an example of a wound with little loss of tissue that heals with primary intention. The skin edges are approximated or closed, and the risk for infection is low. Partial-thickness repairs are done on partial-thickness wounds that are shallow, involving loss of the epidermis and maybe partial loss of the dermis. These wounds heal by regeneration because the epidermis regenerates. Tertiary intention is seen when a wound is left open for several days, and then the wound edges are approximated. Wound closure is delayed until the risk of infection is resolved. A wound involving loss of tissue such as a burn or a pressure ulcer or laceration heals by secondary intention. The wound is left open until it becomes filled with scar tissue. It takes longer for a wound to heal by secondary intention; thus the chance of infection is greater. 11. The nurse is caring for a patient in the burn unit. Which type of wound healing will the nurse consider when planning care for this patient? ANS: B A wound involving loss of tissue such as a burn or a pressure ulcer or laceration heals by secondary intention. The wound is left open until it becomes filled with scar tissue. It takes longer for a wound to heal by secondary intention; thus the chance of infection is greater. A clean surgical incision is an example of a wound with little loss of tissue that heals by primary intention. The skin edges are approximated or closed, and the risk for infection is low. Partial-thickness repair is done on partial-thickness wounds that are shallow, involving loss of the epidermis and maybe partial loss of the dermis. These wounds heal by regeneration because the epidermis regenerates. Tertiary intention is seen when a wound is left open for several days, and then the wound edges are approximated. Wound closure is delayed until the risk of infection is resolved. 12. A nurse is assessing a patient’s wound. Which nursing observation will indicate the wound healed by secondary intention? ANS: D A wound healing by secondary intention takes longer than one healing by primary intention. The wound is left open until it becomes filled with scar tissue. If the scarring is severe, permanent loss of function often occurs. Wounds that heal by primary intention heal quickly with minimal scarring. Scar tissue contains few pigmented cells and has a lighter color than normal skin. 13. The nurse is caring for a patient who has experienced a total abdominal hysterectomy. Which nursing observation of the incision will indicate the patient is experiencing a complication of wound healing? b. Secondary intention c. Tertiary intention d. Primary intention a. Partial-thickness repair b. Secondary intention c. Tertiary intention d. Primary intention a. Minimal loss of tissue function b. Permanent dark redness at site c. Minimal scar tissue d. Scarring that may be severe a. The site is hurting. b. The site is approximated. c. The site has started to itch. 185 A&E I Comprehensive Testbank ANS: D A hematoma is a localized collection of blood underneath the tissues. It appears as swelling, change in color, sensation, or warmth or a mass that often takes on a bluish discoloration. A hematoma near a major artery or vein is dangerous because it can put pressure on the vein or artery and obstruct blood flow. Itching is not a complication. Incisions should be approximated with edges together; this is a sign of normal healing. After surgery, when nerves in the skin and tissues have been traumatized by the surgical procedure, it is expected that the patient will experience pain. 14. A nurse is caring for a postoperative patient. Which finding will alert the nurse to a potential wound dehiscence? ANS: C Patients often report feeling as though something has given way with dehiscence. Dehiscence occurs when an incision fails to heal properly and the layers of skin and tissue separate. It involves abdominal surgical wounds and occurs after a sudden strain such as coughing, vomiting, or sitting up in bed. Evisceration is seen when vital organs protrude through a wound opening. When there is an increase in serosanguineous drainage from a wound in the first few days after surgery, be alert for the potential for dehiscence. Infection is characterized by drainage that is odorous and purulent. 15. A patient has developed a pressure ulcer. Which laboratory data will be important for the nurse to check? ANS: C Normal wound healing requires proper nutrition. Serum proteins are biochemical indicators of malnutrition, and serum albumin is probably the most frequently measured of these parameters. The best measurement of nutritional status is prealbumin because it reflects not only what the patient has ingested but also what the body has absorbed, digested, and metabolized. Zinc and copper are the minerals important for wound healing, not potassium and sodium. Vitamins A and C are important for wound healing, not vitamin E. 16. A nurse is caring for a patient with a wound. Which assessment data will be most important for the nurse to gather with regard to wound healing? ANS: B Oxygen fuels the cellular functions essential to the healing process; the ability to perfuse tissues with adequate amounts of oxygenated blood is critical in wound healing. Pulse oximetry measures the oxygen saturation of blood. Assessment of muscular strength and sensation, although useful for fitness and mobility testing, does not provide any data with regard to wound healing. Sleep, although important for rest and for integration of learning and restoration of cognitive function, does not provide any data with regard to wound healing. d. The site has a mass, bluish in color. a. Protrusion of visceral organs through a wound opening b. Chronic drainage of fluid through the incision site c. Report by patient that something has given way d. Drainage that is odorous and purulent a. Vitamin E b. Potassium c. Albumin d. Sodium a. Muscular strength assessment b. Pulse oximetry assessment c. Sensation assessment d. Sleep assessment 186 A&E I Comprehensive Testbank 17. The nurse is caring for a patient with a healing Stage III pressure ulcer. Upon entering the room, the nurse notices an odor and observes a purulent discharge, along with increased redness at the wound site. What is the next best step for the nurse? ANS: A The patient is showing signs and symptoms associated with infection in the wound. The nurse should complete the assessment: gather all data such as current treatment modalities, medications, vital signs including temperature, and laboratory results such as the most recent complete blood count or white cell count. The nurse can then notify the primary care provider and receive treatment orders for the patient. It is important to notify the charge nurse and consult the wound nurse on the patient’s status and on any new orders. 18. The nurse is collaborating with the dietitian about a patient with a Stage III pressure ulcer. Which nutrient will the nurse most likely increase after collaboration with the dietitian? ANS: B Protein needs are especially increased in supporting the activity of wound healing. The physiological processes of wound healing depend on the availability of protein, vitamins (especially A and C), and the trace minerals of zinc and copper. Wound healing does not require increased amounts of fats or carbohydrates. Vitamin E will not be increased for wound healing. 19. The nurse is completing an assessment on a patient who has a Stage IV pressure ulcer. The wound is odorous, and a drain is currently in place. Which statement by the patient indicates issues with self-concept? ANS: C Body image changes can influence self-concept. The wound is odorous, and a drain is in place. The patient who is asking for a bath and change in linens and states that this is awful gives you a clue that he or she may be concerned about the smell in the room. Factors that affect the patient’s perception of the wound include the presence of scars, drains, odor from drainage, and temporary or permanent prosthetic devices. The patient’s stating that he or she wants to feel better, talking about going home, and caring about what is for dinner could be interpreted as positive statements that indicate progress along the health journey. 20. A patient presents to the emergency department with a laceration of the right forearm caused by a fall. After determining that the patient is stable, what is the next best step for the nurse to take? a. Complete the head-to-toe assessment, including current treatment, vital signs, and laboratory results. b. Notify the health care provider by utilizing Situation, Background, Assessment, and Recommendation (SBAR). c. Consult the wound care nurse about the change in status and the potential for infection. d. Check with the charge nurse about the change in status and the potential for infection. a. Fat b. Protein c. Vitamin E d. Carbohydrate a. “I am so weak and tired. I want to feel better.” b. “I am thinking I will be ready to go home early next week.” c. “I am ready for my bath and linen change right now since this is awful.” d. “I am hoping there will be something good for dinner tonight.” a. Inspect the wound for foreign bodies. 187 A&E I Comprehensive Testbank ANS: B After determining that a patient’s condition is stable, inspect the wound for bleeding. An abrasion will have limited bleeding, a laceration can bleed more profusely, and a puncture wound bleeds in relation to the size and depth of the wound. Address any bleeding issues. Inspect the wound for foreign bodies; traumatic wounds are dirty and may need to be addressed. Determine the size of the wound. A large open wound may expose bone or tissue and be protected, or the wound may need suturing. When the wound is caused by a dirty penetrating object, determine the need for a tetanus vaccination. 21. The nurse is caring for a patient on the medical-surgical unit with a wound that has a drain and a dressing that needs changing. Which action should the nurse take first? ANS: A Because removal of dressings is painful, if often helps to give an analgesic at least 30 minutes before exposing a wound and changing the dressing. The next sequence of events includes gathering supplies for the dressing change, donning gloves, and avoiding the accidental removal of the drain during the procedure. 22. The nurse is caring for a patient who has a wound drain with a collection device. The nurse notices that the collection device has a sudden decrease in drainage. Which action will the nurse take next? ANS: A Because a drainage system needs to be patent, look for drainage flow through the tubing, as well as around the tubing. A sudden decrease in drainage through the tubing may indicate a blocked drain, and you will need to notify the health care provider. The health care provider, not the nurse, determines the need for drain removal and removes drains. Charting the results on the intake and output flow sheet does not take care of the problem. The evacuator may be compressed even when a blockage is present. 23. The nurse is caring for a patient who has a Stage IV pressure ulcer with grafted surgical sites. Which specialty bed will the nurse use for this patient? ANS: B For a patient with newly flapped or grafted surgical sites, the air-fluidized bed will be the best choice; this uses air and fluid support to provide pressure redistribution via a fluid-like medium created by forcing air through beads as characterized by immersion and envelopment. A low-air-loss bed is utilized for prevention or b. Inspect the wound for bleeding. c. Determine the size of the wound. d. Determine the need for a tetanus antitoxin injection. a. Provide analgesic medications as ordered. b. Avoid accidentally removing the drain. c. Don sterile gloves. d. Gather supplies. a. Call the health care provider; a blockage is present in the tubing. b. Chart the results on the intake and output flow sheet. c. Do nothing, as long as the evacuator is compressed. d. Remove the drain; a drain is no longer needed. a. Low-air-loss b. Air-fluidized c. Lateral rotation d. Standard mattress 188 A&E I Comprehensive Testbank treatment of skin breakdown by preventing buildup of moisture and skin breakdown through the use of airflow. A standard mattress is utilized for an individual who does not have actual or potential altered or impaired skin integrity. Lateral rotation is used for treatment and prevention of pulmonary, venous stasis and urinary complications associated with mobility. 24. The nurse is caring for a patient with a pressure ulcer on the left hip. The ulcer is black. Which next step will the nurse anticipate? ANS: C Debridement is the removal of nonviable necrotic (black) tissue. Removal of necrotic tissue is necessary to rid the ulcer of a source of infection, to enable visualization of the wound bed, and to provide a clean base for healing. A wound will not move through the phases of healing if the wound is infected. Documentation occurs after completion of skill. When treating a pressure ulcer, it is important to monitor and reassess the wound at least every 8 hours. Management of drainage will help keep the wound clean, but that is not the next step. 25. The nurse is caring for a patient with a healing Stage III pressure ulcer. The wound is clean and granulating. Which health care provider’s order will the nurse question? ANS: B Clean pressure ulcers with noncytotoxic cleansers such as normal saline, which will not kill fibroblasts and healing tissue. Cytotoxic cleansers such as Dakin’s solution, acetic acid, povidone-iodine, and hydrogen peroxide can hinder the healing process and should not be utilized on clean granulating wounds. Consulting a dietitian for the nutritional needs of the patient, utilizing a low-air-loss therapy unit to decrease pressure, and applying hydrogel dressings to provide a moist environment for healing are all orders that would be appropriate. 26. The nurse is completing an assessment of the patient’s skin’s integrity. Which assessment is the priority? ANS: A Observe pressure points such as bony prominences. The nurse continually assesses the skin for signs of ulcer development. Assessment for tissue pressure damage includes visual and tactile inspection of the skin. Assessment of pulses, breath sounds, and bowel sounds is part of a head-to-toe assessment and could influence the function of the body and ultimately skin integrity; however, this assessment is not a specific part or priority of a skin assessment. 27. The nurse is completing a skin risk assessment using the Braden Scale. The patient has slight sensory impairment, has skin that is rarely moist, walks occasionally, and has slightly limited mobility, along with excellent intake of meals and no apparent problem with friction and shear. Which score will the nurse document for this patient? a. Monitor the wound. b. Document the wound. c. Debride the wound. d. Manage drainage from wound. a. Use a low-air-loss therapy unit. b. Irrigate with Dakin’s solution. c. Apply a hydrogel dressing. d. Consult a dietitian. a. Pressure points b. Breath sounds c. Bowel sounds d. Pulse points 189 A&E I Comprehensive Testbank ANS: C With use of the Braden Scale, the total score is a 20. The patient receives 3 for slight sensory perception impairment, 4 for skin being rarely moist, 3 for walks occasionally, 3 for slightly limited mobility, 4 for intake of meals, and 4 for no problem with friction and shear. 28. The nurse is caring for a surgical patient. Which intervention is most important for the nurse to complete to decrease the risk of pressure ulcers and encourage the patient’s willingness and ability to increase mobility? ANS: D Maintaining adequate pain control (providing analgesic medications) and patient comfort increases the patient’s willingness and ability to increase mobility, which in turn reduces pressure ulcer risks. Although sitting in the chair is beneficial, it does not increase mobility or provide pain control. Explaining the risk of immobility is important for the patient because it may impact the patient’s willingness but not his or her ability. Turning the patient is important for decreasing pressure ulcers but needs to be done every 2 hours and, again, does not influence the patient’s ability to increase mobility. 29. The nurse is caring for a patient with a Stage IV pressure ulcer. Which nursing diagnosis does the nurse add to the care plan? ANS: C After the assessment is completed and the information that the patient has a Stage IV pressure ulcer is gathered, a diagnosis of Impaired skin integrity is selected. Readiness for enhanced nutrition would be selected for an individual with an adequate diet that could be improved. Impaired physical mobility and Chronic pain do not support the current data in the question. 30. The nurse collects the following assessment data: right heel with reddened area that does not blanch. Which nursing diagnosis will the nurse assign to this patient? ANS: B The area on the heel has experienced a decreased supply of blood and oxygen (tissue perfusion), which has resulted in tissue damage. The most appropriate nursing diagnosis with this information is Ineffective a. 15 b. 17 c. 20 d. 23 a. Explain the risks of immobility to the patient. b. Turn the patient every 3 hours while in bed. c. Encourage the patient to sit up in the chair. d. Provide analgesic medication as ordered. a. Readiness for enhanced nutrition b. Impaired physical mobility c. Impaired skin integrity d. Chronic pain a. Imbalanced nutrition: less than body requirements b. Ineffective peripheral tissue perfusion c. Risk for infection d. Acute pain 190 A&E I Comprehensive Testbank peripheral tissue perfusion. Risk for infection, Acute pain, and Imbalanced nutrition do not support the data in the question. 31. The nurse is caring for a patient who is immobile. The nurse wants to decrease the formation of pressure ulcers. Which action will the nurse take first? ANS: C The first step in prevention is to assess the patient’s risk factors for pressure ulcer development. When a patient is immobile, the major risk to the skin is the formation of pressure ulcers. Nursing interventions focus on prevention. Offering favorite fluids, turning, and increasing carbohydrates and fats are not the first steps. Determining risk factors is first so interventions can be implemented to reduce or eliminate those risk factors. 32. The medical-surgical acute care patient has received a nursing diagnosis of Impaired skin integrity. Which health care team member will the nurse consult? ANS: B Refer patients with pressure ulcers to the dietitian for early intervention for nutritional problems. Adequate calories, protein, vitamins, and minerals promote wound healing for the impaired skin integrity. The nurse is the coordinator of care, and collaborating with the dietitian would result in planning the best meals for the patient. The respiratory therapist can be consulted when a patient has issues with the respiratory system. Case management can be consulted when the patient has a discharge need. A chaplain can be consulted when the patient has a spiritual need. 33. The nurse is caring for a patient with a Stage II pressure ulcer and has assigned a nursing diagnosis of Risk for infection. The patient is unconscious and bedridden. The nurse is completing the plan of care and is writing goals for the patient. Which is the best goal for this patient? ANS: D Because the patient has an open wound and the skin is no longer intact to protect the tissue, the patient is at increased risk for infection. The nurse will be assessing the patient for signs and symptoms of infection, including an increase in temperature, an increase in white count, and odorous and purulent drainage from the wound. The patient is unconscious and is unable to communicate the signs and symptoms of infection. It is important for the patient’s family to be able to demonstrate how to care for the wound and wash their hands, but these statements are not goals or outcomes for this nursing diagnosis. 34. The nurse is caring for a group of patients. Which task can the nurse delegate to the nursing assistive personnel? a. Offer favorite fluids. b. Turn the patient every 2 hours. c. Determine the patient’s risk factors. d. Encourage increased quantities of carbohydrates and fats. a. Respiratory therapist b. Registered dietitian c. Case manager d. Chaplain a. The patient will state what to look for with regard to an infection. b. The patient’s family will demonstrate specific care of the wound site. c. The patient’s family members will wash their hands when visiting the patient. d. The patient will remain free of odorous or purulent drainage from the wound. a. Assessing a surgical patient for risk of pressure ulcers 191 A&E I Comprehensive Testbank ANS: B Applying an elastic bandage to a medical-surgical patient can be delegated to the nursing assistive personnel (NAP). Assessing pressure ulcer risk, treating a pressure ulcer, and implementing negative-pressure wound therapy cannot be delegated to an NAP. 35. The nurse is performing a moist-to-dry dressing. The nurse has prepared the supplies, solution, and removed the old dressing. In which order will the nurse implement the steps, starting with the first one? 1. Apply sterile gloves. 2. Cover and secure topper dressing. 3. Assess wound and surrounding skin. 4. Moisten gauze with prescribed solution. 5. Gently wring out excess solution and unfold. 6. Loosely pack until all wound surfaces are in contact with gauze. ANS: B The steps for a moist-to-dry dressing are as follows: (1) Apply sterile gloves; (2) assess appearance of surrounding skin; (3) moisten gauze with prescribed solution. (4) Gently wring out excess solution and unfold; apply gauze as single layer directly onto wound surface. (5) If wound is deep, gently pack dressing into wound base by hand until all wound surfaces are in contact with gauze; (6) cover with sterile dry gauze and secure topper dressing. 36. The nurse is caring for a patient who has suffered a stroke and has residual mobility problems. The patient is at risk for skin impairment. Which initial actions should the nurse take to decrease this risk? ANS: A Use cleansers with nonionic surfactants that are gentle to the skin. After you clean the skin, make sure that it is completely dry. Absorbent pads and garments are controversial and should be considered only when other alternatives have been exhausted. Depending on the needs of the patient, a specialty bed may be needed, but again, this does not provide the initial defense for skin breakdown. Use only products that wick moisture away from the patient’s skin. 37. The nurse is caring for a patient who is at risk for skin impairment. The patient is able to sit up in a chair. The nurse includes this intervention in the plan of care. How long should the nurse schedule the patient to sit in the chair? b. Applying an elastic bandage to a medical-surgical patient c. Treating a pressure ulcer on the buttocks of a medical patient d. Implementing negative-pressure wound therapy on a stable patient a. 4, 3, 1, 5, 6, 2 b. 1, 3, 4, 5, 6, 2 c. 4, 1, 3, 5, 6, 2 d. 1, 4, 3, 5, 6, 2 a. Use gentle cleansers, and thoroughly dry the skin. b. Use therapeutic bed and mattress. c. Use absorbent pads and garments. d. Use products that hold moisture to the skin. a. At least 3 hours b. Less than 2 hours 192 A&E I Comprehensive Testbank ANS: B When patients are able to sit up in a chair, make sure to limit the amount of time to 2 hours or less. The chair sitting time should be individualized. In the sitting position, pressure on the ischial tuberosities is greater than in a supine position. Utilize foam, gel, or an air cushion to distribute weight. Sitting for longer than 2 hours can increase the chance of ischemia. 38. The nurse is caring for a patient who is immobile and is at risk for skin impairment. The plan of care includes turning the patient. Which is the best method for repositioning the patient? ANS: B When repositioning the patient, obtain assistance and utilize a transfer device to lift rather than drag the patient. Sliding the patient into the new position will increase friction. The patient should be placed in a 30- degree lateral position, not a supine position. The head of the bed should be elevated less than 30 degrees to prevent pressure ulcer development from shearing forces. 39. A nurse is assigned most of the patients with pressure ulcers. The nurse leaves the pressure ulcer open to air and does not apply a dressing. To which patient did the nurse provide care? ANS: A Stage I intact pressure ulcers that resolve slowly without epidermal loss over 7 to 14 days do not require a dressing. A composite film, hydrocolloid, or hydrogel can be utilized on a clean Stage II. A hydrocolloid, hydrogel covered with foam, calcium alginate, and gauze can be utilized with a clean Stage III. Hydrogel covered with foam, calcium alginate, and gauze can be utilized with a clean Stage IV. An unstageable wound covered with eschar should utilize a dressing of adherent film or gauze with an ordered solution of enzymes. 40. The nurse is caring for a patient with a wound. The patient appears anxious as the nurse is preparing to change the dressing. Which action should the nurse take? ANS: B Explaining the procedure educates the patient regarding the dressing change and involves him in the care, thereby allowing the patient some control in decreasing anxiety. Telling the patient to close the eyes and turning on the television are distractions that do not usually decrease a patient’s anxiety. If the family is a support system, asking support systems to leave the room can actually increase a patient’s anxiety. 41. The nurse is cleansing a wound site. As the nurse administers the procedure, which intervention should be included? c. No longer than 30 minutes d. As long as the patient remains comfortable a. Place the patient in a 30-degree supine position. b. Utilize a transfer device to lift the patient. c. Elevate the head of the bed 45 degrees. d. Slide the patient into the new position. a. A patient with a clean Stage I b. A patient with a clean Stage II c. A patient with a clean Stage III d. A patient with a clean Stage IV a. Turn on the television. b. Explain the procedure. c. Tell the patient “Close your eyes.” d. Ask the family to leave the room. 193 A&E I Comprehensive Testbank ANS: C Cleanse in a direction from the least contaminated area, such as from the wound or incision, to the surrounding skin. While cleansing surgical or traumatic wounds by applying noncytotoxic solution with sterile gauze or by irrigations is correct, vigorous scrubbing is inappropriate and can cause damage to the skin. Use gentle friction when applying solutions to the skin, and allow irrigation to flow from the least to the most contaminated area. 42. The nurse is caring for a patient after an open abdominal aortic aneurysm repair. The nurse requests an abdominal binder and carefully applies the binder. Which is the best explanation for the nurse to use when teaching the patient the reason for the binder? ANS: D The patient has a large abdominal incision. This incision will need support, and an abdominal binder will support this wound, especially during movement, as well as during deep breathing and coughing. A binder can be used to immobilize a body part (e.g., an elastic bandage applied around a sprained ankle). A binder can be used to prevent edema, for example, in an extremity but in this case is not used to reduce edema at a surgical site. A binder can be used to secure dressings such as elastic webbing applied around a leg after vein stripping. 43. The nurse is caring for a postoperative medial meniscus repair of the right knee. Which action should the nurse take to assist with pain management? ANS: D Ice assists in preventing edema formation, controlling bleeding, and anesthetizing the body part. Elevation (not dependent) assists in preventing edema, which in turn can cause pain. Monitoring vital signs every 15 minutes is routine postoperative care and includes a pain assessment but in itself is not an intervention that decreases pain. Checking the pulses is important to monitor the circulation of the extremity but in itself is not a pain management intervention. 44. The patient has a risk for skin impairment and has a 15 on the Braden Scale upon admission. The nurse has implemented interventions. Upon reassessment, which Braden score will be the best sign that the risk for skin breakdown is removed? a. Allow the solution to flow from the most contaminated to the least contaminated. b. Scrub vigorously when applying noncytotoxic solution to the skin. c. Cleanse in a direction from the least contaminated area. d. Utilize clean gauze and clean gloves to cleanse a site. a. It reduces edema at the surgical site. b. It secures the dressing in place. c. It immobilizes the abdomen. d. It supports the abdomen. a. Monitor vital signs every 15 minutes. b. Check pulses in the right foot. c. Keep the leg dependent. d. Apply ice. a. 12 b. 13 c. 20 d. 23 194 A&E I Comprehensive Testbank ANS: D The best sign is a perfect score of 23. The Braden Scale is composed of six subscales: sensory perception, moisture, activity, mobility, nutrition, and friction and shear. The total score ranges from 6 to 23, and a lower total score indicates a higher risk for pressure ulcer development. The cutoff score for onset of pressure ulcer risk with the Braden Scale in the general adult population is 18. MULTIPLE RESPONSE 1. The nurse is caring for a patient with a surgical incision that eviscerates. Which actions will the nurse take? (Select all that apply.) ANS: A, C, E The presence of an evisceration (protrusion of visceral organs through a wound opening) is a surgical emergency. Immediately place damp sterile gauze over the site, contact the surgical team, do not allow the patient anything by mouth (NPO), observe for signs and symptoms of shock, and prepare the patient for emergency surgery. 2. The nurse is caring for a patient with a wound healing by full-thickness repair. Which phases will the nurse monitor for in this patient? (Select all that apply.) ANS: A, B, C, D The four phases involved in the healing process of a full-thickness wound are hemostasis, inflammatory, proliferative, and maturation. Three components are involved in the healing process of a partial-thickness wound: inflammatory response, epithelial proliferation (reproduction) and migration, and reestablishment of the epidermal layers. 3. The nurse is completing a skin assessment on a medical-surgical patient. Which nursing assessment questions should be included in a skin integrity assessment? (Select all that apply.) ANS: A, B, C, E Changing positions is important for decreasing the pressure associated with long periods of time in the same position. If the patient is able to feel heat or cold and is mobile, she can protect herself by withdrawing from a. Place moist sterile gauze over the site. b. Gently place the organs back. c. Contact the surgical team. d. Offer a glass of water. e. Monitor for shock. a. Hemostasis b. Maturation c. Inflammatory d. Proliferative e. Reproduction f. Reestablishment of epidermal layers a. “Can you easily change your position?” b. “Do you have sensitivity to heat or cold?” c. “How often do you need to use the toilet?” d. “What medications do you take?” e. “Is movement painful?” f. “Have you ever fallen?” 195 A&E I Comprehensive Testbank the source. Knowing toileting habits and any potential for incontinence is important because urine and feces in contact with the skin for long periods can increase skin breakdown. Knowing whether the patient has problems with painful movement will alert the nurse to any potential for decreased movement and increased risk for skin breakdown. Medications and falling are safety risk questions. 4. The nurse is caring for a patient with potential skin breakdown. Which components will the nurse include in the skin assessment? (Select all that apply.) ANS: B, C, D, E Assessment of the skin includes both visual and tactile inspection. Assess for hyperemia and palpate for blanching or nonblaching. Early signs of skin damage include induration, bogginess (less-than-normal stiffness), and increased warmth at the injury site compared to nearby areas. Changes in temperature can indicate changes in blood flow to that area of the skin. Vision is not included in the skin assessment. 5. The nurse is caring for a patient who will have both a large abdominal bandage and an abdominal binder. Which actions will the nurse take before applying the bandage and binder? (Select all that apply.) ANS: A, C, D, F Before applying a bandage or a binder, the nurse has several responsibilities. The nurse would need to inspect the skin for abrasions, edema, and discoloration or exposed wound edges. The nurse also is responsible for covering exposed wounds or open abrasions with a dressing and assessing the condition of underlying dressings and changing if soiled, as well as assessing the skin of underlying areas that will be distal to the bandage. This checks for signs of circulatory impairment, so that a comparison can be made after bandages are applied. Marking the sites of all abrasions is not necessary. Although it is important for the skin to be clean, and even though it may need to be cleaned with a noncytotoxic cleanser, cleansing with hydrogen peroxide can interfere with wound healing. 6. The nurse is updating the plan of care for a patient with impaired skin integrity. Which findings indicate achievement of goals and outcomes? (Select all that apply.) ANS: B, C, D, E Optimal outcomes are to prevent injury to skin and tissues, reduce injury to skin, reduce injury to underlying tissues, and restore skin integrity. Skin intact, nonblanchable erythema absent, no injuries, and presence of a. Vision b. Hyperemia c. Induration d. Blanching e. Temperature of skin a. Cover exposed wounds. b. Mark the sites of all abrasions. c. Assess the condition of current dressings. d. Inspect the skin for abrasions and edema. e. Cleanse the area with hydrogen peroxide. f. Assess the skin at underlying areas for circulatory impairment. a. The patient’s expectations are not being met. b. Skin is intact with no redness or swelling. c. Nonblanchable erythema is absent. d. No injuries to the skin and tissues are evident. e. Granulation tissue is present. 196 A&E I Comprehensive Testbank granulation tissue are all findings indicating achievement of goals and outcomes. The patient’s expectations not being met indicates no progression toward goals/outcomes. MATCHING The nurse is caring for patients who need wound dressings. Match the type of dressing the nurse applies to its description. 1. Gauze 2. Transparent 3. Hydrocolloid 4. Hydrogel 5. Calcium alginate 1.ANS:E2.ANS:C3.ANS:A4.ANS:B5.ANS:D Week 5 Fluid & Electrolytes, Dehydration Chapter 42: Fluid, Electrolyte, and Acid-Base Balance Potter et al.: Fundamentals of Nursing, 9th Edition a. Absorbs drainage through the use of exudate absorbers in the dressing b. Very soothing to the patient and do not adhere to the wound bed c. Barrier to external fluids/bacteria but allows wound to “breathe” d. Manufactured from seaweed and comes in sheet and rope form e. Oldest and most common absorbent dressing 197 A&E I Comprehensive Testbank MULTIPLE CHOICE 1.A patient has dehydration. While planning care, the nurse considers that the majority of the patient’s total water volume exists in with compartment? ANS: A Intracellular (inside the cells) fluid accounts for approximately two thirds of total body water. Extracellular (outside the cells) is approximately one third of the total body water. Intravascular fluid (liquid portion of the blood) and transcellular fluid are two major divisions of the extracellular compartment. 2.The nurse is teaching about the process of passively moving water from an area of lower particle concentration to an area of higher particle concentration. Which process is the nurse describing? ANS: A The process of moving water from an area of low particle concentration to an area of higher particle concentration is known as osmosis. Filtration is mediated by fluid pressure from an area of higher pressure to an area of lower pressure. Diffusion is passive movement of electrolytes or other particles down the concentration gradient (from areas of higher concentration to areas of lower concentration). Active transport requires energy in the form of adenosine triphosphate (ATP) to move electrolytes across cell membranes against the concentration gradient (from areas of lower concentration to areas of higher concentration). 3.The nurse observes edema in a patient who has venous congestion from right heart failure. Which type of pressure facilitated the formation of the patient’s edema? ANS: C Venous congestion increases capillary hydrostatic pressure. Increased hydrostatic pressure causes edema by causing increased movement of fluid into the interstitial area. Osmotic and oncotic pressures involve the concentrations of solutes and can contribute to edema in other situations, such as inflammation or malnutrition. Concentration pressure is not a nursing term. 4.The nurse administers an intravenous (IV) hypertonic solution to a patient. In which direction will the fluid shift? a. Intracellular b. Extracellular c. Intravascular d. Transcellular a. Osmosis b. Filtration c. Diffusion d. Active transport a. Osmotic b. Oncotic c. Hydrostatic d. Concentration a. From intracellular to extracellular b. From extracellular to intracellular c. From intravascular to intracellular 198 A&E I Comprehensive Testbank ANS: A Hypertonic solutions will move fluid from the intracellular to the extracellular (intravascular). A hypertonic solution has a concentration greater than normal body fluids, so water will shift out of cells because of the osmotic pull of the extra particles. Movement of water from the extracellular (intravascular) into cells (intracellular) occurs when hypotonic fluids are administered. Distribution of fluid between intravascular and interstitial spaces occurs by filtration, the net sum of hydrostatic and osmotic pressures. 5.A nurse is preparing to start peripheral intravenous (IV) therapy. In which order will the nurse perform the steps starting with the first one? 1. Clean site. 2. Select vein. 3. Apply tourniquet. 4. Release tourniquet. 5. Reapply tourniquet. 6. Advance and secure. 7. Insert vascular access device. ANS: D The steps for inserting an intravenous catheter are as follows: Apply tourniquet; select vein; release tourniquet; clean site; reapply tourniquet; insert vascular access device; and advance and secure. 6.The nurse is reviewing laboratory results. Which cation will the nurse observe is the most abundant in the blood? ANS: A Sodium is the most abundant cation in the blood. Potassium is the predominant intracellular cation. Chloride is an anion (negatively charged) rather than a cation (positively charged). Magnesium is found predominantly inside cells and in bone. 7.The nurse receives the patient’s most recent blood work results. Which laboratory value is of greatest concern? ANS: B d. From intravascular to interstitial a. 1, 3, 2, 7, 5, 4, 6 b. 1, 3, 2, 5, 7, 6, 4 c. 3, 2, 1, 5, 7, 6, 4 d. 3, 2, 4, 1, 5, 7, 6 a. Sodium b. Chloride c. Potassium d. Magnesium a. Sodium of 145 mEq/L b. Calcium of 15.5 mg/dL c. Potassium of 3.5 mEq/L d. Chloride of 100 mEq/L 199 A&E I Comprehensive Testbank Normal calcium range is 8.4 to 10.5 mg/dL; therefore, a value of 15.5 mg/dL is abnormally high and of concern. The rest of the laboratory values are within their normal ranges: sodium 136 to 145 mEq/L; potassium 3.5 to 5.0 mEq/L; and chloride 98 to 106 mEq/L. 8.The nurse observes that the patient’s calcium is elevated. When checking the phosphate level, what does the nurse expect to see? ANS: B Phosphate will decrease. Serum calcium and phosphate have an inverse relationship. When one is elevated, the other decreases, except in some patients with end-stage renal disease. 9.Four patients arrive at the emergency department at the same time. Which patient will the nurse see first? ANS: A The infant should be seen first. An infant’s proportion of total body water (70% to 80% total body weight) is greater than that of children or adults. Infants and young children have greater water needs and immature kidneys. They are at greater risk for extracellular volume deficit and hypernatremia because body water loss is proportionately greater per kilogram of weight. A teenager with excessive edema from a sprained ankle can wait. A middle-aged adult moaning in pain can wait as can an older adult with a blood pressure of 112/60. 10.The patient has an intravenous (IV) line and the nurse needs to remove the gown. In which order will the nurse perform the steps, starting with the first one? 1. Remove the sleeve of the gown from the arm without the IV. 2. Remove the sleeve of the gown from the arm with the IV. 3. Remove the IV solution container from its stand. 4. Pass the IV bag and tubing through the sleeve. ANS: A Change regular gowns by following these steps for maximum speed and arm mobility: (1) To remove a gown, remove the sleeve of the gown from the arm without the IV line, maintaining the patient’s privacy. (2) Remove the sleeve of the gown from the arm with the IV line. (3) Remove the IV solution container from its stand, and pass it and the tubing through the sleeve. (If this involves removing the tubing from an EID, use the roller clamp to slow the infusion to prevent the accidental infusion of a large volume of solution or medication.) 11.A 2-year-old child is brought into the emergency department after ingesting a medication that causes respiratory depression. For which acid-base imbalance will the nurse most closely monitor this child? a. Increased b. Decreased c. Equal to calcium d. No change in phosphate a. An infant with temperature of 102.2° F and diarrhea for 3 days b. A teenager with a sprained ankle and excessive edema c. A middle-aged adult with abdominal pain who is moaning and holding her stomach d. An older adult with nausea and vomiting for 3 days with blood pressure 112/60 a. 1, 2, 3, 4 b. 2, 3, 4, 1 c. 3, 4, 1, 2 d. 4, 1, 2, 3 200 A&E I Comprehensive Testbank ANS: B Respiratory depression leads to hypoventilation. Hypoventilation results in retention of CO2 and respiratory acidosis. Respiratory alkalosis would result from hyperventilation, causing a decrease in CO2 levels. Metabolic acid-base imbalance would be a result of kidney dysfunction, vomiting, diarrhea, or other conditions that affect metabolic acids. 12.A patient is admitted for a bowel obstruction and has had a nasogastric tube set to low intermittent suction for the past 3 days. Which arterial blood gas values will the nurse expect to observe? ANS: B The patient is losing acid from the nasogastric tube so the patient will have metabolic alkalosis. Lung problems will produce respiratory alkalosis or acidosis. Metabolic acidosis will occur when too much acid is in the body like kidney failure. 13.Which blood gas result will the nurse expect to observe in a patient with respiratory alkalosis? ANS: B Respiratory alkalosis should show an alkalotic pH and decreased CO2 (respiratory) values, with a normal HCO3–. In this case, pH 7.53 is alkaline (normal = 7.35 to 7.45), PaCO2 is 30 (normal 35 to 45 mm Hg), and HCO3–is 24 (normal = 22 to 26 mEq/L). A result of pH 7.60, PaCO2 40 mm Hg, HCO3– 30 mEq/L is metabolic alkalosis. pH 7.35, PaCO2 35 mm Hg, HCO3– 26 mEq/L is within normal limits. pH 7.25, PaCO2 48 mm Hg, HCO3– 23 mEq/L is respiratory acidosis. 14.A nurse is caring for a patient whose ECG presents with changes characteristic of hypokalemia. Which assessment finding will the nurse expect? ANS: B Signs and symptoms of hypokalemia are muscle weakness, abdominal distention, decreased bowel sounds, and cardiac dysrhythmias. Distended neck veins occur in fluid overload. Thready peripheral pulses indicate hypovolemia. Dry mucous membranes and flushed skin are indicative of dehydration and hypernatremia. a. Respiratory alkalosis b. Respiratory acidosis c. Metabolic acidosis d. Metabolic alkalosis a. Respiratory alkalosis b. Metabolic alkalosis c. Metabolic acidosis d. Respiratory acidosis a. pH 7.60, PaCO2 40 mm Hg, HCO3– 30 mEq/L b. pH 7.53, PaCO2 30 mm Hg, HCO3– 24 mEq/L c. pH 7.35, PaCO2 35 mm Hg, HCO3– 26 mEq/L d. pH 7.25, PaCO2 48 mm Hg, HCO3– 23 mEq/L a. Dry mucous membranes b. Abdominal distention c. Distended neck veins d. Flushed skin 201 A&E I Comprehensive Testbank 15.In which patient will the nurse expect to see a positive Chvostek sign? ANS: B A positive Chvostek sign is representative of hypocalcemia or hypomagnesemia. Hypomagnesemia is common with alcohol abuse. Hypocalcemia can be brought on by alcohol abuse and pancreatitis (which also can be affected by alcohol consumption). Burn patients frequently experience extracellular fluid volume deficit. Hyperparathyroidism causes hypercalcemia. Immobility is associated with hypercalcemia. 16.A patient is experiencing respiratory acidosis. Which organ system is responsible for compensation in this patient? ANS: A The kidneys (renal) are responsible for respiratory acidosis compensation. A problem with the respiratory system causes respiratory acidosis, so another organ system (renal) needs to compensate. Problems with the gastrointestinal and endocrine systems can cause acid-base imbalances, but these systems cannot compensate for an existing imbalance. 17.A nurse is caring for a patient with peripheral intravenous (IV) therapy. Which task will the nurse assign to the nursing assistive personnel? ANS: A A nursing assistive personnel (NAP) can record intake and output. An RN cannot delegate regulating flow rate, starting an IV, or changing an IV dressing to an NAP. 18.The nurse is caring for a diabetic patient in renal failure who is in metabolic acidosis. Which laboratory findings are consistent with metabolic acidosis? ANS: A The laboratory values that reflect metabolic acidosis are pH 7.3, PaCO2 36 mm Hg, HCO3– 19 mEq/L. A laboratory finding of pH 7.5, PaCO2 35 mm Hg, HCO3– 35 mEq/L is metabolic alkalosis. pH 7.32, PaCO2 47 a. A 7-year-old child admitted for severe burns b. A 24-year-old adult admitted for chronic alcohol abuse c. A 50-year-old patient admitted for an acute exacerbation of hyperparathyroidism d. A 75-year-old patient admitted for a broken hip related to osteoporosis a. Renal b. Endocrine c. Respiratory d. Gastrointestinal a. Recording intake and output b. Regulating intravenous flow rate c. Starting peripheral intravenous therapy d. Changing a peripheral intravenous dressing a. pH 7.3, PaCO2 36 mm Hg, HCO3– 19 mEq/L b. pH 7.5, PaCO2 35 mm Hg, HCO3– 35 mEq/L c. pH 7.32, PaCO2 47 mm Hg, HCO3– 23 mEq/L d. pH 7.35, PaCO2 40 mm Hg, HCO3– 25 mEq/L 202 A&E I Comprehensive Testbank mm Hg, HCO3– 23 mEq/L is respiratory acidosis. pH 7.35, PaCO2 40 mm Hg, HCO3– 25 mEq/L values are within normal range. 19.The nurse is assessing a patient and finds crackles in the lung bases and neck vein distention. Which action will the nurse take first? ANS: C The patient is in fluid overload. Raising the head of the bed to ease breathing is the first action. Offering calcium-rich foods is for hypocalcemia, not fluid overload. Administering a diuretic is the second action. Increasing fluids is contraindicated and would make the situation worse. 20.A chemotherapy patient has gained 5 pounds in 2 days. Which assessment question by the nurse is mostappropriate? ANS: D A rapid gain in weight usually indicates extracellular volume (ECV) excess if the person began with normal ECV. Asking the patient about urination habits will help determine whether the body is trying to excrete the excess fluid or if renal dysfunction is contributing to ECV excess. This is too rapid a weight gain to be dietary; it is fluid retention. Asking about following a weight loss program will not help determine the cause of the problem. Caloric intake does not account for rapid weight changes. Dry mouth and thirst accompany ECV deficit, which would be associated with rapid weight loss. 21.The health care provider has ordered a hypotonic intravenous (IV) solution to be administered. Which IV bag will the nurse prepare? ANS: A 0.45% sodium chloride is a hypotonic solution. NS and LR are isotonic. D5LR is hypertonic. 22.The health care provider asks the nurse to monitor the fluid volume status of a heart failure patient and a patient at risk for clinical dehydration. Which is the mosteffective nursing intervention for monitoring both of these patients? ANS: C a. Offer calcium-rich foods. b. Administer diuretic. c. Raise head of bed. d. Increase fluids. a. “Are you following any weight loss program?” b. “How many calories a day do you consume?” c. “Do you have dry mouth or feel thirsty?” d. “How many times a day do you urinate?” a. 0.45% sodium chloride (1/2 NS) b. 0.9% sodium chloride (NS) c. Lactated Ringer’s (LR) d. Dextrose 5% in Lactated Ringer’s (D5LR) a. Assess the patients for edema in extremities. b. Ask the patients to record their intake and output. c. Weigh the patients every morning before breakfast. d. Measure the patients’ blood pressures every 4 hours. 203 A&E I Comprehensive Testbank An effective measure of fluid retention or loss is daily weights; each kilogram (2.2 pounds) change is equivalent to 1 liter of fluid gained or lost. This measurement should be performed at the same time every day using the same scale and the same amount of clothing. Although intake and output records are important assessment measures, some patients are not able to keep their own records themselves. Blood pressure can decrease with extracellular volume (ECV) deficit but will not necessarily increase with recent ECV excess (heart failure patient). Edema occurs with ECV excess but not with clinical dehydration. 23.A nurse is caring for a cancer patient who presents with anorexia, blood pressure 100/60, and elevated white blood cell count. Which primary purpose for starting total parenteral nutrition (TPN) will the nurse add to the care plan? ANS: C Total parenteral nutrition is an intravenous solution composed of nutrients and electrolytes to replace the ones the patient is not eating or losing. TPN does not stimulate the appetite. TPN does not contain blood pressure medication or antibiotics. 24.A patient presents to the emergency department with reports of vomiting and diarrhea for the past 48 hours. The health care provider orders isotonic intravenous (IV) therapy. Which IV will the nurse prepare? ANS: C Patients with prolonged vomiting and diarrhea become hypovolemic. A solution to replace extracellular volume is 0.9% sodium chloride, which is an isotonic solution. 0.225% and 0.45% sodium chloride are hypotonic. 3% sodium chloride is hypertonic. 25.A nurse is administering a diuretic to a patient and teaching the patient about foods to increase. Which food choices by the patient will best indicate successful teaching? ANS: B Potatoes and fruits are high in potassium. Milk and cheese are high in calcium. Canned soups and vegetables are high in sodium. Whole grains and dark green leafy vegetables are high in magnesium. 26.The nurse is evaluating the effectiveness of the intravenous fluid therapy in a patient with hypernatremia. Which finding indicates goal achievement? a. Stimulate the patient’s appetite to eat. b. Deliver antibiotics to fight off infection. c. Replace fluid, electrolytes, and nutrients. d. Provide medication to raise blood pressure. a. 0.225% sodium chloride (1/4 NS) b. 0.45% sodium chloride (1/2 NS) c. 0.9% sodium chloride (NS) d. 3% sodium chloride (3% NaCl) a. Milk and cheese b. Potatoes and fresh fruit c. Canned soups and vegetables d. Whole grains and dark green leafy vegetables a. Urine output increases to 150 mL/hr. b. Systolic and diastolic blood pressure decreases. c. Serum sodium concentration returns to normal. 204 A&E I Comprehensive Testbank ANS: C Hypernatremia is diagnosed by elevated serum sodium concentration. Blood pressure is not an accurate indicator of hypernatremia. Emesis and diarrhea will not stop because of intravenous therapy. Urine output is influenced by many factors, including extracellular fluid volume. A large dilute urine output can cause further hypernatremia. 27.The nurse is calculating intake and output on a patient. The patient drinks 150 mL of orange juice at breakfast, voids 125 mL after breakfast, vomits 250 mL of greenish fluid, sucks on 60 mL of ice chips, and for lunch consumes 75 mL of chicken broth. Which totals for intake and output will the nurse document in the patient’s medical record? ANS: A Intake = 150 mL of orange juice, 60 mL of ice chips (but only counts as 30 since ice chips are half of the amount), and 75 mL of chicken broth; 150 + 30 + 75 = 255. Output = 125 mL of urine (void) and 250 mL of vomitus; 125 + 250 = 375. 28.A nurse is assessing a patient. Which assessment finding should cause a nurse to further assess for extracellular fluid volume deficit? ANS: B Physical examination findings of deficit include postural hypotension, tachycardia, thready pulse, dry mucous membranes, and poor skin turgor. Pitting edema indicates that the patient may be retaining excess extracellular fluid. 29.A patient is to receive 1000 mL of 0.9% sodium chloride intravenously at a rate of 125 mL/hr. The nurse is using microdrip gravity drip tubing. Which rate will the nurse calculate for the minute flow rate (drops/min)? ANS: C Microdrip tubing delivers 60 drops/mL. Calculation for a rate of 125 mL/hr using microdrip tubing: (125 mL/1 hr)(60 drops/1 mL)(1 hr/60 min) = 125 drop/min. 30.A nurse begins infusing a 250-mL bag of IV fluid at 1845 on Monday and programs the pump to infuse at 50 mL/hr. At what time should the infusion be completed? d. Large amounts of emesis and diarrhea decrease. a. Intake 255; output 375 b. Intake 285; output 375 c. Intake 505; output 125 d. Intake 535; output 125 a. Moist mucous membranes b. Postural hypotension c. Supple skin turgor d. Pitting edema a. 12 drops/min b. 24 drops/min c. 125 drops/min d. 150 drops/min a. 2300 Monday 205 A&E I Comprehensive Testbank ANS: B 250 mL ÷ 50 mL/hr = 5 hr 1845 + 5 hr = 2345, which would be 2345 on Monday. 31.A nurse is caring for a diabetic patient with a bowel obstruction and has orders to ensure that the volume of intake matches the output. In the past 4 hours, the patient received dextrose 5% with 0.9% sodium chloride through a 22-gauge catheter infusing at 150 mL/hr and has eaten 200 mL of ice chips. The patient also has an NG suction tube set to low continuous suction that had 300-mL output. The patient has voided 400 mL of urine. After reporting these values to the health care provider, which order does the nurse anticipate? ANS: A The total fluid intake and output equals 700 mL, which meets the provider goals. Patients with nasogastric suctioning are at risk for potassium deficit, so the nurse would anticipate a potassium supplement to correct this condition. Remember to record half the volume of ice chips when calculating intake. The other measures would be unnecessary because the net fluid volume is equal. 32.A nurse is caring for a patient who is receiving peripheral intravenous (IV) therapy. When the nurse is flushing the patient’s peripheral IV, the patient reports pain. Upon assessment, the nurse notices a red streak that is warm to the touch. What is the nurse’s initial action? ANS: D The IV site has phlebitis. The nurse should discontinue the IV. The phlebitis score is 3. The site has phlebitis, not infiltration. A moist compress may be needed after the IV is discontinued. 33.A nurse is assisting the health care provider in inserting a central line. Which action indicates the nurse is following the recommended bundle protocol to reduce central line-associated bloodstream infections (CLABSI)? ANS: D A recommended bundle at insertion of a central line is hand hygiene prior to catheter insertion; use of maximum sterile barrier precautions upon insertion; chlorhexidine skin antisepsis prior to insertion and during b. 2345 Monday c. 0015 Tuesday d. 0045 Tuesday a. Add a potassium supplement to replace loss from output. b. Decrease the rate of intravenous fluids to 100 mL/hr. c. Administer a diuretic to prevent fluid volume excess. d. Discontinue the nasogastric suctioning. a. Record a phlebitis grade of 4. b. Assign an infiltration grade. c. Apply moist compress. d. Discontinue the IV. a. Preps skin with povidone-iodine solution. b. Suggests the femoral vein for insertion site. c. Applies double gloving without hand hygiene. d. Uses chlorhexidine skin antisepsis prior to insertion. 206 A&E I Comprehensive Testbank dressing changes; avoidance of the femoral vein for central venous access for adults; and daily evaluation of line necessity, with prompt removal of non-essential lines. Povidone-iodine is not recommended. 34.The nurse is caring for a group of patients. Which patient will the nurse see first? ANS: A The nurse will see the patient with D5W and blood to prevent a medication error. When preparing to administer blood, prime the tubing with 0.9% sodium chloride (normal saline) to prevent hemolysis or breakdown of RBCs. All the rest are normal. A patient with type A blood can receive type O. Type O is considered the universal donor. A patient with a mastectomy should have the IV in the other arm. Potassium chloride should be diluted, and it is never given IV push. 35.A nurse is administering a blood transfusion. Which assessment finding will the nurse report immediately? ANS: B A fever should be reported immediately and the blood transfusion stopped. All other assessment findings are expected. Blood is given to elevate blood pressure, improve pallor, and decrease tachycardia. 36.A nurse has just received a bag of packed red blood cells (RBCs) for a patient. What is the longest time the nurse can let the blood infuse? ANS: C Ideally a unit of whole blood or packed RBCs is transfused in 2 hours. This time can be lengthened to 4 hours if the patient is at risk for extracellular volume excess. Beyond 4 hours there is a risk for bacterial contamination of the blood. 37.A patient has an acute intravascular hemolytic reaction to a blood transfusion. After discontinuing the blood transfusion, which is the nurse’s next action? ANS: D The nurse should first attach new tubing and begin running in normal saline at a rate to keep the vein open, in case any medications need to be delivered through an IV site. The existing tubing should not be used because a. A patient with D5W hanging with the blood b. A patient with type A blood receiving type O blood c. A patient with intravenous potassium chloride that is diluted d. A patient with a right mastectomy and an intravenous site in the left arm a. Blood pressure 110/60 b. Temperature 101.3° F c. Poor skin turgor and pallor d. Heart rate of 100 beats/min a. 30 minutes b. 2 hours c. 4 hours d. 6 hours a. Discontinue the IV catheter. b. Return the blood to the blood bank. c. Run normal saline through the existing tubing. d. Start normal saline at TKO rate using new tubing. 207 A&E I Comprehensive Testbank that would infuse the blood in the tubing into the patient. It is necessary to preserve the IV catheter in place for IV access to treat the patient. After the patient has been assessed and stabilized, the blood can be returned to the blood bank. 38.A nurse is assessing a patient who is receiving a blood transfusion and finds that the patient is anxiously fidgeting in bed. The patient is afebrile and dyspneic. The nurse auscultates crackles in both lung bases and sees jugular vein distention. On which transfusion complication will the nurse focus interventions? ANS: A The signs and symptoms are concurrent with fluid volume overload. Anaphylactic shock would have presented with urticaria, dyspnea, and hypotension. Septicemia would include a fever. A hemolytic reaction would consist of flank pain, chills, and fever. 39.A nurse is preparing to start a blood transfusion. Which type of tubing will the nurse obtain? ANS: C When administering a transfusion you need an appropriate-size IV catheter and blood administration tubing that has a special in-line filter. The patient’s blood should not be mixed with the infusion blood. Air bubbles should not be allowed to enter the blood. The only substance compatible with blood is normal saline; no additives should be mixed with the infusing blood. 40.The nurse is caring for a patient with hyperkalemia. Which body system assessment is the priority? ANS: D Cardiac is the priority. Hyperkalemia places the patient at risk for potentially serious dysrhythmias and cardiac arrest. Potassium balance is necessary for cardiac function. Respiratory is the priority with hypokalemia. Monitoring of gastrointestinal and neurological systems would be indicated for other electrolyte imbalances. 41.Which assessment finding will the nurse expect for a patient with the following laboratory values: sodium 145 mEq/L, potassium 4.5 mEq/L, calcium 4.5 mg/dL? a. Fluid volume overload b. Hemolytic reaction c. Anaphylactic shock d. Septicemia a. Two-way valves to allow the patient’s blood to mix and warm the blood transfusing b. An injection port to mix additional electrolytes into the blood c. A filter to ensure that clots do not enter the patient d. An air vent to let bubbles into the blood a. Gastrointestinal b. Neurological c. Respiratory d. Cardiac a. Weak quadriceps muscles b. Decreased deep tendon reflexes c. Light-headedness when standing up d. Tingling of extremities with possible tetany 208 A&E I Comprehensive Testbank ANS: D This patient has hypocalcemia because the normal calcium range is 8.4 to 10.5 mg/dL. Hypocalcemia causes muscle tetany, positive Chvostek’s sign, and tingling of the extremities. Sodium and potassium values are within their normal ranges: sodium 135 to 145 mEq/L; potassium 3.5 to 5.0 mEq/L. Light-headedness when standing up is a manifestation of ECV deficit or sometimes hypokalemia. Weak quadriceps muscles are associated with potassium imbalances. Decreased deep tendon reflexes are related to hypercalcemia or hypermagnesemia. 42.While the nurse is taking a patient history, the nurse discovers the patient has a type of diabetes that results from a head injury and does not require insulin. Which dietary change should the nurse share with the patient? ANS: C The patient has diabetes insipidus, which places the patient at risk for dehydration and hypernatremia. Dehydration should be prevented by drinking plenty of fluids to replace the extra water excreted in the urine. Foods high in acid are not what causes metabolic acidosis. A reduction in carbohydrates to lower blood sugar will not help a patient with diabetes insipidus but it may help a patient with diabetes mellitus. Calcium-rich dairy products would be recommended for a hypocalcemic patient. MULTIPLE RESPONSE 1.A nurse is selecting a site to insert an intravenous (IV) catheter on an adult. Which actions will the nurse take? (Select all that apply.) ANS: A, C, F The vein should be relatively straight to avoid catheter occlusion. Contraindications to starting an IV catheter are conditions such as mastectomy, AV fistula, and central line in the extremity and should be checked before initiation of IV. Avoid areas of flexion if possible. The nurse should start distally and move proximally, choosing the nondominant arm if possible. The nurse should feel for the best location; a good vein should feel spongy, a rigid vein should be avoided because it might have had previous trauma or damage. 2.Which assessments will alert the nurse that a patient’s IV has infiltrated? (Select all that apply.) ANS: A, C, F a. Reduce the quantity of carbohydrates ingested to lower blood sugar. b. Include a serving of dairy in each meal to elevate calcium levels. c. Drink plenty of fluids throughout the day to stay hydrated. d. Avoid food high in acid to avoid metabolic acidosis. a. Check for contraindications to the extremity. b. Start proximally and move distally on the arm. c. Choose a vein with minimal curvature. d. Choose the patient’s dominant arm. e. Select a vein that is rigid. f. Avoid areas of flexion. a. Edema of the extremity near the insertion site b. Reddish streak proximal to the insertion site c. Skin discolored or pale in appearance d. Pain and warmth at the insertion site e. Palpable venous cord f. Skin cool to the touch 209 A&E I Comprehensive Testbank Infiltration results in skin that is edematous near the IV insertion site. Skin is cool to the touch and may be pale or discolored. Pain, warmth, erythema, a reddish streak, and a palpable venous cord are all symptoms of phlebitis. 3.A nurse is discontinuing a patient’s peripheral IV access. Which actions should the nurse take? (Select all that apply.) ANS: B, E, F The nurse should stop the infusion before removing the IV catheter, so the fluid does not drip on the patient’s skin; keep the catheter parallel to the skin while removing it to reduce trauma to the vein; and apply pressure to the site for 2 to 3 minutes after removal to decrease bleeding from the site. Scissors should not be used because they may accidentally cut the catheter or tubing or may injure the patient. During removal of the IV catheter, light pressure, not firm pressure, is indicated to prevent trauma. Clean gloves are used for discontinuing a peripheral IV access because gloved hands will handle the external dressing, tubing, and tape, which are not sterile. COMPLETION 1.A patient has 250 mL of a jejunostomy feeding with 30 mL of water before and after feeding and 200 mL of urine. Thirty minutes later the patient has 100 mL of diarrhea. At 1300 the patient receives 150 mL of blood and voids another 200 mL. Calculate the patient’s intake. Record your answer as a whole number. _____ mL ANS: 460 The patient’s fluid intake is 250 mL of feeding, 60 mL of water (30 mL before and after), and 150 blood: . Fluid intake includes all liquids that a person eats (e.g., gelatin, ice cream, soup), drinks (e.g., water, coffee, juice), or receives through nasogastric or jejunostomy feeding tubes. IV fluids (continuous infusions and intermittent IV piggybacks) and blood components also are sources of intake. Fluid output includes urine, diarrhea, vomitus, gastric suction, and drainage from postsurgical wounds or other tubes. MATCHING A nurse is monitoring patients for fluid and electrolyte and acid-base imbalances. Match the body’s regulators to the function it provides. 1.Antidiuretic hormone 2.Angiotensin II 3.Aldosterone 4.Atrial natriuretic peptide 5.Bicarbonate 1.ANS:C2.ANS:E3.ANS:B4.ANS:A5.ANS:D a. Wear sterile gloves and a mask. b. Stop the infusion before removing the IV catheter. c. Use scissors to remove the IV site dressing and tape. d. Apply firm pressure with sterile gauze during removal. e. Keep the catheter parallel to the skin while removing it. f. Apply pressure to the site for 2 to 3 minutes after removal. a. Increases excretion of sodium and water b. Reduces excretion of sodium and water c. Reduces excretion of water d. Major buffer in the extracellular fluid e. Vasoconstricts and stimulates aldosterone release 210 A&E I Comprehensive Testbank Week 6 Pain and Sleep Chapter 43: Sleep Potter et al.: Fundamentals of Nursing, 9th Edition MULTIPLE CHOICE 1.The nurse is caring for a young-adult patient on the medical-surgical unit. When doing midnight checks, the nurse observes the patient awake, putting a puzzle together. Which information will the nurse consider to best explain this finding? ANS: D This patient is awake and alert enough to do a puzzle. The individual’s sleep-wake preference is probably late evening. All persons have biological clocks that synchronize their sleep-wake cycle. This explains why some individuals fall asleep in the early evening, whereas others go to bed at midnight or early morning. Waiting to talk with the nurse, being lonely, and noise on the unit may contribute to lack of sleep, but the best explanation for the patient being awake is the biological clock. 2.The nurse is providing an educational session on sleep regulation for new nurses in the Sleep Disorder Treatment Center. Which statement by the nurses will bestindicate that the teaching is effective? ANS: A Sleep involves a sequence of physiological states maintained by the central nervous system. It is associated with changes in the peripheral nervous, endocrine, cardiovascular, respiratory, and muscular systems. A disease process associated with the cranial nerves, urinary pathway, or spinal reflexes may influence a person’s ability to sleep, but the best answer is the central nervous system. 3.The nurse is caring for a patient who is having trouble sleeping. Which action will the nurse take? a. The patient misses family and is lonely. b. The patient was waiting to talk with the nurse. c. The patient has been kept up with the noise on the unit. d. The patient’s sleep-wake cycle preference is late evening. a. “If the patient has a disease process in the central nervous system, it can influence the functions of sleep.” b. “If the patient has a disease process in the cranial nerves, it can influence the functions of sleep.” c. “If the patient has an interruption in the urinary pathways, it can influence the functions of sleep.” d. “If the patient has an interruption in the spinal reflexes, it can influence the functions of sleep.” a. Suggest snug-fitting nightwear. b. Provide a favorite beverage. 211 A&E I Comprehensive Testbank ANS: C Relaxation exercises such as slow, deep breathing for 1 or 2 minutes relieve tension and prepare the body for rest. Instruct patients to wear loose-fitting nightwear. Walking and drinking a favorite beverage would not necessarily encourage sleep. 4.The nurse is caring for a patient in the sleep lab. Which assessment finding indicates to the nurse that the patient is in stage 4 NREM? ANS: C The patient is difficult to arouse, vital signs are significantly lower, and this stage lasts about 15 to 30 minutes. Stage 4 NREM is the deepest stage of sleep. Lighter sleep is seen in stages 1 and 2, where the patient awakens easily. REM sleep is characterized by rapid eye movement. 5.A nurse is teaching the staff about the sleep cycle. Which sequence will the nurse include in the teaching session? ANS: D The cyclical pattern usually progresses from stage 1 through stage 4 of NREM, followed by a reversal from stages 4 to 3 to 2, ending with a period of REM sleep. The others are incorrect sequences. 6.Which nursing observation of the patient in intensive care indicates the patient is sleeping comfortably during NREM sleep? ANS: A During NREM sleep, biological functions slow. During sleep, the heart rate decreases to 60 beats/min or less. The patient experiences decreased respirations, blood pressure, and muscle tone. Heart rates above 60 are too high and respirations of 22 are too high to indicate comfortable NREM sleep. 7.The nurse is teaching a new mother about the sleep requirements of a neonate. Which comment by the patient indicates a correct understanding of the teaching? c. Encourage deep breathing. d. Walk with the patient. a. The patient awakens easily. b. The patient’s eyes rapidly move. c. The patient is difficult to awaken. d. The patient’s vital signs are elevated. a. NREM Stage 1, 2, 3, 4, REM b. NREM Stage 1, 2, 3, 4, 3, 2, 1, REM c. NREM Stage 1, 2, 3, 4, REM, 4, 3, 2 REM d. NREM Stage 1, 2, 3, 4, 3, 2, REM a. Eyes closed, lying quietly, respirations 12, heart rate 60 b. Eyes closed, tossing in bed, respirations 18, heart rate 80 c. Eyes closed, mumbling to self, respirations 16, heart rate 68 d. Eyes closed, lying supine in bed, respirations 22, heart rate 66 a. “I can’t wait to get the baby home to play with the brothers and sisters.” 212 A&E I Comprehensive Testbank ANS: B The patient indicates an understanding when asking the mother to come after the first week. The neonate up to the age of 3 months averages about 16 hours of sleep a day, sleeping almost constantly during the first week. The baby will sleep rather than play. The baby will not be on a sleeping schedule the first week home. The mother will be able to nap since the baby sleeps 16 hours a day. 8.The nurse is discussing lack of sleep with a middle-aged adult. Which area should the nurse most likely assess to determine a possible cause of the lack of sleep? ANS: A During middle adulthood, the total time spent sleeping at night begins to decline. Anxiety, depression, and certain physical illnesses can affect sleep, and women can experience menopausal symptoms. Insomnia is common because of the changes and stresses associated with middle age. Teenagers, caring for pets, and late night television can influence the amount of sleep; however, these are not the most common causes of insomnia in this age group. 9.A single parent is discussing the sleep needs of a preschooler with the nurse. Which information will the nurse share with the parent? ANS: D The preschooler usually has difficulty relaxing or settling down after long, active days. By the age of 5, naps are rare for children, except those for whom a siesta is a custom. Preschoolers frequently awaken during the night. On average, a preschooler needs about 12 hours of sleep. 10.The nurse is having a conversation with an adolescent regarding the need for sleep. The adolescent states that it is common to stay up with friends several nights a week. Which action should the nurse take next? ANS: B b. “I will ask my mom to come after the first week, when the baby is more alert.” c. “I can get the baby on a sleeping schedule the first week while my mom is here.” d. “I won’t be able to nap during the day because the baby will be awake.” a. Anxiety b. Loud teenagers c. Caring for pets d. Late night television a. “Most preschoolers sleep soundly all night long.” b. “It is important that the 5-year-old get a nap every day.” c. “On average, the preschooler needs to sleep 10 hours a night.” d. “Preschoolers may have trouble settling down after a busy day.” a. Talk with the adolescent’s parent about staying up with friends and the need for sleep. b. Discuss with the adolescent sleep needs and the effects of excessive daytime sleepiness. c. Refer the adolescent for counseling about alcohol abuse problems. d. Take no action for this normal occurrence. 213 A&E I Comprehensive Testbank Discussion regarding adolescent sleep needs should first occur with the adolescent. Although it may be common for this adolescent to want to visit with friends and experience activities that go late into the night, these activities can and do impact the hours of sleep and the physical needs of the adolescent, no matter the reason for the late nights, and they do need to be addressed. The nurse will address the adolescent, not the parents. Addressing alcohol abuse problems is not the next step but may be required later. While staying up late may be a normal occurrence for this adolescent, action is required. 11.The nurse is completing an assessment on an older-adult patient who is having difficulty falling asleep. Which condition will the nurse further assess for in this patient? ANS: A Older adults and other individuals who experience depressive mood problems experience delays in falling asleep, earlier appearance of REM sleep, frequent awakening, feelings of sleeping poorly, and daytime sleepiness. A person who is moderately fatigued usually achieves restful sleep, especially if the fatigue is the result of enjoyable work or exercise. Hypertension often causes early-morning awakening and fatigue. Alcohol speeds the onset of sleep. Hypothyroidism decreases stage 4 sleep. 12.The nurse is caring for an adolescent with an appendectomy who is reporting difficulty falling asleep. Which intervention will be most appropriate? ANS: D Discontinuing the soda and chocolate nightly snack will be most beneficial for this patient since it has two factors that will cause difficulty falling asleep. Coffee, tea, colas, and chocolate act as stimulants, causing a person to stay awake or to awaken throughout the night. Personal preference influences the temperature of the room, as well as the lighting of the room. Noise can be a factor in the unit and can awaken the patient, but caffeine can make it difficult to fall asleep. 13.A patient has obstructive sleep apnea. Which assessment is the priority? ANS: C In obstructive sleep apnea, the upper airway becomes partially or completely blocked, diminishing nasal airflow or stopping it. The person still attempts to breathe because the chest and abdominal movement continue, which results in loud snoring and snorting sounds. According to the ABCs of prioritizing care, airway and respiratory status takes priority over gastrointestinal, circulatory, and neurologic functioning. 14.The patient has just been diagnosed with narcolepsy. The nurse teaches the patient about management of the condition. Which information from the patient will cause the nurse to intervene? a. Depression b. Mild fatigue c. Hypertension d. Hypothyroidism a. Close the door to decrease noise from unit activities. b. Adjust temperature in the patient’s room to 21° C (70° F). c. Ensure that the night-light in the patient’s room is working. d. Encourage the discontinuation of a soda and chocolate nightly snack. a. Gastrointestinal function b. Neurological function c. Respiratory status d. Circulatory status a. Takes antidepressant medications 214 A&E I Comprehensive Testbank ANS: C The nurse will intervene about sitting in a hot, stuffy room as this will make the narcolepsy worse so this needs to be corrected. Patients with narcolepsy need to avoid factors that increase drowsiness (e.g., alcohol, heavy meals, exhausting activities, long-distance driving, and long periods of sitting in hot, stuffy rooms). Patients are treated with antidepressants, and management techniques involve scheduling naps no longer than 20 minutes and chewing gum. Additional management techniques include exercise, light high-protein meals, deep breathing, and taking vitamins. 15.The nurse is caring for a patient who has been in holding in the emergency department for 24 hours. The nurse is concerned about the patient’s experiencing sleep deprivation. Which action will be best for the nurse to take? ANS: A The most effective treatment for sleep deprivation is elimination or correction of factors that disrupt the sleep pattern. Obtaining a private room in the medical-surgical unit for the patient will help with decreasing stimuli and promoting more rest than an individual can obtain in an emergency department even with the interventions mentioned. 16.The nurse is completing a sleep assessment on a patient. Which tool will the nurse use to complete the assessment? ANS: A The visual analog scale is utilized for assessing sleep quality. Cataplexy, or sudden muscle weakness during intense emotions such as anger, sadness, or laughter, occurs at any time during the day; there is no cataplexy scale for sleep assessment. A polysomnogram involves the use of EEG, EMG, and EOG to monitor stages of sleep and wakefulness during nighttime sleep; this is used in a sleep laboratory study. Researchers believe that the ascending reticular activating system (RAS) located in the upper brainstem contains special cells that maintain alertness and wakefulness; however, there is no assessment tool called the RAS scale. 17.The nurse is beginning a sleep assessment on a patient. Which question will be most appropriate for the nurse to ask initially? b. Naps shorter than 20 minutes c. Sits in hot, stuffy rooms d. Chews gum a. Expedite the process of obtaining a medical-surgical room for the patient. b. Pull the curtains shut, dim the lights, and decrease the number of visitors. c. Obtain an order for a hypnotic medication to help the patient sleep. d. Ask everyone in the unit to try to be quiet so the patient can sleep. a. Visual analog scale b. Cataplexy scale c. Polysomnogram d. RAS scale a. “What is going on?” b. “How are you sleeping?” c. “Are you taking any medications?” d. “What did you have for dinner last night?” 215 A&E I Comprehensive Testbank ANS: B Sleep is a subjective experience. Only the patient is able to report whether or not it is sufficient and restful. Asking patients how they are sleeping is an introductory question. After this beginning question is asked, problems with sleep such as the nature of the problem, signs and symptoms, onset and duration of the issue, severity, predisposing factors, and the effect on the patient can be assessed. What is going on is too broad and open ended for information about sleep to be obtained specifically. Medications and food intake can be part of the detailed assessment of sleep issues. 18.The nurse adds a nursing diagnosis of ineffective breathing pattern to a patient’s care plan. Which sleep condition caused the nurse to assign this nursing diagnosis? ANS: D Obstructive sleep apnea (OSA) occurs when the muscles or structures of the oral cavity or throat relax during sleep. The upper airway becomes partially or completely blocked, diminishing airflow or stopping it for as long as 30 seconds. The person still attempts to breathe because chest and abdominal movements continue, resulting in snoring or snorting sounds. With narcolepsy, the person feels an overwhelming wave of sleepiness and falls asleep. Insomnia is characterized by chronic difficulty falling asleep. Sleep deprivation is a condition caused by dyssomnia. OSA is the only one of these conditions that results in blockage of the airway and impacts the ability to breathe. 19.The nurse is caring for a postpartum patient. The patient’s labor has lasted over 28 hours within the hospital; the patient has not slept and is disoriented to date and time. Which nursing diagnosis will the nurse document in the patient’s care plan? ANS: D This patient has been deprived of sleep by staying awake during a 28-hour labor. Disorientation is one potential sign of sleep deprivation. In this scenario, there is a clear cause for the patient’s lack of sleep, and it is a one-time episode. Insomnia, on the other hand, is a chronic disorder whereby patients have difficulty falling asleep, awaken frequently, or sleep only for a short time. This scenario does not indicate that this has been a chronic problem for this patient. Although ineffective coping can manifest as a sleep disturbance, clear evidence shows that it was labor that deprived this patient of sleep, not an inability to cope. It could be difficult to care for an infant when sleep deprived; however, this scenario gives no evidence that this mother displays impaired parenting and is not caring adequately for her child or lacks the skills to do so. 20.The patient presents to the clinic with reports of irritability, being sleepy during the day, chronically not being able to fall asleep, and being tired. Which nursing diagnosis will the nurse document in the plan of care? ANS: C a. Insomnia b. Narcolepsy c. Sleep deprivation d. Obstructive sleep apnea a. Insomnia b. Impaired parenting c. Ineffective coping d. Sleep deprivation a. Anxiety b. Fatigue c. Insomnia d. Sleep deprivation 216 A&E I Comprehensive Testbank Insomnia is experienced when the patient has chronic difficulty falling asleep, frequent awakenings from sleep, and/or short sleep or nonrestorative sleep. It is the most common sleep-related complaint and includes symptoms such as irritability, excessive daytime sleepiness, not being able to fall asleep, and fatigue. Anxiety is a vague, uneasy feeling of discomfort or dread accompanied by an autonomic response. Fatigue is an overwhelming sustained sense of exhaustion with decreased capacity for physical and mental work at a usual level. Sleep deprivation is a condition caused by dyssomnia and includes symptoms caused by illness, emotional distress, or medications. 21.The nurse is preparing an older-adult patient’s evening medications. Which treatment will the nurse recognize as relatively safe for difficulty sleeping in older adults? ANS: A Ramelteon (Rozerem), a melatonin receptor agonist, is well tolerated and appears to be effective in improving sleep by improving the circadian rhythm and shortening time to sleep onset. It is safe for long- and short-term use particularly in older adults. The use of benzodiazepines in older adults is potentially dangerous because of the tendency of the drugs to remain active in the body for a longer time. As a result, they also cause respiratory depression, next-day sedation, amnesia, rebound insomnia, and impaired motor functioning and coordination, which leads to increased risk of falls. Caution older adults about using over-the-counter antihistamines because their long duration of action can cause confusion, constipation, and urinary retention. Kava promotes sleep in patients with anxiety; it should be used cautiously because of its potential toxic effects on the liver. 22.The nurse is caring for a patient on the medical-surgical unit who is experiencing an exacerbation of asthma. Which intervention will be most appropriate to help this patient sleep? ANS: A Placing the patient in a semi-Fowler’s position eases the work of breathing. Respiratory disease often interferes with sleep. Patients with chronic lung disease such as emphysema or asthma are short of breath and frequently cannot sleep without two or three pillows to raise their heads. Iron-rich food may help a patient with restless legs syndrome. Providing a snack and encouraging the patient to read may be good interventions for patients, but the most appropriate would be raising the head of the bed. 23.A young mother has been hospitalized for an irregular heartbeat (dysrhythmia). The night nurse makes rounds and finds the patient awake. Which action by the nurse is most appropriate? ANS: D A nurse on the night shift needs to take time to sit and talk with patients unable to sleep. This helps to determine the factors keeping patients awake. Assessment is the first step of the nursing process; therefore assessment needs to be done first and involves ascertaining the cause of the patient’s inability to sleep. Patients who are admitted to the hospital for uncertain diagnoses can be stressed and worried about the testing and a. Ramelteon (Rozerem) b. Benzodiazepine c. Antihistamine d. Kava a. Place bed in semi-Fowler’s position. b. Offer iron-rich foods for meals. c. Provide a snack before bedtime. d. Encourage the patient to read. a. Inform the patient that it is late and time to go to sleep. b. Ask the patient if she would like medication for sleep. c. Recommend a good movie that is on television tonight. d. Take time to sit and talk with the patient about her inability to sleep. 217 A&E I Comprehensive Testbank outcomes. In addition, a young mother can be worried about the care of her children and those caring for the children. This uncertainty and change in routine can cause difficulty in resting or falling asleep. A distraction such as a television may or may not work for the patient. After assessment is completed, a sedative may or may not be in order. Telling the patient that it is late and time to go to sleep is not a therapeutic response for an adult who is under stress. 24.The nurse is evaluating outcomes for the patient with insomnia. Which key principle will the nurse consider during this process? ANS: A With regard to problems with sleep, the patient is the source for evaluating outcomes. The patient is the only one who knows whether sleep problems have improved and what has been successful. Interventions are not the best indicator; achievement of goals according to the patient is the best. Observations do provide needed data, but in the case of insomnia, the patient is the source for evaluating the restfulness of sleep. 25.A patient has sleep deprivation. Which statement by the patient will indicate to the nurse that outcomes are being met? ANS: B Being able to sleep and feeling rested would indicate that outcomes are being met for sleep deprivation. Limiting a nap to 20 minutes is an intervention to promote sleep. Going to sleep within 30 minutes indicates a goal for insomnia. Waking up only once may indicate nocturia is improving but does not relate to sleep deprivation. 26.An older-adult patient is visiting the clinic after a fall during the night. The nurse obtains information on what medications the patient takes. Which medication mostlikely contributed to the patient’s fall? ANS: C The most likely cause is a benzodiazepine. If older patients who were recently continent, ambulatory, and alert become incontinent or confused and/or demonstrate impaired mobility, the use of benzodiazepines needs to be considered as a possible cause. This can contribute to a fall in an older adult. Short-term use of melatonin has been found to be safe, with mild side effects of nausea, headache, and dizziness being infrequent. Iron supplements may be given to patients with restless legs syndrome. Some substances such as L-tryptophan, a natural protein found in foods such as milk, cheese, and meats, promote sleep; while it does promote sleep, it is not the most likely to cause mobility problems. MULTIPLE RESPONSE a. The patient is the best evaluator of sleep. b. The nurse is the best evaluator of sleep. c. Effective interventions are the best evaluators of sleep. d. Observations of the patient are the best evaluators of sleep. a. “I wake up only once a night to go to the bathroom.” b. “I feel rested when I wake up in the morning.” c. “I go to sleep within 30 minutes of lying down.” d. “I only take a 20-minute nap during the day.” a. Melatonin b. L-tryptophan c. Benzodiazepine d. Iron supplement 218 A&E I Comprehensive Testbank 1.The nurse is caring for a patient who has not been able to sleep well while in the hospital, leading to a disrupted sleep-wake cycle. Which assessment findings will the nurse monitor for in this patient? (Select all that apply.) ANS: A, B, C, F The biological rhythm of sleep frequently becomes synchronized with other body functions. Changes in body temperature correlate with sleep pattern. When the sleep-wake cycle becomes disrupted, changes in physiological function such as temperature can occur. Patients can experience decreased appetite, loss of weight, anxiety, restlessness, irritability, and impaired judgment. Gastrointestinal and respiratory/ cardiovascular symptoms such as shortness of breath and chest pain are not symptoms of a disrupted sleep cycle. 2.The nurse is caring for a patient in the intensive care unit who is having trouble sleeping. The nurse explains the purpose of sleep and its benefits. Which information will the nurse include in the teaching session? (Select all that apply.) ANS: A, C, D Sleep contributes to physiological and psychological restoration. NREM sleep contributes to body tissue restoration. It allows the body to rest and conserve energy. This benefits the cardiac system by allowing the heart to beat fewer times each minute. During stage 4, the body releases growth hormone for renewal and repair of specialized cells such as the brain. During NREM sleep, biological functions slow. REM sleep is necessary for brain tissue restoration and cognitive restoration and is associated with a change in cerebral blood flow and increased cortical activity. 3.The patient and the nurse discuss the need for sleep. After the discussion, the patient is able to state factors that hinder sleep. Which statements indicate the patient has a good understanding of the teaching? (Select all that apply.) ANS: A, B, D, E Caffeine, alcohol, and nicotine consumed late in the evening produce insomnia. Worry over personal problems or situations frequently disrupts sleep. Alterations in routines, including changing mealtimes and staying up a. Changes in physiological function such as temperature b. Decreased appetite and weight loss c. Anxiety, irritability, and restlessness d. Shortness of breath and chest pain e. Nausea, vomiting, and diarrhea f. Impaired judgment a. NREM sleep contributes to body tissue restoration. b. During NREM sleep, biological functions increase. c. Restful sleep preserves cardiac function. d. Sleep contributes to cognitive restoration. e. REM sleep decreases cortical activity. a. “Drinking coffee at 7 PM could interrupt my sleep.” b. “Staying up late for a party can interrupt sleep patterns.” c. “Exercising 2 hours before bedtime can decrease relaxation.” d. “Changing the time of day that I eat dinner can disrupt sleep.” e. “Worrying about work can disrupt my sleep.” f. “Taking an antacid can decrease sleep.” 219 A&E I Comprehensive Testbank late at night for social activities, can disrupt sleep. Exercising 2 hours before bedtime actually increases a sense of fatigue and promotes relaxation. Taking an antacid does not decrease sleep. 4.A community health nurse is providing an educational session at the senior center on how to promote sleep. Which practices should the nurse recommend? (Select all that apply.) ANS: B, C, E, F The nurse should instruct the patient to sleep where he or she sleeps best, to use sedatives as a last resort, to decrease fluid intake to cut down on bathroom trips, and, if unable to sleep in 15 to 30 minutes, to get up out of bed. Naps should be eliminated if they are not part of the individual’s routine schedule, and if naps are taken, they should be limited to 20 minutes or less a day. Television can stimulate and disrupt sleep patterns. MATCHING The nurse is caring for a group of patients who have sleeping disruptions. Match the condition to the intervention the nurse will use. 1.Cataplexy 2.Narcolepsy 3.Insomnia 4.Hiatal hernia 5.Sleepwalking 6.Obstructive sleep apnea 1.ANS:C2.ANS:D3.ANS:F4.ANS:B5.ANS:E6.ANS:A Chapter 44: Pain Management Potter et al.: Fundamentals of Nursing, 9th Edition MULTIPLE CHOICE 1.An oriented patient has recently had surgery. Which action is best for the nurse to take to assess this patient’s pain? ANS: B a. Take a nap in the afternoon. b. Sleep where you sleep best. c. Use sedatives as a last resort. d. Watch television right before sleep. e. Decrease fluids 2 to 4 hours before sleep. f. Get up if unable to fall asleep in 15 to 30 minutes. a. Use continuous positive airway pressure. b. Offer a small meal several hours before bedtime. c. Administer antidepressants. d. Administer modafinil (Provigel). e. Do not startle. f. Administer benzodiazepine-like drugs. a. Assess the patient’s body language. b. Ask the patient to rate the level of pain. c. Observe the cardiac monitor for increased heart rate. d. Have the patient describe the effect of pain on the ability to cope. 220 A&E I Comprehensive Testbank One of the most subjective and therefore most useful characteristics for reporting pain is its severity. Therefore, the best way to assess a patient’s pain is to ask the patient to rate the pain. Nonverbal communication, such as body language, is not as effective in assessing pain, especially when the patient is oriented. Heart rate sometimes increases when a patient is in pain, but this is not a symptom that is specific to pain. Pain sometimes affects a patient’s ability to cope, but assessing the effect of pain on coping assesses the patient’s ability to cope; it does not assess the patient’s pain. 2.A nurse is caring for a patient who recently had abdominal surgery and is experiencing severe pain. The patient’s blood pressure is 110/60 mm Hg, and heart rate is 60 beats/min. Additionally, the patient does not appear to be in any distress. Which response by the nurse is most therapeutic? ANS: C Be sure the patient is a partner in making decisions about the best approaches for managing pain. A patient knows the most about his or her pain and is an important partner in selecting successful pain therapies. The nurse must believe that a patient is in pain whenever the patient reports that he or she is in pain, even if the patient does not appear to be in pain. The nurse must be careful to not judge the patient based on vital signs or nonverbal communication and must not assume that the patient is seeking narcotics. The patient is a partner in pain management, so going to get narcotics to treat the pain without consulting with the patient first is not appropriate. 3.A nurse teaches the patient about the gate control theory. Which statement made by a patient reflects a correct understanding about the relationship between the gate control theory of pain and the use of meditation to relieve pain? ANS: A According to this theory, gating mechanisms located along the central nervous system regulate or block pain impulses. Pain impulses pass through when a gate is open and are blocked when a gate is closed. Nonpharmacologic pain-relief measures, such as meditation, work by closing the gates, which keeps pain impulses from coming through. Meditation does not open pain gates or stop pain from occurring. Meditation also does not have an effect on pain neuroregulators. 4.A nurse is planning care for an older-adult patient who is experiencing pain. Which statement made by the nurse indicates the supervising nurse needs to follow up? a. “Your vitals do not show that you are having pain; can you describe your pain?” b. “OK, I will go get you some narcotic pain relievers immediately.” c. “What would you like to try to alleviate your pain?” d. “You do not look like you are in pain.” a. “Meditation controls pain by blocking pain impulses from coming through the gate.” b. “Meditation alters the chemical composition of pain neuroregulators, which closes the gate.” c. “Meditation will help me sleep through the pain because it opens the gate.” d. “Meditation stops the occurrence of pain stimuli.” a. “As adults age, their ability to perceive pain decreases.” b. “Older patients may have low serum albumin in their blood, causing toxic effects of analgesic drugs.” c. “Patients who have dementia probably experience pain, and their pain is not always well controlled.” 221 A&E I Comprehensive Testbank ANS: A Aging does not affect the ability to perceive pain. This misconception must be corrected by the supervising nurse. All the other statements are true and require no follow-up. Opioids are safe to use in older adults as long as they are slowly titrated and the nurse frequently monitors the patient. Patients with dementia most likely experience unrelieved pain because their pain is difficult to assess. Older adults frequently eat poorly, resulting in low serum albumin levels. Many drugs are highly protein bound. In the presence of low serum albumin, more free drug (active form) is available, thus increasing the risk for side and/or toxic effects. 5.The nurse is caring for two patients; both are having a hysterectomy. The first patient is having the hysterectomy after a complicated birth. The second patient has uterine cancer. What will most likely influence the experience of pain for these two patients? ANS: A The degree and quality of pain perceived by a patient are related to the meaning of the pain. The patient’s perception of pain is influenced by psychological factors, such as anxiety and coping, which in turn influence the patient’s experience of pain. Each patient’s experience is different. Neurological factors can interrupt or influence pain perception, but neither of these patients is experiencing alterations in neurological function. The knowledge, attitudes, and beliefs of nurses, health care providers, the surgeon, and other health care personnel about pain affect pain management but do not necessarily influence a patient’s pain perceptions. 6.The nurse is preparing pain medications. To which patient does the nurse anticipate administering an opioid fentanyl patch? ANS: C Transdermal fentanyl (patch), which is 100 times more potent than morphine, is available for opioid-tolerant patients with cancer or chronic pain (prostate cancer). It delivers predetermined doses that provide analgesia for up to 72 hours. The other patients are expected to experience acute pain (fractured femur, cellulitis, and broken hip). Therefore, they will most likely benefit from oral or IV opioids for short-term pain relief. 7.A patient is receiving opioid medication through an epidural infusion. Which action will the nurse take? ANS: B d. “It is safe to administer opioids to older adults as long as you start with small doses and frequently assess the patient’s response to the medication.” a. Meaning of pain b. Neurological factors c. Competency of the surgeon d. Postoperative support personnel a. A 15-year-old adolescent with a fractured femur b. A 30-year-old adult with cellulitis c. A 50-year-old patient with prostate cancer d. An 80-year-old patient with a broken hip a. Restrict fluid intake. b. Label the tubing that leads to the epidural catheter. c. Apply a gauze dressing to the epidural catheter insertion site. d. Ask the nursing assistive personnel to check on the patient at least once every 2 hours. 222 A&E I Comprehensive Testbank To reduce the accidental administration of IV medications into the epidural catheter, the tubing that leads to the epidural catheter needs to be labeled clearly. The epidural insertion site needs to be covered by a transparent dressing to prevent infection and allow the nurse to assess the site. Patients receiving epidural anesthesia need to be monitored every 15 minutes until stabilized and then at least hourly for 12 to 24 hours. 8.A woman is in labor and refuses to receive any sort of anesthesia medication. Which alternative treatment is best for this patient? ANS: D In the case of a patient in labor, relaxation with guided imagery is often an effective supplement for pain management because it provides women with a sense of control over their pain. Relaxation and guided imagery can be used during any phase of health or illness. TENS units are typically used to manage postsurgical and procedural pain. Herbal supplements need to be evaluated for safety during pregnancy. Additionally, some patients consider herbal supplements to be another form of medication, and they are not typically used to control acute pain. A pudendal block is a type of regional anesthesia (injection or infusion of local anesthetics to block a group of sensory nerve fibers); use of it does not respect the patient’s wishes for nonpharmacological pain control. 9.A nurse is teaching a patient about patient-controlled analgesia (PCA). Which statement made by the patient indicates to the nurse that teaching is effective? ANS: B A PCA is a device that allows the patient to determine the level of pain relief delivered, reducing the risk of overdose. The PCA infusion pumps are designed to deliver a specific dose that is programmed to be available at specific time intervals (usually in the range of 8 to 15 minutes) when the patient activates the delivery button. A limit on the number of doses per hour or 4-hour interval may also be set. This can help decrease a patient’s anxiety related to possible overdose. Its use also often eases anxiety because the patient is not reliant on the nurse for pain relief. Other medications, such as oral analgesics, can be given in addition to the PCA machine. One benefit of PCA is that the patient does not need to rely on the nurse to administer pain medication; the patient determines when to take the medication. 10.A nurse is caring for a patient who is experiencing pain following abdominal surgery. Which information is important for the nurse to share with the patient when providing patient education about effective pain management? a. Transcutaneous electrical nerve stimulation (TENS) b. Herbal supplements with analgesic effects c. Pudendal block (regional anesthesia) d. Relaxation and guided imagery a. “I will only need to be on this pain medication.” b. “I feel less anxiety about the possibility of overdosing.” c. “I can receive the pain medication as frequently as I need to.” d. “I need the nurse to notify me when it is time for another dose.” a. “To prevent overdose, you need to wait to ask for pain medication until you begin to experience pain.” b. “You should take your medication after you walk to make sure you do not fall while you are walking.” c. “We should work together to create a schedule to provide regular dosing of medication.” d. “When you experience severe pain, you will need to take oral pain medications.” 223 A&E I Comprehensive Testbank ANS: C One way to maximize pain relief while potentially decreasing opioid use is to administer analgesics around the clock (ATC) rather than on a prn basis. This approach ensures a more constant therapeutic blood level of an analgesic. Working with the patient to design a schedule allows the patient to be a full partner in the care provided. The nurse should not wait until pain is experienced because it takes medications 10-30 minutes to begin to relieve pain. The nurse administers pain medications before painful activities, such as walking, and administers intravenous medications when a patient is having severe pain. 11.A nurse is caring for a patient who recently had spinal surgery. The nurse knows that patients usually experience acute pain following this type of surgery. The patient refuses to get up and walk and is not moving around in the bed. However, the patient is stoic and denies experiencing pain at this time. What most likely explains this patient’s behavior? ANS: B A patient’s culture or beliefs about pain often influence the patient’s expression of pain. In this case, the patient has just had surgery, and the nurse knows that this surgical procedure usually causes patients to experience pain. It is important at this time for the nurse to examine cultural and ethnic factors that are possibly affecting the patient’s lack of expression of pain at this time. Even if surgery corrects neurological factors that create chronic pain, surgery causes pain in the acute period. The patient has not taken any pain medication so this is an unrealistic assumption; most pain medications have standard dosages. The nurse is not allowing personal beliefs to influence pain management because the nurse is attempting to determine the reason why the patient is not verbalizing the experience of pain. 12.A nurse is providing discharge teaching for a patient with a fractured humerus. The patient is going home with hydrocodone. Which important patient education does the nurse provide? ANS: A A common side effect of opioid analgesics is constipation. Therefore, the nurse encourages the patient to drink fluids and eat fiber to prevent constipation. Although medications can be irritating to the stomach, eating a diet high in fat does not prevent gastric ulcers. To best manage pain, the patient needs to take pain medication before painful procedures or activities or before pain becomes severe. As the patient’s pain gets better, the strength of the medications will decrease. IM, IV, and topical analgesics are used for more severe and chronic pain. 13.A patient arrives at the emergency department experiencing a headache and rates the pain as 7 on a 0 to 10 pain scale. Which nonpharmacological intervention does the nurse implement for this patient while awaiting orders for pain medication from the health care provider? a. The surgery successfully cured the patient’s pain. b. The patient’s culture is possibly influencing the patient’s experience of pain. c. The primary health care provider did not prescribe the correct amount of medication. d. The nurse is allowing personal beliefs about pain to influence pain management at this time. a. “You need to drink plenty of fluids and eat a diet high in fiber.” b. “Narcotics can be addictive, so do not take them unless you are in severe pain.” c. “Be sure to eat a meal high in fat before taking the medication, to avoid a stomach ulcer.” d. “As your pain severity lessens, you will begin to give yourself oncedaily intramuscular injections.” a. Reassures the patient that the provider will come to the emergency department soon 224 A&E I Comprehensive Testbank ANS: B The appropriate nonpharmacological pain-management intervention is to quietly play music that the patient finds relaxing. Music diverts a person’s attention away from pain and creates relaxation. Reassessing the patient’s pain scores is done during evaluation. Building the patient’s expectation of the provider’s arrival does not address the patient’s pain. Although yoga promotes relaxation, nurses teach relaxation techniques only when a patient is not experiencing acute pain. Because the patient is having acute pain, this is not an appropriate time to provide patient teaching. 14.A patient who has had type 2 diabetes for 26 years is beginning to experience peripheral neuropathy in the feet and lower leg. The nurse is providing education to the patient to prevent injury to the feet by wearing shoes or slippers when walking. Which statement made by the nurse best explains the rationale for this instruction? ANS: D Any factor that interrupts or influences normal pain reception or perception (e.g., spinal cord injury, peripheral neuropathy, or neurological disease) affects a patient’s awareness of and response to pain. The patient will no longer have protective reflexes to prevent injury to the feet. Wearing shoes prevents the patient from injuring the feet because they protect the feet. Shoes do not block pain perception, and they do not help people adapt to pain. Shoes are not a form of nonpharmacological pain relief. Wearing shoes will not have an effect on opening or closing the pain gates. 15.A nurse is assessing a patient who started to have severe pain 3 days ago. When the nurse asks the patient to describe the pain, the patient states, “The pain feels like it is in my stomach. It is a burning pain, and it spreads out in a circle around the spot where it hurts the most.” Which type of pain does the nurse document the patient is having at this time? ANS: D Visceral pain arises from visceral organs, such as those from the gastrointestinal tract. Visceral pain is diffuse and radiates in several directions and can have a burning quality. Superficial pain has a short duration and is usually a sharp pain arising from the skin. Pain of an unknown cause is called idiopathic pain. Chronic pain lasts longer than 6 months. 16.A patient who had a motor vehicle crash 2 days ago is experiencing pain and is receiving patient-controlled analgesia (PCA). Which assessment finding indicates effective pain management with the PCA? b. Softly plays music that the patient finds relaxing c. Frequently reassesses the patient’s pain scores d. Teaches the patient how to do yoga a. “Wearing shoes blocks pain perception and helps you adapt to pain, which ends up protecting your feet.” b. “Shoes provide nonpharmacological pain relief to people with diabetes and peripheral neuropathy.” c. “The neurological gates open when wearing shoes, which protects your feet.” d. “If you step on something without shoes, you might not feel it; this could possibly cause injury to your foot.” a. Superficial pain b. Idiopathic pain c. Chronic pain d. Visceral pain a. The patient is sleeping and is difficult to arouse. 225 A&E I Comprehensive Testbank ANS: B A level of 2 on a scale of 0 to 10 is evidence of effective pain management. The effectiveness of pain-relief measures is determined by the patient. If the patient is satisfied with the amount of pain relief, then pain measures are effective. A patient who is sleeping and is difficult to arouse is possibly oversedated; the nurse needs to assess this patient further. The amount of medication left in the PCA syringe does not indicate whether pain management is effective or not. Pressing the button shows that the patient knows how to use the PCA but does not evaluate pain management. 17.The nurse is caring for a patient to ease modifiable factors that contribute to pain. Which areas did the nurse focus on with this patient? ANS: B Some examples of modifiable contributors to pain are anxiety and fear. The nurse can take measures to ease the patient’s anxiety and fear related to pain. Age, gender, culture, ethnicity, cognitive abilities, and previous pain experience are all nonmodifiable factors that the nurse can help the patient to understand, but the nurse cannot alter them. 18.The nurse is evaluating the effectiveness of guided imagery for pain management as used for a patient who has second- and third-degree burns and needs extensive dressing changes. Which finding best indicates the effectiveness of guided imagery? ANS: C If the patient needs less pain medication during dressing changes, then guided imagery is helping to manage the patient’s pain. The purpose of guided imagery is to allow the patient to alter the perception of pain. Guided imagery works in conjunction with analgesic medications, potentiating their effects. A rating of 6 on a 0 to 10 scale indicates that the patient is having moderate pain and shows that this patient is not experiencing pain relief at this time. A person who is stoic is not showing feelings, which makes it difficult to know whether or not the patient is experiencing pain. Having to ask for pain medication during the dressing changes indicates the guided imagery is not effective. 19.A nurse is providing medication education to a patient who just started taking ibuprofen. Which information will the nurse include in the teaching session? b. The patient rates pain at a level of 2 on a 0 to 10 scale. c. The patient has sufficient medication left in the PCA syringe. d. The patient presses the control button to deliver pain medication. a. Age and gender b. Anxiety and fear c. Culture and ethnicity d. Previous pain experiences and cognitive abilities a. The patient’s facial expressions are stoic during the procedure. b. The patient rates pain during the dressing change as a 6 on a scale of 0 to 10. c. The patient’s need for analgesic medication decreases during the dressing changes. d. The patient asks for pain medication during the dressing changes only once throughout the procedure. a. Ibuprofen helps to depress the central nervous system to decrease pain perception. b. Ibuprofen reduces anxiety, which will help you cope with your pain. 226 A&E I Comprehensive Testbank ANS: D NSAIDs like ibuprofen likely work by inhibiting the synthesis of prostaglandins to inhibit cellular responses to inflammation. Ibuprofen does not depress the central nervous system, nor does it enhance coping with pain. Opioids bind with opiate receptors to modify perceptions of pain. 20.The nurse has brought a patient the scheduled pain medication. The patient asks the nurse to wait to give pain medication until the time for the dressing change, which is 2 hours away. Which response by the nurse is most therapeutic? ANS: C Additional doses of medication can be given to patients in certain circumstances, as with an extensive dressing change, when the health care provider is notified that more medication is needed. It is the nurse’s responsibility to communicate with the provider and with the patient about a pain-control plan that works for both. By asking to hold off on the dose, the patient is indicating that the dressing changes are extremely painful. The regularly scheduled dose might not be as effective for the patient 2 hours later when the dressing change is scheduled. Oral medications take 30 to 60 minutes to take effect. If the nurse began the dressing change right then, the medication would not have been absorbed yet. The patient has the right to refuse to take a medication. 21.A nurse receives an order from a health care provider to administer hydrocodone and acetaminophen (Vicodin ES 7.5/750), to a patient who is experiencing 8/10 postsurgical pain. The order is to give 2 tablets every 6 hours by mouth as needed for pain. What is the nurse’s next best action? ANS: C The maximum 24-hour dosage for acetaminophen is 4 grams. If the patient took 2 tablets of Vicodin ES every 6 hours, the patient would take in 6 grams of acetaminophen in 24 hours (2 tablets = 750 + 750 = 1500 4 [could have 4 doses in 24 hours every 6 hours] = 6000 mg = 6 g). This exceeds the safe dosage of acetaminophen, so the best action is to question this order. Giving the medication as ordered would possibly result in the patient’s taking more acetaminophen than is considered a safe dose. Acetaminophen overdose can result in liver failure. NSAIDs are used to treat mild to moderate pain. At this moment, the patient is experiencing severe pain. Implementing music therapy is a nursing intervention and is an independent nursing action that can be instituted with pain medication, but the possible acetaminophen dose is the priority. 22.The nurse is caring for a 4-year-old child who has pain. Which technique will the nurse use to best assess pain in this child? c. Ibuprofen binds with opiate receptors to reduce your pain. d. Ibuprofen inhibits the production of prostaglandins. a. “This medication will still be providing you relief at the time of your dressing change.” b. “OK, swallow this pain pill, and I will return in a minute to change your dressing.” c. “Would you like medication to be given for dressing changes in addition to your regularly scheduled medication?” d. “Your medication is scheduled for this time, and I can’t adjust the time for you. I’m sorry, but you must take your pill right now.” a. Give the Vicodin ES to the patient immediately because the patient is experiencing severe pain. b. Ask the health care provider for a nonsteroidal antiinflammatory drug (NSAID) order. c. Ask the health care provider to verify the dosage and frequency of the medication. d. Give the Vicodin ES in addition to playing soothing music for the patient. 227 A&E I Comprehensive Testbank ANS: A The FACES scale assesses pain in children who are verbal. Because a 4-year-old is verbal, this is an appropriate scale to use with this child. Assessing pain intensity in children requires special techniques. Young children often have difficulty expressing their pain. Parents’ statement of pain is not an effective way to assess pain in children because children’s statements are the most important. The 0 to 10 pain scale is too difficult for a 4-year-old child to understand. Previous documentation by nurses will tell you what the child’s pain has been but will not tell you the child’s current pain intensity. 23.A nurse is caring for a group of patients. Which patient will the nurse see first? ANS: A A respiratory rate of 10 indicates respiratory depression. A rare adverse effect of opioids in opioid-naïve patients (patients who have used opioids around the clock for less than approximately 1 week) is respiratory depression. Naloxone (Narcan) may be administered. While the other patients are experiencing pain and do need to be seen, they are not the priority since respirations are not affected. 24.A nurse is caring for a patient with chronic pain. Which statement by the nurse indicates an understanding of pain management? ANS: B Be sure to evaluate after an appropriate period of time. For instance, oral medications usually peak in about 1 hour, whereas IVP medications peak in 15 to 30 minutes. Ask a patient if a medication alleviates the pain when it is peaking. Because oral medications usually peak in about an hour, you need to reassess the patient’s pain within an hour of administration. Nurses must believe any patient report of pain, even if nonverbal communication is not consistent with pain ratings. The patient is the only person who should push the PCA button. Pushing the PCA when a patient is sleeping is dangerous and may lead to narcotic overdose or respiratory depression. Giving the patient a placebo and telling the patient it is medication is unethical. 25.The nurse is assessing how a patient’s pain is affecting mobility. Which assessment question is mostappropriate? a. Use the FACES scale. b. Check to see what previous nurses have charted. c. Ask the parents if they think their child is in pain. d. Have the child rate the level of pain on a 0 to 10 pain scale. a. A patient who received morphine and has a pulse of 62 beats/min, respirations 10 breaths/min, and blood pressure 110/60 mm Hg b. A patient lying very still in bed who reports no pain but is pale with warm, dry skin c. A patient with severe pain who is nauseated and feels like he or she is about to vomit d. A patient writhing and moaning from abdominal pain after abdominal surgery a. “This patient says the pain is a 5 but is not acting like it. I am not going to give any pain medication.” b. “I need to reassess the patient’s pain 1 hour after administering oral pain medication.” c. “It wasn’t time for the patient’s medication, so when it was requested, I gave a placebo.” d. “The patient is sleeping, so I pushed the PCA button.” a. “Have you considered working with a physical therapist?” 228 A&E I Comprehensive Testbank ANS: B Because the nurse is interested in knowing whether the patient’s pain is affecting mobility, the priority assessment question is to ask the patient how the pain affects ability to participate in normal activities of daily living. Although a physical therapist is a good resource to have, especially if pain is severely affecting mobility, considering working with a physical therapist does not describe the effect of pain on the patient’s mobility. Assessing quality of pain and effectiveness of pain medication does not help the nurse to understand how it is affecting the patient’s mobility. 26.The nurse is teaching a student nurse about pain assessment scales. Which statement by the student indicates effective teaching? ANS: A Do not use a pain scale to compare the pain of one patient to that of another. Pain is subjective and cannot be compared to the pain of another patient. Some patients do not express their pain (stoic) or do not wish to take medications to relieve the pain. This does not mean they aren’t in pain. A patient’s behavior is not more reliable than the patient’s report of pain. Pain scales help determine severity or intensity, not quality. 27.The nurse is administering pain medication for several patients. Which patient does the nurse administer medication to first? ANS: D Immediate (STAT) medications need to be given as soon as possible. In addition, this patient is the priority because of the report of severe pain. The other patients need pain medication, but their situations are not as high a priority as that of the patient with the STAT medication order. 28.The nurse is assessing a patient for opioid tolerance. Which finding supports the nurse’s assessment? b. “What activities, if any, has your pain prevented you from doing?” c. “Would you please rate your pain on a scale from 0 to 10 for me?” d. “When does your pain medication typically take effect on your pain?” a. “You cannot use a pain scale to compare the pain of my patient with the pain of your patient.” b. “When patients say they don’t need pain medication, they aren’t in pain.” c. “A patient’s behavior is more reliable than the patient’s report of pain.” d. “Pain assessment scales determine the quality of a patient’s pain.” a. The patient who needs to be premedicated before walking b. The patient who has a PCA running that needs the syringe replaced c. The patient who needs to take a scheduled dose of maintenance pain medication d. The patient who is experiencing 8/10 pain and has an immediate order for pain medication a. The patient needed a substantial dose of naloxone (Narcan). b. The patient needs increasingly higher doses of opioid to control pain. c. The patient no longer experiences sedation from the usual dose of opioid. d. The patient asks for pain medication close to the time it is due around the clock. 229 A&E I Comprehensive Testbank ANS: B Opioid tolerance occurs when a patient needs higher doses of an opioid to control pain. Naloxone (Narcan) is an opioid antagonist that is given to reverse the effects of opioid overdose. Taking pain medications regularly around the clock is an effective way to control pain. The pain medication for this patient is most likely effectively managing the patient’s pain because the patient is not asking for the medication before it is due. A patient no longer experiencing a side effect (sedation) of an opioid does not indicate opioid tolerance. 29.A nurse is caring for a patient with rheumatoid arthritis who is now going to be taking 2 acetaminophen (Tylenol) tablets every 6 hours to control pain. Which part of the patient’s social history is the nurse most concerned about? ANS: A The major adverse effect of acetaminophen is hepatotoxicity (liver toxicity). Because both alcohol and acetaminophen are metabolized by the liver, when taken together, they can cause liver damage. Smoking cigarettes and smoking marijuana are not healthy behaviors, but their effects on health are not affected by acetaminophen. Antianxiety medications can be taken with acetaminophen. 30.The nurse is caring for a patient who suddenly experiences chest pain. What is the nurse’s first priority? ANS: D The nurse’s ability to establish a nursing diagnosis, plan and implement care, and evaluate the effectiveness of care depends on an accurate and timely assessment. The other responses are all interventions; the nurse cannot know which intervention is appropriate until the nurse completes the assessment. 31.The nurse is caring for a group of patients. Which task may the nurse delegate to the nursing assistive personnel (NAP)? ANS: A A massage may be delegated to an NAP. Pain assessment is a nursing function and cannot be delegated to an NAP. Administration of patient-controlled analgesia (PCA) cannot be delegated to an NAP. Assessment of vital signs is a licensed nursing function; the NAP can take vital signs for a patient receiving epidural analgesia. 32.A nurse is caring for a patient with chronic pain from arthritis. Which action is best for the nurse to take? a. Patient drinks 1 to 2 glasses of wine every night. b. Patient smokes 2 packs of cigarettes a day. c. Patient occasionally uses marijuana. d. Patient takes antianxiety medications. a. Call the rapid response team. b. Start an intravenous (IV) line. c. Administer pain-relief medications. d. Ask the patient to rate and describe the pain. a. Administer a back massage to a patient with pain. b. Assessment of pain for a patient reporting abdominal pain. c. Administer patient-controlled analgesia for a postoperative patient. d. Assessment of vital signs in a patient receiving epidural analgesia. a. Give pain medications around the clock. b. Administer pain medication before any activity. c. Give pain medication after the pain is a 7/10 on the pain scale. 230 A&E I Comprehensive Testbank ANS: A When a patient with arthritis has chronic pain, the best way to manage pain is to take medication regularly throughout the day to maintain constant pain relief. “Before any activity” is nonspecific, and the medication may not have time to work before activity. If the patient waits until having pain (7/10) to take the medication, pain relief takes longer. Nonpharmacological measures are used in conjunction with medications unless requested otherwise by the patient. 33.A nurse is caring for a patient who fell on the ice and has connective tissue damage in the wrist and hand. The patient describes the pain as throbbing. Which type of pain does the nurse document in this patient’s medical record? ANS: B Somatic pain comes from bone, joint, connective tissue, or muscle. Visceral pain arises from the visceral (internal) organs such as the GI tract and pancreas. Peripherally generated pain in the peripheral nerves can be caused by polyneuropathies or mononeuropathies. Centrally generated pain results from injury to the central or peripheral nervous system, causing deafferentation or sympathetically maintained pain. 34.The nurse is caring for an infant in the intensive care unit. Which information should the nurse consider when planning care for this patient? ANS: B Infants cannot verbally express their pain, but they do express pain with behavioral cues (facial expressions, crying) and physiological indicators (changes in vital signs). Infants can tolerate analgesics, but proper dosing and close monitoring are essential. Infants and older children have the same sensitivity to pain. Pain can be assessed even though the neonate cannot verbalize; the nurse can observe behavioral clues. Nurses use behavioral cues and physiological responses to assess pain in infants. MULTIPLE RESPONSE 1.The nurse is administering ibuprofen (Advil) to an older patient. Which assessment data causes the nurse to hold the medication? (Select all that apply.) d. Administer pain medication only when nonpharmacological measures have failed. a. Visceral pain b. Somatic pain c. Centrally generated pain d. Peripherally generated pain a. Infants cannot be assessed for pain. b. Infants respond behaviorally and physiologically to painful stimuli. c. Infants cannot tolerate analgesics owing to an underdeveloped metabolism. d. Infants have a decreased sensitivity to pain when compared with older children. a. Patient states allergy to aspirin. b. Patient states joint pain is 2/10 and intermittent. c. Patient reports past medical history of gastric ulcer. d. Patient reports last bowel movement was 4 days ago. e. Patient experiences respiratory depression after administration of an opioid medication. 231 A&E I Comprehensive Testbank ANS: A, C NSAIDs can cause bleeding, especially in the gastrointestinal (GI) tract; therefore, NSAIDs are most likely contraindicated in this patient. Patients with an allergy to aspirin or have asthma are sometimes also allergic to other NSAIDs. The nurse needs to verify that the health care provider is aware of the history of GI bleeding and of allergy to aspirin before administering ibuprofen. NSAIDs do not interfere with bowel function and are used for the treatment of mild to moderate acute intermittent pain. NSAIDs also do not suppress the central nervous system by causing respiratory depression. MATCHING The nurse is assessing the characteristics of a patient’s pain. Match the characteristic to the question a nurse will ask to determine that specific characteristic. 1.Timing 2.Location 3.Severity 4.Quality 5.Aggravating factors 1.ANS:B2.ANS:C3.ANS:A4.ANS:E5.ANS:D Week 7 Concepts related to Oxygenation, Circulation, and Tissue Perfusion, Chronic Obstructive Pulmonary Disease Chapter 41: Oxygenation Potter et al.: Fundamentals of Nursing, 9th Edition MULTIPLE CHOICE a. Could you rate your pain on a scale of 0 to 10? b. How often does it recur? c. Could you point to the area of pain? d. Do certain activities worsen the pain? e. What does the pain feel like? 232 A&E I Comprehensive Testbank 1. A nurse is teaching staff about the conduction of the heart. In which order will the nurse present the conduction cycle, starting with the first structure? 1. Bundle of His 2. Purkinje network 3. Intraatrial pathways 4. Sinoatrial (SA) node 5. Atrioventricular (AV) node ANS: B The conduction system originates with the SA node, the “pacemaker” of the heart. The electrical impulses are transmitted through the atria along intraatrial pathways to the AV node. It assists atrial emptying by delaying the impulse before transmitting it through the Bundle of His and the ventricular Purkinje network. 2. A nurse is teaching the patient with mitral valve problems about the valves in the heart. Starting on the right side of the heart, describe the sequence of the blood flow through these valves. 1. Mitral 2. Aortic 3. Tricuspid 4. Pulmonic ANS: C The blood flows through the valves in the following direction: tricuspid, pulmonic, mitral, and aortic. 3. A nurse explains the function of the alveoli to a patient with respiratory problems. Which information about the alveoli’s function will the nurse share with the patient? ANS: A The alveolus is a capillary membrane that allows gas exchange of oxygen and carbon dioxide during respiration. The alveoli do not store oxygen, regulate tidal volume, or produce hemoglobin. 4. A nurse auscultates heart sounds. When the nurse hears S2, which valves is the nurse hearing close? a. 5, 4, 3, 2, 1 b. 4, 3, 5, 1, 2 c. 4, 5, 3, 1, 2 d. 5, 3, 4, 2, 1 a. 1, 3, 2, 4 b. 4, 3, 2, 1 c. 3, 4, 1, 2 d. 2, 4, 1, 3 a. Carries out gas exchange b. Regulates tidal volume c. Produces hemoglobin d. Stores oxygen a. Aortic and mitral b. Mitral and tricuspid 233 A&E I Comprehensive Testbank ANS: C As the ventricles empty, the ventricular pressures decrease, allowing closure of the aortic and pulmonic valves, producing the second heart sound, S2. The mitral and tricuspid produce the first heart sound, S1. The aortic and mitral do not close at the same time. The mitral and pulmonic do not close at the same time. 5. The nurse is teaching about the process of exchanging gases through the alveolar capillary membrane. Which term will the nurse use to describe this process? ANS: D Diffusion is the process of gases exchanging across the alveoli and capillaries of body tissues. Ventilation is the process of moving gases into and out of the lungs. Surfactant is a chemical produced in the lungs to maintain the surface tension of the alveoli and keep them from collapsing. Perfusion is the ability of the cardiovascular system to carry oxygenated blood to tissues and return deoxygenated blood to the heart. 6. A nurse is caring for a patient who was in a motor vehicle accident that resulted in cervical trauma to C4. Which assessment is the priority? ANS: B Respirations and oxygen saturation are the priorities. Cervical trauma at C3 to C5 usually results in paralysis of the phrenic nerve. When the phrenic nerve is damaged, the diaphragm does not descend properly, thus reducing inspiratory lung volumes and causing hypoxemia. While pulse and blood pressure are important, respirations are the priority. Temperature is not a high priority in this situation. 7. The patient is breathing normally. Which process does the nurse consider is working properly when the patient inspires? ANS: A Inspiration is an active process, stimulated by chemical receptors in the aorta. Reduced arterial oxygen saturation levels indicate hypoxemia, an abnormal finding. Expiration is a passive process that depends on the elastic recoil properties of the lungs, requiring little or no muscle work. Prolonged use of the accessory muscles does not promote effective ventilation and causes fatigue. 8. The home health nurse recommends that a patient with respiratory problems install a carbon monoxide detector in the home. What is the rationale for the nurse’s action? c. Aortic and pulmonic d. Mitral and pulmonic a. Ventilation b. Surfactant c. Perfusion d. Diffusion a. Pulse b. Respirations c. Temperature d. Blood pressure a. Stimulation of chemical receptors in the aorta b. Reduction of arterial oxygen saturation levels c. Requirement of elastic recoil lung properties d. Enhancement of accessory muscle usage a. Carbon monoxide detectors are required by law in the home. 234 A&E I Comprehensive Testbank ANS: B Carbon monoxide binds tightly to hemoglobin; therefore, oxygen is not able to bind to hemoglobin and be transported to tissues, causing hypoxia. A carbon monoxide detector is not required by law, does not signal the cerebral cortex to cease ventilations, and does not combine with oxygen but with hemoglobin to produce a toxin. 9. While performing an assessment, the nurse hears crackles in the patient’s lung fields. The nurse also learns that the patient is sleeping on three pillows to help with the difficulty breathing during the night. Which condition will the nurse most likely observe written in the patient’s medical record? ANS: C Left-sided heart failure results in pulmonary congestion, the signs and symptoms of which include shortness of breath, cough, crackles, and paroxysmal nocturnal dyspnea (difficulty breathing when lying flat). Right-sided heart failure is systemic and results in peripheral edema, weight gain, and distended neck veins. Atrial fibrillation is often described as an irregularly irregular rhythm; rhythm is irregular because of the multiple pacemaker sites. Myocardial ischemia results when the supply of blood to the myocardium from the coronary arteries is insufficient to meet myocardial oxygen demands, producing angina or myocardial infarction. 10. A patient has a myocardial infarction. On which primary blood vessel will the nurse focus care to reduce ischemia? ANS: C A myocardial infarction is the lack of blood flow due to obstruction to the coronary artery, which supplies the heart with blood. The superior vena cava returns blood back to the heart. The pulmonary artery supplies deoxygenated blood to the lungs. The carotid artery supplies blood to the brain. 11. A nurse is teaching a health class about the heart. Which information from the class members indicates teaching by the nurse is successful for the flow of blood through the heart, starting in the right atrium? ANS: C Unoxygenated blood flows through the venae cavae into the right atrium, where it is pumped down to the right ventricle; the blood is then pumped out the pulmonary artery and is returned oxygenated via the pulmonary b. Carbon monoxide tightly binds to hemoglobin, causing hypoxia. c. Carbon monoxide signals the cerebral cortex to cease ventilations. d. Carbon monoxide combines with oxygen in the body and produces a deadly toxin. a. Atrial fibrillation b. Myocardial ischemia c. Left-sided heart failure d. Right-sided heart failure a. Superior vena cava b. Pulmonary artery c. Coronary artery d. Carotid artery a. Right ventricle, left ventricle, left atrium b. Left atrium, right ventricle, left ventricle c. Right ventricle, left atrium, left ventricle d. Left atrium, left ventricle, right ventricle 235 A&E I Comprehensive Testbank vein to the left atrium, where it flows to the left ventricle and is pumped out to the rest of the body via the aorta. 12. The nurse suspects the patient has increased afterload. Which piece of equipment should the nurse obtain to determine the presence of this condition? ANS: C A blood pressure cuff is needed. The diastolic aortic pressure is a good clinical measure of afterload. Afterload is the resistance to left ventricular ejection. In hypertension the afterload increases, making cardiac workload also increase. A pulse oximeter is used to monitor the level of arterial oxygen saturation; it will not help determine increased afterload. While an oxygen cannula may be needed to help decrease the effects of increased afterload, it will not help determine the presence of afterload. A Yankauer suction tip catheter is used to suction the oral cavity. 13. A patient has heart failure and cardiac output is decreased. Which formula can the nurse use to calculate cardiac output? ANS: C Cardiac output can be calculated by multiplying the stroke volume and the heart rate. The other options are not measures of cardiac output. 14. A patient’s heart rate increased from 94 to 164 beats/min. What will the nurse expect? ANS: C With a sustained heart rate greater than 160 beats/min, diastolic filling time decreases, decreasing stroke volume and cardiac output. The hemoglobin level would not be affected. 15. The nurse is careful to monitor a patient’s cardiac output. Which goal is the nurse trying to achieve? ANS: A Cardiac output indicates how much blood is being circulated systemically throughout the body to the periphery. The amount of blood ejected from the left ventricle each minute is the cardiac output. Oxygen status would be determined by pulse oximetry and the presence of cyanosis. Cardiac dysrhythmias are an electrical a. Pulse oximeter b. Oxygen cannula c. Blood pressure cuff d. Yankauer suction tip catheter a. Myocardial contractility × Myocardial blood flow b. Ventricular filling time/Diastolic filling time c. Stroke volume × Heart rate d. Preload/Afterload a. Increase in diastolic filling time b. Decrease in hemoglobin level c. Decrease in cardiac output d. Increase in stroke volume a. To determine peripheral extremity circulation b. To determine oxygenation requirements c. To determine cardiac dysrhythmias d. To determine ventilation status 236 A&E I Comprehensive Testbank impulse monitored through ECG results. Ventilation status is measured by respiratory rate, pulse oximetry, and capnography. Capnography provides instant information about the patient’s ventilation. Ventilation status does not depend solely on cardiac output. 16. A nurse is caring for a group of patients. Which patient should the nurse see first? ANS: A The mask is contraindicated for patients with carbon dioxide retention (hypercapnia) because retention can be worsened; the nurse must see this patient first to correct the problem. All the rest are using correct procedures and do not need to be seen first. A chest tube should not be clamped when ambulating. Clamping a chest tube is contraindicated when ambulating or transporting a patient. Clamping can result in a tension pneumothorax. Use nasotracheal suctioning before pharyngeal suctioning whenever possible. The mouth and pharynx contain more bacteria than the trachea. Keep tracheostomy obturator at bedside with a fresh (new) tracheostomy to facilitate reinsertion of the outer cannula if dislodged. 17. A patient has inadequate stroke volume related to decreased preload. Which treatment does the nurse prepare to administer? ANS: D Preload is affected by the circulating volume; if the patient has decreased fluid volume, it will need to be replaced with fluid or blood therapy. Preload is the amount of blood in the left ventricle at the end of diastole, often referred to as end-diastolic volume. Giving diuretics and vasodilators will make the situation worse. Diuretics causes fluid loss; the patient is already low on fluids or the preload would not be decreased. Vasodilators reduced blood return to the heart, making the situation worse; the patient does not have enough blood and fluid to the heart or the preload would not be decreased. Chest physiotherapy is a group of therapies for mobilizing pulmonary secretions. Chest physiotherapy will not help this cardiovascular problem. 18. A nurse is preparing to suction a patient. The pulse is 65 and pulse oximetry is 94%. Which finding will cause the nurse to stop suctioning? ANS: D Stop when oxygen saturation is 88%. Monitor patient’s vital signs and oxygen saturation during procedure; note whether there is a change of 20 beats/min (either increase or decrease) or if pulse oximetry falls below 90% or 5% from baseline. If this occurs, stop suctioning. A pulse rate of 75 is only 10 beats different from the start of the procedure. A pulse rate of 80 is 15 beats different from the start of suctioning. Oxygen saturation of 91% is not 5% from baseline or below 90%. a. A patient with hypercapnia wearing an oxygen mask b. A patient with a chest tube ambulating with the chest tube unclamped c. A patient with thick secretions being tracheal suctioned first and then orally d. A patient with a new tracheostomy and tracheostomy obturator at bedside a. Diuretics b. Vasodilators c. Chest physiotherapy d. Intravenous (IV) fluids a. Pulse 75 b. Pulse 80 c. Oxygen saturation 91% d. Oxygen saturation 88% 237 A&E I Comprehensive Testbank 19. The patient has right-sided heart failure. Which finding will the nurse expect when performing an assessment? ANS: A Right-sided heart failure results from inability of the right side of the heart to pump effectively, leading to a systemic backup. Peripheral edema, distended neck veins, and weight gain are signs of right-sided failure. Basilar crackles can indicate pulmonary congestion from left-sided heart failure. Cyanosis and chest pain result from inadequate tissue perfusion. 20. A nurse is reviewing the electrocardiogram (ECG) results. Which portion of the conduction system does the nurse consider when evaluating the P wave? ANS: A The P wave represents the electrical conduction through both atria; the SA node initiates electrical conduction through the atria. The AV node conducts down through the bundle of His and the Purkinje fibers to cause ventricular contraction. 21. A nurse teaches a patient about atelectasis. Which statement by the patient indicates an understanding of atelectasis? ANS: B Atelectasis develops when alveoli do not expand. Breathing exercises, especially deep breathing and incentive spirometry, increase lung volume and open the airways, preventing atelectasis. Deep breathing also opens the pores of Kohn between alveoli to allow sharing of oxygen between alveoli. Atelectasis can affect anyone who does not deep breathe. A chest tube is for pneumothorax or hemothorax. It is deep breathing, not hyperventilation, that prevents atelectasis. 22. The nurse is caring for a patient with respiratory problems. Which assessment finding indicates a late sign of hypoxia? ANS: D a. Peripheral edema b. Basilar crackles c. Chest pain d. Cyanosis a. SA node b. AV node c. Bundle of His d. Purkinje fibers a. “Atelectasis affects only those with chronic conditions such as emphysema.” b. “It is important to do breathing exercises every hour to prevent atelectasis.” c. “If I develop atelectasis, I will need a chest tube to drain excess fluid.” d. “Hyperventilation will open up my alveoli, preventing atelectasis.” a. Elevated blood pressure b. Increased pulse rate c. Restlessness d. Cyanosis 238 A&E I Comprehensive Testbank Cyanosis, blue discoloration of the skin and mucous membranes caused by the presence of desaturated hemoglobin in capillaries, is a late sign of hypoxia. Elevated blood pressure, increased pulse rate, and restlessness are early signs of hypoxia. 23. A nurse is caring for a 5-year-old patient whose temperature is 101.2° F. The nurse expects this patient to hyperventilate. Which factor does the nurse remember when planning care for this type of hyperventilation? ANS: C Increased body temperature (fever) increases the metabolic rate, thereby increasing carbon dioxide production. The increased carbon dioxide level stimulates an increase in the patient’s rate and depth of respiration, causing hyperventilation. Anxiety can cause hyperventilation, but this is not the direct cause from a fever. Sleep causes a decreased respiratory drive; hyperventilation speeds up breathing. The cause of the fever in this question is unknown. 24. A nurse is preparing a patient for nasotracheal suctioning. In which order will the nurse perform the steps, beginning with the first step? 1. Insert catheter. 2. Apply suction and remove. 3. Have patient deep breathe. 4. Encourage patient to cough. 5. Attach catheter to suction system. 6. Rinse catheter and connecting tubing. ANS: C The steps for nasotracheal suctioning are as follows: Verify that catheter is attached to suction; have patient deep breathe; insert catheter; apply intermittent suction for no more than 10 seconds and remove; encourage patient to cough; and rinse catheter and connecting tubing with normal saline. 25. A patient has carbon dioxide retention from lung problems. Which type of diet will the nurse most likely suggest for this patient? ANS: A A low-carbohydrate diet is best. Diets high in carbohydrates play a role in increasing the carbon dioxide load for patients with carbon dioxide retention. As carbohydrates are metabolized, an increased load of carbon dioxide is created and excreted via the lungs. A low- or high-caffeine diet is not as important as the carbohydrate load. a. Anxiety over illness b. Decreased drive to breathe c. Increased metabolic demands d. Infection destroying lung tissues a. 1, 2, 3, 4, 5, 6 b. 4, 5, 1, 2, 3, 6 c. 5, 3, 1, 2, 4, 6 d. 3, 1, 2, 5, 4, 6 a. Low-carbohydrate b. Low-caffeine c. High-caffeine d. High-carbohydrate 239 A&E I Comprehensive Testbank 26. A nurse is caring for a patient who is taking warfarin (Coumadin) and discovers that the patient is taking garlic to help with hypertension. Which condition will the nurse assess for in this patient? ANS: C Patients taking warfarin (Coumadin) for anticoagulation prolong the prothrombin time (PT)/international normalized ratio (INR) results if they are taking gingko biloba, garlic, or ginseng with the anticoagulant. The drug interaction can precipitate a life-threatening bleed. Increased cholesterol levels are associated with saturated fat dietary intake. A distended jugular vein and peripheral edema are associated with damage to the right side of the heart. Angina is temporary ischemia of the heart muscle. 27. A nurse is caring for a patient who has poor tissue perfusion as the result of hypertension. When the patient asks what to eat for breakfast, which meal should the nurse suggest? ANS: A A 2000-calorie diet of fruits, vegetables, and low-fat dairy foods that are high in fiber, potassium, calcium, and magnesium and low in saturated and total fat helps prevent and reduce the effects of hypertension. Nonfat yogurt with granola is a good source of calcium, fiber, and potassium; dried apricots add a second source of potassium. Although cereal and a banana provide fiber and potassium, skim milk should be substituted for whole milk to decrease fat. An omelet with sausage and cheese is high in fat. Butter and bacon are high in fat. 28. Upon auscultation of the patient’s chest, the nurse hears a whooshing sound at the fifth intercostal space. What does this finding indicate to the nurse? ANS: B When regurgitation occurs, there is a backflow of blood into an adjacent chamber. For example, in mitral regurgitation the mitral leaflets do not close completely. When the ventricles contract, blood escapes back into the atria, causing a murmur, or “whooshing” sound. The systolic phase begins with ventricular filling and closing of the aortic valve, which is heard as the first heart sound, S1. Orthopnea is an abnormal condition in which a patient uses multiple pillows when reclining to breathe easier or sits leaning forward with arms elevated. 29. A nurse is caring for a patient with chronic obstructive pulmonary disease (COPD) who is receiving 2 L/ min of oxygen. Which oxygen delivery device is most appropriate for the nurse to administer the oxygen? a. Increased cholesterol level b. Distended jugular vein c. Bleeding d. Angina a. A cup of nonfat yogurt with granola and a handful of dried apricots b. Whole wheat toast with butter and a side of bacon c. A bowl of cereal with whole milk and a banana d. Omelet with sausage, cheese, and onions a. The beginning of the systolic phase b. Regurgitation of the mitral valve c. The opening of the aortic valve d. Presence of orthopnea a. Nasal cannula b. Simple face mask c. Non-rebreather mask d. Partial non-rebreather mask 240 A&E I Comprehensive Testbank ANS: A Nasal cannulas deliver oxygen from 1 to 6 L/min. All other devices (simple face mask, non-rebreather mask, and partial non-rebreather mask) are intended for flow rates greater than 6 L/min. 30. The nurse needs to closely monitor the oxygen status of an older-adult patient undergoing anesthesia because of which age-related change? ANS: B Age-related changes in the thorax that occur from ossification of costal cartilage, decreased space between vertebrae, and diminished respiratory muscle strength lead to problems with chest expansion and oxygenation,whereby the patient will have difficulty excreting anesthesia gas. The nurse needs to monitor the patient’s oxygen status carefully to make sure the patient does not retain too much of the drug. Older adults experience alterations in cardiac function as a result of calcification of the conduction pathways, thicker and stiffer heart valves caused by lipid accumulation and fibrosis, and a decrease in the number of pacemaker cells in the SA node. Altered mental status is not a normal age-related change; it indicates possible cardiac and/or respiratory problems. 31. The nurse determines that an older-adult patient is at risk for infection due to decreased immunity. Which plan of care best addresses the prevention of infection for the patient? ANS: B A nursing care plan for preventative health measures should be reasonable and feasible. Keeping up-to-date on vaccinations is important because vaccine reduces the severity of illnesses and serious complications. Determine if and when the patient has had a pneumococcal or influenza (flu) vaccine. This is especially important when assessing older adults because of their increased risk for respiratory disease. Although it is important to finish the full course of antibiotics, it is not a preventative health measure. Scheduling annual tuberculosis skin tests does not address prevention and is an unreliable indictor of tuberculosis in older patients. The exercise routine should be reasonable to increase compliance; exercise is recommended only 3 to 4 times a week for 30 to 60 minutes, and walking, rather than running, is an efficient method. 32. The nurse is caring for a patient with fluid volume overload. Which physiological effect does the nurse most likely expect? ANS: A a. Thinner heart valves cause lipid accumulation and fibrosis. b. Diminished respiratory muscle strength may cause poor chest expansion. c. Alterations in mental status prevent patients’ awareness of ineffective breathing. d. An increased number of pacemaker cells make proper anesthesia induction more difficult. a. Inform the patient of the importance of finishing the entire dose of antibiotics. b. Encourage the patient to stay up-to-date on all vaccinations. c. Schedule patient to get annual tuberculosis skin testing. d. Create an exercise routine to run 45 minutes every day. a. Increased preload b. Increased heart rate c. Decreased afterload d. Decreased tissue perfusion 241 A&E I Comprehensive Testbank Preload refers to the amount of blood in the left ventricle at the end of diastole; an increase in circulating volume would increase the preload of the heart. Afterload refers to resistance; increased pressure would lead to increased resistance, and afterload would increase. A decrease in tissue perfusion would be seen with hypovolemia. A decrease in fluid volume would cause an increase in heart rate as the body is attempting to increase cardiac output. 33. A nurse is caring for a patient with continuous cardiac monitoring for heart dysrhythmias. Which rhythm will cause the nurse to intervene immediately? ANS: A Ventricular tachycardia and ventricular fibrillation are life-threatening rhythms that require immediate intervention. Ventricular tachycardia is a life-threatening dysrhythmia because of the decreased cardiac output and the potential to deteriorate into ventricular fibrillation or sudden cardiac death. Atrial fibrillation is a common dysrhythmia in older adults and is not as serious as ventricular tachycardia. Sinus rhythm is normal. Paroxysmal supraventricular tachycardia is a sudden, rapid onset of tachycardia originating above the AV node. It often begins and ends spontaneously. 34. The patient is experiencing angina pectoris. Which assessment finding does the nurse expect when conducting a history and physical examination? ANS: A Angina pectoris is chest pain that results from limited oxygen supply. Often pain is precipitated by activities such as exercise, stress, and eating a heavy meal and lasts 3 to 5 minutes. Symptoms of angina pectoris are relieved by rest and/or nitroglycerin. Adequate oxygen saturation occurs with rest; inadequate oxygen saturation occurs during exercise. Pain lasting longer than 20 minutes or arm tingling that persists could be a sign of myocardial infarction. 35. A nurse is teaching about risk factors for cardiopulmonary disease. Which risk factor should the nurse describe as modifiable? ANS: A Young and middle-age adults are exposed to multiple cardiopulmonary risk factors: an unhealthy diet, lack of exercise, stress, over-the-counter and prescription drugs not used as intended, illegal substances, and smoking. Reducing these modifiable factors decreases a patient’s risk for cardiac or pulmonary diseases. A nonmodifiable risk factor is family history; determine familial risk factors such as a family history of lung cancer or cardiovascular disease. Other nonmodifiable risk factors include allergies and gender. 36. The nurse is creating a plan of care for an obese patient who is suffering from fatigue related to ineffective breathing. Which intervention best addresses a short-term goal the patient could achieve? a. Ventricular tachycardia b. Atrial fibrillation c. Sinus rhythm d. Paroxysmal supraventricular tachycardia a. Experiences chest pain after eating a heavy meal b. Experiences adequate oxygen saturation during exercise c. Experiences crushing chest pain for more than 20 minutes d. Experiences tingling in the left arm that lasts throughout the morning a. Stress b. Allergies c. Family history d. Gender 242 A&E I Comprehensive Testbank ANS: A To achieve a short-term goal, the nurse should plan a lifestyle change that the patient can make immediately that will have a quick effect. Sleeping on several pillows at night will immediately relieve orthopnea and open the patient’s airway, thereby reducing fatigue. Running 30 minutes a day will improve cardiopulmonary health, but a patient needs to build up exercise tolerance. Smoking cessation is another process that many people have difficulty doing immediately. A more realistic short-term goal would be to gradually reduce the number of cigarettes smoked. Limiting caloric intake can help a patient lose weight, but this is a gradual process and is not reasonable for a short-term goal. 37. A nurse is caring for a patient with left-sided hemiparesis who has developed bronchitis and has a heart rate of 105 beats/min, blood pressure of 156/90 mm Hg, and respiration rate of 30 breaths/min. Which nursing diagnosis is a priority? ANS: B The most important nursing intervention is to maintain airway and circulation for this patient; therefore, Impaired gas exchange is the first nursing priority. Activity intolerance is a concern but is not the priority in this case. Risk for skin breakdown and Risk for infection are also important but do not address an immediate impairment with physiologic integrity. 38. Which nursing intervention is most effective in preventing hospital-acquired pneumonia in an older-adult patient? ANS: C The goal of the nursing action should be the prevention of pneumonia; the action that best addresses this is to cough, turn, and deep breathe to keep secretions from pooling at the base of the lungs. Humidification thins respiratory secretions, making them easier to expel and should be used. Monitoring oxygen status is important but is not a method of prevention. Hydration assists in preventing hospital-acquired pneumonia. The best way to maintain thin secretions is to provide a fluid intake of 1500 to 2500 mL/day unless contraindicated by cardiac or renal status. Restricting fluids is contraindicated in this situation since there is no data indicating cardiac or renal disease. 39. The nurse is assessing a patient with emphysema. Which assessment finding requires further follow-up with the health care provider? a. Sleeping on two to three pillows at night b. Limiting the diet to 1500 calories a day c. Running 30 minutes every morning d. Stopping smoking immediately a. Risk for skin breakdown b. Impaired gas exchange c. Activity intolerance d. Risk for infection a. Discontinue the humidification delivery device to keep excess fluid from lungs. b. Monitor oxygen saturation, and frequently auscultate lung bases. c. Assist the patient to cough, turn, and deep breathe every 2 hours. d. Decrease fluid intake to 300 mL a shift. a. Increased anterior-posterior diameter of the chest b. Accessory muscle used for breathing c. Clubbing of the fingers 243 A&E I Comprehensive Testbank ANS: D Hemoptysis is an abnormal occurrence of emphysema, and further diagnostic studies are needed to determine the cause of blood in the sputum. Clubbing of the fingers, barrel chest (increased anterior-posterior chest diameter), and accessory muscle use are all normal findings in a patient with emphysema. 40. A patient with chronic obstructive pulmonary disease (COPD) asks the nurse why clubbing occurs. Which response by the nurse is most therapeutic? ANS: A Clubbing of the nail bed can occur with COPD and other diseases that cause prolonged oxygen deficiency or chronic hypoxemia. Pursed-lipped breathing helps the alveoli stay open but is not the cause of clubbing. Loss of mental status is not a normal finding with COPD and will not result in clubbing. Low oxygen and not low circulating blood volume is the problem in COPD that results in clubbing. 41. A patient with a pneumothorax has a chest tube inserted and is placed on low constant suction. Which finding requires immediate action by the nurse? ANS: C No bubbling in the suction control chamber indicates an obstruction of the drainage system. An obstruction causes increased pressure, which can cause a tension pneumothorax, which can be life threatening. The nurse needs to determine whether the leak is inside the thorax or in the tubing and act from there. Occasional blood gushes from the lung owing to lung expansion, as during a cough; this is reserve drainage. Drainage over 100 mL/hr after 3 hours of chest tube placement is cause for concern. Yellow purulent drainage indicates an infection that should be reported to the health care provider but is not as immediately life threatening as the lack of bubbling in the suction control chamber. 42. The nurse is caring for a patient with a tracheostomy tube. Which nursing intervention is most effective in promoting effective airway clearance? ANS: B d. Hemoptysis a. “Your disease doesn’t send enough oxygen to your fingers.” b. “Your disease affects both your lungs and your heart, and not enough blood is being pumped.” c. “Your disease will be helped if you pursed-lip breathe.” d. “Your disease often makes patients lose mental status.” a. The patient reports pain at the chest tube insertion site that increases with movement. b. Fifty milliliters of blood gushes into the drainage device after the patient coughs. c. No bubbling is present in the suction control chamber of the drainage device. d. Yellow purulent discharge is seen leaking out from around the dressing site. a. Suctioning respiratory secretions several times every hour b. Administering humidified oxygen through a tracheostomy collar c. Instilling normal saline into the tracheostomy to thin secretions before suctioning d. Deflating the tracheostomy cuff before allowing the patient to cough up secretions 244 A&E I Comprehensive Testbank Humidification from air humidifiers or humidified oxygen tracheostomy collars can help prevent drying of secretions that cause occlusion. Suctioning should be done only as needed; too frequent suctioning can damage the mucosal lining, resulting in thicker secretions. Normal saline should not be instilled into a tracheostomy; research showed no benefit with this technique. The purpose of the tracheostomy cuff is to keep secretions from entering the lungs; the nurse should not deflate the tracheostomy cuff unless instructed to do so by the health care provider. 43. The nurse is educating a student nurse on caring for a patient with a chest tube. Which statement from the student nurse indicates successful learning? ANS: B Correct care of a chest tube involves knowing normal and abnormal functioning of the tube. A constant or intermittent bubbling in the water-seal chamber indicates a leak in the drainage system, and the health care provider must be notified immediately. Stripping the tube is not routinely performed as it increases pressure. If the tubing disconnects from the drainage unit, instruct the patient to exhale as much as possible and to cough. This maneuver rids the pleural space of as much air as possible. Temporarily reestablish a water seal by immersing the open end of the chest tube into a container of sterile water. The chest tube should not be clamped unless necessary; if so, the length of time clamped would be minimal to reduce the risk of pneumothorax. 44. Which coughing technique will the nurse use to help a patient clear central airways? ANS: A The huff cough stimulates a natural cough reflex and is generally effective only for clearing central airways. While exhaling, the patient opens the glottis by saying the word huff. The quad cough technique is for patients without abdominal muscle control such as those with spinal cord injuries. While the patient breathes out with a maximal expiratory effort, the patient or nurse pushes inward and upward on the abdominal muscles toward the diaphragm, causing the cough. With the cascade cough the patient takes a slow, deep breath and holds it for 2 seconds while contracting expiratory muscles. Then he or she opens the mouth and performs a series of coughs throughout exhalation, thereby coughing at progressively lowered lung volumes. This technique promotes airway clearance and a patent airway in patients with large volumes of sputum. Incentive spirometry encourages voluntary deep breathing by providing visual feedback to patients about inspiratory volume. It promotes deep breathing and prevents or treats atelectasis in the postoperative patient. 45. The nurse is suctioning a patient with a tracheostomy tube. Which action will the nurse take? a. “I should clamp the chest tube when giving the patient a bed bath.” b. “I should report if I see continuous bubbling in the water-seal chamber.” c. “I should strip the drains on the chest tube every hour to promote drainage.” d. “I should notify the health care provider first, if the chest tube becomes dislodged.” a. Huff b. Quad c. Cascade d. Incentive spirometry a. Set suction regulator at 150 to 200 mm Hg. b. Limit the length of suctioning to 10 seconds. c. Apply suction while gently rotating and inserting the catheter. d. Liberally lubricate the end of the suction catheter with a water-soluble solution. 245 A&E I Comprehensive Testbank ANS: B Suctioning passes should be limited to 10 seconds to avoid hypoxemia. Suction for a tracheostomy should be set at 100 to 150 mm Hg. Excessive lubrication can clog the catheter or occlude the airway; lubricant is not necessary for oropharyngeal or artificial airway (tracheostomy) suctioning. Suction should never be applied on insertion. 46. The nurse is caring for a patient who needs oxygen via a nasal cannula. Which task can the nurse delegate to the nursing assistive personnel? ANS: A The skill of applying (not adjusting oxygen flow) a nasal cannula or oxygen mask can be delegated to nursing assistive personnel (NAP). The nurse is responsible for assessing the patient’s respiratory system, response to oxygen therapy, and setup of oxygen therapy, including adjustment of oxygen flow rate. 47. The nurse is using a closed suction device. Which patient will be most appropriate for this suctioning method? ANS: D Closed suctioning is most often used on patients who require invasive mechanical ventilation to support their respiratory efforts because it permits continuous delivery of oxygen while suction is performed and reduces the risk of oxygen desaturation. In this case, the acute respiratory distress syndrome requires mechanical ventilation. In the presence of epiglottitis, croup, laryngospasm, or irritable airway, the entrance of a suction catheter via the nasal route causes intractable coughing, hypoxemia, and severe bronchospasm, necessitating emergency intubation or tracheostomy. The 5-year-old child with asthma would benefit from an inhaler. A chest tube is needed for the pneumothorax. 48. While the nurse is changing the ties on a tracheostomy collar, the patient coughs, dislodging the tracheostomy tube. Which action will the nurse take first? ANS: B The nurse’s first priority is to establish a stable airway by inserting a spare trach into the patient’s airway; ideally an obturator should be used. The nurse could activate the emergency response team if the patient is still unstable after the tracheostomy is placed. A patient with a tracheostomy breathes through the tube, not the nose or mouth; a face mask would not be an effective method of getting air into the lungs. Manually occluding pressure over the tracheostomy site is not appropriate and would block the patient’s only airway. MULTIPLE RESPONSE a. Applying the nasal cannula b. Adjusting the oxygen flow c. Assessing lung sounds d. Setting up the oxygen a. A 5-year-old with excessive drooling from epiglottitis b. A 5-year-old with an asthma attack following severe allergies c. A 24-year-old with a right pneumothorax following a motor vehicle accident d. A 24-year-old with acute respiratory distress syndrome requiring mechanical ventilation a. Press the emergency response button. b. Insert a spare tracheostomy with the obturator. c. Manually occlude the tracheostomy with sterile gauze. d. Place a face mask delivering 100% oxygen over the nose and mouth. 246 A&E I Comprehensive Testbank 1. A nurse is following the Ventilator Bundle standards to prevent ventilator-associated pneumonia. Which strategies is the nurse using? (Select all that apply.) ANS: B, C, E The key components of the Institute for Healthcare Improvement (IHI) Ventilator Bundle are: Elevation of the head of the bed (HOB)—elevation is 30 to 45 degrees Daily “sedation vacations” and assessment of readiness to extubate Peptic ulcer disease prophylaxis Deep venous thrombosis prophylaxis Daily oral care with chlorhexidine Monitor cuff pressure frequently to ensure that there is an adequate seal to prevent aspiration of secretions is also included. Sterile technique is used for suctioning when on ventilators. Heart failure prophylaxis is not a component. 2. A nurse is teaching a community health promotion class and discusses the flu vaccine. Which information will the nurse include in the teaching session? (Select all that apply.) ANS: A, F Annual (yearly) flu vaccines are recommended for all people 6 months and older. The inactivated flu vaccine should be given to these individuals with chronic health problems and those 50 and older. People with a known hypersensitivity to eggs or other components of the vaccine should consult their health care provider before being vaccinated. There is a flu vaccine made without egg proteins that is approved for adults 18 years of age and older. Adults with an acute febrile illness should schedule the vaccination after they have recovered. The live, attenuated nasal spray vaccine is given to people from 2 through 49 years of age if they are not pregnant or do not have certain long-term health problems such as asthma; heart, lung, or kidney disease; diabetes; or anemia. 3. A nurse is caring for a patient with sleep apnea. Which types of ventilator support should the nurse be prepared to administer for this patient? (Select all that apply.) a. Head of bed elevation to 90 degrees at all times b. Daily oral care with chlorhexidine c. Cuff monitoring for adequate seal d. Clean technique when suctioning e. Daily “sedation vacations” f. Heart failure prophylaxis a. It is given yearly. b. It is given in a series of four doses. c. It is safe for children allergic to eggs. d. It is safe for adults with acute febrile illnesses. e. The nasal spray is given to people over 50. f. The inactivated flu vaccine is given to people over 50. a. Assist-control (AC) b. Pressure support ventilation (PSV) c. Bilevel positive airway pressure (BiPAP) d. Continuous positive airway pressure (CPAP) 247 A&E I Comprehensive Testbank ANS: C, D Ventilatory support is achieved using a variety of modes, including continuous positive airway pressure (CPAP) and bilevel positive airway pressure (BiPAP). The purpose of CPAP and BiPAP is to maintain a positive airway pressure and improve alveolar ventilation. This prevents or treats atelectasis by inflating the alveoli, reducing pulmonary edema by forcing fluid out of the lungs back into circulation, and improving oxygenation in those with sleep apnea. AC, PSV, and SIMV are invasive mechanical ventilation and are not routinely used on patients with sleep apnea. AC delivers a set tidal volume (VT) with each breath, regardless of whether the breath was triggered by the patient or the ventilator. Synchronized intermittent mandatory ventilation like AC delivers a minimum number of fully assisted breaths per minute that are synchronized with the patient’s respiratory effort. Any breaths taken between volume-cycled breaths are not assisted; the volume of these breaths is determined by the patient’s strength, effort, and lung mechanics. PSV mode is often combined with SIMV mode: inspiratory pressure is added to spontaneous breaths to overcome the resistance of the endotracheal tube or to help increase the volume of the patient’s spontaneous breaths. Chapter 28: Obstructive Pulmonary Diseases Lewis: Medical-Surgical Nursing, 10th Edition MULTIPLE CHOICE 1. The nurse teaches a patient with chronic bronchitis about a new prescription for Advair Diskus (combined fluticasone and salmeterol). Which action by the patient would indicate to the nurse that teaching about medication administration has been successful? a. The patient shakes the device before use. b. The patient rapidly inhales the medication. c. The patient attaches a spacer to the Diskus. d. The patient performs huff coughing after inhalation. ANS: B The patient should inhale the medication rapidly. Otherwise the dry particles will stick to the tongue and oral mucosa and not get inhaled into the lungs. Advair Diskus is a dry powder inhaler; shaking is not recommended. Spacers are not used with dry powder inhalers. Huff coughing is a technique to move mucus into larger airways to expectorate. The patient should not huff cough or exhale forcefully after taking Advair in order to keep the medication in the lungs. 2. The nurse teaches a patient how to administer formoterol (Perforomist) through a nebulizer. Which action by the patient indicates good understanding of the teaching? a. The patient attaches a spacer before using the inhaler. b. The patient coughs vigorously after using the inhaler. c. The patient removes the facial mask when misting stops. d. The patient activates the inhaler at the onset of expiration. ANS: C A nebulizer is used to administer aerosolized medication. A mist is seen when the medication is aerosolized, and when all of the medication has been used, the misting stops. The other options refer to inhaler use. Coughing vigorously after inhaling and activating the inhaler at the onset of expiration are both incorrect techniques when using an inhaler. 3. A patient is scheduled for spirometry. Which action should the nurse take to prepare the patient for this procedure? a. Give the rescue medication immediately before testing. b. Administer oral corticosteroids 2 hours before the procedure. e. Synchronized intermittent mandatory ventilation (SIMV) 248 A&E I Comprehensive Testbank c. Withhold bronchodilators for 6 to 12 hours before the examination. d. Ensure that the patient has been NPO for several hours before the test. ANS: C Bronchodilators are held before spirometry so that a baseline assessment of airway function can be determined. Testing is repeated after bronchodilator use to determine whether the decrease in lung function is reversible. There is no need for the patient to be NPO. Oral corticosteroids should be held before spirometry. Rescue medications (which are bronchodilators) would not be given until after the baseline pulmonary function was assessed. 4. Which information will the nurse include in the asthma teaching plan for a patient being discharged? a. Use the inhaled corticosteroid when shortness of breath occurs. b. Inhale slowly and deeply when using the dry powder inhaler (DPI). c. Hold your breath for 5 seconds after using the bronchodilator inhaler. d. Tremors are an expected side effect of rapidly acting bronchodilators. ANS: D Tremors are a common side effect of short-acting b2-adrenergic (SABA) medications and not a reason to avoid using the SABA inhaler. Inhaled corticosteroids do not act rapidly to reduce dyspnea. Rapid inhalation is needed when using a DPI. The patient should hold the breath for 10 seconds after using inhalers. 5. The emergency department nurse is evaluating the effectiveness of therapy for a patient who has received treatment during an asthma attack. Which assessment finding is the best indicator that the therapy has been effective? a. No wheezes are audible. b. O2 saturation is >90%. c. Accessory muscle use has decreased. d. Respiratory rate is 16 breaths/minute. ANS: B The goal for treatment of an asthma attack is to keep the O2 saturation above 90%. The other patient data may occur when the patient is too fatigued to continue with the increased work of breathing required in an asthma attack. 6. A patient seen in the asthma clinic has recorded daily peak flow rates that are 75% of the baseline. Which action will the nurse plan to take next? a. Increase the dose of the leukotriene inhibitor. b. Teach the patient about the use of oral corticosteroids. c. Administer a bronchodilator and recheck the peak flow. d. Instruct the patient to keep the scheduled follow-up appointment. ANS: C The patient’s peak flow reading indicates that the condition is worsening (yellow zone). The patient should take the bronchodilator and recheck the peak flow. Depending on whether the patient returns to the green zone, indicating well-controlled symptoms, the patient may be prescribed oral corticosteroids or a change in dosing of other medications. Keeping the next appointment is appropriate, but the patient also needs to be taught how to control symptoms now and use the bronchodilator. 7. The nurse teaches a patient who has asthma about peak flow meter use. Which action by the patient indicates that teaching was successful? a. The patient inhales rapidly through the peak flow meter mouthpiece. 249 A&E I Comprehensive Testbank b. The patient takes montelukast (Singulair) for peak flows in the red zone. c. The patient calls the health care provider when the peak flow is in the green zone. d. The patient uses an albuterol (Ventolin HFA) inhaler for peak flows in the yellow zone. ANS: D Readings in the yellow zone indicate a decrease in peak flow. The patient should use short-acting b2- adrenergic (SABA) medications. Readings in the green zone indicate good asthma control. The patient should exhale quickly and forcefully through the peak flow meter mouthpiece to obtain the readings. Readings in the red zone do not indicate good peak flow, and the patient should take a fast-acting bronchodilator and call the health care provider for further instructions. Singulair is not indicated for acute attacks but rather is used for maintenance therapy. 8. A young adult patient who denies any history of smoking is seen in the clinic with a new diagnosis of chronic obstructive pulmonary disease (COPD). The nurse should plan to teach the patient about a. a1-antitrypsin testing. c. use of the nicotine patch. b. leukotriene modifiers. d. continuous pulse oximetry. ANS: A When COPD occurs in young patients, especially without a smoking history, a genetic deficiency in a1- antitrypsin should be suspected. Because the patient does not smoke, a nicotine patch would not be ordered. There is no indication that the patient requires continuous pulse oximetry. Leukotriene modifiers would be used in patients with asthma, not with COPD. 9. The nurse is caring for a patient with chronic obstructive pulmonary disease (COPD). Which information obtained from the patient would prompt the nurse to consult with the health care provider before administering the prescribed theophylline? a. The patient reports a recent 15-lb weight gain. b. The patient denies shortness of breath at present. c. The patient takes cimetidine (Tagamet HB) daily. d. The patient complains of coughing up green mucus. ANS: C Cimetidine interferes with the metabolism of theophylline, and concomitant administration may lead rapidly to theophylline toxicity. The other patient information would not affect whether the theophylline should be administered or not. 10. The home health nurse is visiting a patient with chronic obstructive pulmonary disease (COPD). Which nursing action is appropriate to implement for a nursing diagnosis of impaired breathing pattern related to anxiety? a. Titrate O2 to keep saturation at least 90%. b. Teach the patient how to use pursed-lip breathing. c. Discuss a high-protein, high-calorie diet with the patient. d. Suggest the use of over-the-counter sedative medications. ANS: B Pursed-lip breathing techniques assist in prolonging the expiratory phase of respiration and decrease air trapping. There is no indication that the patient requires O2 therapy or an improved diet. Sedative medications should be avoided because they decrease respiratory drive. 11. A patient with chronic obstructive pulmonary disease (COPD) has a nursing diagnosis of imbalanced nutrition: less than body requirements. Which intervention would be most appropriate for the nurse to include in the plan of care? a. Encourage increased intake of whole grains. 250 A&E I Comprehensive Testbank b. Increase the patient’s intake of fruits and fruit juices. c. Offer high-calorie protein snacks between meals and at bedtime. d. Assist the patient in choosing foods with high vegetable content. ANS: C Eating small amounts more frequently (as occurs with snacking) will increase caloric intake by decreasing the fatigue and feelings of fullness associated with large meals. Patients with COPD should rest before meals. Foods that have a lot of texture such as whole grains may take more energy to eat and get absorbed and lead to decreased intake. Although fruits, juices, and minerals are not contraindicated, foods high in protein are a better choice. 12. The nurse interviews a patient with a new diagnosis of chronic obstructive pulmonary disease (COPD). Which information is most specific in confirming a diagnosis of chronic bronchitis? a. The patient tells the nurse about a family history of bronchitis. b. The patient indicates a 30 pack-year cigarette smoking history. c. The patient reports a productive cough for 3 months every winter. d. The patient denies having respiratory problems until the past 12 months. ANS: C A diagnosis of chronic bronchitis is based on a history of having a productive cough for 3 months for at least 2 consecutive years. There is no family tendency for chronic bronchitis. Although smoking is the major risk factor for chronic bronchitis, a smoking history does not confirm the diagnosis. 13. The nurse teaches a patient about pursed-lip breathing. Which action by the patient would indicate to the nurse that further teaching is needed? a. The patient inhales slowly through the nose. b. The patient puffs up the cheeks while exhaling. c. The patient practices by blowing through a straw. d. The patient’s ratio of inhalation to exhalation is 1:3. ANS: B The patient should relax the facial muscles without puffing the cheeks while doing pursed-lip breathing. The other actions by the patient indicate a good understanding of pursed-lip breathing. 14. Which finding by the nurse for a patient with a nursing diagnosis of impaired gas exchange will be most useful in evaluating the effectiveness of treatment? a. Even, unlabored respirations c. Absence of wheezes or crackles b. Pulse oximetry reading of 92% d. Respiratory rate of 18 breaths/min ANS: B For the nursing diagnosis of impaired gas exchange, the best data for evaluation are arterial blood gases (ABGs) or pulse oximetry. The other data may indicate either improvement or impending respiratory failure caused by fatigue. 15. The nurse is caring for a patient with cor pulmonale. The nurse should monitor the patient for which expected finding? a. Chest pain c. Peripheral edema b. Finger clubbing d. Elevated temperature ANS: C Cor pulmonale causes clinical manifestations of right ventricular failure, such as peripheral edema. The other clinical manifestations may occur in the patient with other complications of chronic obstructive pulmonary disease but are not indicators of cor pulmonale. 251 A&E I Comprehensive Testbank 16. The nurse is admitting a patient diagnosed with an acute exacerbation of chronic obstructive pulmonary disease (COPD). How should the nurse determine the appropriate O2 flow rate? a. Minimize O2 use to avoid O2 dependency. b. Maintain the pulse oximetry level at 90% or greater. c. Administer O2 according to the patient’s level of dyspnea. d. Avoid administration of O2 at a rate of more than 2 L/min. ANS: B The best way to determine the appropriate O2 flow rate is by monitoring the patient’s oxygenation either by arterial blood gases (ABGs) or pulse oximetry. An O2 saturation of 90% indicates adequate blood O2 level without the danger of suppressing the respiratory drive. For patients with an exacerbation of COPD, an O2 flow rate of 2 L/min may not be adequate. Because O2 use improves survival rate in patients with COPD, there is no concern about O2 dependency. The patient’s perceived dyspnea level may be affected by other factors (e.g., anxiety) besides blood O2 level. 17. A patient hospitalized with chronic obstructive pulmonary disease (COPD) is being discharged home on O2 therapy. Which instruction should the nurse include in the discharge teaching? a. Travel is not possible with the use of O2 devices. b. O2 flow should be increased if the patient has more dyspnea. c. O2 use can improve the patient’s prognosis and quality of life. d. Storage of O2 requires large metals tanks that each last 4 to 6 hours. ANS: C The use of home O2 improves quality of life and prognosis. Because increased dyspnea may be a symptom of an acute process such as pneumonia, the patient should notify the health care provider rather than increasing the O2 flow rate if dyspnea becomes worse. O2 can be supplied using liquid, storage tanks, or concentrators, depending on individual patient circumstances. Travel is possible using portable O2 concentrators. 18. A patient is receiving 35% O2 via a Venturi mask. To ensure the correct amount of O2 delivery, which action by the nurse is important? a. Teach the patient to keep the mask on during meals. b. Keep the air entrainment ports clean and unobstructed. c. Give a high enough flow rate to keep the bag from collapsing. d. Drain moisture condensation from the corrugated tubing every hour. ANS: B The air entrainment ports regulate the O2 percentage delivered to the patient, so they must be unobstructed. The other options refer to other types of O2 devices. A high O2 flow rate is needed when giving O2by partial rebreather or nonrebreather masks. Draining O2 tubing is necessary when caring for a patient receiving mechanical ventilation. The mask can be removed or changed to a nasal cannula at a prescribed setting when the patient eats. 19. Postural drainage with percussion and vibration is ordered twice daily for a patient with chronic bronchitis. Which intervention should the nurse include in the plan of care? a. Schedule the procedure 1 hour after the patient eats. b. Maintain the patient in the lateral position for 20 minutes. c. Give the prescribed albuterol (Ventolin HFA) before the therapy. d. Perform percussion before assisting the patient to the drainage position. ANS: C 252 A&E I Comprehensive Testbank Bronchodilators are administered before chest physiotherapy. Postural drainage, percussion, and vibration should be done 1 hour before or 3 hours after meals. Patients remain in each postural drainage position for 5 minutes. Percussion is done while the patient is in the postural drainage position. 20. The nurse develops a teaching plan to help increase activity tolerance at home for an older adult with severe chronic obstructive pulmonary disease (COPD). Which instructions would be appropriate for the nurse to include in the plan of care? a. Stop exercising when you feel short of breath. b. Walk until pulse rate exceeds 130 beats/minute. c. Limit exercise to activities of daily living (ADLs). d. Walk 15 to 20 minutes a day at least 3 times/week. ANS: D Encourage the patient to walk 15 to 20 minutes a day at least three times a week with gradual increases. Shortness of breath is normal with exercise and not an indication that the patient should stop. Limiting exercise to ADLs will not improve the patient’s exercise tolerance. A 70-yr-old patient should have a pulse rate of 120 beats/min or less with exercise (80% of the maximal heart rate of 150 beats/min ). 21. A patient with severe chronic obstructive pulmonary disease (COPD) tells the nurse, “I wish I were dead! I’m just a burden on everybody.” Based on this information, which nursing diagnosis is most appropriate? a. Complicated grieving related to expectation of death b. Chronic low self-esteem related to physical dependence c. Ineffective coping related to unknown outcome of illness d. Deficient knowledge related to lack of education about COPD ANS: B The patient’s statement about not being able to do anything for himself or herself supports this diagnosis. Although deficient knowledge, complicated grieving, and ineffective coping may also be appropriate diagnoses for patients with COPD, the data for this patient do not support these diagnoses. 22. A patient with chronic obstructive pulmonary disease (COPD) has poor gas exchange. Which action by the nurse would support the patient’s ventilation? a. Have the patient rest in bed with the head elevated to 15 to 20 degrees. b. Encourage the patient to sit up at the bedside in a chair and lean forward. c. Ask the patient to rest in bed in a high-Fowler’s position with the knees flexed. d. Place the patient in the Trendelenburg position with pillows behind the head. ANS: B Patients with COPD improve the mechanics of breathing by sitting up in the “tripod” position. Resting in bed with the head elevated in a semi-Fowler’s position would be an alternative position if the patient was confined to bed, but sitting in a chair allows better ventilation. The Trendelenburg position or sitting upright in bed with the knees flexed would decrease the patient’s ability to ventilate well. 23. A 55-yr-old patient with increasing dyspnea is being evaluated for a possible diagnosis of chronic obstructive pulmonary disease (COPD). When teaching a patient about pulmonary spirometry for this condition, what is the most important question the nurse should ask? a. “Are you claustrophobic?” b. “Are you allergic to shellfish?” c. “Have you taken any bronchodilators today?” d. “Do you have any metal implants or prostheses?” ANS: C 253 A&E I Comprehensive Testbank Spirometry will help establish the COPD diagnosis. Bronchodilators should be avoided at least 6 hours before the test. Spirometry does not involve being placed in an enclosed area such as for magnetic resonance imaging (MRI). Contrast dye is not used for spirometry. The patient may still have spirometry done if metal implants or prostheses are present because they are contraindications for an MRI. 24. A young adult patient with cystic fibrosis (CF) is admitted to the hospital with increased dyspnea. Which intervention should the nurse include in the plan of care? a. Schedule a sweat chloride test. b. Arrange for a hospice nurse visit. c. Place the patient on a low-sodium diet. d. Perform chest physiotherapy every 4 hours. ANS: D Routine scheduling of airway clearance techniques is an essential intervention for patients with CF. A sweat chloride test is used to diagnose CF, but it does not provide any information about the effectiveness of therapy. There is no indication that the patient is terminally ill. Patients with CF lose excessive sodium in their sweat and require high amounts of dietary sodium. 25. A patient in the clinic with cystic fibrosis (CF) reports increased sweating and weakness during the summer months. Which action by the nurse would be most appropriate? a. Teach the patient signs of hypoglycemia. b. Have the patient add dietary salt to meals. c. Suggest decreasing intake of dietary fat and calories. d. Instruct the patient about pancreatic enzyme replacements. ANS: B Added dietary salt is indicated whenever sweating is excessive, such as during hot weather, when fever is present, or from intense physical activity. The management of pancreatic insufficiency includes pancreatic enzyme replacement of lipase, protease, and amylase (e.g., Pancreaze, Creon, Ultresa, Zenpep) administered before each meal and snack. This patient is at risk for hyponatremia based on reported symptoms. Adequate intake of fat, calories, protein, and vitamins is important. Fat-soluble vitamins (vitamins A, D, E, and K) must be supplemented because they are malabsorbed. Use of caloric supplements improves nutritional status. Hyperglycemia caused by pancreatic insufficiency is more likely to occur than hypoglycemia. 26. A young adult female patient with cystic fibrosis (CF) tells the nurse that she is considering getting married and wondering about having children. Which initial response by the nurse is best? a. “Are you aware of the normal lifespan for patients with CF?” b. “Would like more information to help you with that decision?” c. “Many women with CF do not have difficulty conceiving children.” d. “You will need to have genetic counseling before making a decision.” ANS: B The nurse’s initial response should be to assess the patient’s knowledge level and need for information. Although the lifespan for patients with CF is likely to be shorter than normal, it would not be appropriate for the nurse to address this as the initial response to the patient’s comments. The other responses have accurate information, but the nurse should first assess the patient’s understanding about the issues surrounding pregnancy. 27. A patient with chronic obstructive pulmonary disease (COPD) has coarse crackles throughout the lung fields and a chronic, nonproductive cough. Which nursing intervention will be most effective? a. Change the O2 flow rate to the highest prescribed rate. b. Teach the patient to use the Flutter airway clearance device. c. Reinforce the ongoing use of pursed-lip breathing techniques. 254 A&E I Comprehensive Testbank d. Teach the patient about consistent use of inhaled corticosteroids. ANS: B Airway clearance devices assist with moving mucus into larger airways, where it can more easily be expectorated. The other actions may be appropriate for some patients with COPD, but they are not indicated for this patient’s problem of thick mucus secretions. 28. The nurse provides dietary teaching for a patient with chronic obstructive pulmonary disease (COPD) who has a low body mass index (BMI). Which patient statement indicates that the teaching has been effective? a. “I will drink lots of fluids with my meals.” b. “I can have ice cream as a snack every day.” c. “I will exercise for 15 minutes before meals.” d. “I will decrease my intake of meat and poultry.” ANS: B High-calorie foods such as ice cream are an appropriate snack for patients with COPD. Fluid intake of 3 L/day is recommended, but fluids should be taken between meals rather than with meals to improve oral intake of solid foods. The patient should avoid exercise for an hour before meals to prevent fatigue while eating. Meat and dairy products are high in protein and are good choices for the patient with COPD. 29. Which instruction should the nurse include in an exercise teaching plan for a patient with chronic obstructive pulmonary disease (COPD)? a. “Avoid upper body exercise to prevent dyspnea.” b. “Stop exercising if you start to feel short of breath.” c. “Use the bronchodilator before you start to exercise.” d. “Breathe in and out through the mouth while you exercise.” ANS: C Use of a bronchodilator before exercise improves airflow for some patients and is recommended. Shortness of breath is normal with exercise and not a reason to stop. Patients should be taught to breathe in through the nose and out through the mouth (using a pursed-lip technique). Upper-body exercise can improve the mechanics of breathing in patients with COPD. 30. The nurse completes an admission assessment on a patient with asthma. Which information given by patient is indicates a need for a change in therapy? a. The patient uses albuterol (Ventolin HFA) before aerobic exercise. b. The patient says that the asthma symptoms are worse every spring. c. The patient’s heart rate increases after using the albuterol (Ventolin HFA) inhaler. d. The patient’s only medications are albuterol (Ventolin HFAl) and salmeterol (Serevent). ANS: D Long-acting b2-agonists should be used only in patients who also are using an inhaled corticosteroid for longterm control. Salmeterol should not be used as the first-line therapy for long-term control. Using a bronchodilator before exercise is appropriate. The other information given by the patient requires further assessment by the nurse but is not unusual for a patient with asthma. 31. The nurse takes an admission history on a patient with possible asthma who has new-onset wheezing and shortness of breath. Which information may indicate a need for a change in therapy? a. The patient has chronic inflammatory bowel disease. b. The patient has a history of pneumonia 6 months ago. 255 A&E I Comprehensive Testbank c. The patient takes propranolol (Inderal) for hypertension. d. The patient uses acetaminophen (Tylenol) for headaches. ANS: C b-Blockers such as propranolol can cause bronchospasm in some patients with asthma. The other information will be documented in the health history but does not indicate a need for a change in therapy. 32. A patient newly diagnosed with asthma is being discharged. The nurse anticipates including which topic in the discharge teaching? Use of long-acting b-adrenergic medications c. Self-administration of inhaled corticosteroids a. b. Side effects of sustained-release theophylline d. Complications associated with O2 therapy ANS: C Inhaled corticosteroids are more effective in improving asthma than any other drug and are indicated for all patients with persistent asthma. The other therapies would not typically be first-line treatments for newly diagnosed asthma. 33. A patient with cystic fibrosis (CF) has blood glucose levels that are consistently between 180 to 250 mg/ dL. Which nursing action will the nurse plan to implement? a. Discuss the role of diet in blood glucose control. b. Evaluate the patient’s use of pancreatic enzymes. c. Teach the patient about administration of insulin. d. Give oral hypoglycemic medications before meals. ANS: C The glucose levels indicate that the patient has developed CF-related diabetes, and insulin therapy is required. Because the etiology of diabetes in CF is inadequate insulin production, oral hypoglycemic agents are not effective. Patients with CF need a high-calorie diet. Inappropriate use of pancreatic enzymes would not be a cause of hyperglycemia in a patient with CF. 34. The nurse assesses a patient with a history of asthma. Which assessment finding indicates that the nurse should take immediate action? a. Pulse oximetry reading of 91% b. Respiratory rate of 26 breaths/min c. Use of accessory muscles in breathing d. Peak expiratory flow rate of 240 L/min ANS: C Use of accessory muscle indicates that the patient is experiencing respiratory distress, and rapid intervention is needed. The other data indicate the need for ongoing monitoring and assessment but do not suggest that immediate treatment is required. 35. A patient who has been experiencing an asthma attack develops bradycardia and a decrease in wheezing. Which action should the nurse take first? a. Notify the health care provider. b. Document changes in respiratory status. c. Encourage the patient to cough and deep breathe. d. Administer IV methylprednisolone (Solu-Medrol). ANS: A 256 A&E I Comprehensive Testbank The patient’s assessment indicates impending respiratory failure, and the nurse should prepare to assist with intubation and mechanical ventilation after notifying the health care provider. IV corticosteroids require several hours before having any effect on respiratory status. The patient will not be able to cough or deep breathe effectively. Documentation is not a priority at this time. OBJ: Special Questions: Prioritization 36. A patient who is experiencing an acute asthma attack is admitted to the emergency department. Which assessment should the nurse complete first? a. Listen to the patient’s breath sounds. b. Ask about inhaled corticosteroid use. c. Determine when the dyspnea started. d. Obtain the forced expiratory volume (FEV) flow rate. ANS: A Assessment of the patient’s breath sounds will help determine how effectively the patient is ventilating and whether rapid intubation may be necessary. The length of time the attack has persisted is not as important as determining the patient’s status at present. Most patients having an acute attack will be unable to cooperate with an FEV measurement. It is important to know about the medications the patient is using but not as important as assessing the breath sounds. 37. Which assessment finding in a patient who has received omalizumab (Xolair) is most important to report immediately to the health care provider? a. Pain at injection site c. Peak flow reading 75% of normal b. Flushing and dizziness d. Respiratory rate 24 breaths/minute ANS: B Flushing and dizziness may indicate that the patient is experiencing an anaphylactic reaction, and immediate intervention is needed. The other information should also be reported, but do not indicate possibly lifethreatening complications of omalizumab therapy. 38. The nurse in the emergency department receives arterial blood gas results for four recently admitted patients with obstructive pulmonary disease. The results for which patient will require the most rapid action by the nurse? a. pH 7.28, PaCO2 50 mm Hg, and PaO2 58 mm Hg b. pH 7.48, PaCO2 30 mm Hg, and PaO2 65 mm Hg c. pH 7.34, PaCO2 33 mm Hg, and PaO2 80 mm Hg d. pH 7.31, PaCO2 58 mm Hg, and PaO2 64 mm Hg ANS: A The pH, PaCO2, and PaO2 indicate that the patient has severe uncompensated respiratory acidosis and hypoxemia. Rapid action will be required to prevent increasing hypoxemia and correct the acidosis. OBJ: Special Questions: Prioritization | Special Questions: Multiple Patients 39. Which nursing action for a patient with chronic obstructive pulmonary disease (COPD) could the nurse delegate to experienced unlicensed assistive personnel (UAP)? a. Obtain O2 saturation using pulse oximetry. b. Monitor for increased O2 need with exercise. c. Teach the patient about safe use of O2 at home. d. Adjust O2 to keep saturation in prescribed parameters. ANS: A 257 A&E I Comprehensive Testbank UAP can obtain O2 saturation (after being trained and evaluated in the skill). The other actions require more education and a scope of practice that licensed practical/vocational nurses (LPN/LVNs) or registered nurses (RNs) would have. OBJ: Special Questions: Delegation 40. The clinic nurse makes a follow-up telephone call to a patient with asthma. The patient reports having a baseline peak flow reading of 600 L/min, and the current peak flow is 420 L/min. Which action should the nurse take first? a. Tell the patient to go to the hospital emergency department. b. Instruct the patient to use the prescribed albuterol (Ventolin HFA). c. Ask about recent exposure to any new allergens or asthma triggers. d. Question the patient about use of the prescribed inhaled corticosteroids. ANS: B The patient’s peak flow is 70% of normal, indicating a need for immediate use of short-acting b2-adrenergic SABA medications. Assessing for correct use of medications or exposure to allergens is also appropriate, but would not address the current decrease in peak flow. Because the patient is currently in the yellow zone, hospitalization is not needed. OBJ: Special Questions: Prioritization 41. The nurse reviews the medication administration record (MAR) for a patient having an acute asthma attack. Which medication should the nurse administer first? a. Methylprednisolone (Solu-Medrol) 60 mg IV b. Albuterol (Ventolin HFA) 2.5 mg per nebulizer c. Salmeterol (Serevent) 50 mcg per dry-powder inhaler (DPI) d. Ipratropium (Atrovent) 2 puffs per metered-dose inhaler (MDI) ANS: A Albuterol is a rapidly acting bronchodilator and is the first-line medication to reverse airway narrowing in acute asthma attacks. The other medications work more slowly. OBJ: Special Questions: Prioritization 42. The nurse receives a change-of-shift report on the following patients with chronic obstructive pulmonary disease (COPD). Which patient should the nurse assess first? a. A patient with loud expiratory wheezes b. A patient with a respiratory rate of 38 breaths/min c. A patient who has a cough productive of thick, green mucus d. A patient with jugular venous distention and peripheral edema ANS: B A respiratory rate of 38/min indicates severe respiratory distress, and the patient needs immediate assessment and intervention to prevent possible respiratory arrest. The other patients also need assessment as soon as possible, but they do not need to be assessed as urgently as the patient with tachypnea. OBJ: Special Questions: Prioritization | Special Questions: Multiple Patients 43. Which finding in a patient hospitalized with bronchiectasis is most important to report to the health care provider? a. Cough productive of bloody, purulent mucus b. Scattered crackles and wheezes heard bilaterally c. Complaint of sharp chest pain with deep breathing d. Respiratory rate 28 breaths/minute while ambulating ANS: A 258 A&E I Comprehensive Testbank Hemoptysis may indicate life-threatening hemorrhage, and should be reported immediately to the health care provider. The other findings are frequently noted in patients with bronchiectasis and may need further assessment but are not indicators of life-threatening complications. COMPLETION 1. A patient with asthma has a personal best peak expiratory flow rate (PEFR) of 400 L/min. When explaining the asthma action plan, the nurse will teach the patient that a change in therapy is needed when the PEFR is less than ___ L/minute ANS: 320 A PEFR less than 80% of the personal best indicates that the patient is in the yellow zone where changes in therapy are needed to prevent progression of the airway narrowing. Week 8 Diabetes Mellitus Chapter 48: Diabetes Mellitus Lewis: Medical-Surgical Nursing, 10th Edition MULTIPLE CHOICE 1. Which statement by a nurse to a patient newly diagnosed with type 2 diabetes is correct? a) Insulin is not used to control blood glucose in patients with type 2 diabetes. b) Complications of type 2 diabetes are less serious than those of type 1 diabetes. c) Changes in diet and exercise may control blood glucose levels in type 2 diabetes. 259 A&E I Comprehensive Testbank d) Type 2 diabetes is usually diagnosed when the patient is admitted with a hyperglycemic coma. ANS: C For some patients with type 2 diabetes, changes in lifestyle are sufficient to achieve blood glucose control. Insulin is frequently used for type 2 diabetes, complications are equally severe as for type 1 diabetes, and type 2 diabetes is usually diagnosed with routine laboratory testing or after a patient develops complications such as frequent yeast infections. 2. A patient screened for diabetes at a clinic has a fasting plasma glucose level of 120 mg/dL (6.7 mmol/L). The nurse will plan to teach the patient about a) self-monitoring of blood glucose. b) using low doses of regular insulin. c) lifestyle changes to lower blood glucose. d) effects of oral hypoglycemic medications. ANS: C The patient’s impaired fasting glucose indicates prediabetes, and the patient should be counseled about lifestyle changes to prevent the development of type 2 diabetes. The patient with prediabetes does not require insulin or oral hypoglycemics for glucose control and does not need to self-monitor blood glucose. 3. A 28-yr-old male patient with type 1 diabetes reports how he manages his exercise and glucose control. Which behavior indicates that the nurse should implement additional teaching? a) The patient always carries hard candies when engaging in exercise. b) The patient goes for a vigorous walk when his glucose is 200 mg/dL. c) The patient has a peanut butter sandwich before going for a bicycle ride. d) The patient increases daily exercise when ketones are present in the urine. ANS: D When the patient is ketotic, exercise may result in an increase in blood glucose level. Patients with type 1 diabetes should be taught to avoid exercise when ketosis is present. The other statements are correct. 4. The nurse is assessing a 22-yr-old patient experiencing the onset of symptoms of type 1 diabetes. To which question would the nurse anticipate a positive response? a) “Are you anorexic?” b) “Is your urine dark colored?” c) “Have you lost weight lately?” d) “Do you crave sugary drinks?” 260 A&E I Comprehensive Testbank ANS: C Weight loss occurs because the body is no longer able to absorb glucose and starts to break down protein and fat for energy. The patient is thirsty but does not necessarily crave sugar-containing fluids. Increased appetite is a classic symptom of type 1 diabetes. With the classic symptom of polyuria, urine will be very dilute. 5. A patient with type 2 diabetes is scheduled for a follow-up visit in the clinic several months from now. Which test will the nurse schedule to evaluate the effectiveness of treatment for the patient? a) Fasting blood glucose b) Oral glucose tolerance c) Glycosylated hemoglobin d) Urine dipstick for glucose ANS: C The glycosylated hemoglobin (A1C) test shows the overall control of glucose over 90 to 120 days. A fasting blood level indicates only the glucose level at one time. Urine glucose testing is not an accurate reflection of blood glucose level and does not reflect the glucose over a prolonged time. Oral glucose tolerance testing is done to diagnose diabetes but is not used for monitoring glucose control after diabetes has been diagnosed. 6. The nurse is assessing a 55-yr-old female patient with type 2 diabetes who has a body mass index (BMI) of 31 kg/m 2 .Which goal in the plan of care is most important for this patient? a) The patient will reach a glycosylated hemoglobin level of less than 7%. b) The patient will follow a diet and exercise plan that results in weight loss. c) The patient will choose a diet that distributes calories throughout the day. d) The patient will state the reasons for eliminating simple sugars in the diet. ANS: A The complications of diabetes are related to elevated blood glucose and the most important patient outcome is the reduction of glucose to near-normal levels. A BMI of 30?9?kg/m 2 or above is considered obese, so the other outcomes are appropriate but are not as high in priority. 7. A patient who has type 1 diabetes plans to swim laps for an hour daily at 1:00 PM. The clinic nurse will plan to teach the patient to a) check glucose level before, during, and after swimming. b) delay eating the noon meal until after the swimming class. c) increase the morning dose of neutral protamine Hagedorn (NPH) insulin. d) time the morning insulin injection so that the peak occurs while swimming. 261 A&E I Comprehensive Testbank ANS: A The change in exercise will affect blood glucose, and the patient will need to monitor glucose carefully to determine the need for changes in diet and insulin administration. Because exercise tends to decrease blood glucose, patients are advised to eat before exercising. Increasing the morning NPH or timing the insulin to peak during exercise may lead to hypoglycemia, especially with the increased exercise. 8. The nurse determines a need for additional instruction when the patient with newly diagnosed type 1 diabetes says which of the following? a) “I will need a bedtime snack because I take an evening dose of NPH insulin.” b) “I can choose any foods, as long as I use enough insulin to cover the calories.” c) “I can have an occasional beverage with alcohol if I include it in my meal plan.” d) “I will eat something at meal times to prevent hypoglycemia, even if I am not hungry.” ANS: B Most patients with type 1 diabetes need to plan diet choices very carefully. Patients who are using intensified insulin therapy have considerable flexibility in diet choices but still should restrict dietary intake of items such as fat, protein, and alcohol. The other patient statements are correct and indicate good understanding of the diet instruction. 9. To assist an older patient with diabetes to engage in moderate daily exercise, which action is most important for the nurse to take? a) Determine what types of activities the patient enjoys. b) Remind the patient that exercise improves self-esteem. c) Teach the patient about the effects of exercise on glucose level. d) Give the patient a list of activities that are moderate in intensity. ANS: A Because consistency with exercise is important, assessment for the types of exercise that the patient finds enjoyable is the most important action by the nurse in ensuring adherence to an exercise program. The other actions may be helpful but are not the most important in improving compliance. 10. Which statement by the patient indicates a need for additional instruction in administering insulin? 1. “I need to rotate injection sites among my arms, legs, and abdomen each day.” 2. “I can buy the 0.5-mL syringes because the line markings will be easier to see.” 3. “I do not need to aspirate the plunger to check for blood before injecting insulin.” 4. “I should draw up the regular insulin first, after injecting air into the NPH bottle.” ANS: A Rotating sites is no longer recommended because there is more consistent insulin absorption 262 A&E I Comprehensive Testbank when the same site is used consistently. The other patient statements are accurate and indicate that no additional instruction is needed. 11. Which patient action indicates good understanding of the nurse’s teaching about administration of aspart (NovoLog) insulin? a) The patient avoids injecting the insulin into the upper abdominal area. b) The patient cleans the skin with soap and water before insulin administration. c) The patient stores the insulin in the freezer after administering the prescribed dose. d) The patient pushes the plunger down while removing the syringe from the injection site. ANS: B Cleaning the skin with soap and water is acceptable. Insulin should not be frozen. The patient should leave the syringe in place for about 5 seconds after injection to be sure that all the insulin has been injected. The upper abdominal area is one of the preferred areas for insulin injection. 12. A patient receives aspart (NovoLog) insulin at 8:00 AM. At which time would the nurse anticipate the highest risk for hypoglycemia? a) 10:00 AM b) 12:00 AM c) 2:00 PM d) 4:00 PM ANS: A The rapid-acting insulins peak in 1 to 3 hours. The patient is not at a high risk for hypoglycemia at the other listed times, although hypoglycemia may occur. 13. Which patient action indicates a good understanding of the nurse’s teaching about the use of an insulin pump? a) The patient programs the pump for an insulin bolus after eating. b) The patient changes the location of the insertion site every week. c) The patient takes the pump off at bedtime and starts it again each morning. d) The patient plans a diet with more calories than usual when using the pump. ANS: A In addition to the basal rate of insulin infusion, the patient will adjust the pump to administer a bolus after each meal, with the dosage depending on the oral intake. The insertion site should be changed every 2 or 3 days. There is more flexibility in diet and exercise when an insulin pump is used, but it does not provide for consuming a higher calorie diet. The pump will deliver a basal insulin rate 24 hours a day. 14. A patient with diabetes is starting on intensive insulin therapy. Which type of insulin will the nurse discuss using for mealtime coverage? 263 A&E I Comprehensive Testbank a) Lispro (Humalog) b) Glargine (Lantus) c) Detemir (Levemir) d) NPH (Humulin N) ANS: A Rapid- or short-acting insulin is used for mealtime coverage for patients receiving intensive insulin therapy. NPH, glargine, or detemir will be used as the basal insulin. 15. Which information will the nurse include when teaching a patient who has type 2 diabetes about glyburide ? a) Glyburide decreases glucagon secretion from the pancreas. b) Glyburide stimulates insulin production and release from the pancreas. c) Glyburide should be taken even if the morning blood glucose level is low. d) Glyburide should not be used for 48 hours after receiving IV contrast media. ANS: B The sulfonylureas stimulate the production and release of insulin from the pancreas. If the glucose level is low, the patient should contact the health care provider before taking glyburide because hypoglycemia can occur with this class of medication. Metformin should be held for 48 hours after administration of IV contrast media, but this is not necessary for glyburide. Glucagon secretion is not affected by glyburide. 16. The nurse has been teaching a patient with type 2 diabetes about managing blood glucose levels and taking glipizide (Glucotrol). Which patient statement indicates a need for additional teaching? a) “If I overeat at a meal, I will still take the usual dose of medication.” b) “Other medications besides the Glucotrol may affect my blood sugar.” c) “When I am ill, I may have to take insulin to control my blood sugar.” d) “My diabetes won’t cause complications because I don’t need insulin.” ANS: D The patient should understand that type 2 diabetes places the patient at risk for many complications and that good glucose control is as important when taking oral agents as when using insulin. The other statements are accurate and indicate good understanding of the use of glipizide. 17. When a patient who takes metformin (Glucophage) to manage type 2 diabetes develops an allergic rash from an unknown cause, the health care provider prescribes prednisone. The nurse will anticipate that the patient may a) need a diet higher in calories while receiving prednisone. 264 A&E I Comprehensive Testbank b) develop acute hypoglycemia while taking the prednisone. c) require administration of insulin while taking prednisone. d) have rashes caused by metformin-prednisone interactions. ANS: C Glucose levels increase when patients are taking corticosteroids, and insulin may be required to control blood glucose. Hypoglycemia is not a side effect of prednisone. Rashes are not an adverse effect caused by taking metformin and prednisone simultaneously. The patient may have an increased appetite when taking prednisone but will not need a diet that is higher in calories. 18. A hospitalized diabetic patient received 38 U of NPH insulin at 7:00 AM. At 1:00 PM, the patient has been away from the nursing unit for 2 hours, missing the lunch delivery while awaiting a chest x-ray. To prevent hypoglycemia, the best action by the nurse is to a) save the lunch tray for the patient’s later return to the unit. b) ask that diagnostic testing area staff to start a 5% dextrose IV. c) send a glass of milk or orange juice to the patient in the diagnostic testing area. d) request that if testing is further delayed, the patient be returned to the unit to eat. ANS: D Consistency for mealtimes assists with regulation of blood glucose, so the best option is for the patient to have lunch at the usual time. Waiting to eat until after the procedure is likely to cause hypoglycemia. Administration of an IV solution is unnecessarily invasive for the patient. A glass of milk or juice will keep the patient from becoming hypoglycemic but will cause a rapid rise in blood glucose because of the rapid absorption of the simple carbohydrate in these items. 19. The nurse identifies a need for additional teaching when the patient who is self-monitoring blood glucose a) washes the puncture site using warm water and soap. b) chooses a puncture site in the center of the finger pad. c) hangs the arm down for a minute before puncturing the site. d) says the result of 120 mg indicates good blood sugar control. ANS: B The patient is taught to choose a puncture site at the side of the finger pad because there are fewer nerve endings along the side of the finger pad. The other patient actions indicate that teaching has been effective. 20. The nurse is preparing to teach a 43-yr-old man who is newly diagnosed with type 2 diabetes about home management of the disease. Which action should the nurse take first? a) Ask the patient’s family to participate in the diabetes education program. b) Assess the patient’s perception of what it means to have diabetes mellitus. 265 A&E I Comprehensive Testbank c) Demonstrate how to check glucose using capillary blood glucose monitoring. d) Discuss the need for the patient to actively participate in diabetes management. ANS: B Before planning teaching, the nurse should assess the patient’s interest in and ability to selfmanage the diabetes. After assessing the patient, the other nursing actions may be appropriate, but planning needs to be individualized to each patient. 21. An unresponsive patient with type 2 diabetes is brought to the emergency department and diagnosed with hyperosmolar hyperglycemic syndrome (HHS). The nurse will anticipate the need to 1. give 50% dextrose. c. initiate O2 by nasal cannula. 2. insert an IV catheter. d. administer glargine (Lantus) insulin. ANS: B HHS is initially treated with large volumes of IV fluids to correct hypovolemia. Regular insulin is administered, not a long-acting insulin. There is no indication that the patient requires O2. Dextrose solutions will increase the patient’s blood glucose and would be contraindicated. 22. A 26-yr-old female with type 1 diabetes develops a sore throat and runny nose after caring for her sick toddler. The patient calls the clinic for advice about her symptoms and a blood glucose level of 210 mg/dL despite taking her usual glargine (Lantus) and lispro (Humalog) insulin. The nurse advises the patient to a) use only the lispro insulin until the symptoms are resolved. b) limit intake of calories until the glucose is less than 120 mg/dL. c) monitor blood glucose every 4 hours and notify the clinic if it continues to rise. d) decrease intake of carbohydrates until glycosylated hemoglobin is less than 7%. ANS: C Infection and other stressors increase blood glucose levels and the patient will need to test blood glucose frequently, treat elevations appropriately with lispro insulin, and call the health care provider if glucose levels continue to be elevated. Discontinuing the glargine will contribute to hyperglycemia and may lead to diabetic ketoacidosis (DKA). Decreasing carbohydrate or caloric intake is not appropriate because the patient will need more calories when ill. Glycosylated hemoglobin testing is not used to evaluate short-term alterations in blood glucose. 23. The health care provider suspects the Somogyi effect in a 50-yr-old patient whose 6:00 AM blood glucose is 230 mg/dL. Which action will the nurse teach the patient to take? a) Avoid snacking at bedtime. b) Increase the rapid-acting insulin dose. c) Check the blood glucose during the night d) Administer a larger dose of long-acting insulin. 266 A&E I Comprehensive Testbank ANS: C If the Somogyi effect is causing the patient’s increased morning glucose level, the patient will experience hypoglycemia between 2:00 and 4:00 AM. The dose of insulin will be reduced, rather than increased. A bedtime snack is used to prevent hypoglycemic episodes during the night. 24. Which action should the nurse take after a patient treated with intramuscular glucagon for hypoglycemia regains consciousness? a) Assess the patient for symptoms of hyperglycemia. b) Give the patient a snack of peanut butter and crackers. c) Have the patient drink a glass of orange juice or nonfat milk. d) Administer a continuous infusion of 5% dextrose for 24 hours. ANS: B Rebound hypoglycemia can occur after glucagon administration, but having a meal containing complex carbohydrates plus protein and fat will help prevent hypoglycemia. Orange juice and nonfat milk will elevate blood glucose rapidly, but the cheese and crackers will stabilize blood glucose. Administration of IV glucose might be used in patients who were unable to take in nutrition orally. The patient should be assessed for symptoms of hypoglycemia after glucagon administration. 25. Which question during the assessment of a patient who has diabetes will help the nurse identify autonomic neuropathy? a) “Do you feel bloated after eating?” b) “Have you seen any skin changes?” c) “Do you need to increase your insulin dosage when you are stressed?” d) “Have you noticed any painful new ulcerations or sores on your feet?” ANS: A Autonomic neuropathy can cause delayed gastric emptying, which results in a bloated feeling for the patient. The other questions are also appropriate to ask but would not help in identifying autonomic neuropathy. 26. Which information will the nurse include in teaching a female patient who has peripheral arterial disease, type 2 diabetes, and sensory neuropathy of the feet and legs? a) Choose flat-soled leather shoes. b) Set heating pads on a low temperature. c) Use callus remover for corns or calluses. d) Soak feet in warm water for an hour each day. ANS: A The patient is taught to avoid high heels and that leather shoes are preferred. The feet should be washed, but not soaked, in warm water daily. Heating pad use should be avoided. Commercial 267 A&E I Comprehensive Testbank callus and corn removers should be avoided. The patient should see a specialist to treat these problems. 27. Which finding indicates a need to contact the health care provider before the nurse administers metformin (Glucophage)? a) The patient’s blood glucose level is 174 mg/dL. b) The patient is scheduled for a chest x-ray in an hour. c) The patient has gained 2 lb (0.9 kg) in the past 24 hours. d) The patient’s blood urea nitrogen (BUN) level is 52 mg/dL. ANS: D The BUN indicates possible renal failure, and metformin should not be used in patients with renal failure. The other findings are not contraindications to the use of metformin. 28. A patient who has diabetes and reported burning foot pain at night receives a new prescription. Which information should the nurse teach the patient about amitriptyline ? a) Amitriptyline decreases the depression caused by your foot pain. b) Amitriptyline helps prevent transmission of pain impulses to the brain. c) Amitriptyline corrects some of the blood vessel changes that cause pain. d) Amitriptyline improves sleep and makes you less aware of nighttime pain. ANS: B Tricyclic antidepressants (TCAs) decrease the transmission of pain impulses to the spinal cord and brain. TCAs also improve sleep quality and are used for depression, but that is not the major purpose for their use in diabetic neuropathy. The blood vessel changes that contribute to neuropathy are not affected by TCAs. 29. A patient who has type 2 diabetes is being prepared for an elective coronary angiogram. Which information would the nurse anticipate might lead to rescheduling the test? a) The patient’s most recent A1C was 6.5%. b) The patient’s blood glucose is 128 mg/dL. c) The patient took the prescribed metformin today. d) The patient took the prescribed captopril this morning. ANS: C To avoid lactic acidosis, metformin should be discontinued a day or 2 before the coronary angiogram and should not be used for 48 hours after IV contrast media are administered. The other patient data will also be reported but do not indicate any need to reschedule the procedure. 30. Which action by a patient indicates that the home health nurse’s teaching about glargine and regular insulin has been successful? a) The patient administers the glargine 30 minutes before each meal. 268 A&E I Comprehensive Testbank b) The patient’s family prefills the syringes with the mix of insulins weekly. c) The patient discards the open vials of glargine and regular insulin after 4 weeks. d) The patient draws up the regular insulin and then the glargine in the same syringe. ANS: C Insulin can be stored at room temperature for 4 weeks. Glargine should not be mixed with other insulins or prefilled and stored. Short-acting regular insulin is administered before meals, and glargine is given once daily. 31. A patient with diabetes rides a bicycle to and from work every day. Which site should the nurse teach the patient to use to administer the morning insulin? a. thigh. b. buttock. c. abdomen. d. upper arm. ANS: C Patients should be taught not to administer insulin into a site that will be exercised because exercise will increase the rate of absorption. The thigh, buttock, and arm are all exercised by riding a bicycle. 32. The nurse is interviewing a new patient with diabetes who takes rosiglitazone (Avandia). Which information would the nurse anticipate resulting in the health care provider discontinuing the medication? a) The patient’s blood pressure is 154/92. b) The patient’s blood glucose is 86 mg/dL. c) The patient reports a history of emphysema. d) The patient has chest pressure when walking. ANS: D Rosiglitazone can cause myocardial ischemia. The nurse should immediately notify the health care provider and expect orders to discontinue the medication. A blood glucose level of 86 mg/ dL indicates a positive effect from the medication. Hypertension and a history of emphysema do not contraindicate this medication. 33. The nurse is taking a health history from a 29-yr-old pregnant patient at the first prenatal visit. The patient reports that she has no personal history of diabetes, but her mother has diabetes. Which action will the nurse plan to take? a) Teach the patient about administering regular insulin. b) Schedule the patient for a fasting blood glucose level. c) Teach about an increased risk for fetal problems with gestational diabetes. d) Schedule an oral glucose tolerance test for the twenty-fourth week of pregnancy. 269 A&E I Comprehensive Testbank ANS: B Patients at high risk for gestational diabetes should be screened for diabetes on the initial prenatal visit. An oral glucose tolerance test may also be used to check for diabetes, but it would be done before the twenty-fourth week. Teaching plans would depend on the outcome of a fasting blood glucose test and other tests. 34. A 27-yr-old patient admitted with diabetic ketoacidosis (DKA) has a serum glucose level of 732 mg/dL and serum potassium level of 3.1 mEq/L. Which action prescribed by the health care provider should the nurse take first? a) Place the patient on a cardiac monitor. b) Administer IV potassium supplements. c) Ask the patient about home insulin doses. d) Start an insulin infusion at 0.1 units/kg/hr. ANS: A Hypokalemia can lead to potentially fatal dysrhythmias such as ventricular tachycardia and ventricular fibrillation, which would be detected with electrocardiogram (ECG) monitoring. Because potassium must be infused over at least 1 hour, the nurse should initiate cardiac monitoring before infusion of potassium. Insulin should not be administered without cardiac monitoring because insulin infusion will further decrease potassium levels. Discussion of home insulin and possible causes can wait until the patient is stabilized. 35. A patient with diabetic ketoacidosis is brought to the emergency department. Which prescribed action should the nurse implement first? a) Infuse 1 L of normal saline per hour. b) Give sodium bicarbonate 50 mEq IV push. c) Administer regular insulin 10 U by IV push. d) Start a regular insulin infusion at 0.1 units/kg/hr. ANS: A The most urgent patient problem is the hypovolemia associated with diabetic ketoacidosis (DKA), and the priority is to infuse IV fluids. The other actions can be done after the infusion of normal saline is initiated. 36. A patient who was admitted with diabetic ketoacidosis secondary to a urinary tract infection has been weaned off an insulin drip 30 minutes ago. The patient reports feeling lightheaded and sweaty. Which action should the nurse take first? a) Infuse dextrose 50% by slow IV push. b) Administer 1 mg glucagon subcutaneously. c) Obtain a glucose reading using a finger stick. d) Have the patient drink 4 ounces of orange juice. 270 A&E I Comprehensive Testbank ANS: C The patient’s clinical manifestations are consistent with hypoglycemia, and the initial action should be to check the patient’s glucose with a finger stick or order a stat blood glucose. If the glucose is low, the patient should ingest a rapid-acting carbohydrate, such as orange juice. Glucagon or dextrose 50% might be given if the patient’s symptoms become worse or if the patient is unconscious. 37. A female patient is scheduled for an oral glucose tolerance test. Which information from the patient’s health history is important for the nurse to communicate to the health care provider regarding this test? a) The patient uses oral contraceptives. b) The patient runs several days a week. c) The patient has been pregnant three times. d) The patient has a family history of diabetes. ANS: A Oral contraceptive use may falsely elevate oral glucose tolerance test (OGTT) values. Exercise and a family history of diabetes both can affect blood glucose but will not lead to misleading information from the OGTT. History of previous pregnancies may provide informational about gestational glucose tolerance but will not lead to misleading information from the OGTT. 38. Which laboratory value reported to the nurse by the unlicensed assistive personnel (UAP) indicates an urgent need for the nurse’s assessment of the patient? a) Bedtime glucose of 140 mg/dL b) Noon blood glucose of 52 mg/dL c) Fasting blood glucose of 130 mg/dL d) 2-hr postprandial glucose of 220 mg/dL ANS: B The nurse should assess the patient with a blood glucose level of 52 mg/dL for symptoms of hypoglycemia and give the patient a carbohydrate-containing beverage such as orange juice. The other values are within an acceptable range or not immediately dangerous for a patient with diabetes. 39. When a patient with type 2 diabetes is admitted for a cholecystectomy, which nursing action can the nurse delegate to a licensed practical/vocational nurse (LPN/LVN)? 1. Communicate the blood glucose level and insulin dose to the circulating nurse in surgery. 2. Discuss the reason for the use of insulin therapy during the immediate postoperative period. 271 A&E I Comprehensive Testbank 3. Administer the prescribed lispro (Humalog) insulin before transporting the patient to surgery. 4. Plan strategies to minimize the risk for hypoglycemia or hyperglycemia during the postoperative period. ANS: C LPN/LVN education and scope of practice includes administration of insulin. Communication about patient status with other departments, planning, and patient teaching are skills that require RN education and scope of practice. 40. An active 32-yr-old male who has type 1 diabetes is being seen in the endocrine clinic. Which finding indicates a need for the nurse to discuss a possible a change in therapy with the health care provider? a) Hemoglobin A1C level of 6.2% b) Blood pressure of 140/88 mmHg c) Heart rate at rest of 58 beats/minute d) High density lipoprotein (HDL) level of 65 mg/dL ANS: B To decrease the incidence of macrovascular and microvascular problems in patients with diabetes, the goal blood pressure is usually 130/80 mm Hg. An A1C less than 6.5%, a low resting heart rate (consistent with regular aerobic exercise in a young adult), and an HDL level of 65 mg/ dL all indicate that the patient’s diabetes and risk factors for vascular disease are well controlled. 41. A 30-yr-old patient has a new diagnosis of type 2 diabetes. The nurse will discuss the need to schedule a dilated eye examination 1. every 2 years. c. when the patient is 39 years old. 2. as soon as possible. d. within the first year after diagnosis. ANS: B Because many patients have some diabetic retinopathy when they are first diagnosed with type 2 diabetes, a dilated eye examination is recommended at the time of diagnosis and annually thereafter. Patients with type 1 diabetes should have dilated eye examinations starting 5 years after they are diagnosed and then annually. 42. After the nurse has finished teaching a patient who has a new prescription for exenatide (Byetta), which patient statement indicates that the teaching has been effective? a) “I may feel hungrier than usual when I take this medicine.” b) “I will not need to worry about hypoglycemia with the Byetta.” c) “I should take my daily aspirin at least an hour before the Byetta.” d) “I will take the pill at the same time I eat breakfast in the morning.” 272 A&E I Comprehensive Testbank ANS: C Because exenatide slows gastric emptying, oral medications should be taken at least 1 hour before the exenatide to avoid slowing absorption. Exenatide is injected and increases feelings of satiety. Hypoglycemia can occur with this medication. 43. A few weeks after an 82-yr-old patient with a new diagnosis of type 2 diabetes has been placed on metformin (Glucophage) therapy and taught about appropriate diet and exercise, the home health nurse makes a visit. Which finding should the nurse promptly discuss with the health care provider? a) Hemoglobin A1C level is 7.9%. b) Last eye examination was 18 months ago. c) Glomerular filtration rate is decreased. d) Patient has questions about the prescribed diet. ANS: C The decrease in renal function may indicate a need to adjust the dose of metformin or change to a different medication. In older patients, the goal for A1C may be higher in order to avoid complications associated with hypoglycemia. The nurse will plan on scheduling the patient for an eye examination and addressing the questions about diet, but the area for prompt intervention is the patient’s decreased renal function. 44. The nurse has administered 4 oz of orange juice to an alert patient whose blood glucose was 62 mg/dL. Fifteen minutes later, the blood glucose is 67 mg/dL. Which action should the nurse take next? a) Give the patient 4 to 6 oz more orange juice. b) Administer the PRN glucagon (Glucagon) 1 mg IM. c) Have the patient eat some peanut butter with crackers. d) Notify the health care provider about the hypoglycemia. ANS: A The “rule of 15” indicates that administration of quickly acting carbohydrates should be done two or three times for a conscious patient whose glucose remains less than 70 mg/dL before notifying the health care provider. More complex carbohydrates and fats may be used after the glucose has stabilized. Glucagon should be used if the patient’s level of consciousness decreases so that oral carbohydrates can no longer be given. 45. Which nursing action can the nurse delegate to experienced unlicensed assistive personnel (UAP) who are working in the diabetic clinic? a) Measure the ankle-brachial index. b) Check for changes in skin pigmentation. c) Assess for unilateral or bilateral foot drop. 273 A&E I Comprehensive Testbank d) Ask the patient about symptoms of depression. ANS: A Checking systolic pressure at the ankle and brachial areas and calculating the ankle-brachial index is a procedure that can be done by UAP who have been trained in the procedure. The other assessments require more education and critical thinking and should be done by the registered nurse (RN). 46. After change-of-shift report, which patient will the nurse assess first? e) A 19-yr-old patient with type 1 diabetes who was admitted with possible dawn f) phenomenon g) A 35-yr-old patient with type 1 diabetes whose most recent blood glucose reading h) was 230 mg/dL i) A 60-yr-old patient with hyperosmolar hyperglycemic syndrome who has poor j) skin turgor and dry oral mucosa k) A 68-yr-old patient with type 2 diabetes who has severe peripheral neuropathy and l) complains of burning foot pain ANS: C The patient’s diagnosis of HHS and signs of dehydration indicate that the nurse should rapidly assess for signs of shock and determine whether increased fluid infusion is needed. The other patients also need assessment and intervention but do not have life-threatening complications. 47. After change-of-shift report, which patient should the nurse assess first? a) A 19-yr-old patient with type 1 diabetes who has a hemoglobin A1C of 12% b) A 23-yr-old patient with type 1 diabetes who has a blood glucose of 40 mg/dL c) A 40-yr-old patient who is pregnant and whose oral glucose tolerance test is 202 mg/dL d) A 50-yr-old patient who uses exenatide (Byetta) and is complaining of acute abdominal pain ANS: B Because the brain requires glucose to function, untreated hypoglycemia can cause unconsciousness, seizures, and death. The nurse will rapidly assess and treat the patient with low blood glucose. The other patients also have symptoms that require assessments or interventions, but they are not at immediate risk for life-threatening complications. MULTIPLE RESPONSE 1. To monitor for complications in a patient with type 2 diabetes, which tests will the nurse in the diabetic clinic schedule at least annually (select all that apply)? a) Chest x-ray b) Blood pressure 274 A&E I Comprehensive Testbank c) Serum creatinine d) Urine for microalbuminuria e) Complete blood count (CBC) f) Monofilament testing of the foot ANS: B, C, D, F Blood pressure, serum creatinine, urine testing for microalbuminuria, and monofilament testing of the foot are recommended at least annually to screen for possible microvascular and macrovascular complications of diabetes. Chest x-ray and CBC might be ordered if the patient with diabetes presents with symptoms of respiratory or infectious problems but are not routinely included in screening. OTHER 1. In which order will the nurse take these steps to prepare NPH 20 units and regular insulin 2 units using the same syringe? (Put a comma and a space between each answer choice [A, B, C, D, E]). a. Rotate NPH vial. b. Withdraw regular insulin. c. Withdraw 20 units of NPH. d. Inject 20 units of air into NPH vial. e. Inject 2 units of air into regular insulin vial. ANS: A, D, E, B, C When mixing regular insulin with NPH, it is important to avoid contact between the regular insulin and the additives in the NPH that slow the onset, peak, and duration of activity in the longer-acting insulin. 275 A&E I Comprehensive Testbank Week 9 Hypertension, Stroke Chapter 32: Hypertension Lewis: Medical-Surgical Nursing, 10th Edition MULTIPLE CHOICE 1. Which action will the nurse in the hypertension clinic take to obtain an accurate baseline blood pressure (BP) for a new patient? a. Deflate the BP cuff at a rate of 5 to 10 mm Hg per second. b. Have the patient sit in a chair with the feet flat on the floor. c. Assist the patient to the supine position for BP measurements. d. Obtain two BP readings in the dominant arm and average the results. ANS: B The patient should be seated with the feet flat on the floor. The BP is obtained in both arms, and the results of the two arms are not averaged. The patient does not need to be in the supine position. The cuff should be deflated at 2 to 3 mm Hg per second. 2. The nurse obtains the following information from a patient newly diagnosed with prehypertension. Which finding is most important to address with the patient? a. Low dietary fiber intake b. No regular physical exercise c. Drinks a beer with dinner every night d. Weight is 5 pounds above ideal weight ANS: B The recommendations for preventing hypertension include exercising aerobically for 30 minutes most days of the week. A weight that is 5 pounds over the ideal body weight is not a risk factor for hypertension. The Dietary Approaches to Stop Hypertension (DASH) diet is high in fiber, but increasing fiber alone will not prevent hypertension from developing. The patient’s alcohol intake is within guidelines and will not increase the hypertension risk. 3. Which action should the nurse take when giving the initial dose of oral labetalol to a patient with hypertension? a. Encourage the use of hard candy to prevent dry mouth. b. Teach the patient that headaches often occur with this drug. c. Instruct the patient to call for help if heart palpitations occur. d. Ask the patient to request assistance before getting out of bed. ANS: D 276 A&E I Comprehensive Testbank Labetalol decreases sympathetic nervous system activity by blocking both a- and b-adrenergic receptors, leading to vasodilation and a decrease in heart rate, which can cause severe orthostatic hypotension. Heart palpitations, dry mouth, dehydration, and headaches are possible side effects of other antihypertensives. 4. After the nurse teaches the patient with stage 1 hypertension about diet modifications that should be implemented, which diet choice indicates that the teaching has been most effective? a. The patient avoids eating nuts or nut butters. b. The patient restricts intake of chicken and fish. c. The patient drinks low-fat milk with each meal. d. The patient has two cups of coffee in the morning. ANS: C For the prevention of hypertension, the Dietary Approaches to Stop Hypertension (DASH) recommendations include increasing the intake of calcium-rich foods. Caffeine restriction and decreased protein intake are not included in the recommendations. Nuts are high in beneficial nutrients and 4 to 5 servings weekly are recommended in the DASH diet. 5. A patient has just been diagnosed with hypertension and has been started on captopril . Which information is most important to include when teaching the patient about this drug? a. Include high-potassium foods such as bananas in the diet. b. Increase fluid intake if dryness of the mouth is a problem. c. Change position slowly to help prevent dizziness and falls. d. Check blood pressure (BP) in both arms before taking the drug. ANS: C The angiotensin-converting enzyme (ACE) inhibitors frequently cause orthostatic hypotension, and patients should be taught to change position slowly to allow the vascular system time to compensate for the position change. Increasing fluid intake may counteract the effect of the drug, and the patient is taught to use gum or hard candy to relieve dry mouth. The BP should be taken in the nondominant arm by newly diagnosed patients in the morning, before taking the drug, and in the evening. Because ACE inhibitors cause potassium retention, increased intake of high-potassium foods is inappropriate. 6. Propranolol (Inderal) is prescribed for a patient diagnosed with hypertension. The nurse should consult with the health care provider before giving this drug when the patient reveals a history of a. daily alcohol use. c. reactive airway disease. b. peptic ulcer disease. d. myocardial infarction (MI). ANS: C Nonselective b-blockers block b1- and b2-adrenergic receptors and can cause bronchospasm, especially in patients with a history of asthma. b-Blockers will have no effect on the patient’s peptic ulcer disease or alcohol use. b-Blocker therapy is recommended after MI. 7. A 56-yr-old patient who has no previous history of hypertension or other health problems suddenly develops a blood pressure (BP) of 198/110 mm Hg. After reconfirming the BP, it is appropriate for the nurse to tell the patient that a. a BP recheck should be scheduled in a few weeks. b. dietary sodium and fat content should be decreased. c. diagnosis, treatment, and ongoing monitoring will be needed. d. there is an immediate danger of a stroke, requiring hospitalization. ANS: C 277 A&E I Comprehensive Testbank A sudden increase in BP in a patient older than age 50 years with no previous hypertension history or risk factors indicates that the hypertension may be secondary to some other problem. The BP will need treatment and ongoing monitoring. If the patient has no other risk factors, a stroke in the immediate future is unlikely. There is no indication that dietary salt or fat intake have contributed to this sudden increase in BP, and reducing intake of salt and fat alone will not be adequate to reduce this BP to an acceptable level. 8. Which action will be included in the plan of care when the nurse is caring for a patient who is receiving nicardipine (Cardene) to treat a hypertensive emergency? a. Organize nursing activities so that the patient has undisturbed sleep for 8 hours at night. b. Keep the patient NPO to prevent aspiration caused by nausea and possible vomiting. c. Assist the patient up in the chair for meals to avoid complications associated with immobility. d. Use an automated noninvasive blood pressure machine to obtain frequent measurements. ANS: D Frequent monitoring of BP is needed when the patient is receiving rapid-acting IV antihypertensive medications. This can be most easily accomplished with an automated BP machine or arterial line. The patient will require frequent assessments, so allowing 8 hours of undisturbed sleep is not reasonable. When patients are receiving IV vasodilators, bed rest is maintained to prevent decreased cerebral perfusion and fainting. There is no indication that this patient is nauseated or at risk for aspiration, so an NPO status is unnecessary. 9. The nurse has just finished teaching a hypertensive patient about the newly prescribed drug, ramipril (Altace). Which patient statement indicates that more teaching is needed? a. “The medication may not work well if I take aspirin.” b. “I can expect some swelling around my lips and face.” c. “The doctor may order a blood potassium level occasionally.” d. “I will call the doctor if I notice that I have a frequent cough.” ANS: B Angioedema occurring with angiotensin-converting enzyme (ACE) inhibitor therapy is an indication that the ACE inhibitor should be discontinued. The patient should be taught that if any swelling of the face or oral mucosa occurs, the health care provider should be immediately notified because this could be life threatening. The other patient statements indicate that the patient has an accurate understanding of ACE inhibitor therapy. 10. During change-of-shift report, the nurse obtains the following information about a hypertensive patient who received the first dose of nadolol (Corgard) during the previous shift. Which information indicates that the patient needs immediate intervention? a. The patient’s pulse has dropped from 68 to 57 beats/min. b. The patient complains that the fingers and toes feel quite cold. c. The patient has developed wheezes throughout the lung fields. d. The patient’s blood pressure (BP) reading is now 158/91 mm Hg. ANS: C The most urgent concern for this patient is the wheezes, which indicate that bronchospasm (a common adverse effect of the noncardioselective b-blockers) is occurring. The nurse should immediately obtain an O2 saturation measurement, apply supplemental O2, and notify the health care provider. The mild decrease in heart rate and complaint of cold fingers and toes are associated with b-receptor blockade but do not require any change in therapy. The BP reading may indicate that a change in medication type or dose may be indicated. However, this is not as urgently needed as addressing the bronchospasm. 278 A&E I Comprehensive Testbank 11. An older patient has been diagnosed with possible white coat hypertension. Which planned action by the nurse best addresses the suspected cause of the hypertension? a. Instruct the patient about the need to decrease stress levels. b. Teach the patient how to self-monitor and record BPs at home. c. Schedule the patient for regular blood pressure (BP) checks in the clinic. d. Inform the patient and caregiver that major dietary changes will be needed. ANS: B In the phenomenon of “white coat” hypertension, patients have elevated BP readings in a clinical setting and normal readings when BP is measured elsewhere. Having the patient self-monitor BPs at home will provide a reliable indication about whether the patient has hypertension. Regular BP checks in the clinic are likely to be high in a patient with white coat hypertension. There is no evidence that this patient has elevated stress levels or a poor diet, and those factors do not cause white coat hypertension. 12. Which blood pressure (BP) finding by the nurse indicates that no changes in therapy are needed for a 48- yr-old patient with newly diagnosed hypertension? a. 98/56 mm Hg c. 128/92 mm Hg b. 128/76 mm Hg d. 142/78 mm Hg ANS: B The 8th Joint National Committee’s recommended goal for antihypertensive therapy for a 30- to 59-yr-old patient with hypertension is a BP below 140/90 mm Hg. The BP of 98/56 mm Hg may indicate overtreatment of the hypertension and an increased risk for adverse drug effects. The other two blood pressures indicate a need for modifications in the patient’s treatment. 13. Which information is most important for the nurse to include when teaching a patient with newly diagnosed hypertension? a. Most people are able to control BP through dietary changes. b. Annual BP checks are needed to monitor treatment effectiveness. c. Hypertension is usually asymptomatic until target organ damage occurs. d. Increasing physical activity alone controls blood pressure (BP) for most people. ANS: C Hypertension is usually asymptomatic until target organ damage has occurred. Lifestyle changes (e.g., physical activity, dietary changes) are used to help manage BP, but drugs are needed for most patients. Home BP monitoring should be taught to the patient and findings checked by the health care provider frequently when starting treatment for hypertension and then every 3 months when stable. 14. The nurse on the intermediate care unit received change-of-shift report on four patients with hypertension. Which patient should the nurse assess first? a. 48-yr-old with a blood pressure of 160/92 mm Hg who reports chest pain b. 52-yr-old with a blood pressure of 198/90 mm Hg who has intermittent claudication c. 50-yr-old with a blood pressure of 190/104 mm Hg who has a creatinine of 1.7 mg/dL d. 43-yr-old with a blood pressure of 172/98 mm Hg whose urine shows microalbuminuria ANS: A 279 A&E I Comprehensive Testbank The patient with chest pain may be experiencing acute myocardial infarction and rapid assessment and intervention are needed. The symptoms of the other patients also show target organ damage but are not indicative of acute processes. 15. The nurse is reviewing the laboratory test results for a patient who has recently been diagnosed with hypertension. Which result is most important to communicate to the health care provider? a. Serum creatinine of 2.8 mg/dL c. Serum hemoglobin of 14.7 g/dL b. Serum potassium of 4.5 mEq/L d. Blood glucose level of 96 mg/dL ANS: A The elevated serum creatinine indicates renal damage caused by the hypertension. The other laboratory results are normal. 16. A patient with a history of hypertension treated with a diuretic and an angiotensin-converting enzyme (ACE) inhibitor arrives in the emergency department complaining of a severe headache and nausea and has a blood pressure (BP) of 238/118 mm Hg. Which question should the nurse ask to follow up on these findings? a. “Have you recently taken any antihistamines?” b. “Have you consistently taken your medications?” c. “Did you take any acetaminophen (Tylenol) today?” d. “Have there been recent stressful events in your life?” ANS: B Sudden withdrawal of antihypertensive medications can cause rebound hypertension and hypertensive crisis. Although many over-the-counter medications can cause hypertension, antihistamines and acetaminophen do not increase BP. Stressful events will increase BP but not usually to the level seen in this patient. 17. The nurse is assessing a patient who has been admitted to the intensive care unit (ICU) with a hypertensive emergency. Which finding is most important to report to the health care provider? a. Urine output over 8 hours is 250 mL less than the fluid intake. b. The patient cannot move the left arm and leg when asked to do so. c. Tremors are noted in the fingers when the patient extends the arms. d. The patient complains of a headache with pain at level 7 of 10 (0 to 10 scale). ANS: B The patient’s inability to move the left arm and leg indicates that a stroke may be occurring and will require immediate action to prevent further neurologic damage. The other clinical manifestations are also likely caused by the hypertension and will require rapid nursing actions, but they do not require action as urgently as the neurologic changes. 18. A patient with hypertension who has just started taking atenolol (Tenormin) returns to the health clinic after 2 weeks for a follow-up visit. The blood pressure (BP) is unchanged from the previous visit. Which action should the nurse take first? a. Tell the patient why a change in drug dosage is needed. b. Ask the patient if the medication is being taken as prescribed. c. Inform the patient that multiple drugs are often needed to treat hypertension. d. Question the patient regarding any lifestyle changes made to help control BP. ANS: B Because nonadherence with antihypertensive therapy is common, the nurse’s initial action should be to determine whether the patient is taking the atenolol as prescribed. The other actions also may be implemented, but these would be done after assessing patient adherence with the prescribed therapy. 280 A&E I Comprehensive Testbank 19. The registered nurse (RN) is caring for a patient with a hypertensive crisis who is receiving sodium nitroprusside . Which nursing action can the nurse delegate to an experienced licensed practical/vocational nurse (LPN/LVN)? a. Evaluate effectiveness of nitroprusside therapy on blood pressure (BP). b. Assess the patient’s environment for adverse stimuli that might increase BP. c. Titrate nitroprusside to decrease mean arterial pressure (MAP) to 115 mm Hg. d. Set up the automatic noninvasive BP machine to take readings every 15 minutes. ANS: D LPN/LVN education and scope of practice include the correct use of common equipment such as automatic noninvasive blood pressure machines. The other actions require advanced nursing judgment and education, and should be done by RNs. 20. The charge nurse observes a new registered nurse (RN) doing discharge teaching for a patient with hypertension who has a new prescription for enalapril (Vasotec). The charge nurse will need to intervene if the new RN tells the patient to a. increase the dietary intake of high-potassium foods. b. make an appointment with the dietitian for teaching. c. check the blood pressure (BP) at home at least once a day. d. move slowly when moving from lying to sitting to standing. ANS: A The ACE inhibitors cause retention of potassium by the kidney, so hyperkalemia is a possible adverse effect. The other teaching by the new RN is appropriate for a patient with newly diagnosed hypertension who has just started therapy with enalapril. 21. Which assessment finding for a patient who is receiving IV furosemide (Lasix) to treat stage 2 hypertension is most important to report to the health care provider? a. Blood glucose level of 175 mg/dL b. Serum potassium level of 3.0 mEq/L c. Orthostatic systolic BP decrease of 12 mm Hg d. Most recent blood pressure (BP) reading of 168/94 mm Hg ANS: B Hypokalemia is a frequent adverse effect of the loop diuretics and can cause life-threatening dysrhythmias. The health care provider should be notified of the potassium level immediately and administration of potassium supplements initiated. The elevated blood glucose and BP also indicate a need for collaborative interventions but will not require action as urgently as the hypokalemia. An orthostatic drop of 12 mm Hg will require intervention only if the patient is symptomatic. 22. Which nursing action should the nurse take first to assist a patient with newly diagnosed stage 1 hypertension in making needed dietary changes? a. Collect a detailed diet history. b. Provide a list of low-sodium foods. c. Help the patient make an appointment with a dietitian. d. Teach the patient about foods that are high in potassium. ANS: A The initial nursing action should be assessment of the patient’s baseline dietary intake through a thorough diet history. The other actions may be appropriate, but assessment of the patient’s baseline should occur first. 281 A&E I Comprehensive Testbank 23. The nurse is caring for a 70-yr-old patient who uses hydrochlorothiazide and enalapril (Norvasc) but whose self-monitored blood pressure (BP) continues to be elevated. Which patient information may indicate a need for a change? a. Patient takes a daily multivitamin tablet. b. Patient checks BP daily just after getting up. c. Patient drinks wine three to four times a week. d. Patient uses ibuprofen (Motrin) treat osteoarthritis. ANS: D Because use of nonsteroidal antiinflammatory drugs (NSAIDs) can prevent adequate BP control, the patient may need to avoid the use of ibuprofen. A multivitamin tablet will help supply vitamin D, which may help lower BP. BP decreases while sleeping, so self-monitoring early in the morning will result in obtaining pressures that are at their lowest. The patient’s alcohol intake is not excessive. SHORT ANSWER 1. The nurse obtains a blood pressure of 176/82 mm Hg for a patient. What is the patient’s mean arterial pressure (MAP)? ANS: 113 mm Hg MAP = (SBP + 2 DBP)/3 Chapter 57: Stroke Lewis: Medical-Surgical Nursing, 10th Edition MULTIPLE CHOICE 1. After a patient experienced a brief episode of tinnitus, diplopia, and dysarthria with no residual effects, the nurse anticipates teaching the patient about a. cerebral aneurysm clipping. c. oral low-dose aspirin therapy. b. heparin intravenous infusion. d. tissue plasminogen activator (tPA). ANS: C The patient’s symptoms are consistent with transient ischemic attack (TIA), and drugs that inhibit platelet aggregation are prescribed after a TIA to prevent a stroke. Continuous heparin infusion is not routinely used after TIA or with acute ischemic stroke. The patient’s symptoms are not consistent with a cerebral aneurysm. tPA is used only for acute ischemic stroke, not for TIA. 2. A patient is being admitted with a possible stroke. Which information from the assessment indicates that the nurse should consult with the health care provider before giving the prescribed aspirin? a. The patient has dysphasia. b. The patient has atrial fibrillation. c. The patient reports that symptoms began with a severe headache. d. The patient has a history of brief episodes of right-sided hemiplegia. ANS: C A sudden onset headache is typical of a subarachnoid hemorrhage, and aspirin is contraindicated. Atrial fibrillation, dysphasia, and transient ischemic attack are not contraindications to aspirin use. 282 A&E I Comprehensive Testbank 3. A patient with a stroke experiences facial drooping on the right side and right-sided arm and leg paralysis. When admitting the patient, which clinical manifestation will the nurse expect to find? a. Impulsive behavior b. Right-sided neglect c. Hyperactive left-sided tendon reflexes d. Difficulty comprehending instructions ANS: D Right-sided paralysis indicates a left-brain stroke, which will lead to difficulty with comprehension and use of language. The left-side reflexes are likely to be intact. Impulsive behavior and neglect are more likely with a right-side stroke. 4. During the change of shift report, a nurse is told that a patient has an occluded left posterior cerebral artery. The nurse will anticipate that the patient may have a. dysphasia. c. visual deficits. b. confusion. d. poor judgment. ANS: C Visual disturbances are expected with posterior cerebral artery occlusion. Aphasia occurs with middle cerebral artery involvement. Cognitive deficits and changes in judgment are more typical of anterior cerebral artery occlusion. 5. When teaching about clopidogrel (Plavix), the nurse will tell the patient with cerebral atherosclerosis a. to monitor and record the blood pressure daily. b. to call the health care provider if stools are tarry. c. that clopidogrel will dissolve clots in the cerebral arteries. d. that clopidogrel will reduce cerebral artery plaque formation. ANS: B Clopidogrel inhibits platelet function and increases the risk for gastrointestinal bleeding, so patients should be advised to notify the health care provider about any signs of bleeding. The medication does not lower blood pressure, decrease plaque formation, or dissolve clots. 6. A patient with carotid atherosclerosis asks the nurse to describe a carotid endarterectomy. Which response by the nurse is accurate? a. “The obstructing plaque is surgically removed from inside an artery in the neck.” b. “The diseased portion of the artery in the brain is replaced with a synthetic graft.” c. “A wire is threaded through an artery in the leg to the clots in the carotid artery, and the clots are removed.” d. “A catheter with a deflated balloon is positioned at the narrow area, and the balloon is inflated to flatten the plaque.” ANS: A In a carotid endarterectomy, the carotid artery is incised, and the plaque is removed. The response beginning, “The diseased portion of the artery in the brain is replaced” describes an arterial graft procedure. The answer beginning, “A catheter with a deflated balloon is positioned at the narrow area” describes an angioplasty. The final response beginning, “A wire is threaded through the artery” describes the mechanical embolus removal in cerebral ischemia (MERCI) procedure. 7. A patient admitted with possible stroke has been aphasic for 3 hours, and his current blood pressure (BP) is 174/94 mm Hg. Which order by the health care provider should the nurse question? a. Keep head of bed elevated at least 30 degrees. 283 A&E I Comprehensive Testbank b. Infuse normal saline intravenously at 75 mL/hr. c. Start a labetalol drip to keep BP less than 140/90 mm Hg. d. Administer tissue plasminogen activator (tPA) intravenously per protocol. ANS: C Because elevated BP may be a protective response to maintain cerebral perfusion, antihypertensive therapy is recommended only if mean arterial pressure (MAP) is greater than130 mm Hg or systolic pressure is greater than 220 mm Hg. Fluid intake should be 1500 to 2000 mL/day to maintain cerebral blood flow. The head of the bed should be elevated to at least 30 degrees unless the patient has symptoms of poor tissue perfusion. tPA may be administered if the patient meets the other criteria for tPA use. 8. A patient arrives in the emergency department with hemiparesis and dysarthria that started 2 hours previously, and health records show a history of several transient ischemic attacks (TIAs). The nurse anticipates preparing the patient for a. surgical endarterectomy. b. transluminal angioplasty. c. intravenous heparin drip administration. d. tissue plasminogen activator (tPA) infusion. ANS: D The patient’s history and clinical manifestations suggest an acute ischemic stroke, and a patient who is seen within 4.5 hours of stroke onset is likely to receive tPA (after screening with a CT scan). Heparin administration in the emergency phase is not indicated. Emergent carotid transluminal angioplasty or endarterectomy is not indicated for the patient who is having an acute ischemic stroke. 9. A female patient who had a stroke 24 hours ago has expressive aphasia. An appropriate nursing intervention to help the patient communicate is to a. ask questions that the patient can answer with “yes” or “no.” b. develop a list of words that the patient can read and practice reciting. c. have the patient practice her facial and tongue exercises with a mirror. d. prevent embarrassing the patient by answering for her if she does not respond. ANS: A Communication will be facilitated and less frustrating to the patient when questions that require a “yes” or “no” response are used. When the language areas of the brain are injured, the patient might not be able to read or recite words, which will frustrate the patient without improving communication. Expressive aphasia is caused by damage to the language areas of the brain, not by the areas that control the motor aspects of speech. The nurse should allow time for the patient to respond. 10. For a patient who had a right hemisphere stroke, the nurse anticipates planning interventions to manage a. impaired physical mobility related to right-sided hemiplegia. b. risk for injury related to denial of deficits and impulsiveness. c. impaired verbal communication related to speech-language deficits. d. ineffective coping related to depression and distress about disability. ANS: B The patient with right-sided brain damage typically denies any deficits and has poor impulse control, leading to risk for injury when the patient attempts activities such as transferring from a bed to a chair. Right-sided brain damage causes left hemiplegia. Left-sided brain damage typically causes language deficits. Left-sided brain damage is associated with depression and distress about the disability. 11. When caring for a patient with a new right-sided homonymous hemianopsia resulting from a stroke, which intervention should the nurse include in the plan of care? 284 A&E I Comprehensive Testbank a. Apply an eye patch to the right eye. b. Approach the patient from the right side. c. Place needed objects on the patient’s left side. d. Teach the patient that the left visual deficit will resolve. ANS: C During the acute period, the nurse should place objects on the patient’s unaffected side. Because there is a visual defect in the right half of each eye, an eye patch is not appropriate. The patient should be approached from the left side. The visual deficit may not resolve, although the patient can learn to compensate for the defect. 12. A left-handed patient with left-sided hemiplegia has difficulty feeding himself. Which intervention should the nurse include in the plan of care? a. Provide a wide variety of food choices. b. Provide oral care before and after meals. c. Assist the patient to eat with the right hand. d. Teach the patient the “chin-tuck” technique. ANS: C Because the patient has difficulty feeding himself, the appropriate interventions will focus on teaching the patient to use the right hand for self-feeding. The other interventions are appropriate for patients with other etiologies for the imbalanced nutrition. 13. A patient has a ruptured cerebral aneurysm and subarachnoid hemorrhage. Which intervention will the nurse include in the plan of care? a. Apply intermittent pneumatic compression stockings. b. Assist to dangle on edge of bed and assess for dizziness. c. Encourage patient to cough and deep breathe every 4 hours. d. Insert an oropharyngeal airway to prevent airway obstruction. ANS: A The patient with a subarachnoid hemorrhage usually has minimal activity to prevent cerebral vasospasm or further bleeding and is at risk for venous thromboembolism. Activities such as coughing and sitting up that might increase intracranial pressure or decrease cerebral blood flow are avoided. Because there is no indication that the patient is unconscious, an oropharyngeal airway is inappropriate. 14. A patient will attempt oral feedings for the first time after having a stroke. The nurse should assess the gag reflex and then a. order a varied pureed diet. c. assist the patient into a chair. b. assess the patient’s appetite. d. offer the patient a sip of juice. ANS: C The patient should be as upright as possible before attempting feeding to make swallowing easier and decrease aspiration risk. To assess swallowing ability, the nurse should initially offer water or ice to the patient. Pureed diets are not recommended because the texture is too smooth. The patient may have a poor appetite, but the oral feeding should be attempted. 15. A male patient who has right-sided weakness after a stroke is making progress in learning to use the left hand for feeding and other activities. The nurse observes that when the patient’s wife is visiting, she feeds and dresses him. Which nursing diagnosis is most appropriate for the patient? a. Interrupted family processes related to effects of illness of a family member b. Situational low self-esteem related to increasing dependence on spouse for care c. Disabled family coping related to inadequate understanding by patient’s spouse 285 A&E I Comprehensive Testbank d. Impaired nutrition: less than body requirements related to hemiplegia and aphasia ANS: C The information supports the diagnosis of disabled family coping because the wife does not understand the rehabilitation program. There are no data supporting low self-esteem, and the patient is attempting independence. The data do not support an interruption in family processes because this may be a typical pattern for the couple. There is no indication that the patient has impaired nutrition. 16. Several weeks after a stroke, a 50-yr-old male patient has impaired awareness of bladder fullness, resulting in urinary incontinence. Which nursing intervention should be planned to begin an effective bladder training program? a. Limit fluid intake to 1200 mL daily to reduce urine volume. b. Assist the patient onto the bedside commode every 2 hours. c. Perform intermittent catheterization after each voiding to check for residual urine. d. Use an external “condom” catheter to protect the skin and prevent embarrassment. ANS: B Developing a regular voiding schedule will prevent incontinence and may increase patient awareness of a full bladder. A 1200-mL fluid restriction may lead to dehydration. Intermittent catheterization and use of a condom catheter are appropriate in the acute phase of stroke, but should not be considered solutions for long-term management because of the risks for urinary tract infection and skin breakdown. 17. A patient who has a history of a transient ischemic attack (TIA) has an order for aspirin 160 mg daily. When the nurse is administering medications, the patient says, “I don’t need the aspirin today. I don’t have a fever.” Which action should the nurse take? a. Document that the aspirin was refused by the patient. b. Tell the patient that the aspirin is used to prevent a fever. c. Explain that the aspirin is ordered to decrease stroke risk. d. Call the health care provider to clarify the medication order. ANS: C Aspirin is ordered to prevent stroke in patients who have experienced TIAs. Documentation of the patient’s refusal to take the medication is an inadequate response by the nurse. There is no need to clarify the order with the health care provider. The aspirin is not ordered to prevent aches and pains. 18. A patient in the clinic reports a recent episode of dysphasia and left-sided weakness at home that resolved after 2 hours. The nurse will anticipate teaching the patient about a. tPA. c. warfarin (Coumadin). b. aspirin . d. nimodipine ANS: B After a transient ischemic attack, patients typically are started on medications such as aspirin to inhibit platelet function and decrease stroke risk. tPA is used for acute ischemic stroke. Coumadin is usually used for patients with atrial fibrillation. Nimodipine is used to prevent cerebral vasospasm after a subarachnoid hemorrhage. 19. A patient with a left-brain stroke suddenly bursts into tears when family members visit. The nurse should a. use a calm voice to ask the patient to stop the crying behavior. b. explain to the family that depression is normal following a stroke. c. have the family members leave the patient alone for a few minutes. d. teach the family that emotional outbursts are common after strokes. ANS: D 286 A&E I Comprehensive Testbank Patients who have left-sided brain stroke are prone to emotional outbursts that are not necessarily related to the emotional state of the patient. Depression after a stroke is common, but the suddenness of the patient’s outburst suggests that depression is not the major cause of the behavior. The family should stay with the patient. The crying is not within the patient’s control, and asking the patient to stop will lead to embarrassment. 20. Which stroke risk factor for a 48-yr-old male patient in the clinic is most important for the nurse to address? a. The patient is 25 lb above the ideal weight. b. The patient drinks a glass of red wine with dinner daily. c. The patient’s usual blood pressure (BP) is 170/94 mm Hg. d. The patient works at a desk and relaxes by watching television. ANS: C Hypertension is the single most important modifiable risk factor. People who drink more than 1 (for women) or 2 (for men) alcoholic beverages a day may increase their risk for hypertension. Physical inactivity and obesity contribute to stroke risk but not as much as hypertension. 21. A patient in the emergency department with sudden-onset right-sided weakness is diagnosed with an intracerebral hemorrhage. Which information about the patient is most important to communicate to the health care provider? a. The patient’s speech is difficult to understand. b. The patient’s blood pressure (BP) is 144/90 mm Hg. c. The patient takes a diuretic because of a history of hypertension. d. The patient has atrial fibrillation and takes warfarin (Coumadin). ANS: D The use of warfarin probably contributed to the intracerebral bleeding and remains a risk factor for further bleeding. Administration of vitamin K is needed to reverse the effects of the warfarin, especially if the patient is to have surgery to correct the bleeding. The history of hypertension is a risk factor for the patient but has no immediate effect on the patient’s care. The BP of 144/90 indicates the need for ongoing monitoring but not for any immediate change in therapy. Slurred speech is consistent with a left-sided stroke, and no change in therapy is indicated. 22. A patient with left-sided weakness that started 60 minutes earlier is admitted to the emergency department and diagnostic tests are ordered. Which test should be done first? a. Complete blood count (CBC) b. Chest radiograph (chest x-ray) c. Computed tomography (CT) scan d. 12-Lead electrocardiogram (ECG) ANS: C Rapid screening with a noncontrast CT scan is needed before administration of tissue plasminogen activator (tPA), which must be given within 4.5 hours of the onset of clinical manifestations of the stroke. The sooner the tPA is given, the less brain injury. The other diagnostic tests give information about possible causes of the stroke and do not need to be completed as urgently as the CT scan. 23. Nurses in change-of-shift report are discussing the care of a patient with a stroke who has progressively increasing weakness and decreasing level of consciousness. Which patient problem do they determine has the highest priority for the patient? a. Impaired physical mobility related to weakness b. Disturbed sensory perception related to brain injury c. Risk for impaired skin integrity related to immobility d. Risk for aspiration related to inability to protect airway 287 A&E I Comprehensive Testbank ANS: D Protection of the airway is the priority of nursing care for a patient having an acute stroke. The other diagnoses are also appropriate, but interventions to prevent aspiration are the priority at this time. 24. Which information about the patient who has had a subarachnoid hemorrhage is most important to communicate to the health care provider? a. The patient complains of having a stiff neck. b. The patient’s blood pressure (BP) is 90/50 mm Hg. c. The patient reports a severe and unrelenting headache. d. The cerebrospinal fluid (CSF) report shows red blood cells (RBCs). ANS: B To prevent cerebral vasospasm and maintain cerebral perfusion, BP needs to be maintained at a level higher than 90 mm Hg systolic after a subarachnoid hemorrhage. A low BP or drop in BP indicates a need to administer fluids and/or vasopressors to increase the BP. An ongoing headache, RBCs in the CSF, and a stiff neck are all typical clinical manifestations of a subarachnoid hemorrhage and do not need to be rapidly communicated to the health care provider. 25. The nurse is caring for a patient who has been experiencing stroke symptoms for 60 minutes. Which action can the nurse delegate to a licensed practical/vocational nurse (LPN/LVN)? a. Assess the patient’s gag and cough reflexes. b. Determine when the stroke symptoms began. c. Administer the prescribed short-acting insulin. d. Infuse the prescribed IV metoprolol (Lopressor). ANS: C Administration of subcutaneous medications is included in LPN/LVN education and scope of practice. The other actions require more education and scope of practice and should be done by the registered nurse (RN). 26. After receiving change-of-shift report on the following four patients, which patient should the nurse see first? a. A 60-yr-old patient with right-sided weakness who has an infusion of tPA prescribed b. A 50-yr-old patient who has atrial fibrillation and a new order for warfarin (Coumadin) c. A 30-yr-old patient with a subarachnoid hemorrhage 2 days ago who has nimodipine scheduled d. A 40-yr-old patient who experienced a transient ischemic attack yesterday who has a dose of aspirin due ANS: A tPA needs to be infused within the first few hours after stroke symptoms start in order to be effective in minimizing brain injury. The other medications should also be given as quickly as possible, but timing of the medications is not as critical. 27. The nurse is caring for a patient who has just returned after having left carotid artery angioplasty and stenting. Which assessment information is of most concern to the nurse? a. The pulse rate is 102 beats/min. b. The patient has difficulty speaking. c. The blood pressure is 144/86 mm Hg. d. There are fine crackles at the lung bases. ANS: B Small emboli can occur during carotid artery angioplasty and stenting, and the aphasia indicates a possible stroke during the procedure. Slightly elevated pulse rate and blood pressure are not unusual because of anxiety 288 A&E I Comprehensive Testbank associated with the procedure. Fine crackles at the lung bases may indicate atelectasis caused by immobility during the procedure. The nurse should have the patient take some deep breaths. 28. A 70-yr-old female patient with left-sided hemiparesis arrives by ambulance to the emergency department. Which action should the nurse take first? a. Take the patient’s blood pressure. b. Check the respiratory rate and effort. c. Assess the Glasgow Coma Scale score. d. Send the patient for a computed tomography (CT) scan. ANS: B The initial nursing action should be to assess the airway and take any needed actions to ensure a patent airway. The other activities should take place quickly after the ABCs (airway, breathing, and circulation) are completed. 29. The home health nurse is caring for an 81-yr-old who had a stroke 2 months ago. Based on information shown in the accompanying figure from the history, physical assessment, and physical and occupational therapy, which problem is the highest priority? History Physical Assessment Physical/Occupational Therapy · Well controlled type 2 diabetes for 10 years · Married 45 years; spouse has heart failure and chronic obstructive pulmonary disease · Oriented to time, place, person · Speech clear ·Minimal left leg weakness · Uses cane with walking · Spouse does household cleaning and cooking and assists patient with bathing and dressing a. Risk for hypoglycemia c. Risk for caregiver role strain b. Impaired transfer ability d. Ineffective health maintenance ANS: C The spouse’s household and patient care responsibilities, in combination with chronic illnesses, indicate a high risk for caregiver role strain. The nurse should further assess the situation and take appropriate actions. The data about the control of the patient’s diabetes indicates that ineffective health maintenance and risk for unstable blood glucose are not priority concerns at this time. Because the patient is able to ambulate with a cane, the nursing diagnosis of impaired transfer ability is not supported. OTHER 1. A 63-yr-old patient who began experiencing right arm and leg weakness is admitted to the emergency department. In which order will the nurse implement these actions included in the stroke protocol? (Put a comma and a space between each answer choice [A, B, C, D].) a. Obtain CT scan without contrast. b. Infuse tissue plasminogen activator (tPA). c. Administer oxygen to keep O2 saturation >95%. d. Use National Institute of Health Stroke Scale to assess patient. ANS: 289 A&E I Comprehensive Testbank C, D, A, B The initial actions should be those that help with airway, breathing, and circulation. Baseline neurologic assessments should be done next. A CT scan will be needed to rule out hemorrhagic stroke before tPA can be administered. Week 10 Documentation and Informatics Chapter 26: Documentation and Informatics Potter et al.: Fundamentals of Nursing, 9th Edition MULTIPLE CHOICE 1. A nurse preceptor is working with a student nurse. Which behavior by the student nurse will require the nurse preceptor to intervene? a. The student nurse reads the patient’s plan of care. 290 A&E I Comprehensive Testbank ANS: C When you are a student in a clinical setting, confidentiality and compliance with the Health Insurance Portability and Accountability Act (HIPAA) are part of professional practice. When a student nurse shares patient information with a friend, confidentiality and HIPAA standards have been violated, causing the preceptor to intervene. You can review your patients’ medical records only to seek information needed to provide safe and effective patient care. For example, when you are assigned to care for a patient, you need to review the patient’s medical record and plan of care. You do not share this information with classmates and you do not access the medical records of other patients on the unit. 2. A nurse exchanges information with the oncoming nurse about a patient’s care. Which action did the nurse complete? ANS: A Whether the transfer of patient information occurs through verbal reports, electronic or written documents, you need to follow some basic principles. Reports are exchanges of information among caregivers. A patient’s electronic medical record or chart is a confidential, permanent legal documentation of information relevant to a patient’s health care. Nurses document referrals (arrangements for the services of another care provider). Nurses use acuity ratings to determine the hours of care and number of staff required for a given group of patients every shift or every 24 hours. 3. A nurse is auditing and monitoring patients’ health records. Which action is the nurse taking? ANS: A The auditing and monitoring of patients’ health records involve nurses periodically auditing records to determine the degree to which standards of care are met and identifying areas needing improvement and staff development. The mistakes in documentation that commonly result in malpractice include failing to record nursing actions; this is the aspect of legal documentation. The financial billing or reimbursement purpose involves diagnosis-related groups (DRGs) as the basis for establishing reimbursement for patient care. For research purposes, the researcher compares the patient’s recorded findings to determine whether the new method was more effective than the standard protocol. Data analysis contributes to evidence-based nursing practice and quality health care. 4. After providing care, a nurse charts in the patient’s record. Which entry will the nurse document? b. The student nurse reviews the patient’s medical record. c. The student nurse shares patient information with a friend. d. The student nurse documents medication administered to the patient. a. A verbal report b. An electronic record entry c. A referral d. An acuity rating a. Determining the degree to which standards of care are met by reviewing patients’ health records b. Realizing that care not documented in patients’ health records still qualifies as care provided c. Basing reimbursement upon the diagnosis-related groups documented in patients’ records d. Comparing data in patients’ records to determine whether a new treatment had better outcomes than the standard treatment a. Appears restless when sitting in the chair b. Drank adequate amounts of water 291 A&E I Comprehensive Testbank ANS: D A factual record contains descriptive, objective information about what a nurse observes, hears, palpates, and smells. Objective data is obtained through direct observation and measurement (skin pale and cool). For example, “B/P 80/50, patient diaphoretic, heart rate 102 and regular.” Avoid vague terms such as appears, seems, or apparently because these words suggest that you are stating an opinion, do not accurately communicate facts, and do not inform another caregiver of details regarding behaviors exhibited by the patient. Use of exact measurements establishes accuracy. For example, a description such as “Intake, 360 mL of water” is more accurate than “Patient drank an adequate amount of fluid.” 5. A nurse has provided care to a patient. Which entry should the nurse document in the patient’s record? ANS: C Use of exact measurements establishes accuracy. Charting that an abdominal wound is “approximated, 5 cm in length without redness, drainage, or edema,” is more descriptive than “large abdominal incision healing well.” Include objective data to support subjective data, so your charting is as descriptive as possible. Avoid using generalized, empty phrases such as “status unchanged” or “had good day.” It is essential to avoid the use of unnecessary words and irrelevant details or personal opinions. “Patient is hard to care for” is a personal opinion and should be avoided. It is also a critical comment that can be used as evidence for nonprofessional behavior or poor quality of care. Just chart, “Refuses all treatments and medications.” 6. A preceptor is working with a new nurse on documentation. Which situation will cause the preceptor to follow up? ANS: B Chart consecutively, line by line (not every other line); every other line is incorrect and must be corrected by the preceptor. If space is left, draw a line horizontally through it, and place your signature and credentials at the end. Every other line should not be left blank. All the other behaviors are correct and need no follow-up. Documenting only for yourself is an appropriate behavior. End each entry with signature and title/credentials. For computer documentation, keep your password to yourself. 7. A nurse is charting on a patient’s record. Which action will the nurse take that is accurate legally? ANS: A c. Apparently is asleep with eyes closed d. Skin pale and cool a. Status unchanged, doing well b. Patient seems to be in pain and states, “I feel uncomfortable.” c. Left knee incision 1 inch in length without redness, drainage, or edema d. Patient is hard to care for and refuses all treatments and medications. Family is present. a. The new nurse documents only for self. b. The new nurse charts consecutively on every other line. c. The new nurse ends each entry with signature and title. d. The new nurse keeps the password secure. a. Charts legibly b. States the patient is belligerent c. Writes entry for another nurse d. Uses correction fluid to correct error 292 A&E I Comprehensive Testbank Record all entries legibly. Do not write personal opinions (belligerent). Enter only objective and factual observations of patient’s behavior; quote all patient comments. Do not erase, apply correction fluid, or scratch out errors made while recording. Chart only for yourself. 8. A nurse wants to find all the pertinent patient information in one record, regardless of the number of times the patient entered the health care system. Which record should the nurse find? ANS: B The term electronic health record/EHR is increasingly used to refer to a longitudinal (lifetime) record of all health care encounters for an individual patient by linking all patient data from previous health encounters. An electronic medical record (EMR) is the legal record that describes a single encounter or visit created in hospitals and outpatient health care settings that is the source of data for the EHR. There are no such terms as electronic charting record or electronic problem record that record the lifetime information of a patient. 9. A nurse has taught the patient how to use crutches. The patient went up and down the stairs using crutches with no difficulties. Which information will the nurse use for the “I” in PIE charting? ANS: B A second progress note method is the PIE format. The narrative note includes P—Nursing diagnosis, I— Intervention, and E—Evaluation. The intervention is “Demonstrated use of crutches.” “Patient went up and down stairs” and “Used crutches with no difficulties” are examples of E. “Deficient knowledge regarding crutches” is P. 10. A nurse wants to find the daily weights of a patient. Which form will the nurse use? ANS: D Within a computerized documentation system, flow sheets and graphic records allow you to quickly and easily enter assessment data about a patient, such as vital signs, admission and or daily weights, and percentage of meals eaten. In the problem-oriented medical record, the database section contains all available assessment information pertaining to the patient (e.g., history and physical examination, nursing admission history and ongoing assessment, physical therapy assessment, laboratory reports, and radiologic test results). Many computerized documentation systems have the ability to generate a patient care summary document that you review and sometimes print for each patient at the beginning and/or end of each shift; it includes information such as basic demographic data, health care provider’s name, primary medical diagnosis, and current orders. Health care team members monitor and record the progress made toward resolving a patient’s problems in progress notes. 11. A nurse is a member of an interdisciplinary team that uses critical pathways. According to the critical pathway, on day 2 of the hospital stay, the patient should be sitting in the chair. It is day 3, and the patient cannot sit in the chair. What should the nurse do? a. Electronic medical record b. Electronic health record c. Electronic charting record d. Electronic problem record a. Patient went up and down stairs b. Demonstrated use of crutches c. Used crutches with no difficulties d. Deficient knowledge related to never using crutches a. Database b. Progress notes c. Patient care summary d. Graphic record and flow sheet 293 A&E I Comprehensive Testbank ANS: C A variance occurs when the activities on the critical pathway are not completed as predicted or the patient does not meet expected outcomes. An example of a negative variance is when a patient develops pulmonary complications after surgery, requiring oxygen therapy and monitoring with pulse oximetry. A positive variance occurs when a patient progresses more rapidly than expected (e.g., use of a Foley catheter may be discontinued a day early). When a nurse is using the problem-oriented medical record, after analyzing data, health care team members identify problems and make a single problem list. A third format used for notes within a POMR is focus charting. It involves the use of DAR notes, which include D—Data (both subjective and objective), A— Action or nursing intervention, and R—Response of the patient (i.e., evaluation of effectiveness). 12. A nurse needs to begin discharge planning for a patient admitted with pneumonia and a congested cough. When is the best time the nurse should start discharge planning for this patient? ANS: A Ideally, discharge planning begins at admission. Right before discharge is too late for discharge planning. After the congestion is treated is also too late for discharge planning. Usually the primary care provider writes the order too close to discharge, and nurses do not need an order to begin the teaching that will be needed for discharge. By identifying discharge needs early, nursing and other health care professionals begin planning for discharge to the appropriate level of care, which sometimes includes support services such as home care and equipment needs. 13. A patient is being discharged home. Which information should the nurse include? ANS: B Discharge documentation includes medications, diet, community resources, follow-up care, and who to contact in case of an emergency or for questions. A patient’s acuity level, usually determined by a computer program, is based on the types and numbers of nursing interventions (e.g., intravenous [IV] therapy, wound care, or ambulation assistance) required over a 24-hour period. Many computerized documentation systems include standardized care plans or clinical practice guidelines (CPGs) to facilitate the creation and documentation of a nursing and or interprofessional plan of care. Each CPG facilitates safe and consistent care for an identified problem by describing or listing institutional standards and evidence-based guidelines that are easily accessed and included in a patient’s electronic health record. Verbal orders occur when a health care provider gives therapeutic orders to a registered nurse while they are standing in proximity to one another. 14. A nurse developed the following discharge summary sheet. Which critical information should the nurse add? a. Add this data to the problem list. b. Focus chart using the DAR format. c. Document the variance in the patient’s record. d. Report a positive variance in the next interdisciplinary team meeting. a. Upon admission b. Right before discharge c. After the congestion is treated d. When the primary care provider writes the order a. Acuity level b. Community resources c. Standardized care plan d. Signature for verbal order TOPIC DISCHARGE SUMMARY 294 A&E I Comprehensive Testbank ANS: C List actual time of discharge, mode of transportation, and who accompanied the patient for discharge summary information. Clinical decision support systems (CDSSs) are computerized programs used within the health care setting, to aid and support clinical decision making. The knowledge base within a CDSS contains rules and logic statements that link information required for clinical decisions in order to generate tailored recommendations for individual patients that are presented to nurses as alerts, warnings, or other information for consideration. A nurse completes a nursing history form when a patient is admitted to a nursing unit, not when the patient is discharged. SOAP notes are not given to patients who are being discharged. SOAP notes are a type of documentation style. 15. A home health nurse is preparing for an initial home visit. Which information should be included in the patient’s home care medical record? ANS: D Information in the home care medical record includes patient assessment, referral and intake forms, interprofessional plan of care, a list of medications, and reports to third-party payers. An interprofessional plan of care is used rather than a nursing process form. A step-by-step skills manual and a list of possible procedures are not included in the record. 16. A nurse in a long-term care setting that is funded by Medicare and Medicaid is completing standardized protocols for assessment and care planning for reimbursement. Which task is the nurse completing? ANS: A Medication Diet Activity level Follow-up care Wound care Phone numbers When to call the doctor Time of discharge a. Clinical decision support system b. Admission nursing history c. Mode of transportation d. SOAP notes a. Nursing process form b. Step-by-step skills manual c. A list of possible procedures d. Reports to third-party payers a. A minimum data set b. An admission assessment and acuity level c. A focused assessment/specific body system d. An intake assessment form and auditing phase 295 A&E I Comprehensive Testbank The Resident Assessment Instrument (RAI), which includes the Minimum Data Set (MDS) and the Care Area Assessment (CAA), is the data set that is federally mandated for use in long-term care facilities by CMS. MDS assessment forms are completed upon admission, and then periodically, within specific guidelines and time frames for all residents in certified nursing homes. The MDS also determines the reimbursement level under the prospective payment system. A focused assessment is limited to a specific body system. An admission assessment and acuity level is performed in the hospital. An intake form is for home health. There is no such thing as an auditing phase in an assessment intake. 17. A nurse is charting. Which information is critical for the nurse to document? ANS: C Nursing interventions and treatments (e.g., medication administration) must be documented. Avoid using generalized, empty phrases such as “status unchanged” or “had good day.” Do not document retaliatory or critical comments about a patient, like demanding and argumentative. Family is poor is not critical information to chart. 18. A nurse is completing an OASIS data set on a patient. The nurse works in which area? ANS: A Nurses use two different data sets to document the clinical assessments and care provided in the home care setting, the Outcome and Assessment Information Set (OASIS) and the Omaha System. The intensive care unit does not use the OASIS data set. The long-term health care setting includes skilled nursing facilities (SNFs) in which patients receive 24-hour day care. 19. A nurse is preparing to document a patient who has chest pain. Which information is critical for the nurse to include? ANS: D You need to ensure the information within a recorded entry or a report is complete, containing appropriate and essential information (pain of 8). Document subjective and objective assessment. While pupils equal and reactive to light is data, it does not relate to the chest pain; this information would be critical for a head injury. Derogatory or inappropriate comments about the patient or family (“pain”) is not appropriate. This kind of language can be used as evidence for nonprofessional behavior or poor quality of care. Avoid using generalized, empty phrases like “poor results.” Use complete, concise descriptions. 20. A nurse is providing care to a group of patients. Which situation will require the nurse to obtain a telephone order? a. The patient had a good day with no complaints. b. The family is demanding and argumentative. c. The patient received a pain medication, Lortab. d. The family is poor and had to go on welfare. a. Home health b. Intensive care unit c. Skilled nursing facility d. Long-term care facility a. The family is a “pain.” b. Pupils equal and reactive to light c. Had poor results from the pain medication d. Sharp pain of 8 on a scale of 1 to 10 a. As the nurse and health care provider leave a patient’s room, the primary care provider gives the nurse an order. 296 A&E I Comprehensive Testbank ANS: B Telephone orders and verbal orders (VO) usually occur at night or during emergencies (blood pressure dropping); they should be used only when absolutely necessary and not for the sake of convenience. Because the time is 1 AM (0100 military time) and the health care provider is not present, the nurse will need to call the health care provider for a telephone order. A VO involves the health care provider giving orders to a nurse while they are standing in proximity to one another. Just reading an order that is correctly written in the chart does not require a telephone order. 21. A nurse obtained a telephone order from a primary care provider for a patient in pain. Which chart entry should the nurse document? ANS: C The nurse receiving a TO or VO enters the complete order into the computer using the computerized provider order entry (CPOE) software or writes it out on a physician’s order sheet for entry in the computer as soon as possible. After you have taken the order, read the order back, using the “read back” process, and document that you did this to provide evidence that the information received (such as call back instructions and/or therapeutic orders) was verified with the provider. An example follows: “10/16/2015 (08:15), Change IV fluid to Lactated Ringers with Potassium 20 mEq/L to run at 125 mL/hr. TO: Dr. Knight/J. Woods, RN, read back.” VO stands for verbal order, not telephone order. The health care provider’s name and read back must be included in the chart entry. 22. A nurse is teaching the staff about informatics. Which information from the staff indicates the nurse needs to follow up? ANS: C When the staff make an incorrect statement, then the nurse needs to follow up. Competence in informatics is not the same as computer competency. All the rest are correct information so the nurse does not need to follow up. To become competent in informatics, you need to be able to use evolving methods of discovering, retrieving, and using information in practice. This means that you learn to recognize when information is needed and have the skills to find, evaluate, and use that information effectively. Nursing informatics is a b. At 0100, a patient’s blood pressure drops from 120/80 to 90/50, and the incision dressing is saturated with blood. c. At 0800, the nurse and health care provider make rounds, and the primary care provider tells the nurse a diet order. d. A nurse reads an order correctly as written by the health care provider in the patient’s medical record. a. 12/16/20XX 0915 Morphine, 2 mg, IV every 4 hours for incisional pain. VO Dr. Day/J. Winds, RN, read back. b. 12/16/20XX 0915 Morphine, 2 mg, IV every 4 hours for incisional pain. TO J. Winds, RN, read back. c. 12/16/20XX 0915 Morphine, 2 mg, IV every 4 hours for incisional pain. TO Dr. Day/J. Winds, RN, read back. d. 12/16/20XX 0915 Morphine, 2 mg, IV every 4 hours for incisional pain. TO J. Winds, RN. a. To be proficient in informatics, a nurse should be able to discover, retrieve, and use information in practice. b. A nurse needs to know how to find, evaluate, and use information effectively. c. If a nurse has computer competency, the nurse is competent in informatics. d. Nursing informatics is a recognized specialty area of nursing practice. 297 A&E I Comprehensive Testbank specialty that integrates the use of information and computer technology to support all aspects of nursing practice, including direct delivery of care, administration, education, and research. 23. A hospital is using a computer system that allows all health care providers to use a protocol system to document the care they provide. Which type of system/design will the nurse be using? ANS: C One design model for Nursing Clinical Information Systems (NCIS) is the protocol or critical pathway design. This design facilitates interdisciplinary management of information because all health care providers use evidence-based protocols or critical pathways to document the care they provide. The knowledge base within a CDSS contains rules and logic statements that link information required for clinical decisions in order to generate tailored recommendations for individual patients, which are presented to nurses as alerts, warnings, or other information for consideration. The nursing process design is the most traditional design for an NCIS. This design organizes documentation within well-established formats such as admission and postoperative assessments, problem lists, care plans, discharge planning instructions, and intervention lists or notes. Computerized provider order entry (CPOE) systems allow health care providers to directly enter orders for patient care into the hospital’s information system. 24. A nurse wants to reduce data entry errors on the computer system. Which action should the nurse take? ANS: D To increase accuracy and decrease unnecessary duplication, many health care agencies keep records or computers near a patient’s bedside to facilitate immediate documentation of information as it is collected. A good system requires frequent, random changes in personal passwords to prevent unauthorized persons from tampering with records. When using a health care agency computer system, it is essential that you do not share your computer password with anyone under any circumstances. You destroy all papers containing personal information immediately after you use them. Taking nursing notes home is a violation of the Health Insurance Portability and Accountability Act (HIPAA) and confidentiality. 25. Which entry will require follow-up by the nurse manager? a. Clinical decision support system b. Nursing process design c. Critical pathway design d. Computerized provider order entry system a. Use the same password all the time. b. Share password with only one other staff member. c. Print out and review computer nursing notes at home. d. Chart on the computer immediately after care is provided. 0800 Patient states, “Fell out of bed.” Patient found lying by bed on the floor. Legs equal in length bilaterally with no distortion, pedal pulses strong, leg strength equal and strong, no bruising or bleeding. Neuro checks within normal limits. States, “Did not pass out.” Assisted back to bed. Call bell within reach. Bed monitor on. ——————-Jane More, RN 0810 Notified primary care provider of patient’s status. New orders received. ——————-Jane More, RN 0815 Portable x-ray of L hip taken in room. States, “I feel fine.” ——————-Jane More, RN 0830 Incident report completed and placed on chart. ——————-Jane More, RN a. 0800 298 A&E I Comprehensive Testbank ANS: D Do not include any reference to an incident in the medical record; therefore, the nurse manager must follow up. A notation about an incident report in a patient’s medical record makes it easier for a lawyer to argue that the reference makes the incident report part of the medical record and therefore subject to attorney review. When an incident occurs, document an objective description of what happened, what you observed, and the follow-up actions taken, including notification of the patient’s health care provider in the patient’s medical record. Remember to evaluate and document the patient’s response to the incident. 26. A patient has a diagnosis of pneumonia. Which entry should the nurse chart to help with financial reimbursement? ANS: A Accurately documenting services provided, including the supplies and equipment used in a patient’s care, clarifies the type of treatment a patient received. This documentation also supports accurate and timely reimbursement to a health care agency and/or patient. None of the other options had equipment or supplies listed. Avoid using generalized, empty phrases such as “status unchanged” or “had good day.” Do not enter personal opinions—stating that the patient is cooperative is a personal opinion and should be avoided. “Finally, patient had no complaints” is a critical comment about the patient and if charted can be used as evidence of nonprofessional behavior or poor quality of care. 27. A nurse is teaching the staff about health care reimbursement. Which information should the nurse include in the teaching session? ANS: A Nurses’ documentation practices in home health, long-term care, and hospitals can determine reimbursement for health care. A “near miss” is an incident where no property was damaged and no patient or personnel were injured, but given a slight shift in time or position, damage or injury could have easily occurred. A clinical information system (CIS) does not have to be installed by 2014 to obtain reimbursement. CIS programs include monitoring systems; order entry systems; and laboratory, radiology, and pharmacy systems. Diagnosisrelated groups (DRGs) are the basis for establishing reimbursement for patient care, not HIPAA. Legislation to protect patient privacy regarding health information is the Health Insurance Portability and Accountability Act (HIPAA). b. 0810 c. 0815 d. 0830 a. Used incentive spirometer to encourage coughing and deep breathing. Lung congested upon auscultation in lower lobes bilaterally. Pulse oximetry 86%. Oxygen per nasal cannula applied at 2 L/min per standing order. b. Cooperative, patient coughed and deep breathed using a pillow as a splint. Stated, “felt better.” Finally, patient had no complaints. c. Breathing without difficulty. Sitting up in bed watching TV. Had a good day. d. Status unchanged. Remains stable with no abnormal findings. Checked every 2 hours. a. Home health, long-term care, and hospital nurses’ documentation can affect reimbursement for health care. b. A clinical information system must be installed by 2014 to obtain health care reimbursement. c. A “near miss” helps determine reimbursement issues for health care. d. HIPAA is the basis for establishing reimbursement for health care. 299 A&E I Comprehensive Testbank 28. A nurse is discussing the advantages of a nursing clinical information system. Which advantage should the nurse describe? ANS: B Advantages associated with the nursing information system include reduced errors of omission; better access to information (not more time to read charts); enhanced quality of documentation; reduced, not increased, hospital costs; increased nurse job satisfaction; compliance with requirements of accrediting agencies (e.g., TJC); and development of a common, not varied, clinical database. MULTIPLE RESPONSE 1. Which behaviors indicate the student nurse has a good understanding of confidentiality and the Health Insurance Portability and Accountability Act (HIPAA)? (Select all that apply.) ANS: D, E When you are a student in a clinical setting, confidentiality and compliance with HIPAA are part of professional practice. Reading the progress notes of an assigned patient’s record and giving a change-of-shift report to the oncoming nurse about the patient are behaviors that follow HIPAA and confidentiality guidelines. Do not share information with other patients or health care team members who are not caring for a patient. Not only is it unethical to view medical records of other patients, but breaches of confidentiality lead to disciplinary action by employers and dismissal from work or nursing school. To protect patient confidentiality, ensure that written materials used in your student clinical practice do not include patient identifiers (e.g., room number, date of birth, demographic information), and never print material from an electronic health record for personal use. 2. A nurse is describing the purposes of a health care record to a group of nursing students. Which purposes will the nurse include in the teaching session? (Select all that apply.) ANS: A, B, C, E, F A patient’s record is a valuable source of data for all members of the health care team. Its purposes include interdisciplinary communication, legal documentation, financial billing (reimbursement), education, research, a. Varied clinical databases b. Reduced errors of omission c. Increased hospital costs d. More time to read charts a. Writes the patient’s room number and date of birth on a paper for school b. Prints/copies material from the patient’s health record for a graded care plan c. Reviews assigned patient’s record and another unassigned patient’s record d. Gives a change-of-shift report to the oncoming nurse about the patient e. Reads the progress notes of assigned patient’s record f. Discusses patient care with the hospital volunteer a. Communication b. Legal documentation c. Reimbursement d. Nursing process e. Research f. Education 300 A&E I Comprehensive Testbank and auditing/monitoring. Nursing process is a way of thinking and performing nursing care; it is not a purpose of a health care record. 3. A nurse is developing a plan to reduce data entry errors and maintain confidentiality. Which guidelines should the nurse include? (Select all that apply.) ANS: B, D, E, F When faxing, use programmed speed-dial keys to eliminate the chance of a dialing error and misdirected information. An automatic sign-off is a safety mechanism that logs a user off the computer system after a specified period of inactivity. Disciplinary action, including loss of employment, occurs when nurses or other health care personnel inappropriately access patient information. All papers containing PHI (e.g., Social Security number, date of birth or age, patient’s name or address) must be destroyed immediately after you use or fax them. Most agencies have shredders or locked receptacles for shredding and incineration. Strong passwords use combinations of letters, numbers, and symbols that are difficult to guess. A firewall is a combination of hardware and software that protects private network resources (e.g., the information system of the hospital) from outside hackers, network damage, and theft or misuse of information and should not be bypassed. MATCHING A nurse is using focused charting. Match the chart entry to the correct letter of the acronym. 1.D 2.A 3.R 1.ANS:C2.ANS:A3.ANS:B Week 11 Nutrition, Dysphagia Chapter 45: Nutrition Potter et al.: Fundamentals of Nursing, 9th Edition MULTIPLE CHOICE a. Bypass the firewall. b. Implement an automatic sign-off. c. Create a password with just letters. d. Use a programmed speed-dial key when faxing. e. Impose disciplinary actions for inappropriate access. f. Shred papers containing personal health information (PHI). a. Applied oxygen, stayed with patient, and instructed to slow breathing. b. Patient states, “feel better,” respirations 16 with O2 saturations 96%. c. Patient states, “can’t catch my breath and chest hurts.” Confused. 301 A&E I Comprehensive Testbank 1.A nurse is teaching about the energy needed at rest to maintain life-sustaining activities for a specific period of time. What is the nurse discussing? ANS: B The basal metabolic rate (BMR) is the energy needed at rest to maintain life-sustaining activities for a specific period of time. The resting energy expenditure (REE), or resting metabolic rate, is the amount of energy an individual needs to consume over a 24-hour period for the body to maintain all of its internal working activities while at rest. Nutrients are the elements necessary for body processes and function. Nutrient density is the proportion of essential nutrients to the number of kilocalories. High–nutrient density foods provide a large number of nutrients in relation to kilocalories. 2.In general, when a patient’s energy requirements are completely met by kilocalorie (kcal) intake in food, which assessment finding will the nurse observe? ANS: C In general, when energy requirements are completely met by kilocalorie (kcal) intake in food, weight does not change. When kilocalories ingested exceed a person’s energy demands, the individual gains weight. If kilocalories ingested fail to meet a person’s energy requirement, the individual loses weight. Fluid, not kilocalories, causes daily weight fluctuations. 3.A nurse is asked how many kcal per gram fats provided. How should the nurse answer? ANS: D Fats (lipids) are the most calorie-dense nutrient, providing 9 kcal/g. Carbohydrates and protein provide 4 kcal/ g. 4.A nurse is teaching a patient about proteins that must be obtained through the diet and cannot be synthesized in the body. Which term used by the patient indicates teaching is successful? ANS: D a. Resting energy expenditure (REE) b. Basal metabolic rate (BMR) c. Nutrient density d. Nutrients a. Weight increases. b. Weight decreases. c. Weight does not change. d. Weight fluctuates daily. a. 3 b. 4 c. 6 d. 9 a. Amino acids b. Triglycerides c. Dispensable amino acids d. Indispensable amino acids 302 A&E I Comprehensive Testbank The body does not synthesize indispensable amino acids, so these need to be provided in the diet. The simplest form of protein is the amino acid. The body synthesizes dispensable amino acids. Triglycerides are made up of three fatty acids attached to a glycerol. 5.A nurse is caring for a patient with a postsurgical wound. When planning care, which goal will be the priority? ANS: C When intake of nitrogen is greater than output, the body is in positive nitrogen balance. Positive nitrogen balance is required for growth, normal pregnancy, maintenance of lean muscle mass and vital organs, and wound healing. Negative nitrogen balance occurs when the body loses more nitrogen than the body gains. Neutral nitrogen balance occurs when gain equals loss and is not optimal for tissue healing. There is no such term as dependent nitrogen balance. 6.In providing diet education for a patient on a low-fat diet, which information is important for the nurse to share? ANS: D Most animal fats have high proportions of saturated fatty acids, whereas vegetable fats have higher amounts of unsaturated and polyunsaturated fatty acids. Linoleic acid, an unsaturated fatty acid, is the only essential fatty acid in humans. Diet recommendations include limiting saturated fat to less than 7% and trans fat to less than 1%. 7.A patient has a decreased gag reflex, left-sided weakness, and drooling. Which action will the nurse take when feeding this patient? ANS: B Have the patient flex the head slightly to a chin-down position to help prevent aspiration. If the patient has unilateral weakness, teach him or her and the caregiver to place food in the stronger side of the mouth. Provide a 30-minute rest period before eating and position the patient in an upright, seated position in a chair or raise the head of the bed to 90 degrees. Thin liquids such as water and fruit juice are difficult to control in the mouth and are more easily aspirated. 8.The patient has been diagnosed with cardiovascular disease and placed on a low-fat diet. The patient asks the nurse, “How much fat should I have? I guess the less fat, the better.” Which information will the nurse include in the teaching session? a. Reduce dependent nitrogen balance. b. Maintain negative nitrogen balance. c. Promote positive nitrogen balance. d. Facilitate neutral nitrogen balance. a. Polyunsaturated fats should be less than 7% of the total calories. b. Trans fat should be less than 7% of the total calories. c. Unsaturated fats are found mostly in animal sources. d. Saturated fats are found mostly in animal sources. a. Position in semi-Fowler’s. b. Flex head with chin tuck. c. Place food on left side. d. Offer fruit juice. a. Cholesterol intake needs to be less than 300 mg/day. 303 A&E I Comprehensive Testbank ANS: D Deficiency occurs when fat intake falls below 10% of daily nutrition. While keeping cholesterol below 300 mg is correct according to the American Heart Association, it does not answer the patient’s question about fat. Various types of fatty acids have significance for health and for the incidence of disease and are referred to in dietary guidelines. Linoleic acid and arachidonic acid are important for metabolic processes but are manufactured by the body when linoleic acid is available from the diet. 9.The nurse is describing the ChooseMyPlate program to a patient. Which statement from the patient indicates successful learning? ANS: A ChooseMyPlate serves as a basic guide for making food choices for a healthy lifestyle. The ChooseMyPlate program was developed by the U.S. Department of Agriculture to replace the MyFoodPyramid program. It helps balance calories but does not provide specific calories of food. These guidelines are for Americans over the age of 2 years. These guidelines are provided for health, not sickness. 10.The nurse is teaching a health class about the ChooseMyPlate program. Which guidelines will the nurse include in the teaching session? ANS: D The ChooseMyPlate program includes guidelines for balancing calories; decreasing portion size; increasing healthy foods; increasing water consumption; and decreasing fats, sodium, and sugars. It does not balance sodium and potassium. 11.The nurse is providing nutrition education to a Korean patient using the five food groups. In doing so, what should be the focus of the teaching? ANS: C As a nurse, consider the food preferences of patients from different racial and ethnic groups, vegetarians, and others when planning diets. Initiation of a balanced diet is more important than conversion to what may be considered an American diet. Ethnic food choices may be just as nutritious as American choices. Foods should be chosen for their nutritive value and should not be compared with the American diet. b. Fats have no significance in health and the incidence of disease. c. All fats come from external sources so this can be easily controlled. d. Deficiencies occur when fat intake falls below 10% of daily nutrition. a. “I can use this to make healthy lifestyle food choices.” b. “I can use this to count specific calories of food.” c. “I can use this for my baby girl.” d. “I can use this when I am sick.” a. Balancing sodium and potassium b. Decreasing water consumption c. Increasing portion size d. Balancing calories a. Discouraging the patient’s ethnic food choices b. Changing the patient’s diet to a more conventional American diet c. Including racial and ethnic practices with food preferences of the patient d. Comparing the patient’s ethnic preferences with American dietary choices 304 A&E I Comprehensive Testbank 12.A nurse is teaching a nutrition class about the different daily values. When teaching about the referenced daily intakes (RDIs), which information should the nurse include? ANS: A The RDIs are the first set, comprising protein, vitamins, and minerals based on the RDA. The daily reference values (DRVs) make up the second set and consist of nutrients such as total fat, saturated fat, cholesterol, carbohydrates, fiber, sodium, and potassium. Combined, both sets make up the daily values used on food labels. Daily values did not replace RDAs but provided a separate, more understandable format for the public. Daily values are based on percentages of a diet consisting of 2000 kcal/day for adults and children 4 years or older. 13.The nurse is planning care for a group of patients. Which task will the nurse assign to the nursing assistive personnel? ANS: A The skill of measuring blood glucose level after skin puncture (capillary puncture) can be delegated to nursing assistive personnel. The other skills cannot be delegated. A nurse must measure a nasoenteric tube for insertion, pH in gastrointestinal aspirate, and patient’s risk for aspiration. 14.In teaching mothers-to-be about infant nutrition, which instruction should the nurse provide? ANS: D Breast milk or formula provides sufficient nutrition for the first 4 to 6 months of life. Infants should not have regular cow’s milk during the first year of life. It is too concentrated for an infant’s kidneys to manage, increases the risk of milk product allergies, and is a poor source of iron and vitamins C and E. Furthermore, children under 1 year of age should never ingest honey and corn syrup products because they are potential sources of the botulism toxin, which increases the risk of infant death. 15.When planning care for an adolescent who plays sports, which modification should the nurse include in the care plan? a. Have values for protein, vitamins, and minerals b. Are based on percentages of fat, cholesterol, and fiber c. Have replaced recommended daily allowances (RDAs) d. Are used to develop diets for chronic illnesses requiring 1800 cal/day a. Measuring capillary blood glucose level b. Measuring nasoenteric tube for insertion c. Measuring pH in gastrointestinal aspirate d. Measuring the patient’s risk for aspiration a. Supplement breast milk with corn syrup. b. Give cow’s milk during the first year of life. c. Add honey to infant formulas for increased energy. d. Provide breast milk or formula for the first 4 to 6 months. a. Increasing carbohydrates to 55% to 60% of total intake b. Providing vitamin and mineral supplements c. Decreasing protein intake to 0.75 g/kg/day d. Limiting water before and after exercise 305 A&E I Comprehensive Testbank ANS: A Sports and regular moderate to intense exercise necessitate dietary modification to meet increased energy needs for adolescents. Carbohydrates, both simple and complex, are the main source of energy, providing 55% to 60% of total daily kilocalories. Protein needs increase to 1 to 1.5 g/kg/day. Fat needs do not increase. Adequate hydration is very important. Adolescents need to ingest water before and after exercise to prevent dehydration, especially in hot, humid environments. Vitamin and mineral supplements are not required, but intake of iron-rich foods is required to prevent anemia. 16.In providing prenatal care to a pregnant patient, what does the nurse teach the expectant mother? ANS: C Folic acid intake is particularly important for DNA synthesis and growth of red blood cells. Inadequate intake may lead to fetal neural tube defects, anencephaly, or maternal megaloblastic anemia. Protein intake throughout pregnancy needs to increase to 60 grams daily. Calcium intake is especially critical in the third trimester, when fetal bones mineralize. Prenatal care usually includes vitamin and mineral supplementation to ensure daily intakes; however, pregnant women should not take additional supplements beyond prescribed amounts. 17.The patient is an 80-year-old male who is visiting the clinic today for a routine physical examination. The patient’s skin turgor is fair, but the patient reports fatigue and weakness. The skin is warm and dry, pulse rate is 116 beats/min, and urinary sodium level is slightly elevated. Which instruction should the nurse provide? ANS: A Thirst sensation diminishes, leading to inadequate fluid intake or dehydration; the patient should be encouraged to drink more water/fluids. Symptoms of dehydration in older adults include confusion, weakness, hot dry skin, furrowed tongue, and high urinary sodium. Milk continues to be an important food for older woman and men, who need adequate calcium to protect against osteoporosis; the patient’s problem is dehydration, not osteoporosis. Caution older adults to avoid grapefruit and grapefruit juice because these will decrease absorption of many drugs. The patient needs fluids not calories; drinking calorie-dense fluids is unnecessary. 18.The nurse is assessing a patient for nutritional status. Which action will the nurse take? ANS: D Combine multiple objective measures with subjective measures related to nutrition to adequately screen for nutritional problems. Using a single objective measure is ineffective in predicting risk of nutritional problems. a. Calcium intake is especially important in the first trimester. b. Protein intake needs to decrease to preserve kidney function. c. Folic acid is needed to help prevent birth defects and anemia. d. Extra vitamins and minerals should be taken as much as possible. a. Drink more water to prevent further dehydration. b. Drink more calorie-dense fluids to increase caloric intake. c. Drink more milk and dairy products to decrease the risk of osteoporosis. d. Drink more grapefruit juice to enhance vitamin C intake and medication absorption. a. Forego the assessment in the presence of chronic disease. b. Use the Mini Nutritional Assessment for pediatric patients. c. Choose a single objective tool that fits the patient’s condition. d. Combine multiple objective measures with subjective measures. 306 A&E I Comprehensive Testbank Chronic disease and increased metabolic requirements are risk factors for the development of nutritional problems; these patients may be in critical need of this assessment. The Mini Nutritional Assessment is used for screening older adults in home care programs, nursing homes, and hospitals. 19.The patient has a calculated body mass index (BMI) of 34. How will the nurse classify this finding? ANS: D BMI greater than 30 is defined as obesity. BMI between 25 and 30 is classified as overweight. BMI from 18.5 to 24.9 is normal. BMI under 18.5 is underweight. 20.A nurse is caring for patients with dysphagia. Which patient has neurogenic dysphagia? ANS: D Stroke is the only cause of dysphagia in this list that is considered neurogenic. Myasthenia gravis and muscular dystrophy are considered myogenic in origin, whereas benign peptic stricture is considered obstructive. 21.The patient has H. pylori. Which action should the nurse take? ANS: C H. pylori, a bacterium that causes up to 85% of peptic ulcers, is confirmed by laboratory tests or a biopsy during endoscopy. Antibiotics treat and control the bacterial infection. Avoidance of wheat and oats are required for patients with celiac disease who must follow a gluten-free diet. Encourage patients to avoid foods that increase stomach acidity and pain such as caffeine, decaffeinated coffee, frequent milk intake, citric acid juices, and certain seasonings (hot chili peppers, chili powder, black pepper). Discourage smoking, alcohol, aspirin, and nonsteroidal antiinflammatory drugs (NSAIDs). 22.In determining malnourishment in a patient, which assessment finding is consistent with this disorder? ANS: C Spoon-shaped nails, koilonychia, is an indication of poor nutrition. All the others are normal findings. Lips should be moist, conjunctivae should be pink, and hair should not be easily plucked. a. Normal weight b. Underweight c. Overweight d. Obese a. A patient with benign peptic stricture b. A patient with muscular dystrophy c. A patient with myasthenia gravis d. A patient with stroke a. Encourage avoidance of wheat and oats. b. Encourage milkshakes as a nutritious snack. c. Encourage completion of antibiotic therapy. d. Encourage nonsteroidal antiinflammatory drugs. a. Moist lips b. Pink conjunctivae c. Spoon-shaped nails d. Not easily plucked hair 307 A&E I Comprehensive Testbank 23.A nurse is preparing to administer an enteral feeding. In which order will the nurse implement the steps, starting with the first one? 1. Elevate head of bed to at least 30 degrees. 2. Check for gastric residual volume. 3. Flush tubing with 30 mL of water. 4. Verify tube placement. 5. Initiate feeding. ANS: C The steps for an enteral feeding are as follows: Place patient in high-Fowler’s position or elevate head of bed to at least 30 (preferably 45) degrees; verify tube placement; check for gastric residual volume; flush tubing with 30 mL of water; and initiate feeding. 24.The patient is admitted with facial trauma, including a broken nose, and has a history of esophageal reflux and of aspiration pneumonia. With which tube will the nurse most likely administer the feeding? ANS: B Patients with gastroparesis or esophageal reflux or with a history of aspiration pneumonia may require placement of tubes beyond the stomach into the intestine. The jejunostomy tube is the only tube in the list that is beyond the stomach and is not contraindicated by facial trauma. The nasogastric tube and the PEG tube are placed in the stomach, and placement could lead to aspiration. The nasointestinal tube and the nasogastric tube may be contraindicated by facial trauma and the broken nose. 25.The nurse is preparing to insert a nasogastric tube in a patient who is semiconscious. To determine the length of the tube needed to be inserted, how should the nurse measure the tube? ANS: D Measure distance from the tip of the nose to the earlobe to the xiphoid process of the sternum. This approximates the distance from the nose to the stomach in 98% of patients. For duodenal or jejunal placement, an additional 20 to 30 cm is required. 26.Before giving the patient an intermittent gastric tube feeding, what should the nurse do? a. 4, 2, 1, 5, 3 b. 2, 4, 1, 3, 5 c. 1, 4, 2, 3, 5 d. 2, 1, 4, 5, 3 a. Nasogastric tube b. Jejunostomy tube c. Nasointestinal tube d. Percutaneous endoscopic gastrostomy (PEG) tube a. From the tip of the nose to the earlobe b. From the tip of the earlobe to the xiphoid process c. From the tip of the earlobe to the nose to the xiphoid process d. From the tip of the nose to the earlobe to the xiphoid process a. Make sure that the tube is secured to the gown with a safety pin. b. Inject air into the stomach via the tube and auscultate. 308 A&E I Comprehensive Testbank ANS: C Be sure that the formula is at room temperature. Cold formula causes gastric cramping and discomfort because the mouth and the esophagus do not warm the liquid. Do not use safety pins. Safety pins can become unfastened and may cause harm to the patient. Auscultation is no longer considered a reliable method for verification of tube placement because a tube inadvertently placed in the lungs, pharynx, or esophagus transmits sound similar to that of air entering the stomach. Gastric fluid of patient who has fasted for at least 4 hours usually has a pH of 1 to 4, especially when the patient is not receiving gastric-acid inhibitor. 27.A small-bore feeding tube is placed. Which technique will the nurse use to best verify tube placement? ANS: A At present, the most reliable method for verification of placement of small-bore feeding tubes is x-ray examination. Aspiration of contents and pH testing are not infallible. The nurse would need a more precise indicator to help differentiate the source of tube feeding aspirate. Auscultation is no longer considered a reliable method for verification of tube placement because a tube inadvertently placed in the lungs, pharynx, or esophagus transmits sound similar to that of air entering the stomach. 28.The nurse is concerned about pulmonary aspiration when providing the patient with an intermittent tube feeding. Which action is the priority? ANS: B A major cause of pulmonary aspiration is regurgitation of formula. The nurse needs to verify tube placement and elevate the head of the bed 30 to 45 degrees during feedings and for 2 hours afterward. While observing the color of gastric contents is a component, it is not the priority component; pH is the primary component. The addition of blue food coloring to enteral formula to assist with detection of aspirate is no longer used. Do not hang formula longer than 4 to 8 hours. Formula becomes a medium for bacterial growth after that length of time. 29.The patient is to receive multiple medications via the nasogastric tube. The nurse is concerned that the tube may become clogged. Which action is best for the nurse to take? ANS: D c. Have the tube feeding at room temperature. d. Check to make sure pH is at least 5. a. X-ray b. pH testing c. Auscultation d. Aspiration of contents a. Observe the color of gastric contents. b. Verify tube placement before feeding. c. Add blue food coloring to the enteral formula. d. Run the formula over 12 hours to decrease overload. a. Instill nonliquid medications without diluting. b. Irrigate the tube with 60 mL of water after all medications are given. c. Mix all medications together to decrease the number of administrations. d. Check with the pharmacy for availability of the liquid forms of medications. 309 A&E I Comprehensive Testbank Use liquid medications when available to prevent tube occlusion. Irrigate with 30 mL of water before and after each medication per tube. Completely dissolve crushed medications in liquid if liquid medication is not available. Read pharmacological information on compatibility of drugs and formula before mixing medications. 30.The patient has just started on enteral feedings, and the patient is reporting abdominal cramping. Which action will the nurse take next? ANS: A One possible cause of abdominal cramping is a rapid increase in rate or volume. Lowering the rate of delivery may increase tolerance. Another possible cause of abdominal cramping is the use of cold formula. The nurse should warm the formula to room temperature. High-fat formulas are also a cause of abdominal cramping. Consult with the health care provider regarding prokinetic medication for increasing gastric motility for delayed gastric emptying. 31.The patient has just been started on an enteral feeding and has developed diarrhea after being on the feeding for 2 hours. What does the nurse suspect is the mostlikely cause of the diarrhea? ANS: C Hyperosmolar formulas can cause diarrhea or formula intolerance. If that is the case, the solution is to lower the rate, dilute the formula, or change to an isotonic formula. Antibiotics destroy normal intestinal flora and disturb the internal ecology, allowing for Clostridium difficiletoxin buildup. However, this takes time (more than 2 hours), and no indication suggests that this patient is on antibiotics. Bacterial contamination of the feeding usually occurs when feedings are left hanging for longer than 8 hours. 32.A patient develops a foodborne disease from Escherichia coli. When taking a health history, which food item will the nurse most likely find the patient ingested? ANS: B Undercooked ground beef is the usual food source for Escherichia coli. Botulism is associated with improperly home-canned foods. Soft cheese is the usual food source for listeriosis. Custards are associated with salmonellosis and Staphylococcus. 33.The nurse is caring for a patient receiving total parenteral nutrition (TPN). Which action will the nurse take? a. Slow the rate of tube feeding. b. Instill cold formula to “numb” the stomach. c. Change the tube feeding to a high-fat formula. d. Consult with the health care provider about prokinetic medication. a. Antibiotic therapy b. Clostridium difficile c. Formula intolerance d. Bacterial contamination a. Improperly home-canned food b. Undercooked ground beef c. Soft cheese d. Custard a. Run lipids for no longer than 24 hours. b. Take down a running bag of TPN after 36 hours. c. Clean injection port with alcohol 5 seconds before and after use. 310 A&E I Comprehensive Testbank ANS: D During central venous catheter dressing changes, always use a sterile mask and gloves, and assess insertion sites for signs and symptoms of infection. To avoid infection, change the TPN infusion tubing every 24 hours, and do not hang a single container of PN for longer than 24 hours or lipids longer than 12 hours. 34.The patient is having at least 75% of nutritional needs met by enteral feeding, so the health care provider has ordered the parenteral nutrition (PN) to be discontinued. However, the nurse notices that the PN infusion has fallen behind. What should the nurse do? ANS: C Sudden discontinuation of PN can cause hypoglycemia. PN must be tapered off. Usually, 10% dextrose is infused when PN solution is suddenly discontinued. Too rapid administration of hypertonic dextrose (PN) can result in an osmotic diuresis and dehydration. If an infusion falls behind schedule, the nurse should not increase the rate in an attempt to catch up. 35.The patient is on parenteral nutrition and is lethargic. The patient reports thirst and headache and has had increased urination. Which problem does the nurse prepare to address? ANS: A Signs and symptoms of hyperglycemia are thirst, headache, lethargy, and increased urination. Hypocapnia is not associated with parenteral nutrition. Hypercapnia increases oxygen consumption and increases CO2 levels. Ventilator-dependent patients are at greatest risk for this. Hypoglycemia is characterized by diaphoresis, shakiness, confusion, and loss of consciousness. 36.In providing diabetic teaching for a patient with type 1 diabetes mellitus, which instructions will the nurse provide to the patient? ANS: B The diabetic patient should limit saturated fat to less than 7% of total calories and cholesterol intake to less than 200 mg/day. Type 1 diabetes requires both insulin and dietary restrictions for optimal control. Nonnutritive sweeteners can be eaten as long as the recommended daily intake levels are followed. 37.The patient with cardiovascular disease is receiving dietary instructions from the nurse. Which information from the patient indicates teaching is successful? d. Wear a sterile mask when changing the central venous catheter dressing. a. Increase the rate to get the volume caught up before discontinuing. b. Stop the infusion as ordered. c. Taper infusion gradually. d. Hang 5% dextrose. a. Hyperglycemia b. Hypoglycemia c. Hypercapnia d. Hypocapnia a. Insulin is the only consideration that must be taken into account. b. Saturated fat should be limited to less than 7% of total calories. c. Nonnutritive sweeteners can be used without restriction. d. Cholesterol intake should be greater than 200 mg/day. 311 A&E I Comprehensive Testbank ANS: C American Heart Association guidelines recommend limiting saturated fat to less than 7%, trans fat to less than 1%, and cholesterol to less than 300 mg/day. Diet therapy includes eating fish at least 2 times per week and eating whole grain high-fiber foods. Maintaining a prescribed carbohydrate intake is necessary for diabetes mellitus. 38.The nurse is providing home care for a patient diagnosed with acquired immunodeficiency syndrome (AIDS). Which dietary intervention will the nurse add to the care plan? ANS: A Small, frequent, nutrient-dense meals that limit fatty foods and overly sweet foods are easier to tolerate. Restorative care of malnutrition resulting from AIDS focuses on maximizing kilocalories and nutrients. Patients benefit from eating cold foods and drier or saltier foods with fluid in between. 39.A patient is on a full liquid diet. Which food item choice by the patient will cause the nurse to intervene? ANS: D Mashed potatoes and gravy are on a dysphagia, mechanical soft, soft and regular diet but are not components of a full liquid diet. The nurse will need to provide teaching on what is allowed on the diet. Custard, frozen yogurt, and pureed vegetables are all on a full liquid diet. 40.A nurse is caring for a group of patients. Which patient will the nurse see first? ANS: B The nurse should see the patient with total parenteral nutrition that has the same tubing for 26 hours. To prevent infection, change the TPN infusion tubing every 24 hours. Change the administration system every 72 a. Maintain a prescribed carbohydrate intake. b. Eat fish at least 5 times per week. c. Limit trans fat to less than 1%. d. Avoid high-fiber foods. a. Provide small, frequent nutrient-dense meals for maximizing kilocalories. b. Prepare hot meals because they are more easily tolerated by the patient. c. Avoid salty foods and limit liquids to preserve electrolytes. d. Encourage intake of fatty foods to increase caloric intake. a. Custard b. Frozen yogurt c. Pureed vegetables d. Mashed potatoes and gravy a. Patient receiving total parenteral nutrition of 2-in-1 for 50 hours b. Patient receiving total parenteral nutrition infusing with same tubing for 26 hours c. Patient receiving continuous enteral feeding with same feeding bag for 12 hours d. Patient receiving continuous enteral feeding with same tubing for 24 hours 312 A&E I Comprehensive Testbank hours when infusing a 2-in-1 solution and every 24 hours for a 3-in-1 solution. Change bag and use a new administration set every 24 hours for a continuous enteral feeding. While the patient with the continuous enteral feeding has the same tubing for 24 hours, it has not extended the time like the total parenteral nutrition has. 41.The nurse is preparing to check the gastric aspirate for pH. Which equipment will the nurse obtain? ANS: B Cone-tipped or Asepto syringe is needed for testing of gastric aspirate for pH; these syringes are better than a Luer-Lok syringe. Clean gloves are needed, not sterile or double. MULTIPLE RESPONSE 1.A nurse is teaching a health class about the nutritional requirements throughout the life span. Which information should the nurse include in the teaching session? (Select all that apply.) ANS: A, B, E An infant usually doubles birth weight at 4 to 5 months and triples it at 1 year. Toddlers exhibit strong food preferences and become picky eaters. Older adults often experience a decrease in taste cells that alters food flavor and may decrease intake. Toddlers need to avoid hot dogs and grapes, not school-age children. The lactating woman needs 500 kcal/day above the usual allowance because the production of milk increases energy requirements. 2.The patient is asking the nurse about the best way to stay healthy. The nurse explains to the patient which teaching points? (Select all that apply.) ANS: A, C, D Recommendations include maintaining body weight in a healthy range; increasing physical activity and decreasing sedentary activities; increasing intake of fruits, vegetables, whole grain products, and fat-free or low-fat milk; eating moderate amount of lean meats, poultry, and eggs; keeping fat intake between 20% and 35% of total calories, with most fats coming from polyunsaturated or monounsaturated fatty acids (most meats contain saturated fatty acids); and choosing prepared foods with little salt while at the same time eating potassium-rich foods. 3.When assessing patient with nutritional needs, which patients will require follow-up from the nurse? (Select all that apply.) a. 10-mL Luer-Lok syringe b. Asepto syringe c. Sterile gloves d. Double gloves a. Infants triple weight at 1 year. b. Toddlers become picky eaters. c. School-age children need to avoid hot dogs and grapes. d. Breastfeeding women need an additional 750 kcal/day. e. Older adults have altered food flavor from a decrease in taste cells. a. Increase physical activity. b. Keep total fat intake to 10% or less. c. Maintain body weight in a healthy range. d. Choose and prepare foods with little salt. e. Increase intake of meat and other high-protein foods. 313 A&E I Comprehensive Testbank ANS: A, C, D The nurse should follow up with the tetracycline, diverticulitis, and enteral feeding. Tetracycline has decreased drug absorption with milk and antacids and has decreased nutrient absorption of calcium from binding. Nutritional treatment for diverticulitis includes a moderate- or low-residue diet until the infection subsides. Afterward, prescribing a high-fiber diet for chronic diverticula problems ensues. A patient with a gastric residual volume of 500 mL needs to have the feeding withheld and reassessed for tolerance to feedings. All the rest are normal and expected and do not require follow-up. Patients manage irritable bowel syndrome by increasing fiber, reducing fat, avoiding large meals, and avoiding lactose or sorbitol-containing foods for susceptible individuals. Initiate consultation with a speech-language pathologist for swallowing exercises and techniques to improve swallowing and reduce risk of aspiration for a patient with dysphagia. 4.To honor cultural values of patients from different ethnic/religious groups, which actions demonstrate culturally sensitive care by the nurse? (Select all that apply.) ANS: A, C, E The Jewish religion fasts 24 hours on Yom Kippur and must adhere to kosher food preparation methods. Hinduism requires no meats or fish. Muslims do not eat pork. Mormons do not drink caffeinated or alcoholic drinks. Week 12 Care of the Surgical Patient Chapter 50: Care of Surgical Patients Potter et al.: Fundamentals of Nursing, 9th Edition MULTIPLE CHOICE 1. The nurse is caring for a surgical patient, when the family member asks what perioperative nursing means. How should the nurse respond? a. A patient with infection taking tetracycline with milk b. A patient with irritable bowel syndrome increasing fiber c. A patient with diverticulitis following a high-fiber diet daily d. A patient with an enteral feeding and 500 mL of gastric residual e. A patient with dysphagia being referred to a speech-language pathologist a. Allows fasting on Yom Kippur for a Jewish patient b. Allows caffeine drinks for a Mormon patient c. Serves no ham products to a Muslim patient d. Serves kosher foods to a Christian patient e. Serves no meat or fish to a Hindu patient a. Perioperative nursing occurs in preadmission testing. b. Perioperative nursing occurs primarily in the postanesthesia care unit. c. Perioperative nursing includes activities before, during, and after surgery. d. Perioperative nursing includes activities only during the surgical procedure. 314 A&E I Comprehensive Testbank ANS: C Perioperative nursing care occurs before, during, and after surgery. Preadmission testing occurs before surgery and is considered preoperative. Nursing care provided during the surgical procedure is considered intraoperative, and in the postanesthesia care unit, it is considered postoperative. All of these are parts of the perioperative phase, but each individual phase does not explain the term completely. 2. The nurse is caring for a patient who is scheduled to undergo a surgical procedure. The nurse is completing an assessment and reviews the patient’s laboratory tests and allergies and prepares the patient for surgery. In which perioperative nursing phase is the nurse working? ANS: B Reviewing the patient’s laboratory tests and allergies is done before surgery in the preoperative phase. Perioperative means before, during, and after surgery. Intraoperative means during the surgical procedure in the operating suite; postoperative means after the surgery and could occur in the postanesthesia care unit, in the ambulatory surgical area, or on the hospital unit. 3. The nurse is caring for a patient in the postanesthesia care unit. The patient has developed profuse bleeding from the surgical site, and the surgeon has determined the need to return to the operative area. How will the nurse classify this procedure? ANS: D An emergency procedure must be done immediately to save a life or preserve the function of a body part. An example would be repair of a perforated appendix, repair of a traumatic amputation, or control of internal hemorrhaging. An urgent procedure is necessary for a patient’s health and often prevents additional problems from developing. An example would be excision of a cancerous tumor, removal of a gallbladder for stones, or vascular repair for an obstructed artery. An elective procedure is performed on the basis of the patient’s choice; it is not essential and is not always necessary for health. An example would be a bunionectomy, plastic surgery, or hernia reconstruction. A major procedure involves extensive reconstruction or alteration in body parts; it poses great risks to well-being. An example would be a coronary artery bypass or colon resection. 4. The nurse is caring for a patient in preadmission testing. The patient has been assigned a physical status classification by the American Society of Anesthesiologists of ASA III. Which assessment will support this classification? ANS: C An ASA III rating is a patient with a severe systemic disease, such as poorly controlled hypertension with an implanted pacemaker. ASA I is a normal healthy patient with no major illnesses or conditions. ASA II is a patient with mild systemic disease. ASA V is a moribund patient who is not expected to survive without the operation and includes patients with ruptured abdominal/thoracic aneurysm or massive trauma. a. Perioperative b. Preoperative c. Intraoperative d. Postoperative a. Major b. Urgent c. Elective d. Emergency a. Normal, healthy patient b. Denial of any major illnesses or conditions c. Poorly controlled hypertension with implanted pacemaker d. Moribund patient not expected to survive without the operation 315 A&E I Comprehensive Testbank 5. The patient has presented to the ambulatory surgery center to have a colonoscopy. The patient is scheduled to receive moderate sedation (conscious sedation) during the procedure. How will the nurse interpret this information? ANS: B Moderate sedation (conscious sedation) is used routinely for procedures that do not require complete anesthesia but rather a depressed level of consciousness. Not all patients who are treated on an outpatient basis receive moderate sedation. Regional anesthesia such as local anesthesia provides loss of sensation in an area of the body. General anesthesia is used for patients who need to be immobile and to not remember the surgical procedure. 6. The nurse is caring for a patient in the postanesthesia care unit who has undergone a left total knee arthroplasty. The anesthesia provider has indicated that the patient received a left femoral peripheral nerve block. Which assessment will be an expected finding for this patient? ANS: A Induction of regional anesthesia results in loss of sensation in an area of the body—in this case, the left leg. The peripheral nerve block influences the portions of sensory pathways that are anesthetized in the targeted area of the body. Decreased pulse, toes cool to touch, and cyanosis are indications of decreased blood flow and are not expected findings. Reports of pain in the left foot may indicate that the block is not working or is subsiding and is not an expected finding in the immediate postoperative period. 7. The nurse is preparing a patient for surgery. Which goal is a priority for assessing the patient before surgery? ANS: B The goal of the preoperative assessment is to identify a patient’s normal preoperative function and the presence of any risks to recognize, prevent, and minimize possible postoperative complications. Gathering appropriate equipment, planning care, and educating the patient and family are all important interventions that must be provided for the surgical patient; they are part of the nursing process but are not the priority reason/goal for completing an assessment of the surgical patient. 8. The nurse is completing a medication history for the surgical patient in preadmission testing. Which medication should the nurse instruct the patient to hold (discontinue) in preparation for surgery according to protocol? a. The procedure results in loss of sensation in an area of the body. b. The procedure requires a depressed level of consciousness. c. The procedure will be performed on an outpatient basis. d. The procedure necessitates the patient to be immobile. a. Sensation decreased in the left leg b. Patient report of pain in the left foot c. Pulse decreased at the left posterior tibia d. Left toes cool to touch and slightly cyanotic a. Plan for care after the procedure. b. Establish a patient’s baseline of normal function. c. Educate the patient and family about the procedure. d. Gather appropriate equipment for the patient’s needs. a. Warfarin b. Vitamin C 316 A&E I Comprehensive Testbank ANS: A Medications such as warfarin or aspirin alter normal clotting factors and thus increase the risk of hemorrhaging. Discontinue at least 48 hours before surgery. Acetaminophen is a pain reliever that has no special implications for surgery. Vitamin C actually assists in wound healing and has no special implications for surgery. Prednisone is a corticosteroid, and dosages are often temporarily increased rather than held. 9. The nurse is prescreening a surgical patient in the preadmission testing unit. The medication history indicates that the patient is currently taking an anticoagulant. Which action should the nurse take when consulting with the health care provider? ANS: B INR, PT (prothrombin time), APTT (activated partial thromboplastin time), and platelet counts reveal the clotting ability of the blood. Anticoagulants can be utilized for different conditions, but its action is to increase the time it takes for the blood to clot. This action can put the surgical patient at risk for bleeding tendencies. Typically, if at all possible, this medication is held several days before a surgical procedure to decrease this risk. Chest x-ray, BUN, and Na are diagnostic screening tools for surgery but are not specific to anticoagulants. 10. The nurse is encouraging the postoperative patient to utilize diaphragmatic breathing. Which priority goal is the nurse trying to achieve? ANS: B After surgery, patients may have reduced lung volume and may require greater effort to cough and deep breathe; inadequate lung expansion can lead to atelectasis and pneumonia. Purposely utilizing diaphragmatic breathing can decrease this risk. During general anesthesia, the lungs are not fully inflated during surgery and the cough reflex is suppressed, so mucus collects within airway passages. Diaphragmatic breathing does not manage pain; in some cases, if splinting and pain medications are not given, it can cause pain. Diaphragmatic breathing does not reduce healing time or decrease thrombus formation. Better, more effective interventions are available for these situations. 11. The nurse is caring for a postoperative patient on the medical-surgical floor. Which activity will the nurse encourage to prevent venous stasis and the formation of thrombus? ANS: C After general anesthesia, circulation slows, and when the rate of blood slows, a greater tendency for clot formation is noted. Immobilization results in decreased muscular contractions in the lower extremities; these c. Prednisone d. Acetaminophen a. Ask for a radiological examination of the chest. b. Ask for an international normalized ratio (INR). c. Ask for a blood urea nitrogen (BUN). d. Ask for a serum sodium (Na). a. Manage pain b. Prevent atelectasis c. Reduce healing time d. Decrease thrombus formation a. Diaphragmatic breathing b. Incentive spirometry c. Leg exercises d. Coughing 317 A&E I Comprehensive Testbank promote venous stasis. Coughing, diaphragmatic breathing, and incentive spirometry are utilized to decrease atelectasis and pneumonia. 12. The nurse is caring for a preoperative patient. The nurse teaches the principles and demonstrates leg exercises for the patient. The patient is unable to perform leg exercises correctly. What is the nurse’s best next step? ANS: B If the patient is unable to perform leg exercises, the nurse should look for circumstances that may be impacting the patient’s ability to learn. In this case, the patient can be anticipating the upcoming surgery and may be experiencing anxiety. The patient may also be in pain or may be fatigued; both of these can affect the ability to learn. Evaluation of educational methods may be needed, but in this case, principles and demonstrations are being utilized. Asking anyone “why” can cause defensiveness and may not help in attaining the answer. The nurse is aware that the patient is unable to do the exercises. Moving forward without ascertaining that learning has occurred will not help the patient in meeting goals. 13. Which nursing assessment will indicate the patient is performing diaphragmatic breathing correctly? ANS: A Positioning the hands along the borders of the rib cage allows the patient to feel movement of the chest and abdomen as the diaphragm descends and the lungs expand. As the patient takes a deep breath and slowly exhales, the middle fingers will touch while the chest wall contracts. The fingers will separate as the chest wall expands. The patient will feel normal downward movement of the diaphragm during inspiration and normal upward movement during expiration. 14. The nurse is caring for a postoperative patient with an abdominal incision. The nurse provides a pillow to use during coughing. Which activity is the nurse promoting? ANS: B Deep breathing and coughing exercises place additional stress on the suture line and cause discomfort. Splinting incisions with hands and a pillow provides firm support and reduces incisional pull. Providing a pillow during coughing does not provide distraction or reduce anxiety. Providing a pillow does not provide pain relief. Coughing can increase anxiety because it can cause pain. Analgesics provide pain relief. 15. The nurse is encouraging a reluctant postoperative patient to deep breathe and cough. Which explanation can the nurse provide that may encourage the patient to comply? a. Encourage the patient to practice at a later date. b. Assess for the presence of anxiety, pain, or fatigue. c. Ask the patient why exercises are not being done. d. Evaluate the educational methods used to educate the patient. a. Hands placed on the border of the rib cage with fingers extended will touch as the chest wall contracts. b. Hands placed on the chest wall with fingers extended will separate as the chest wall contracts. c. The patient will feel upward movement of the diaphragm during inspiration. d. The patient will feel downward movement of the diaphragm during expiration. a. Pain relief b. Splinting c. Distraction d. Anxiety reduction 318 A&E I Comprehensive Testbank ANS: D Deep breathing and coughing expel retained anesthetic gases and facilitate a patient’s return to consciousness. Although it is correct that a patient may experience atelectasis and pneumonia if deep breathing and coughing are not performed, the way this is worded sounds threatening and could be communicated in a more therapeutic manner. Deep breathing and coughing are encouraged every 2 hours while the patient is awake. Just clearing the throat does not remove mucus from deeper airways. 16. The nurse and the nursing assistive personnel are assisting a postoperative patient to turn in bed. To assist in minimizing discomfort, which instruction should the nurse provide to the patient? ANS: D Instruct the patient to place the right hand over the incisional area to splint it, providing support and minimizing pulling during turning. Closing one’s eyes, holding one’s breath, and holding the nurse’s shoulders do not help support the incision during a turn. 17. The nurse is preparing to assist the patient in using the incentive spirometer. Which nursing intervention should the nurse provide first? ANS: A Performing hand hygiene reduces microorganisms and should be performed first. Placing the patient in the correct position such as high Fowler’s for the typical postoperative patient or reverse Trendelenburg for the bariatric patient would be the next step in the process. Demonstration of use of the mouthpiece followed by the instruction to inhale slowly would be the last step in this scenario. 18. The nurse and the nursing assistive personnel (NAP) are caring for a group of postoperative patients who need turning, coughing, deep breathing, incentive spirometer, and leg exercises. Which task will the nurse assign to the NAP? ANS: D The nurse can delegate to the NAP to encourage patients to practice postoperative exercises regularly after instruction and to inform the nurse if the patient is unwilling to perform these exercises. The skills of a. “If you don’t deep breathe and cough, you will get pneumonia.” b. “You will need to cough only a few times during this shift.” c. “Let’s try clearing the throat because that will work just as well.” d. “Deep breathing and coughing will clear out the anesthesia.” a. “Close your eyes and think about something pleasant.” b. “Hold your breath and count to three.” c. “Grab my shoulders with your hands.” d. “Place your hand over your incision.” a. Perform hand hygiene. b. Explain use of the mouthpiece. c. Instruct the patient to inhale slowly. d. Place in the reverse Trendelenburg position. a. Teach postoperative exercises. b. Do nothing associated with postoperative exercises. c. Document in the medical record when exercises are completed. d. Inform the nurse if the patient is unwilling to perform exercises. 319 A&E I Comprehensive Testbank demonstrating and teaching postoperative exercises and documenting are not within the scope of practice for the nursing assistant. Doing nothing is not appropriate. 19. The nurse is providing preoperative teaching for the ambulatory surgery patient who will be having a cyst removed from the right arm. Which will be the best explanation for diet progression after surgery? ANS: A Patients usually receive a normal diet the first evening after surgery unless they have undergone surgery on GI structures. Implement diet intake while judging the patient’s response. For example, provide clear liquids such as water, apple juice, broth, or tea after nausea subsides. If the patient tolerates liquids without nausea, advance the diet as ordered. There is no need to stay on ice chips for several hours or clear liquids for 24 hours after this procedure. Putting a time frame on the progression is too prescriptive. Progression should be adjusted for the patient’s needs. 20. The nurse explains the pain relief measures available after surgery during preoperative teaching for a surgical patient. Which comment from the patient indicates the need for additional education on this topic? ANS: B Anesthesia will be provided during the procedure itself, and the patient should not experience pain during the procedure; however, this will not minimize the pain after surgery. Pain management is utilized after the postoperative phase. Inform the patient of interventions available for pain relief, including medication, relaxation, and distraction. The patient needs to know and understand how to take the medications that the health care provider will prescribe postoperatively. During the stay in the facility, the level of pain is frequently assessed by the nurses. Coordinating pain medication with postoperative exercises helps to minimize discomfort and allows the exercises to be more effective. 21. The nurse is making a preoperative education appointment with a patient. The patient asks if a family member should come to the appointment. Which is the best response by the nurse? ANS: C Including family members in perioperative education is advisable. Often a family member is a coach for postoperative exercises when the patient returns from surgery. If anxious relatives do not understand routine postoperative events, it is likely that their anxiety will heighten the patient’s fears and concerns. Preoperative preparation of family members before surgery helps to minimize anxiety and misunderstanding. An additional person is needed at the appointment if at all possible, and he or she needs to be involved in the process, not just a. “Start with clear liquids, soup, and crackers. Advance to a normal diet as tolerated.” b. “Stay with ice chips for several hours. After that, you can have whatever you want.” c. “Stay on clear liquids for 24 hours. Then you can progress to a normal diet.” d. “Start with clear liquids for 2 hours and then full liquids for 2 hours. Then progress to a normal diet.” a. “I will be asked to rate my pain on a pain scale.” b. “I will have minimal pain because of the anesthesia.” c. “I will take the pain medication as the provider prescribes it.” d. “I will take my pain medications before doing postoperative exercises.” a. “There is no need for an additional person at the appointment.” b. “Your family can come and wait with you in the waiting room.” c. “We recommend including family members at this appointment.” d. “It is required that you have a family member at this appointment.” 320 A&E I Comprehensive Testbank waiting in the waiting room; however, it is certainly not a requirement for actually completing the surgery that someone comes to this appointment. 22. The nurse is reviewing the surgical consent with the patient during preoperative education and finds the patient does not understand what procedure will be completed. What is the nurse’s best next step? ANS: A Surgery cannot be legally or ethically performed until the patient fully understands the need for a procedure and all the implications. It is the surgeon’s responsibility to explain the procedure, associated risks, benefits, alternatives, and possible complications. It is important for the nurse to pause with preoperative education to notify the health care provider of the patient’s questions. It is not within the nurse’s scope to explain the procedure. The nurse can certainly reinforce what the health care provider has explained, but the information needs to come from the health care provider. It is not prudent to ask a patient to sign a form for a procedure that he/she does not understand. 23. During preoperative assessment for a 7:30 AM (0730) surgery, the nurse finds the patient drank a cup of coffee this morning. The nurse reports this information to the anesthesia provider. Which action does the nurse anticipate next? ANS: A The recommendations before nonemergent procedures requiring general and regional anesthesia or sedation/ analgesia include fasting from intake of clear liquids for 2 or more hours. A delay in or cancellation of surgery will be in order for this case. Questions regarding components of the coffee, asking why, and evaluating the preoperative education may all be items to be addressed, especially from a performance improvement perspective, but at this time in caring for this patient, a delay or cancellation is in order to prevent aspiration. 24. The nurse has administered a preoperative medication to the patient going to surgery. Which action will the nurse take next? ANS: B Once a preoperative medication has been administered, instruct the patient to call for help when getting out of bed to prevent falls. For patient safety, explain the purpose of a preoperative medication and its effects. Notifying the operating suite that the medication has been given may be part of a facilities procedure but is not the best next step. It is important to have the patient sign consents before the patient has received medication that may make him/her drowsy. Wasting unused medication according to policy is important but is not the best next step. 25. The nurse has completed a preoperative assessment for a patient going to surgery and gathers assessment data. Which will be the most important next step for the nurse to take? a. Notify the health care provider about the patient’s question. b. Explain the procedure that will be completed. c. Continue with preoperative education. d. Ask the patient to sign the form. a. A delay in or cancellation of surgery b. Questions regarding components of the coffee c. Additional questions about why the patient had coffee d. Instructions to determine what education was provided in the preoperative visit a. Notify the operating suite that the medication has been given. b. Instruct the patient to call for help to go to the restroom. c. Waste any unused medication according to policy. d. Ask the patient to sign the consent for surgery. 321 A&E I Comprehensive Testbank ANS: A The most important step is notifying the operating suite of the patient’s latex allergy. Many products that contain latex are used in the operating suite and the postanesthesia care unit (PACU). When preparing for a patient with this allergy, special considerations are required from preparation of the room to the types of tubes, gloves, drapes, and instruments utilized. Obtaining vital signs, documenting, and administering medications are all part of the process and should be done—with the latex allergy in mind. However, making sure that the patient has a safe environment is the first step. 26. The nurse is preparing a patient for a surgical procedure on the right great toe. Which action will be mostimportant to include in this patient’s preparation? ANS: C Because errors have occurred in the past with patients undergoing the wrong surgery on the wrong site, the universal protocol guidelines have been implemented and are used with all invasive procedures. Part of this protocol includes marking the operative site with indelible ink. Knowing where the family is during a procedure, placing the patient in a clean gown, and asking the patient to remove all hairpins and cosmetics are important but are not most important in this list of items. 27. The circulating nurse is caring for a patient intraoperatively. Which primary role of the circulating nurse will be implemented? ANS: B The circulating nurse is an RN who remains unscrubbed and uses the nursing process in the management of patient care activities in the OR suite. The circulating nurse also manages patient positioning, antimicrobial skin preparation, medications, implants, placement and function of intermittent pneumatic compression (IPC) devices, specimens, warming devices and surgical counts of instruments, and dressings. The RN first assistant collaborates with the surgeon by handling and cutting tissue, using instruments and medical devices, providing exposure of the surgical area and hemostasis, and suturing. The scrub nurse, who can be a registered nurse, a licensed practical nurse, or a surgical technologist, maintains the sterile field, assists with applying the sterile drapes, and hands sterile instruments and supplies to the surgeon. 28. The nurse is caring for a patient in the preoperative holding area of an ambulatory surgery center. Which nursing action will be most appropriate for this area? a. Notify the operating suite that the patient has a latex allergy. b. Document that the patient had a bath at home this morning. c. Administer the ordered preoperative intravenous antibiotic. d. Ask the nursing assistive personnel to obtain vital signs. a. Place the patient in a clean surgical gown. b. Ask the patient to remove all hairpins and cosmetics. c. Ascertain that the surgical site has been correctly marked. d. Determine where the family will be located during the procedure. a. Suturing the surgical incision in the OR suite b. Managing patient care activities in the OR suite c. Assisting with applying sterile drapes in the OR suite d. Handing sterile instruments and supplies to the surgeon in the OR suite 322 A&E I Comprehensive Testbank ANS: D The temperature in the preoperative holding area and in adjacent operating suites is usually cold. Offer the patient an extra warm blanket. Counts are taken by the circulating and scrub nurses in the operating room. Emptying a urinary drainage bag and checking the surgical dressing occur in the postanesthesia care unit, not in the holding area. 29. The nurse is caring for a patient in the operating suite. Which outcome will be most appropriate for this patient at the end of the intraoperative phase? ANS: A A primary focus of intraoperative care is to prevent injury and complications related to anesthesia, surgery, positioning, and equipment use, including use of the electrical cautery grounding pad for the prevention of burns. The perioperative nurse is an advocate for the patient during surgery and protects the patient’s dignity and rights at all times. Signs and symptoms of infection do not have the time to present during the intraoperative phase. During the intraoperative phase, the patient is anesthetized and unconscious and typically has an endotracheal tube that prevents conversation. Nausea, vomiting, and pain typically begin in the postoperative phase of the experience. 30. The nurse is concerned about the skin integrity of the patient in the intraoperative phase of surgery. Which action will the nurse take to minimize skin breakdown? ANS: B Although it may be necessary to place a patient in an unusual position, try to maintain correct alignment and protect the patient from pressure, abrasion, and other injuries. Special mattresses, use of foam padding, and attachments to the operating suite table provide protection for the extremities and bony prominences. Bathing before surgery helps to decrease the number of microbes on the skin. Periodically adjusting the patient during the surgical procedure is impractical and can present a safety issue with regard to maintaining sterility of the field and maintaining an airway. Measuring the time the patient is in one position may help with monitoring the situation but does not prevent skin breakdown. 31. The nurse is assessing a postoperative patient with a history of obstructive sleep apnea for airway obstruction. Which assessment finding will best alert the nurse to this complication? a. Count the sterile surgical instruments. b. Empty the urinary drainage bag. c. Check the surgical dressing. d. Apply a warm blanket. a. The patient will be free of burns at the grounding pad. b. The patient will be free of nausea and vomiting. c. The patient will be free of infection. d. The patient will be free of pain. a. Encouraging the patient to bathe before surgery b. Securing attachments to the operating table with foam padding c. Periodically adjusting the patient during the surgical procedure d. Measuring the time a patient is in one position during surgery a. Drop in pulse oximetry readings b. Moaning with reports of pain c. Shallow respirations d. Disorientation 323 A&E I Comprehensive Testbank ANS: A One of the greatest concerns after general anesthesia is airway obstruction, especially in patients with obstructive sleep apnea. A drop in oxygen saturation by pulse oximetry is a sign of airway obstruction in patients with obstructive sleep apnea. Weak pharyngeal/laryngeal muscle tone from anesthetics; secretions in the pharynx, bronchial tree, or trachea; and laryngeal or subglottic edema also contribute to airway obstruction. In the postanesthetic patient, the tongue is a major cause of airway obstruction. Shallow respirations are indicative of respiratory depression. Moaning and reports of pain are common in all surgical patients and are an expected event. Disorientation is common when first awakening from anesthesia but can be a sign of hypoxia. 32. The nurse is caring for a patient in the operating suite who is experiencing hypercarbia, tachypnea, tachycardia, premature ventricular contractions, and muscle rigidity. Which condition does the nurse suspect the patient is experiencing? ANS: A A life-threatening, rare complication of anesthesia is malignant hyperthermia. Malignant hyperthermia causes hypercarbia, tachycardia, tachypnea, premature ventricular contractions, unstable blood pressure, cyanosis, skin mottling, and muscular rigidity. It often occurs during anesthesia induction. Hypoxia would manifest with decreased oxygen saturation as one of its signs and symptoms. Fluid imbalance would be assessed with intake and output and can manifest with tachycardia and blood pressure fluctuations but does not have muscle rigidity. Hemorrhage can manifest with tachycardia and decreased blood pressure, along with a thready pulse. Usually some sign or symptom of blood loss is noted (e.g., drains, incision, orifice, and abdomen). 33. The nurse is caring for a postoperative patient who has had a minimally invasive carpel tunnel repair. The patient has a temperature of 97° F and is shivering. Which reason will the nurse most likely consider as the primary cause when planning care? ANS: A The operating suite and recovery room environments are extremely cool. The patient’s anesthetically depressed level of body function results in lowering of metabolism and a fall in body temperature. Although the patient is dressed in a gown and there are air currents in the operating room, these are not the primary reasons for the low temperature. Also, the patient in this type of case does not have a large open body cavity to contribute to heat loss. 34. The nurse is monitoring a patient in the postanesthesia care unit (PACU) for postoperative fluid and electrolyte imbalance. Which action will be most appropriate for the nurse to take? ANS: C a. Malignant hyperthermia b. Fluid imbalance c. Hemorrhage d. Hypoxia a. Anesthesia lowers metabolism. b. Surgical suites have air currents. c. The patient is dressed only in a gown. d. The large open body cavity contributed to heat loss. a. Encourage copious amounts of water. b. Start an additional intravenous (IV) line. c. Measure and record all intake and output. d. Weigh the patient and compare with preoperative weight. 324 A&E I Comprehensive Testbank Accurate recording of intake and output assesses renal and circulatory function. Measure and record all sources of intake and output. Encouraging copious amounts of water in a postoperative patient might encourage nausea and vomiting. In the PACU, it is impractical to weigh the patient while waking from surgery, but in the days afterward, it is a good assessment parameter for fluid imbalance. Starting an additional IV is not necessary and is not important at this juncture. 35. The nurse is caring for a patient in the postanesthesia care unit. The patient asks for a bedpan and states to the nurse, “I feel like I need to go to the bathroom, but I can’t.” Which nursing intervention will be most appropriate initially? ANS: A Depending on the surgery, some patients do not regain voluntary control over urinary function for 6 to 8 hours after anesthesia. Palpate the lower abdomen just above the symphysis pubis for bladder distention. Another option is to use a bladder scan or ultrasound to assess bladder volume. The nurse must assess before deciding if the patient can try again. Not everyone feels as if they need to go but can’t after surgery. Calling the health care provider is not the initial best action. The nurse needs to have data before calling the provider. 36. The postanesthesia care unit (PACU) nurse transports the inpatient surgical patient to the medical-surgical floor. Before leaving the floor, the medical-surgical nurse obtains a complete set of vital signs. What is the rationale for this nursing action? ANS: B Before the PACU nurse leaves the acute care area, the staff nurse assuming care for the patient takes a complete set of vital signs to compare with PACU findings. Minor vital sign variations normally occur after transporting the patient. The PACU nurse reviews the patient’s information with the medical-surgical nurse, including the surgical and PACU course, physician orders, and the patient’s condition. While vital signs may or may not be the first action in a head-to-toe assessment, this is not the rationale for this situation. While following policy or ascertaining that the floor nurse checks on the patient are good reasons for safe care, they are not the best rationale for obtaining vital signs. 37. The nurse is caring for a patient who will undergo a removal of a lung lobe. Which level of care will the patient require immediately post procedure? ANS: B a. Assess the patient for bladder distention. b. Encourage the patient to wait a minute and try again. c. Inform the patient that everyone feels this way after surgery. d. Call the health care provider to obtain an order for catheterization. a. This is done to complete the first action in a head-to-toe assessment. b. This is done to compare and monitor for vital sign variation during transport. c. This is done to ensure that the medical-surgical nurse checks on the postoperative patient. d. This is done to follow hospital policy and procedure for care of the surgical patient. a. Acute care—medical-surgical unit b. Acute care—intensive care unit c. Ambulatory surgery d. Ambulatory surgery—extended stay 325 A&E I Comprehensive Testbank Patients undergoing extensive surgery and requiring anesthesia of long duration recover slowly. If a patient is undergoing major surgery such as a procedure on the lung, a stay in the hospital and specifically in the intensive care unit is required to monitor for potential risks to well-being. This patient would require more care than can be provided on a medical-surgical unit. It is not appropriate for this type of patient to go home after the procedure or to stay in an extended stay area of an ambulatory surgery area because of the complexity and associated risks. 38. The nurse is caring for a group of patients. Which patient will the nurse see first? ANS: A For patients who have had eye, intracranial, or spinal surgery, coughing may be contraindicated because of the potential increase in intraocular or intracranial pressure. The nurse will need to see this patient first to control the cough and intraocular pressure. All the rest are normal postoperative patients. Leg exercise should not be performed on the operative leg with vascular surgery. A patient after knee surgery should receive heparin and be wearing intermittent pneumatic compression devices; while the nurse will check on the patient, it does not have to be first. Monitoring vital signs after surgery is required and this is the standard schedule. 39. The nurse demonstrates postoperative exercises for a patient. In which order will the nurse instruct the patient to perform the exercises? 1. Turning 2. Breathing 3. Coughing 4. Leg exercises ANS: A The sequence of exercises is leg exercises, turning, breathing, and coughing. MULTIPLE RESPONSE 1. The nurse is participating in a “time-out.” In which activities will the nurse be involved? (Select all that apply.) ANS: A, B, C a. A patient who had cataract surgery is coughing. b. A patient who had vascular repair of the right leg is not doing right leg exercises. c. A patient after knee surgery is wearing intermittent pneumatic compression devices and receiving heparin. d. A patient after surgery has vital signs taken every 15 minutes twice, every 30 minutes twice, hourly for 2 hours then every 4 hours. a. 4, 1, 2, 3 b. 1, 2, 3, 4 c. 2, 3, 4, 1 d. 3, 1, 4, 2 a. Verify the correct site. b. Verify the correct patient. c. Verify the correct procedure. d. Perform “time-out” after surgery. e. Perform the actual marking of the operative site. 326 A&E I Comprehensive Testbank A time-out is performed just before starting the procedure for final verification of the correct patient, procedure, site, and any implants. The marking and time-out most commonly occur in the holding area, just before the patient enters the OR. The individual performing surgery and who is accountable for it must personally mark the site, and the patient must be involved if possible. 2. The nurse is using a forced air warmer for a surgical patient preoperatively. Which goals is the nurse trying to achieve? (Select all that apply.) ANS: B, D, E Evidence suggests that pre-warming for a minimum of 30 minutes may reduce the occurrence of hypothermia. Prevention of hypothermia (core temperature < 36° C) helps to reduce complications such as shivering, cardiac arrest, blood loss, SSI, pressure ulcers, and mortality. 3. The nurse is caring for a postoperative patient with an incision. Which actions will the nurse take to decrease wound infections? (Select all that apply.) ANS: A, E Performing hand hygiene before and after contact with the patient helps to decrease the number of microorganisms and break the chain of infection. Maintaining blood glucose levels at less than 150 mg/dL has resulted in decreased wound infection. Removing unwanted hair by clipping instead of shaving decreases the numbers of nicks and cuts caused by a razor and the potential for the introduction of microbes. The patient is postoperative; administration of an antibiotic 60 minutes before the surgical incision supports the defense against infection preoperatively. Providing a bath and linen change daily is positive but is not necessarily important for infection control. Many surgeons prefer to change surgical dressings the first time so they can inspect the incisional area, but this is done before 2 days postoperatively. 4. The nurse is preparing for a patient who will be going to surgery. The nurse screens for risk factors that can increase a person’s risks in surgery. What risk factors are included in the nurse’s screening? (Select all that apply.) ANS: A, C, D, E a. Induce shivering. b. Reduce blood loss. c. Induce pressure ulcers. d. Reduce cardiac arrests. e. Reduce surgical site infection. a. Maintain normoglycemia. b. Use a straight razor to remove hair. c. Provide bath and linen change daily. d. Perform first dressing change 2 days postoperatively. e. Perform hand hygiene before and after contact with the patient. f. Administer antibiotics within 60 minutes before surgical incision. a. Age b. Race c. Obesity d. Nutrition e. Pregnancy f. Ambulatory surgery 327 A&E I Comprehensive Testbank Very young and old patients are at risk during surgery because of immature or declining physiological status. Normal tissue repair and resistance to infection depend on adequate nutrients. Obesity increases surgical risk by reducing respiratory and cardiac function. During pregnancy, the concern is for the mother and the developing fetus. Because all major systems of the mother are affected during pregnancy, risks for operative complications are increased. Race and ambulatory surgery are not risks associated with a surgical procedure. 5. The nurse is providing preoperative education and reviews with the patient what it will be like to be in the surgical environment. Which points should the nurse include in the teaching session? (Select all that apply.) ANS: B, D, E The surgical staff is dressed in special clothing, hats, and masks—all for infection control. Families are not allowed in the operating suite for several reasons, which include infection control and sterility. The nurse is there as the coordinator and patient advocate during a surgical procedure. The rooms are very bright so everyone can see, and the operating table is very uncomfortable for the patient. 6. The operating room nurse is providing a hand-off report to the postanesthesia care unit (PACU) nurse. Which components will the operating room nurse include? (Select all that apply.) ANS: A, B, E, F The surgical teams report will include topics such as the type of anesthesia provided, vital sign trends, intraoperative medications, IV fluids, estimated blood and urine loss, and pertinent information about the surgical wound (e.g., dressings, tubes, drains). When the patient enters the PACU, the nurse and members of the surgical team discuss his or her status. A standardized approach or tool for hand-off communications assists in providing accurate information about a patient’s care, treatment and services, current condition, and any recent or anticipated changes. The hand-off is interactive, multidisciplinary, and done at the patient’s bedside, allowing for a communication exchange that gives caregivers the chance to dialogue and ask questions. Insurance data and family location are unnecessary. 7. The nurse is caring for a group of postoperative patients on the surgical unit. Which patient assessments indicate the nurse needs to follow up? (Select all that apply.) a. The operative suite will be very dark. b. The family is not allowed in the operating suite. c. The operating table or bed will be comfortable and soft. d. The nurses will be there to assist you through this process. e. The surgical staff will be dressed in special clothing with hats and masks. a. IV fluids b. Vital signs c. Insurance data d. Family location e. Anesthesia provided f. Estimated blood loss a. Patient with abdominal surgery has patent airway. b. Patient with knee surgery has approximated incision. c. Patient with femoral artery surgery has strong pedal pulse. d. Patient with lung surgery has 20 mL/hr of urine output via catheter. e. Patient with bladder surgery has bloody urine within the first 12 hours. 328 A&E I Comprehensive Testbank ANS: D, F Thready pulse, low blood pressure, and urine output of 20 mL/hr need to have follow-up by the nurse. Hemorrhage results in a fall in blood pressure; elevated heart and respiratory rates; thready pulse; cool, clammy, pale skin; and restlessness. Notify the surgeon if these changes occur. If the patient has a urinary catheter, there should be a continuous flow of urine of approximately 30 to 50 mL/hr in adults; this patient requires follow-up since the output is 20 mL/hr. All the rest are normal findings. A patent airway, a strong distal pulse, and approximated incision are all normal findings. Surgery involving portions of the urinary tract normally causes bloody urine for at least 12 to 24 hours, depending on the type of surgery. f. Patient with appendix surgery has thready pulse and blood pressure is 90/60. 329 [Show More]

Last updated: 1 year ago

Preview 1 out of 329 pages

Reviews( 0 )

$24.00

Add to cart

Instant download

Can't find what you want? Try our AI powered Search

OR

GET ASSIGNMENT HELP
146
0

Document information


Connected school, study & course


About the document


Uploaded On

Mar 18, 2021

Number of pages

329

Written in

Seller


seller-icon
Aplus exams

Member since 3 years

2 Documents Sold


Additional information

This document has been written for:

Uploaded

Mar 18, 2021

Downloads

 0

Views

 146

Document Keyword Tags

Recommended For You


$24.00
What is Browsegrades

In Browsegrades, a student can earn by offering help to other student. Students can help other students with materials by upploading their notes and earn money.

We are here to help

We're available through e-mail, Twitter, Facebook, and live chat.
 FAQ
 Questions? Leave a message!

Follow us on
 Twitter

Copyright © Browsegrades · High quality services·